0% found this document useful (0 votes)
5K views499 pages

Ocular Conditions and Management Strategies

This case describes a 6.5-year-old male with a right eye growth. On exam, a 2mm gelatinous mass with neovascularization was found on the right conjunctiva extending onto the cornea. This presentation is classic for conjunctival intraepithelial neoplasia (CIN). CIN is the most common conjunctival malignancy in the US and usually presents as an elevated gelatinous mass in the interpalpebral area. Treatment involves surgical excision and cryotherapy due to the lesion's premalignant nature. The patient's medication, phenytoin, can cause nystagmus as a known side effect.

Uploaded by

gigaslayerx
Copyright
© © All Rights Reserved
We take content rights seriously. If you suspect this is your content, claim it here.
Available Formats
Download as PDF, TXT or read online on Scribd
0% found this document useful (0 votes)
5K views499 pages

Ocular Conditions and Management Strategies

This case describes a 6.5-year-old male with a right eye growth. On exam, a 2mm gelatinous mass with neovascularization was found on the right conjunctiva extending onto the cornea. This presentation is classic for conjunctival intraepithelial neoplasia (CIN). CIN is the most common conjunctival malignancy in the US and usually presents as an elevated gelatinous mass in the interpalpebral area. Treatment involves surgical excision and cryotherapy due to the lesion's premalignant nature. The patient's medication, phenytoin, can cause nystagmus as a known side effect.

Uploaded by

gigaslayerx
Copyright
© © All Rights Reserved
We take content rights seriously. If you suspect this is your content, claim it here.
Available Formats
Download as PDF, TXT or read online on Scribd

2.5.

CASE 9

.57

acute IK includ es salmon patch lesions (strom al neovasc ularizat ion), stromal

edema , conjunct ival inj ect ion , anterior chamber reaction a nd keratic precipi

tates. C hronic cases ca n res ult in stroma l scarring, ghost vessels and irregula r

astigmat ism.

Question 2: correct answer a - inherited (autosomal dominant)

Posterior poly morphous dystrophy a nd Fuch's endotheli a l dystrophy are au

tosomal dominant inherited co nditions. l\Iacu la r dystrophy is an a utosoma l

recessive inherited con dit.ion. Co ngeni ta l sy philis is the most common cause of

intersti t ia l kera titi s.

Question 3: correct answer d - pseudotumor ce rebri

Int.erst itial kerat it is, Hu tc hinson 's teet h and deafness co mp r ise a clinical t ri ad

(Hutchinson 'S triad) found in congen ita l syphilis patients . Neuro-op hthalmic

findings s uch as pap illedem a a nd optic neuritis occur in tertiary syphilis .

Question 4: correct answer a - topical hypertonic solution qid

Most patients with poste rior polymo rphous dystrophy do not require treat

ment; however , pat ie nts shou ld be monitored closely for gla ucoma . If the pa

tient develops corneal ed ema (as in this case ), treat ment p rotocol is simila r

to tha t of Fuch 's endotheli a l dys trophy. At this patient's follovv-up visit , go

ni oscopy should be performed to insure t hat no anterior chamber abnormalities

are present.

Fuch's endothelial dystrophy trea tment options include topica l sodium

chloride (5%) dro ps (qid) a nd ointment (q hs) , ha ir dryer dehy dratio n (5-10

minutes each morning) and a ntiglaucoma medi cat ions (if JOP 20 or hi gher ) (4).

Ruptured corneal bullae should be t reated as a co rneal abrasion. Significant

pain or vision loss warrants co ns ult for co rneal transplant (penetrating kerato

plasty) .

Disciform keratitis treat ment req uires predforte qid a nd prophylactic t reat

ment with viroptic qid; see herpes simplex case for mo re details.

Macular dystrophy t reat ment involves a corneal transplant or PTK; this

is us ually required in the 3rd decad e, as the patient sta rts to develop vi sion

loss (2). Recurrence after keratoplasty is uncommon (4).

Inte rstitial keratitis trea.tment, during act ive cases, is wi th to pical steroids.

Management should include testing or referral for und erlying systemic etiology.

If kerato uveit.is is present, t reatment includ es to pical steroid s (pred forte acetate

1% every 2 to 4 hours , then taper ) a nd cycloplegics (cyclope ntalate 1% qid ) (2).

Old, inad ive interst itial keratitis can be treated wit.h a corneal transplant.

Question 5: correct answer c - chlorpronlazine

CHA PTER 2. CONJ UNCTIVA/C ORNEA

58

Recall that chlorpromazine can res ult in pigment deposition on the corneal
endotheli um.

2.6

Case 10

Demographics
Age/race /gender: 6.5 year-old white male; reti red welder
Chief complaint: growth o n right eye
History of present illness
Location: OD

Severity: unknow n

Nature of onset: gradual

Duration: 2-3 years

Secondary complaints/symptoms: forei gn-body sensation in right eye


Family ocular history
mother: age-related macular degeneration
Pati e nt medical history: seizures, controlled with medication
Medications take n by patient: phenytoin
Patie nt allergy history: NKDA
Review of systems
N eurologic: seizures
Mental status
Orientation: ori ented to time, place , a nd person
Mood: a ppropriate
Affect: app ropriate
Clinical findings

BVA:
~I--~II~D
=i~s~ta-n-c-e~~N
I ~e ar-'1

2.6. CASE 10

59

Pupils: PERRL , negative APD

EOMs: full , no rest ri ctions , nystagmus superi orly OD, OS

Confrontation fi e lds: full to finger co unting OD, OS

Slit lamp

lids/lashes /adnexa: unremarkable OD, OS


co njunct iva: OD: 2m m x 2mm elevated, gelatinous mass wit h neo
vascul a rization. Normal OS.
cornea: OD: mass from conjunctiva extend s 1 mm onto nasal cornea.
Normal OS.
anterior chamb e r: d eep and quiet OD, OS
iris: normal OD , OS
lens: mild nuclear sclerosis OD, OS
vitreous: posteri or vitreous detachment OD. clear OS.
lOPs: 16 mmHg OD , 15 mmHg OS @ 2:30 PfI'I by applanation tonom
etry
Fundus OD
C / D: 0.30 H /V wit h healthy rim tiss ue
macula: normal
posterior pole: normal
periphery: inferior lattice degeneration with no atrophic holes
Fundus OS

C/D: 0.35 H / V wi t h heal t hy ri m ti ssue


macula: normal
posterior pole: normal
periphery: inferior/tempo rallattice degener ation with no atrop hic
holes
Blood pressure: 117/ 65 Il1mHg, right a rm , sitting
Pulse: 68 bpm , regular
Question 1: Which of the following is the MOST likely diagnosis of
this patient 's ocular condition?
a . racial mela nosis
b. primary acquireclmelanosis (PAi'vI)
c. conjunctival nevus
d. conju nctival intraepitheli al neoplasia (eIN)
e . ly mphoma
f. pyogenic gran uloma

60

CHAPTER 2.

CONJUNCTIVA/CORNEA

Question 2: Malignant melanomas of the conjunctiva most commonly


arise from which of the following?
a. pre-exist ing nevus
b. primary acquired melanosis (PAM)
c. de novo
d. trau ma
Question 3: Which of the following listed is a known ocular side
effect of this patient's medication?
a. retinoschisis
b. cataract
c. nystagmus
d. color vision defects
Question 4: Which of the following is the MOST appropriate treat
ment/ management for this patient's ocular condition?
a. curettage
b. topica l antibiotic-steroid drops
c. re fer for surgica l excision with supplementa l cryotherapy
d. slit lamp photograph and monitor again in 6 months
Question 5: Which of the following m e dications can cause a retinal
detachment?
a. tropica mide
b. bropheniramine
c. amiodarone
d. pilocarpine

ANSWERS
Question 1: correct answer d - conjunctival intraepithelial neoplasia

(eIN)

This case gives t he classic presentation of eIN (also called carcinoma in sit u).

This condition , alt hough rare , is the most co mmon conjunctival ma lignancy

in the Uni ted St ates (15). eIN is a unilateraJ, premaligna nt condition (to

squamous cell carcinoma). 95% of cases are found at the limbus, within

t he interpalpe bral fissure (5). Presentation can vary, but the most common

appearance is a n elevated, gelatinous mass with neovasculari zat ion ; app roxi

mately 10% of cases exhibit leukoplakia (keratinization) (10).

2.6. CASE 10

61

Summary of other answer choices

Conjunctival squamous cell carcinomas are rare, slow


growing , malignant tumors th a t typica lly present in elderly, Cau
casian (90%) males (81 %); this malignancy is associated with ul
traviolet radi a tion and is most co mrnonly deri ved fr om CIN (6) .

Racial melanosis is a benign condition, common in African Ameri cans , with


no malignant pot ential It is cha rac t erized by flat , pat chy areas of pigmen
tati on, most con centrated at the limbus, that a re usuall y bilateral a nd often
asy mmetric.
PAM is acquired pig mentation, common in elderly white patients, that is ra re,
unilateral , and has prema ligna nt potential ; 30% of cases progress to malignant
melanoma (5). Th e patches, which can be located anyw here on t he conjun c
ti va, a re usuall y flat wit h indistinct margins . P Ard res ul ts from a proliferation
of intraepitheli a l melanocytes (10). PA?lI can be benign with no pote ntial for
malign a ncy - biopsy is warra nted to determine whether m alignant.
Conjunctival nevi represent a benign proliferation of melanocy tes that presents
a round pub erty or early adulthood (within 1st two decades of life). The lesions
a re rare , ty pically unila teral and fl a t (or slightly elevated) and occasionally
non-pig mented (30%) (1 ) . At puberty, it is not uncommon for the size and
darkness of nevi to increase . The most common location for a conjunctival
nev us is the juxtalimbal area, followed by the plica and ca runcle (1) . If the
nevus is located in a n unu s ua l area (e.g. cornea, forniceal conjuncti va) , has
irregular borders, quickly in creases in size, or is s uspicious in color, a biopsy
is warranted. Althoug h nevi rarely become ma.lignan t, approximately 33% of
conjunctival melanomas arise from nevi; 20% of these cases result in death (5) .
Conjunctival IYITlphomas are smoot h, fleshy, s ubconj unctival salmo n- colored
patches that typi cally occur in middle-aged adul ts. Prognosis varies de pending
on whet her lesion is benign (e .g. reactive ly mphoid hy perpl asia ) or malignant
(e .g. Non-Hodgkin 's ly mphoma , mucosa associated ly mphoid ti ss ue (MALT)) .
Pyogenic granulomas are fl eshy vascul ar (red ), gr a nulomato us masses (he
ma ngiomas) that typica lly result from trauma (often very minor ) or s urgery (1) (6).
P yoge nic gra nulomas typically grow very rapidly over a period of a few weeks
to an average size of a ha lf a n inch (14).
Question 2: correct answer b - primary acquire d melanosis (PAM)
IVlalignant luelanoITlas of th e conjunctiva ca n be pigmented or non-pigme nted
a nd most commonly arise from P AM (50 - 75% of cases) (6) , less commonly
from a pre- existing ne vus (33% of cases) (5) a nd, rarely, de novo. The most
impor tant prognosti c indi cator for progression to malig na ncy is thi ckness of

62

CH APTER 2.

CONJUNCTIVA/CORNEA

t he lesion (5). These lesion s are rare (much less common t han a rare choroidal

mela noma) but are more da ngero us (more likely to metastasize) than con

junct ival squamo ll s cell carcinomas. Recall tha t mela nomas occur beca use o f

un controlled proliferation of mela nocytes.

Question 3: correct answer c - nystagmus

The follo wing is a brief s ummary of drugs th at can result in cata racts, nystag

mus a nd color vision defects (3):

cataracts: chlorpromaz ine (stell ate cararacts), a miodarone (lens de


posits) , miotics (lens vac uoles), gold salts (gold deposits) and corticos
teroids (posterior subca psul a r cataracts)
nystagmus: phe ny toin (dilantin) , phenobarbital (lum ina l), salicylates
blue/ye llow color vision defects : digoxin (digitalis)

Questi on 4: correct answer c - refer for surgical excision with sup

plemental cryotherapy (2).

PAM should be followed every 6 months I"ith cl inical documentation (e.g. slit

lam p photographs); if patches become nod ular , les ion is li kely malign a nt (2).

Treat ment involves surgica l exc ision with cryotherapy (2) .

Conjunctival lymphomas requ ire tissue biopsy wit h immunohi stoc hemical

studies due to wid e spect rum of disease outcomes (2) (6) . If malignant , external

beam radiatio n of conjuncti va is utilized for t reatment (2 ).

P yogenic granulomas trea tment opt ions include topical antibioti c-steroid

drops (e.g. tobradex) or excision (4.).

Conjunctival nevi and racial melanosis do no t requi re treatment. If t he ne

vus becomes a maligna nt melano ma, treatme nt is lo cal excision with cryother

apy.

Any malig na nt conjunct ival lesion wit h metastasis (e.g. ly mph


node in vol vement) warra nts a dditiona l treatment wit h chemother
apy.

Questi o n 5: correct answer d - pilocarp ine

Pilocarpine and other miotics can cause retinal breaks , leading to a detachment.

11

2.7

11

white

student,

ocular irritation

Chief
History of

illness

Location: OD
severe
Nature of onset:
Duration:
tated

more irri

ago

tient
on eye
and condition is
lens wearer, discontinued lenses 8 mont hs ago;
a child
ocular
mother: snowflake cataracts
Patient medical history: diabetes mellitns
of

1), controlled
I}t

:i'vledications taken

70

history: NKDA

Patient

medical history
Inother:
Review of systems
Endocrine: mild

moderate

Mental status
Orientation: oriented to
Mood:

persOll

qid

insulin;

64

CHAPTER 2. CONJUNCTIVA/CORNEA

Affect: appropriate
Clinical findings
Distance I Near

Pupils: PERRL, negat ive APD

EOI'vIs: full,

110

restrictions

Confrontation fields: full to finger co unting OD , OS

Slit lamp

lids/lashes/adnexa: 1+ sleeving at base of lashes OD, OS


conjunctiva: OD: 2+ sectoral inject ion , OS: trace diffuse injection
cornea: OD: geographic co rn eal ulcer with heaped- up vesicu lar le
sions surrounding the defec ts. as: normal
anterior chamber: dee p and quiet aD , Os
iris: normal OD, as
lens: clear OD , OS
vitreous: clear OD, as
lOPs: 16 mmHg aD , 15 mmHg Os
etry

2:30 PM by applanatio n tonom

Fundus OD
C / D: 0.20 H/V with healthy rim tissue
macula: normal
posterior pole: normal
periphery: unremarka ble
Fundus OS
C/D: 0.25 H/ V with healthy rim tiss ue
macula: normal
posterior pole: normal
periphery: unrema rkable
Blood pressure: ll5/65 mmHg, right arm , sitt ing
Pulse: 68 bpm , regular
Question 1: Which of the following is the MOST likely diagnosis of
this patient's ocular condition?
a. Herpes Simplex Virus (HSV) epitheliopathy
b. Acant hamoeba

2.7. CASE 11

65

c. Recurrent corneal erosion


d. Th)'geson's superficia l pun ctate kerato pat hy
e . Exposure kera topat hy

Question 2: What is the :MOST likely cause of this condition?

a . idiopathic etiology
b. di a betes
c. previou s tra uma
d. viral ill fect ion

Question 3: Which of the following conditions is NOT a recognized


associa ted complication of HSV?

a . bl epharoconjunctivi tis
b. keratouveitis
c. disciform keratitis
d. mac ular edema

Question 4: Which of the following c onditions can result in corneal


hypoesthesia?

a. herpes simplex virus (HSV ) epitheliopathy


b. aca nthamoeba
c. recurrent corn eal erosion
d. expos ure kera topathy
e.

t"lNO

of a bove

f. t hree of t he above

Question 5: vVhich of the following is the MOST appropriate treat


ment/management for this patient's ocular condition?

a. mild topica l steroids, 1 drop 4x's daily for 1 week


b. a nti-par asitic medicati ons
c. aggressive lubri cation with artificial tears, 1 drop every 2 hours for 1 week
d . viroptic solu t ion , 1 drop every 2 hours

CHAPTER 2.

66

CONJUNCTIVA/CORNEA

Question 6: Which of the following conditions is NOT a cause of


exposure keratopathy?
a. nocturnal lagophthalmos
b. floppy eyeli d sy ndrome
c. thyroid eye disease
d. entropion
e. eyelid co loboma

f. Bell's palsy

ANSWERS
Question 1: correct answer a - Herpes Simplex Virus (HSV) e pithe
liopathy
This case provides a classic desc ription of Herpes Simp lex ker at it is; a yo ung
patient wit h recent stress (fin als) with a hi story of cold sores experiencing a
sore, pa inful eye. The patient 's cond ition was wo rsening as a result o f predforte
acetate treat ment started 4 days ago. The following is a summary of important
cl inical poin ts regarding Herpes Simplex Virus (HSV) (7) (l6):
HSV is the 2nd most common ca use (trauma is first ) of corneal blindness
in the Uni ted States (5); abou t 50.000 new cases of HSV occur each
year (7) .
90% of adults obtain virus from prim a ry infection (o ften asy mptomatic)
as a child (7), age 6 months to 5 years , wit h mild virus-type symptoms (l ).
After this init ia l infec tion, th e virus hi des within t he t rigeminal (gasse
rian) gangl ion.
25 % cha nce of havi ng recurrence a fter first epithelial infection ; risk in
creases to 40 - 4-5% a ft.er 2nd recur rence (7) .
Rec urrent HSV infections (reactivatio n from latent infection in trigemi
nal ganglion) can be triggered by physical a nd emotional stress, incl uding
(non-exhaustive list) s un expos ure, fever, st ress, tra uma, immunosuppres
sion (6) .
Two common ty pes: I and II; bot h can cause ocular infect ion but Type 1
is significantly more co mmon. Type I causes infect ions "above the belt"
and is t ransmitted by close contact . T y pe II causes infect ions "below
the belt" (genital) and is t ransmitted sex ua lly; neo nates can be infected
during vaginal deli very (2) .
Hypoest hesia of t he a ffected cornea (as tested \vith Q-t ip) can often a id
in diagnosis.
It is rare to need lab work-u p for diagnosis of op hthalmic cases of Herpes
Simpl ex; if necessary, corneal scra pings can be eva luat.ed with Giemsa
stain and , if positive, will reveal mul tinucleated g ia nt ce lls (l6 ) .

2.7. CASE 11

67

Summary of other answer choices


Acantham o eba is a parasite found in wa ter , existin g as a cyst oj' trophozoite.
Initially, the co ndition s tart s with epi theli al defects that range from SPK to
whorl-like defec ts to pseudodend ritic lesions; in these initial stages, the pain
is often severe and out of proport ion to appearance o f infi a mmation. P at chy,
anterior stromal infil t rat.es become confiuent over a 2-3 mo nth period a nd ca n
form a ring ulcer (hallm ark of aca nthamoeba) . U p to 90% of cases are initially
misdiagnosed as HSV. (10)
Recurrent corneal erosions and the early st.ages of a HSV dendri te can have
a simil ar appeara nce. Cotton swa b testing is helpful in differen t ia ting the two
conditions; if res ults are non-conclusive , it is ad v isable to initi ally treat t he
epithelial defect as an erosi on a nd foll ow the condition carefully ove r the nex t
few days to see if a dendrite form s . Ini tia l trea tment wit h viropti c (in non-HSV
cases) ca n increase toxi city to the epithelium (due to thimerosol preserva tive)
and decrease corneal regenera tion and healing.
Thygeson's is a rare type of chronic keratitis with an unknown etiology, pos
sibly vira l (6). The classic presenta tion includes small, bilateral (90%) (10),
mul t iple, asy mmetric gray-white clusters of corneal lesions (ty pically central in
location) in a white, quiet eye with no conjunctival inj ec tion or anterior cham
ber reaction (7). These lesions resemble s ubepithe lial infi lt rates, b ut are more
superficial , less orga nized and are duller in color (18) ; they are often cen tral,
slightly elevated and stain minimally wi th flu orescein (2) (4) . Pati ent s oft en
have chronic complaints of foreign-body sensati on a nd tea ring. The condition
is most common in t he 2nd or 3rd decad e; exacer bat ions a nd remi ssions wit h
no seri ous seq ulae are coml11on.

In the minori ty of cases , Thygeson's is unilatera l a nd can easil y


be confu sed with berpes simplex disease . In t hese cases, compare
t he degree o f conjunctiva l injecti on; a Thygeson' s eye is white or
has t ra ce inj ection, whereas the HSV eye is usua lly grade II or
more inj ected. Corneal sensi tivity test ing can a lso be used to help
differen t iate th ese concli tio ns (7).

Exposure keratopathy , similar to HSV , commonly results in reduced corn eal


sensiti vity. This condition is ca used by abnormal or in complete lid clos ure a nd
results in kerat opathy t hat can ra nge from mild SPK to ulcera ti on. The SPK
is most common within t he inferi or 1/ 3 or interpalpebral region of tbe corn ea.
Pati ents typi cally report symptoms th a t a re worse in the morning, including
r edness, foreign body se nsat.ion and burning.

68

CHA PTER 2.

CONJUNCTIVA / CORNEA

Question 2: correct answer d - viral infection

j\lost patient.s obtain HSV rrom viral infection as a child; others are infected

by direct co nta ct . The patient's diabetes may prolong the healing time for

this infec tion. The pat ient's history of t ra uma would be a risk factor for t he

development of recurrent corneal erosion .

Question 3: correct answer d - macular edema

He rpes Simplex co mmonly presents in one or more of the following forms (16):

Epithelial keratitis - varies in appeara nce from SPK to dendri t ic ulcer


to geographic ulcer. The edges of an HSV dendrite (act ive vira l cells)
stain well with rose bengal, center ulceration stains well wit h fluorescein.
Disciform keratitis - disc-shaped st romal edema (due to endot heliit is)
that may have vVessely ring of infil trates and fin e KP's on the endothe
lium. If not co ntrolled can lead to scarring and seve re vision loss.
Blepharoconjunctivitis - vesicles and /o r crusting on eyelid and pe
riorbital area. Conj unctivitis is acute, unilateral with follicles, watery
discharge and prea uricular ly mphadenopathy.
Keratouveitis - cl assic presentation is marked corneal edema with ante
rior cham ber reaction and KP's on th e endothelium; hypopyo n, rubeosis
a nd elevated rop may be present (10).
Question 4: correct answer e - two of above
HSV and expos ure keratopathy can cause hypoesthesia, as determined with
cotto n-swab Q-tip sensiti vity testing.
Question 5: correct answer d - viroptic solution, 1 drop every 2
hours
Herpes Simplex AND Herpes Zoster Ophthalmicus treat ment pearls
include the following:
Herpes simplex epithelia l keratitis req uires trea t ment with viroptic ev
ery 2 hours (9 times daily for .5-7 days, then 5 tim es daily for 7 more
days) (6). Cycloplegics should be utili zed with cornea l involvement or
for a ny associat.ed ante rior cha mber reaction.

Cortico s teroids shou ld NOT be utilized for treat ment of epithe


lial disease but provide significant benefit with stromal kerati tis.

2.7. CASE 11

69
stromal keratitis
and
treatment
to 2-:3 times per
it

twice
can be
reduce the risk of recurrence
zoster is
of varicella infection
bolsters the immune system and
zoster disease (7).

5 x
for

be utilized p.l'
treatment revolves around
includes

can be utilized for treatment


last for 1 to
then
recurrences

(10)
Treatment for other conditions included in this case are covered
the text
other cases.

,-,,,,.C>V'VH

6: correct answer d

Conditions that affect lid closure should be included within the


1ist) nocturnal
Bell's

Jack. Clinical
Heinmann. 1999.

ilth

\\loburn: Butterworth and

70

CHAPTER 2. CONJUNCTIVA/ CORNEA

[2] Rapuano, Christopher J. Heng, ' ;Vee-Jin. Co lor Atlas and Synopsis of Clin
ical Ophthalmology. Will s Eye Hospit.al. Singapore: McGrmv-Hill , 2003.
[3] Bartlett, Jimmy D., Jaanus, Siret D. Clinica.l Ocular Pharm acology.
Boston: Butterworth , 2008 .
[4]

Friedbert , j\ Iark A. Rapuano, Christopher J . The V,Tills Eye IVIanual, 3rd


edition. Philadel phia: Lippincott ,,,iilli a ms and Wilkins, 1999.

[5] Tamesis, Richard R. Ophthalmology Board Review. 2nd Edition. IVIcGraw


Hill, 2006.
[6] Friedman , Neil J. Kaiser , Peter K. The j\Iassachuse tts Eye and Ear In fir
ma ry. 3rd Edition. Elsevier , 2009.
[7] Thomas, Randall. Melton, Ron. http: // www.eyeupdate.com/
[8]

http://www. emedicine.com/oph/topi c85 .htm.ParagAIVIajmud ar .MD.


Allergic Conjuncti vitis. June 30t h, 2008.

[9]

Fitzpatrick, Thomas B. Color Atlas a nd Synopsis of clinical Dermatology.


Com mon and Serio us Diseases, 4th ed. McG ra w-Hill, 200l.

[10] Friedman, N. Kaiser, P. Tratt ler, W. Review of Ophthalmology. Philadel


phia: Elsevier, 200 5.
[11] http://emedicine. med scape.com / a rt icl e/ 1197057 -overview
[12] Charles M. Kr afchak. J\Iutations in TCF8 Cause Posterior Polymorphous
Cornea l Dystrophy a nd Ectopic Expression of COL4A.3 by Cornea.l En
do t helia l Cells. Am J Hum Genet. 200 5 November; 77(.5): 694-708.
[13 ] http: //journals.lww.com. T it le of Salzmann Nodular Degeneration
[14] Dermatologic diseases, http://www.aocd .org
[15] http: // bjo.bmj. co m/content/ 89/ 9/ 1221.1 .extract1
[16] Ch eat ha m, K. Cheatham, M. 'Nood, 1<. KMK Part One Basic Science
R ev iew Guide. 4th edition. 2009
[17] Pa va n-La ngston , Deborah. Manual of Ocular Di agnosis a nd Therapy, 6t h
eel . Philadelphi a: Lippincott Williams and Wilkins, 2008.
[1 8] http://legacy.revoptom.com/ handbook/sect3j .htm
[19] www.com ea texas.com

Chapter 3

Lens/C
Post-Op

/IOL/P
ive Care

by Kyle M.

O.D., :M.S., F.A.A.O.

71

3.1. CASE 12

3.1

73

Case 12

Demographics
Age/race/gender: 75 year-old white male; retired
Chief complaint: gradual vision loss
History of present illness
Location: OU

Severity: severe

Nature of onset: started 6 months ago

Duration: constant

Exacerbations/remissions: none

Accompanying signs/ symptonls: occasional float ers

Secondary complaints/symptoms: both eyes occasionally water


Patient ocular history: last eye exam was 1 year ago; cataracts noted at
previous examinat ion
Pati ent medical history: heart disease (had bypass at age 65), rheumatoid
arthritis
Medications taken by patient: hydroxychloroquine
Patient allergy history: seasonal a llergies; NKDA
Family medical history: unremarkable
Review of systems
Musculoskeletal: stiff joints

Cardiovasc ular: hypertension

Menta l status
Orientation: oriented to time, pla.ce , a nd person
lVlood: appropriate
Affect: appropriate
Clinical findings

CHAPTER 3. LENS/CATARACT/ OPERATIVE CARE

74

Entering VA:
" Distance

Near

BVA:
1'---'I 'I--~R~e~fr
-a-c~t~io-n---r
I~D~i~
s ~ta
-n-c e~1

-2.00 - 1. 00 x 100

-2.00 - .75 x 089

Pupils: PERRL, negative APD


EOMs: full , no restrict ions
Confronta tion fie lds : full to finger co unt ing OD , OS
Slit lamp
lid s/ las hes/ adnexa: 2+ meibomian itis OD , OS
conjunc tiva: normal OD, OS
cornea: OD : 1+ fingerprin t lesions at level of basement membrane
with slightly elevated areas of epithelium and corresponding
negat ive stain ing - located superior to li ne of sight; 1+ inferior
SPK OD, OS
a nte rior chamber: deep a nd q uiet OD, OS

iris: normal OD , OS

lens : mild nuclear opacity OD, 1+ nuclear opacity OS

vitreo us: clear OD, OS

lOPs: 12 mmHg OD , 14 mmHg OS


etry

2:30 PM by app lanation tonom

Fundus aD
C / D : 0. 10 H/V with healt hy rim tissue
mac ula : t race RP E mott ling, 1 disc dia meter superior t o mac\lla
posterior pole: norma l
p e riphery: unrema rkable
Fundus as
C/D: O.15H/V wit h healt hy rim t iss ue

m ac ula : normal

pos t e rior pole: normal

p e riphery: unrema rkab le

Blood pres sure: 125/84 mmHg, right arm , sitt ing


Puls e: 68 bpm , regul ar

3.1. CASE 12

7.5

Question 1: Which of the following is the MOST likely cause of the


patie nt's chief complaint?
a. superfi cial punct.ate keratitis (SPK)
b. anterior basement membrane dystrophy
c. nuclear sclerosis
d. refractive error
e . macular degeneration

f. pri ma ry open angle glaucoma

Question 2: Which of the following listed is the MOST likely cause


of the patient's secondary complaint?
a. nuclear sclerosis
b. posterior blephar it is
c. refractive error
d. macul opathy

Question 3: Which of the following is the MOST appropriate treat


ment/ manageme nt for the patient's chie f complaint?
a. prescription for new spectacles
b. art ifi cial tears qid
c. referral for cataract extraction
d. re ferral to rheumatologist

Question 4: Which of the following conditions


with infantile catarac ts?

IS

NOT associated

a. galactosemia
b. rubella
c. hypogly cemia.
d. Von Rippel-Lindau disease

Question 5: Which of the following drugs does NOT cause whorl


keratopathy?
a.. a miodarone
b . ch loroquine
c. hydroxychloroqu ine
d . tamox.i fen
e. indomet hacin

f. isotretinoin

76

CHAPTER 3.

CARE

worsens, which of the

is

to occur?

shift in
b.

shift in

c. distance

will be

than near vision

near vision will be worse than distance vision

ANSWERS
1: correct answer d - refractive error
in this case

and
chief

of other answer choices


to int.erfere wit.h
in
the refraction would not have

open
at the advanced

does not result in any


The

result in vision

most cornmon anterior corneal


females and is thous:ht to be

3.2. CASE 13

77
exhihits

fluorescein

and

water. Pos
secretion and

The

3: correct answer a - prescription for new


1
4: correct answer d

Von

disease

5: correct answer f - isotretinoin

that cause whorl


amiodarone indomethacin
u"."~PJU

include CHAI-T:
tamoxifen.

6: correct answer a -

rnVrlTllr

shift in

are

as

13

25
Chief

droopy

of present illness
Location: 0 C

unknown

78

CHAPTER 3.

CARE

Nature of onset:
Duration:

Family medical
mother: diabetes
Ale:

diabetes

l\1edications taken
'JKDA

Patient
Review of

Ivlusculoskeletal: absent tendon reflexes


of

frontal

Ivlental status
Orientation: oriented to tillle,

and person

Mood: appropriate

Affect:

Clinical nnulIlg::s

APD

EOl\1s; external

OD, OS

Confrontation fields: full

OS

Slit
2mm

OS
cornea: clear
OS
anterior chamber:
iris: normal OD, OS

and
mnlticoiored

OS

cardiac l1ly

3.2. CASE 13

79

lOPs: 16 mmHg OD, 15 mmHg OS @ 2:30 PM by applanation tonom


etry
Fundus OD
C/D: 0.10 H/V with healthy rim tissue
macula: normal
posterior pole: 2+ diffuse pigmentary retinopathy
periphery: unremarkable
Fundus OS
C/D: 0.15H/V with healthy rim tissue
m.acula: normal
posterior pole: 1 + diffuse pigmentary retinopathy
periphery: unremarkable
Blood pressure: 11.5/65 mmHg, right arm, sitting
Pulse: 68 bpm, regular
Question 1: Which of the following systemic conditions could explain
all of the the ocular findings in this case?
a. Vlilson's disease
b. myotonic dystrophy
c. diabetes
d. atopic dermatitis
Question 2: What is the MOST likely etiology of this patient's con
dition?
a. inherited disorder (autosomal recessive)
b. inherited disorder (autosomal dominant)
c. decreased number of melanosomes
d. increased activity of tyrosinase
Question 3: Which of the following conditions results in tremor of
the wrist?
a. \Vilson's disease
b. myotonic dystrophy
c. diabetes
d. atopic dermatitis

80

CHAPTER 3. LENS/CATARACT/OPERATIVE CARE

Question 4: Repeated utilization of X-rays are MOST likely to result


in which of the following?
a. posterior subcapsular cataract
b. nuclear cataract
c. cortical cataract
d. anterior subcapsular cataract
Question 5: Which of the following cataracts is MOST likely to form
in a diabetic with uncontrolled blood sugar levels?
a. posterior subcapsular cataract
b. nuclear cataract
c. cortical cataract
d. anterior subcapsular cataract
Question 6: Which of the following conditions is MOST likely to
cause a "sunflower cataract"?
a. V/ilson's disease
b. myotonic dystrophy
c. diabetes
d. atopic derma ti tis

ANSWERS
Question 1: correct answer b - myotonic dystrophy

This patient has miotic pupils, ptosis, external ophthalmoplegia, "Christmas

tree" cataracts and pigmentary retinopathy, findings characteristic of myotonic

dystrophy. This condition results in progressive muscle wasting and weakness,

particularly in the lower legs, hands, neck, and face. Presentation is most

common during the third decade with myotonic features and ptosis (3).

Summary of other answer choices


Wilson's disease, also known as hepatolenticular degeneration, is an inher
ited disorder (a u tosomal recessive) that results in a deficiency in the enzyme
ceruloplasmin; this leads to an increase in copper deposition (throughout the
body) that is most concentrated in the liver, brain, and cornea (Kayser
Fleischer Ring) (8). The condition is characterized by asterixis (tremor of
the wrist), basal ganglia degeneration. cirrhosis, corneal deposits, cataracts,
carcinoma and dementia (2) (1) (8).

3.3. CASE 14

81

Diabetes is thc most co mmon ca use of presenile cata racts. This patient has

normal hemoglobin Ale levels and no retinopathy a nd the cataract described

in this case is not consiste nt wit.h cata racts that result from diabet.es. Lens

opacification in diabetics res ul ts from increased sorbitol pro du ction ; t his cre ates

a n osmotic gradicnt fa voring water movement into the lens fibers and swelling,

d a mage , and cataract form ation res ults.

Atopic dermatiti s, if severe, ca n result in cataract formation (10% of pa

tients); t his typically occurs betwee n t he ages of 15 and 30 (3). Two types

of cataracts a re most common: shie ld cata ract (consists of a dense anterior

subcaps ula r plaque) and posterior subcaps ular cataract (PSC).

Question 2: correct answer b - inherited disorder (autosomal domi

n a nt)

Ivlyoto ni c dystrophy is a n autosomal dominant inherited disorder .

Question 3: correct a nswer a - Wilson's disease

Asterixis , t remor of the wri st, can occur in \Vil so n's disease . l'vIyotoni c dys tro

phy ofte n a ffects the hands, ma king release of g rip difficult (3).

Quest ion 4: corr ect answer a - posterior subcapsular cataract

Posterior subca ps ular cataracts are commo nly a result of steroid use an d x-rays .

Question 5: correct answer c - cortical cataract

Acutely high blood sugar levels class ically g ive rise to bil ateral, multiple , gray

whi te cortical lens opacities described as "s nowflake opacit ies. " In most diabet

ics, chron ic, progressive, nuclear a nd cort ical cataracts are most common (4)

Question 6: correct answer a - Wils on 's disease

Lens opacities in Wilson 's disease occur in 10% of patients (4) and are gree n

brow n "sunfl ower" cataracts (3). Myotoni c dystrophy often results in sma ll ,

iridesce nt , mul t ico lored crys tals (a ppear like "Christ mas tree" cat aracts) be hi nd

the anter ior ca.psule; PSC cata racts can develop later (3) (7). As mentioned

above , diabetics can have "s nowflake opacit ies" and ato pic dermatitis patients

can have "s hield" cataracts.

3.3

Case 14

Demographics
Age/ race/gender: 75 year old v,hite-male; retired

82

CHAPTER 3. LENS/CATARACT/ OPERATIVE CARE

Chief complaint: glare at night when driving


History of present illness
Location: OD

> OS

Severity: mod erate


Nature of onset: grad ual
Duration: 5 years
Secondary complaints /s ymptoms: blurry visio n in t he right eye
Patient ocular history: uveitis in rigbt eye, one episode , occurred 30 years
ago
Family ocular history
mother: "lazy eye", Fuch's endothelial dystrophy
Patient medical history: benign prostatic hypertrophy
Medications taken by p a tient: terazosin; digoxin
Patient allergy history: NKDA
Review of systems
Genitourinary: frequently bas to urinate
Mental status
Orientation: not oriented to time, place, and person; pati ent smells of
alcohol
Mood: appropriate
Affect: appropriate
Clinical findings
Entering VA:
" Distance I Near

BVA:
'I---'IIr=R~e~fr-a-c~~
ti on-'~D~is~t-a-n-ce-'I
1

-3.50 DS

-2.00 DS

Pupils: PERRL , negative APD

EOMs: gaze nystagmus at 45 degrees from fixation OD , OS

Confrontation fields: full to finger counting OD , OS

3.3. CASE 14

83

Slit lamp
lids/lashes/adnexa: unremarkable OD, OS
conjunctiva: normal OD , OS
cornea: clear OD , OS
anterior chamber: deep and quiet OD, OS
iris: normal OD , OS
lens: OD: 3+ nuclear sclerosis. OS: 1 + nuclear sclerosis
vitreous: clear OD, OS
lOPs: 16 mmHg OD , 15 mmHg OS @ 2:30 PM by applanation tonom
etry
Fundus OD - li mited view due to cloudy media
C/D: 0.10 H/ V with heal thy rim tissue
macula: norma l
posterior pole: normal
periphery: unremarkable
Fundus OS
C/D: 0.15 H/ V with healthy rim tissue
macula: normal
posterior pole: norma l
periphery: unremarkable

Blood pressure: 115/ 65 mmHg, right arm , sitting

Pulse: 68 bpm , reg ular

Question 1: Which of the following is the MOST likely source ofthis


patient's chief complaint?
a. ac ute-angle closure glaucoma
b. medication related
c. Fuch 's endothelial dystrophy
d. cataract
Question 2: Which of the following medicat ions does NOT need to
be discontinued prior to cataract surgery?
a. coumadin
b. xalatan
c. flomax

d. sildenafil

84

CHAPTER 3. LENS/ CATARACT/ OPERATIVE CARE

Question 3: \Vhich of the following systemic medications can cause


complaints of dimming vision and/or flickering lights?
a. digoxin
b. phenytoin
c. oral contraceptives
d. isoniazid

Question 4: Prior to referring this patient for cataract surgery, which


of the following tests could be us eful in assessing the structural in
t egrity of the posterior segment ?
a. a-sca n ul t rasonog rap hy
b. b-scan ultrasonog rap hy
c. potential ac ui ty meter (PArd)
d. brightness acuity meter (BAT)

Question 5: During calculation of IOL lens power, the technician


accidentally inputs 26 mm, instead of 24 mm, for the p a tient's axial
length measurement . How many diopters of error is expected from
this mistake?
a. 6 D

b. 4 D
c. 3 D

d. 12 D

Question 6: Which of the following could explain the p a tient 's nys
tagmus ?
a. blurry acuity in t he rig ht eye
b. alcohol
c. terazosin
d. nuclear sclerosis

ANSWERS
Question 1: correct answer d - cataract
Differentia ls for gla re or halos aro und lights (non-exh a us t ive) include cataracts ,
ac ute a ngle closure gla ucoma, corneal edema (often from corneal endothelial
disease), corneal haze and med icat io ns (d igox in , chloroquine) (5 ). This pat ient
is tak ing digoxin , but t he pat ient reports that t he gla re is worse in t he right
eye, which matches t he signifi cant cataract asy mmetry noted in t he case.

3.4. CASE 15

85

Question 2: correct answer d - sildenafil


P atients ta king sildenafil (v iag ra) have a slightly increased risk of developing
non-art.eri t ic ischemic optic neuropat.hy (NArO N); however, cata ract su rgery is
not known to increase t he risk of ocu la r compli catio ns for pati ents taking the
med icatio n. Reca ll t hat flomax can ca use intra operative floppy iris sy ndrome.
P atients are advised to discontinue Warfarin approximately 96 to 115 hours (4
doses) prior to cataract surgery (6). Since t his patient has a history of uveitis, it
would be prudent to discont inue xalatan prior to surgery to avoid the increased
risk of cystoid macular edema.
Question 3: corre ct answer a - digoxin

Digoxin can cause blue/:yellow co lor defects , retrobulbar optic neuritis and en

top ic phenomenon Csnowy visio n ," dimming vision and flickering lig hts) (10).

Question 4: correct answer b - b-scan ultrasonography

B-SC3n ultrasound helps deter mine if posterior segment abnormalities are present

whe n cataracts (or other opacities) are too dense for fundu s exa mination. A

sca n ultrasound is used to meas ure axial lengt h for calculation of rOL power.

BAT testing is used to assess glare disability. PAM testing is used to assess

v is ual acuity through mild to moderate ocular medi a for determination of vi

s ual acuity potential as if no opaciti es were present; this a llows the cl inician to

assess t he risk and benefits of cataract surgery a nd a ids in setting app ropriate

post-surgical acuity expecta t ions for t he pati ent.

Question 5: correct answer a - 6 D

The average axia l lengt h is 24 mm; a 1 mm error in axial length measure ment

correspond s to a 3D error in calculated lens power ; t illiS, a 2 mm error would

correspond to 6 D (6).

Question 6: correct answer b - alcohol

Alcohol can affect smooth purs ui ts , resulting in fa ilure of horizontal gaze nys

tagmus testing.

3.4

Case 15

Demographics
Age/race/gender: 21 year old , asia n female , engineering stude nt
Chief complaint: wants to get refractive surgery, wants to be free of glasses
a nd contact lenses

86

CHAPTER 3.

LE~YS/CATARACT/OPERATIVE

CARE

Secondary complaints/symptoms: some dryness wit h co ntact lens wear


Patient o cular history: myopia
Patient medical history: type 1 diabetic , last Hba l c= 8.5 %
Medications taken by patient: insulin
Clinical findings
Habitual Spectacle Rx 1 year old

OD: -7S0DS 20/ 20


OS: -800DS 20/20

Subjective refraction

OD: -800DS 20/20

OS: -800DS 20/ 20

Pupils: PERRL, no apd, 8mm d iameter in t he dal"k

Slit lamp

lids/lashes/adnexa: clea r OU

conjunctiva: clear OU

cornea: tbut 4 seconds, inferior SPK OU

anterior chamber: deep OU

iris: no nvi OU

lens: clear Olj

vitreous: cleal" OU

Internals:
Fundus OD
pel'iphery: la tt ice wit hout holes inferior, no tea rs or detachments

OD
Fundus OS wnl , no tears or detachments OS
Questions:
Question 1: Which of the following characteristics / clinical findings
could potentially limit this patie nt's candidacy for refractive surgery
and/ or the patient's leve l of satisfaction with the results?
<:1.

the pupil size

b. diabetes
c. lattice degeneration
d. stability of refr act ive error
e. patient expectations

3.I CASE 15

87

f. dry eye
g. all of the above
h. two of the above
Question 2: This patient decides to undergo LASIK surgery. The
central pachyr:netry readings are 600um OU. Is the cornea thick
enough to proceed?
a. No, clear lens extraction should be done instead
b. No, PRJ< is the ollly surgery which would work for this patjent
c. Yes

d. Refractive surgery is contraindicated for this patient based on corneal


thickness
Question 3: At the 1 day follow-up visit, the patient notes the vision
seems hazy and foreign body sensation is present. Slit lamp exam
ination reveals fine, white, grainy cells in the cornea. What is the
most likely diagnosis?
a. epithelial illgrowth
b. corneal ectasia
c. corneal infection
d. DLK (diffuse lamellar keratitis)
e. wrillkled flap
f. anterior basement, membrane dystrophy
Question 4: The patients condition in question 3 clears up by the
end of the first week. The residual refractive error is -l.OOD OU.
The patient wants an enhancement. \\Then should an enhancement
be recommended, if at all, for this patient?
a. The patient does not fall withill re-treatrnent criteria range
b. Right a,vay while the flap can still be moved
c. \I\lait for 2 years
d. VVait for at least 3 months, 6 months is preferred
Question 5: This patient develops glaucoma later in life. How will
LASIK affect the lOP measw'ements?
a. no effect
b. will give a false low reading
c. Ivill give a false high reading

88

CHAPTER 3. LENS/CATARACT/OPERATIVE CARE

Answers:
Question 1: correct answe r g - all of the above All of the answers

could potent ially limit this patient's candidacy for refract ive surgery and/or

the patient 's level of satisfaction with the results.

Large pupils in dim light may result in increased aberration s and halos with

night dri ving.

Diabetics whose blood sugar is not well co ntrolled may have flu ctuations in

refractive error which may make the amount of treatment needed unclear. This

pati ents last Ha1 c indicated suboptimal blood sugar control.

Lattice dege neration/retina thinning may increase t he risk of ret inal tears dur

in g or after s urgery. A clear link between LASIK a nd vitreoretinopathy is not

defi niti ve, yet, caution should be exercised (14).

It is generally reco mmended to have a stab le refraction for at least one year

prior to the s urgery.

The patie nt states she wants to be free of glasses or contacts. A patient needs

a realist ic expectation of the surgery. The expectation should be a decreased

dependence on corrective lenses, no t complete freedom from le nses .

During the su rgery, corneal nerves are tem porarily damaged resulting in dry

eye as a common complication. If the eyes are dry prior to the surgery, this

problem may be exacerbated. Patients may experience decreased vision as a

result of dryness. Post-operative management requires aggress ive lubrication.

Absolute contraindi cations for LASIK: pregnan cy, under age 18, and corneal

disease.

Question 2: correct answer c - yes For LAS H\: su rgery, 250um must

remain under the flap to minimi ze compli ca tions. The flap is approximately

160-200um thick. The ablation depth is approximately 15um/diopter (18) .

This patient is a -SD myope wh ich means the a blation depth is 15x8= 120um.

The thickness of the cornea required for t his patient:

250um (remaining under flap)+200um (flap thickness)+120um (t,issue ablated)=

570um

This patient s ho uld be fine with LASII< with a 600U111 thick cornea.

Types of Refractive Surgery (17)

Radial Kerato metry (RK): This procedure is no longe r done. It co nsisted


of radial in cisions made with a diamond knife in the corneal stroma to
fl atten it, thus reducing myopia. The precision of the final refractive error
was difficult to control and was s ubop timal. It was common for patients
to end up hyperopic.
Photorefractive Keratectomy (PRK): Thi s procedure uses an excimer laser

3.4. CASE 15

89

to ablate the cornea for reshaping to correct for myopia, hyperopia and/or
as tigma tism. The laser esse nti aJl y creates a corneal abra sion whose hea l
ing is closely monitored and tit.rated with topical s teroid s. This ha s a
s lower recovery a nd more discomfort than LA SIK. On the other hand ,
P RK has less complications and is ch eaper than LASH-\:.
Laser In Situ Keratomileusis (LASIK): During this p roced ure, a flap is
created with a mi crokera tome, an excimer laser a bla tes the cornea, and
the flap is reattached. The healing time is s hor te r tha n for PRK a nd
is less pa inful. LASII\: can be used to correct hyperopia, myopia and
ast.igmatism although the treat ment ranges are la rger for myopia.
Intralase: Thi s is t he same procedure as LASIK but the Aap is made with a
laser instea d of a microke rat ome.
LASEK: This procedure is also like LASII{ but the flap is created using dilute
alcohol instead o f a mi crokeratome . Thi s procedure and Intralase avoid
microkeratome malfunctions.
Intrastromal Corneal Rings (ICR): During this procedure, ringts are in
serted in t he corneal periphery to flatt en the cornea. lCR is reversible
an d is o ften used in kerato conu s.
Clear Lens Extraction: This is essentially cat.a ract s urgery on a clear lens
where the 10L is chose n to a lter the eyes refractive power. This is a n
option for pati ents with corneas not t hick enough for LASIIC No accom
modation will remain a fter surgery unless a multi-fo ca l lOP or Crystalens
(accommodating 1OL) is used.
Intraocular lens implantation in a phakic eye: This allows for a lterat ion
of th e eyes refractive power without removing the abilit.y to accommod ate
which occurs in clear lens extrac tion. The implanted lens can be put in
t. he ante rior or posterior chamber.
Astigmatic keratotomy (AK): Circumferential inc isions are made to rel a.."
the cornea in the steepest meridian.
Conductive Keratoplasty (CK): T his uses radio frequ ency energy to shrink
the peripheral cornea. CK is used to treat presbyo pia a nd low hy pe ropi a
and th e res ul ts are not permanent. (15)
Wave-front guided (also refe rred to as custom cornea) ablation is available
wit h the goal of red uced a berration s.
Question 3: correct answer d - DLK DLK, also known as Sand of the
Sahara, is an infrequent inflamma tory, non-infecti ous react ion a t the lamellar
interface of the flap. It usua lly begins in the days right after s urgery. The
patient can be symptomatic and may need to be treated with topi ca l steroids

CHAPTER 3. LENS/CATARACT/OPERATIVE CARE

90

and, if severe enough, oral steroids and/or the flip lifted and irrigated. Tlus
can lead to vision loss if not properly man aged . The etiology is unk nown (16).

Refractive surgery potential complications/patient complaints


dry eye
glare and halos
reduced low contrast visual acuity
infection
retinal detac hment
residual refractive e rror
irreg ular astigma tism
corneal haze
increased aberratio ns
corneal ectasia- abnormal bulging forward of a thin cornea, need 250u l11
under flap in LA SII<, 400um for PRK to nunimize this risk
microkeratome complica tions (LASII<)
flap complications- free caps, button holes, corneal perforat ion, irregular
flaps, wrinkled flap (macro striae)
DLK
epithelial ingrowth- occurs at aro und 1 month, milky, white deposits at
the interface
Question 4: correct answer d Earli est retreatment at 3 months for LASIK
and PRK , 6 months is preferred. This will allow for stabilization of t he refrac
tive error. In the mean time, correct with glasses or contacts.
Recommended retreatment criteria
Refractive error 20.7.5 diopters from target in an unhappy patient
Uncorrected vis ual acuity ::;20/30 in the distance eye in an unhappy pa
tient
Astigmatism >0.75 D causing symptoms s uch as ghosting (17)
Question 5: correct answer b - will give a false low reading vVith the
removal of corneal tiss ue, t he cornea is thinner than it was pre-operatively. A
thinner cornea does not require as much force to applanate (flatten), therefore ,
a falsely lower lOP would be measured.

3.5. CA.SE 16

91

Note on long-term management of refractive surgery patients- Former


myopes are still at risk for retinal thinning , former hyperopes are still a t risk for
angle clos ure, and lOP readin gs are fal sely low after refractive surgery. P at ient
education on eye protection during contact sports to avoid dislodging of the
corneal fiap in LASIK (17).

3.5

Case 16

Demographics
Age/race/gender: 66 year old asian male
Chief complaint: here for 1 week post-op cataract s urgery (phacoemulsifica
tion) visit for OD
History of present illness
Character/signs/symptoms: scrat chiness which has gotten better since
day 1
Location: OD
Severity: minimal
Secondary complaints/ symptoms: slig ht blur
Patient ocular history: cataract OS, pseudoexfoliation syndrome OU
Patient medical history: high cholest erol
Medications taken by patient: pred forte qicl OD , vigamox qicl OD, lipitor
Patient allergy history: peN
Clinical findings
VA: OD sc 20;'30 , OS cc 20/40
Pupils: reactive O U, no APD
Slit lamp
lids/lashes/adnexa: clear OU
conjunctiva: tr diffuse injection OD, clear os
cornea: tr mi crocystic edema OD , negative Seidel sign, clear OS
anterior chamber: 1+ cell OD , deep chamber, quiet OS
iris: PXF deposits on pupillary margin

lens:

io!

centered and clear OD, moderate

I1S

and cortical

OS

vitreous:

no

lOPs:
Internals:
Fundus OU

CL2
macula: Aat
wnl
periphery:

or detachments

Cataract surgery overview


Types of cataract surgery

This is not

ECCE

remains. This is only


incision to remove the lens
induce

Phacoemulsification: The
removed and
broken up and sucked out. This
smaller
cornea, and sutures are not
This
most
at this time.

remains but lens is


usually at the
is
far the

93

16

3,,5,

IOL options
surgeon

on

visual demands, and

.. both eyes for


.. rnonovisioll
.. multifoeal IOL

..

reSTOR and

rOL (i,e,

..

IOL
fixed IOL-

lost in

.. anterior chamber 10L-

.. iris fixed

arise
lOL is stitched

no

..
..

the iris at the

a second surgery is done to insert an

current

TOL
I01, is

Questions:
1:
for Seidel
one visit. If it is present, the
the
EXCEPT:

at the day
is at increased risk for all of

a.
b,
iris

d, choroidal detachrnent

e. shallow anterior chamber

f all of the

2: This
has elevated lOP. 'Vhich of the
likely NOT a potential cause of the elevation in this
a. retained viscoelastic
b. steroid

material

d,

block

e, red

cells in tra becular meslnvork

IS

94

CHAPTER 3.

CARE

Which of the
treatments would be the most
INITIAL choice to treat this
elevated lOP?
a.

b. Diamox

c. timolol
d. laser PI

no treatment needed

4: At the one month


the
is
new
well. The
returns at threo rnonths post
op and notes vision has decreased. Fundus examination reveals cys
toid macular edema (CME). What is the standard
treat
ment for this condition?
a.

steroids

b.

steroids and

NSAIDS

and

c.

d. PO steroids

e. laser treatment

Answers:
1: correct answer f - all of the choices can occur with a
wound leak

VhHCULU,,-, bacteria
to the inside
and can result in a
microbial infection. There
0.02-0.05 percent. chance of
with cataract surgery (11). It
one or as late
several months.

severe
loss of
anterior chamber reaction with a
immediate

severe
mucus
antibiotic treat-

ment, poor
The wound

and loss of aqueous


and a shallow
is low eye pressure which does not tend to
than

8.5. CASE 16
when lOP is low. Fluid accumulates
in the
the choroid and
a
choroidal detachment. The choroid will appear elevated (DDx: RD or
steroids and
choroidals- the choroidal det<1chments from
sides are so elevated
that they
refer back to surgeon for
up into the \vound site alld the

To check for
and watch for

incision
out of the wound.

Possible treatments for wound leak


Pressure

or

Consult

surgeon

correct answer d - pupillary block


IOP
the
post-op
the
are reactive and the anterior ,~lt'UWJC;L
IS
block ca n occur due to a
of the aq lleous flow
posterior chamber to anterior chAmber
the 10L or
the vitreous. The 10L
is in
and there is no vitreous
block
not
correct.
Comments

Oll

from the iris, may be


Red blood
the trahecular meshwork.

Other

cataract surgery

Subluxation of IOL Rare, PXF has


occurs in :'Iiarfan's syndrome

sur"ery and can


'"

(11) (13):
risk due to weak lens zonules,

CHAPTER 3.

CA.RE

Ptosis

which is

had retrobulbar anesthesia


occurs if the
done now. This should resolve

Ivluro
is intact.
under the choroid which
This

3: correct answer c - tim.olol


treated to decrease the
of
should be

should
An aqueous suppressant.
inhibitor, or
choice.

the pred forte to a soft steroid (such


an JOP

Question 4: correct answer c


CME after cataract
termed Irvine Gass
around 3 months. It is
anterior segment inflammation
which is made
with the
of the natural
This inflammation causes
breakdown of the blood-retinal barrier and
results in vessel
Fluid accumulates in the outer
steroids and NSAIDS. If this is not
some

3.5. CASE 16

97

Othe r p oss ible inte rme diate / late cataract s urgery complications

(11 ) (1.3)
Ptosis- trauma to levator during s urgery
Diplopia
monocular: uncorrected refractive error / IOL dislocation/dry eye/ ast ig mat ism/ mac ula
problem/corneal irregularity
binocu lar: uncovered a pre-exist ing st rabism us, decompensated pho
ria, surg ical trauma- treat with prism or st rabismus surgery
Elevated l OP
Corneal decompensat ioll
C hron ic a nterio r uveitis- retain ed lens fr agments, IOL rubbing on iris
PCO (posterior capsule opacificat ion)- result of cell proliferation o n pos

terio r capsule and / or ElschIlig pea rls whi ch a re a nterior le ns epithelium

cells whi ch pro liferate to posteri or ca psule

Symptoms: simi lar to cataracts

Frequency : up to 50 % in 5 years (12)

Treatment: YAG caps ulotomy, risk of RD

Ret in a l detachmellt- 1% ri sk, occ urs within first 6 months (ll) , may

includ e Schaffer's sign (pigment in vitreous), emergent referral

Incr eased risk: axial myopia (>25mm), previous retinal tear or de tac h

mellt in either eye, family history, lattice degeneration

UGH sy ndro me (Uveit is , G laucoma, Hyphema)- usuall y with a nterior

cha mber IOL- ha p tic rub o n iri s causing hyphema and uveit is in the

a nterior cha mber which clog t he trabec ular meshwork and cause elevated

lOP. Treatment: steroids, ocu la r hypotensives, possible IOL excha nge

M ore Post-operative Management Information


P ost-op Drops used
Antibiotic such a Vigamox
Steroid such as Preel ForLe- will need to be t apered
Some surgeons may a lso give a topical NSAID
Note: If patient has gla ucoma , these drops should be continued

unless a prostagla ndin analog is used. In that case, a different drop

may be prescribed or no oc ula r hy potell sive drop may used d uring

the healing period.

CHAPTER 3. LENS/CATARA CT/ OPERATIVE CARE

98

Lega l pearl Clear documentation from t he surgeon should be


made when they release t he pat ient back fo r post-op care

General post-op follow-up frequency One day, o ne week, one month , an d


3-6 months (case-by-case basis)
Pos t-op glasses/refractive error
Glasses are typically prescr ibed at t.he one mont h visit .

Do not presc ribe, yet, if sutures may be cut in the fut.ure. If needed, cut

suture in steepest meridi an (90 degree from the axis). Wait unti l a round

4-6 weeks post-op.

The refractive error pl a nned may be emmetropia or some surgeons a im

for a small myopic prescrip t.ion.

Patient. will have no natural accommod ation rem aining.

References
[1] Noble, Joh n, Ed. Textbook of Prima ry Care i'vIedicine, 3rd eel. St. Louis,
l\'Iosby, 2001
[2] Bhusha n, Vikas, Le, Tao , Amin, Chirag. F irst Aiel for the USMLE Step 1.
New York, McGraw-Hill, 2003.
[3]

Kanski, J ack. Clinical Ophtha lmology 4th ed. Woburn: Butterwor t h a nd


I-rein mann , 1999.

[4] Rap ua no, Christopher J. Heng, Wee-Jin. Color Atl as and Synopsis of Clin
ical Ophthal mology. Wills Eye Hospital. Singapore: i'vIcGr3\v-H ill, 2003.
[5]

Friedbert , r\llark A. Rap ua no, Christopher J. The iVill s Eye Manua l, 3rd
edition. Philadelphia : Lippincott \Villiams a nd Wilkins, 1999.

[6]

Tamesis, Richard R. Ophthalmology Board Rev iew ., 2nd Edi tion.


I\'IcGraw-Hill , 2006 .

[7] Fri eelman , Neil J. Kaiser, P eter K. The fllassachusetts Eye anel Ea r Infir
mary. 3rel Editi on . Elsevier , 2009.

18] Cheatham, K. Cheatham, M. VI/ooel, K. KMK Part One Basic Sc ience


Rev iew G ui de . 4th edi t ioll. 2009
[9] Pavan-Langston, Deborah. Manual of Oc ular Diagnosis and Therapy, 6th
ed. Ph iladelp hia: Lippincott iiVilliams a nd Wilkins, 2008.

3.5. CASE 16

99

[10] Bartlett, Jimmy D. , Jaa nu s, Siret D. Clinical Ocular Pharmacology.


Boston: Butterworth, 2008.
[11] AOA cataract surgery clinical practice guidelines
[12] Apple D, Solomon K, Tetz M, et al. Posterior capsule opacification. Surv
Ophthalmol 1992; 37( 2):73-116.
[13] www.eyeweb.org, cataract surgery section
[14] Arevalo , et al. Rhegmatogenous retinal detachment after LASIK for the
correction of myopia. Ret ina. 2000;20(4) :338-41.
[15] Grosvenor, T. Primary Care Optometry, Fifth edition. Butterworth,
Heinemann, El se vier, 2007.
[16] Hoffman R , Fine H, Packer M. Incidence and outcomes of LASII( with
diffuse lamellar keratitis treated with topica l and oral corticosteroids. J
Cataract Rerract Surg 2003:29.
[17] AOA Clinical Practice recommendations for co-management of refractive
surgery, 'Nviw.aoa.org/documents/Co-f..,lanageme nt.pdf
[18] Machat J, Slade S , Probst L. The Art of Lasik , Slack Incorporated , 1998.
[19] Roy, F. Fraunfelder, F , Fraunfeld er F. Roy and Fraunfelder's current ocu
lar therapy. Elsevier, 2007.
[20] Rainer G, et al Intraocular pressure rise after smal! incis ion cataract
surgery: a randomisecl intraindi viclual comparison of two dispersive vis
coelastic agents. Br J Ophthalmol 2001 ;85 : 139-142.

Chapter

isclera/

M. Cheatham,

lera/Uvea

F.A.A.O.

101

103

4.1. CASE 17

4.1

Case 17

Demographics
Age/race/gender: 30 year old. white male; unemployed
Chief complaint: ocular pain
History of present illness
Location:

au

Severity: severe
Nature of onset: 1 week ago
Exacerbations/ Remissions: no hist.ory of previo us episode
Duration: consta lit
Secondary complaints/symptoms: photophobia
Family ocular history
father: retinal detachment at age 32
Patient medical history: ulcerative colitis diagnosed 4 years ago
IVledications taken by patient: none
Patient allergy history: NKDA
Family medical history
mother: rheu matoid a rthritis
Review of systems
Gastrointestinal: intermittent diarrhea , const ipation as well as feeling
bloated after meals
Mental status
Orientation: oriented to time , place, and perso n
l\1ood: approp riate
Affect: appropriate
Clinical findings

104

os

Confrontation fields: full to


Slit

1 + collarettes around base of lashes OD>OS


+ diffuse
OS:
diffuse

cornea: clear OD, OS


anterior chamber: OD:
flare, OS: 2+
iris: normal
OS
lens: OD: .\Iittendorf's dot. Clear OS
vitreous: clear
OS
lOPs: 16

OS @ 2:30 PM

flare

tOllom-

Fundus OD

with

rim tissue

macula: normal

normal

unremarkable

Fundus OS

0.15 HjV with


macula: normal

pole: normal

unremarkable

Blood pressure:

rim tissue

anIl,

Pulse: 68

Which of the
ocular condition?
il.

uveitis

b. scleritis
c.

d. corneal abrasion

recurrent corneal erosion


f. corneal ulcer

is the MOST

of

4. 1. CASE 17

105

Question 2: Which of the following describes the pathophysiology of


this condition?
a. breakdown of t.he blood aqueous ba rrier
b. breakdown of the blood r et inal barrier
c. t.rauma leading to ruptured iris blood vessels
d. broken attachment between corneal basal cells and underlying basement
membrane
Question 3: Which of the following is the most likely cause of the
patient's condition?
a. rheumato id ar t hriti s
b. ulcerative coli tis
c. stapb a Ltreus
d. byperopia
Question 4: Which of the following tests could be beneficial in con
firming the etiology ?
a. PPD test
b. Chest X-ray
c. Colonoscopy
d. Angiotensin converting enzyme (ACE)
e. Human Leukocyte Antige n (HLA B27)
f. Rapid card reage nt (RPR) test

g. Venereal disease research laboratory (VD RL) test


h. F luorescent treponemal an tibody a bsorption (FTA-ABS) test
Question 5: In uveitis, does the lOP in the involved eye increase ,
decrease or remain the same?
a . lOP can increase or decrease, dep ending on pathophysiology
b. lOP decreases
c. lOP increases
d. lOP remains the same
Question 6: Which of the following is the most appropriate treatment
for this patient's condition?
a. art ificia l tears qid
b. zymar q 2hrs

CHAPTER<l.

106
acetate q

Predforte

q 12hrs

d. PreclforLe 1%

ANSWERS
1: correct answer a - uveitis
of severe ocular
it. is
list of conditions to consider within the
in
scleritis and corneal
in the
these

On cases
tant
have a
differential

Uveitis can be acute, chronic


sified

and clas

Common symptoms of acute uveitis


and decreased vision. The hallmark
of
uveitis "l'ihite blood cells in the anterior chamber.
number of different
circumlimbal
[OP in the involved eye,

on the absence or presence


uveitis
often

of other answer choices


Scleritis

the most common

inflammation of the sclera in which


of
disease
(1);
of those
vascular disease with rheumatoid arthritis
Overall, the condition is
common in women and
Scleritis is characterized
'with
thin sclera

result in severe,
conditions and indurle uveitis,
, cataracts and corneal involvement. These
eral; recall that

and Gout

tuberculosis,

(4) (1).

4.1.

107

CASE 17

a blue hue when observed under natural


vessels do .:.JOT blanch with 2.5%

Anterior scleritis occurs in 98% of cases,


Ant.erior scleritis can be divided into

cat.e

Scleritis
most
form
conditions (4) .

.. Nodular

..

immobile nodule

with inftarnmation: worst form of scleritis,


of
tient.s may die within a
years as result of
autoimmune disease.
of
lose vision
; 60% have ocular
anterior uveitis,
cataract and
chronic rheumatoid
is. The condition
lack of symptoms and minimal
of scleral
with exposure

No corneal abnormalities were noted in this case, which removes the


the other answer choices.
not described
state

of

that are characteristic of

2:

correct answer a - breakdown of the blood aqueous

barrier
Uveitis results from breakdown of
blood aqueous barrier. which consists
of
in three locations:
iris vessels and Schlemm's canaL This breakdown leads to a
lows white blood cells
and
to enter the anterior charn
ber. \Alhen
\:VBe's at.each to the corneal endothelium
referred to

CHAPTER 4.

108

contain cluster of
Busacca's
white in color,

Question 3: correct answer b

ulcerative colitis

The most
tient
and
the most common uveitis
keratic

non-

anterior uveitis

Common
exhaustive

and

disease (IBD) - IBD includes Crohn's


Uveitis is
bilateral with
uveitis
with pT)'Qr,n("'"

"

with
iri
recurrent
most commOll in Asians and :'vIiddle

" Behcet's disease


tis and mouth and
Easterners.
disease -

"

have tick bite


iritis with recurrent,

KP's and

Juvenile rheumatoid arthritis


of uveitis in children. The condition is
toid factor
cause

uveitis inclucle
vit

4.1. CASE 17

109

Histoplasmosis - mu ltifocal "punched-out" yellow-white lesi ons. Com


mon in Ohio- J\'Iississippi Ri ver Valley; triad of per ipapillary atrophy, mul
tifocallesions in periphery, a nd maculopathy (including choro idal neovas
cular ization ) .
Sarcoidosis - Gra nulomato us pa nuveitis with retinal vasc ulitis (5); dif
fuse vit ritis or white , fluffy opa cities in inferior vitreous ("co tton-ba ll
opacities") or yellow-white exudates caused by periphlebit is ("cand le-wax
droppings") a nd shea thing around retinal veins.
Syphilis - Posterior uveitis occurs in secondary syp hilis (5). Causes a n
a cute mul tifocal chorioretini t is and \'it ritis (pa nuvei tis). Call ed "great
mimic ," because it can be confused with seve ral ot her condi t ions. Other
signs include salt and pepper fun d us and flame-shaped ret inal hemor
rhages . Recall that sy philis ca n be congenital or acquired.
Pars Planitis - Int.ermediate uveit is, bilat.eral in yo ung patients, clumps
of cell s on inferior pars plana ("snow-banking").
Cytomegalovirus (im munoco mpromised patients) - white patc hes of
necroti c retina wit h hemorrhagic retiniti s, vascul ar s heathing. Seen in
AIDS pat ients with CD4 co unts of less than 50.
Rheumatoid arthritis is a leading cause of scleri t is, no t uveitis. Staph
aureus and Staph epidermidis co ncent.rate within the perior bital area a nd
a re common culprits for blepharitis, bacterial conjunctivitis and ot her ocul ar
pat hology. Hyperopia, especially in Asians, is a risk fact or for angle-closure
glaucoma.
Question 4: correct answer c - colonoscopy

A colonoscopy would be helpful in evaluating inflammatory bowel disease

findi ngs, th e etiology of t his patient 's uveitis. Purified protein derivative test

(PP D) is used to diagllose tuberc ulosis; a read ing of 15 mm o r more of indura

tion is co nsidered abnormal (positive) in heal t hy indi viduals; 5mm or more

of induration is ab norma l in immunocompromised patients. Recall t hat most

pat ients wit h t. uberc ulosis wi ll be from Sou theast Asia or Africa and the condi

tion is often spread from coworkers, relatives, spouses, etc. The most common

systemi c compla int for tuberculosis is night sweats.

Chest x-ray fin dings would be help ful in d iag nosing sarcoidosis a nd tuber

culosis . Sarcoid osi s patients are classically middle-aged , Afri ca n American

(10: 1) (7) femal es. These patients <Ire often asymptomatic, but can have
com plaints of breathing difficulties (after li tt le exert ion), dry cougb and Ull
usua l ras hes. A lowe r back x-ray (sac ro-iliac) is used to evaluate ankylosing
spondylitis.
Angiotensin converting enzyme (ACE) testi ng ca n be helpful in eva luat
ing for sarcoid osis. However , up to 40% of ac t ive cases of sarcoid can have a
normal ACE result (7) . The most acc urate way to diagnose the con dit.ion is
with a chest x-ray.

110

CHAPTER 4. EPISCLERA/SCLERA/UVEA

The HLA-B27 test can be ordered to help determine uveit is etiology. The
three most common know n cond it ions (most cases are idi opa thic) that cause
acute , a nterior, non-gra nulomatous uveitis are a nkylosing s pondy litis, inflam
matory bowel di sease a nd Reiter 's sy ndrome. T he fo llowing is the condition
fo llowed by t he percentage of positive HLA B27 results for each : ankylos
ing spondy li tis (9 0%), Reiter 's syndrome (85 - 95%), inflammatory bowel
disease (60%) (5) . Since t he most C01111110n causes of ac ute, anterior, non
granulomatous uveit is a re commonly HLA-B27 positive , ordering the test for
etiology purposes sho uld be in question beca use , regardless of t he resul t, it does
not help in differenti at ing between t he most com111on causes of th e condi tion .

Notewor t hy positive HLA B-27 co nditions include CRAP:


Crohn 's disease, Rei ter 's sy ndrome, Anky Josing spondylitis,
P soriatic arth ri t is.

RPR and VDRL testing is used to determine if a n active syphil is infection


is occurring. FTA-ABS test ing dete rmines 1Nhet her a patient currently has or
has ever had syphili s in their lifetime; if a patient has ever had syphilis, the
FTA-ABS will be positi ve for life.
Question 5 : correct a n swer a pending on p at hophysiology

rop

can increase or d ecrease, de

In the early stages of uveitis, lOP typi ca lly decre ases; however, as described
below, a n increase in lOP can also occur and can be a sign that th e eye is
improving, or can be the resul t o f very serious compli cations.
Initially, infla mmation of t he ci lia ry body in uveit is leads to a decrease in
aqueous humor productio n. lOP can a lso increase in t hese cases as a result of
whi te blood cell s clogging t he t rabecula r mes hwork (TlvI) or inflammation to
the T iVI itself. Reca ll that aqueous humor is produced in the non- pigmented
epithelium of the ciliary body t hro ugh the processes of diffusion, ultrafil t ratio n
and active secretion; the latter of wh ich utili zes carbonic anhyd rase (to make
bicarbo nate) a nd Nal{ ATPase enzy me to crea te a io nic gradie nt t hat leads to
80% of aqueous humor produc tion.
During treatment , an increase in lOP often occurs . In many cases, t his can
simply be a steroid response. However, another cause is tha t the eye is improv
ing as a res ult of treatment; as steroid treatment decreases infl ammation wit hin
the cilia ry body and aqueous humor product ion returns to no rmal , lOP can
rise because the Ti\I o utflow has not returned to normal yet (cells still block
ing TM or T i\I fibers sti ll inflamed). T he major conce rn in uveiti s patients is
not when lOP d ecreases, but rather when rop in crea.ses and glau coma follows;
this ca n occur as a resul t of posterior sy nechiae (PS ) a nd perip hera l anterior

4.2. CASE 18

111
becomes inflamed and
meshwork
to

6: correct answer c - P1'edfo1'te 1% acetate q


q 12h1's

overall

of treatment in

Resolve

ocular issues.

Hornat

two-fold:

make the
tion.
with

cases
a referral to internal medicine
is indicated.

Scleritis treatment

include the

disorder

evaluation and treatment of


oral

","'->11

J'el

agents

oral steroids or

or
steroids
contraindica ted

or risk of

not effective and sub-Tenon's steroid

are
and

with IV steroids
Treatment lor the other differentials
the text.

4.2

18

III

this case

covered

other cases

CHA PTER

112

4.

EPISCLERA / SCLERA/ UVEA

Age/race/gender: 22 yeaT-old black female; stud ent


Chief complaint: blurry vision
History of present illness
Location: OU

Severity: mild

Nature of onset: gradual

Duration: 2-3 mont hs

Family ocular history


dad: uveitis
Patient medical history: heart murmur
Medications taken by patient: brophenira mine; used prn
Patient allergy history: NKDA ; pollen
Family medica l history:
mother: breast cancer; diagnosed at age 55
father: Crohn 's disease
Review of systems: unremarkable
Mental status
Orientation: oriented to time, pl ace, a nd person
Mood: appropria te
Affect: a ppropriate
Clinical findings
Entering Visual Acuity:
Distance
Near
1

BVA:
'I --""II-cR=-e--=fl-'a-c-t-o-io-n-r--=D:-:i"-s-tan-c-e---'

I ~~ II

-2. 75 DS
-1.75 DS

20/ 15
20/15

Pupils: PERRL, negative APD


EOMs: full, no restrictions

11.2. CASE 18

1I3
00, OS

Confrontation fields: full to


Slit lamp

blepharitis

OS

conjunctiva: normal
cornea:

OS
that

lens: clear
OS
vitreous: clear OD, OS
lOPs: 16 mrnHg OD,
etry

OS

2:30 PJ\I by

tonom

Fundus OD
C/D: 0.10
macula: Ilormal
posterior
periphery:
Fundus OS

tissue

0.15 H/V with


macula: normal
posterior pole: normal
periphery: unremarkable

Blood pressure: 1
Pulse: 68 bpm, regular
Which of the following is the MOST likely diagnosis of
iris lesion?
b.
nodule

d.

of the iris

use

following conditions is LEAST


nodule formation?
a.
b. sarcoidosis

c. irritable bowel
d.

to

CHA PTER 4 . EPISCLERA/ SCLERA/ UVEA

114

Question 3: Which of the following is a vision thre atening complica


tion of this p a tient's condition?
a. met astas is t o fovea
b. seco nd a ry gla ucoma
c. growt h of lesion into the line of sight
d. seco ndary cataract format io n
Question 4: A metastatic lesion of the iris would be likely to cause
all of the following EXCEPT?
a. dec rease in

rop

b. ru beosis
c. ante rior uveitis
d. pseudohy popyon
Question 5: Which of the following is NOT true?
a. omega-3 and omega-6 fat ty ac ids must be ingest ed through the diet
b. omega-3 fatty acids are less prevalent in the diet
c. omega-3 fatty acids are a nt i-inflammatory
d. omega- 6 fatty acids are a nt i-infl ammatory

ANSWERS
Question 1: correct answer a - iris cyst

Iris cysts are uncommon , unil a teral lesions which ca n be primary (most co m

mon ) or secondary (after tra uma , surgery, or from strong miotic medications).

Prima ry lesions are usua lly peripheral (5) and ca n ori ginate from the iris pi g

ment epi thelium (most c0111 mon) (1) and stroma; t hey do not typica lly grow

in size a nd are asy mptoma tic. The classic clini cal presentation is a globula r,

da rk-brown structure t hat trans illuminates; t heir clini cal significance lies in

t he fac t tha t they ca n be very similar in appeara nce to iris and cilia ry body

t umors (1) .

Summary of other answer choices


Iris nevus is a common condit ion (50% of popul ation (5)) caused by a benign
proliferation of mela nocytes; lesions a re pig mented a nd ca n be single or mul t iple
a nd a re typica lly flat (or slightly elevated ). Size is almost always less tha n 3
mm (5) (1).

CASE 18

115

Lisch nodules are

Koeppe nodules
tions of
the

hamartomas that
of l\'eurofibro
inferior
small,
cells

on

Metastatic lesions of the

2: correct i:U1SWer c - irritable bowel

inflammation.

3: correct answer b - secondary

with t.hese lesions.


illvolvement.

4: correct answer a - decrease in lOP


anterior

5: correct answer d-

acids are

116

4.3

CHA PTER 4. EPISCLERA/ SCLERA /U VEA

Case 19

Demographics
Age/race/gender: 37 year-old white femal e; teacher
Chief complaint: ocular irritation
History of present illness
Location: OD
Severity: unknown
Nature of onset: ac ute
Duration: 4 days ago
Accompanying signs/symptoms: no pain, no co mpla ints of blurry
vision
S econdary complaints/symptoms: dry eyes
Family ocular history
mother: catar acts
Patient medical history: rheumatoid ar thritis; acid reflux
Medications taken by pati e nt: plaquenil , protonix
Patient allergy history: NKDA
Family m e dical history
mother: hyperthyroidism
Review of systems
Musculoskeletal: joint pain
Mental status
Orie ntation: oriented to time, place, a nd person
Mood: appropriate
Affect: app ropriate
Clinical findings

BVAr:__-rr~~__-'~__,
I

Distance

N ear

4,3, CASE 19

117
APD

EOMs:

os

Confrontation fields: full to


Slit lamp

cornea:
SPK
anterior chamber: deep and
iris: normal OD" OS

lens: clear

OS

OD,OS

vitreous: clear

lOPs: 1
Fundus

OS 'Q! 2:30

aD

0,10 HjV with


macula: normal

pole: llormal

Fundus

P~I

rim tissue

as

: 0,15 HjV with


macula: llormal

pole; normal

rim tissue

unremarkable

Blood pressure: 1

ann,

Pulse:

Question 1: \Vhich of the


this
ocular condition?
a. nodular

e>n,e'c-"e>r,

c.

d.
nod ular sc:leri tis

is the MOST likely

U.UA"'~~V,J""

of

118

CHAPTER -1. EPISCLERA/SCLERA/UVEA

Question 2: What is the MOST likely cause of this condition?


a. idiopat hic
b. rheumatoid art hritis
c. thyroid disease
d. ult.raviolet s un damage

Question 3: Scleritis differs from episcleritis by all of the following


EXCEPT:
a. scleritis usually has 1110re diffuse vasc ula r inflammation
b. scleritis is usuall y 1110re acute in onset
c. scleritis is usua lly found in a chronically ill patient
d. scleriti s has a bluish-red hue (rather t han red)
e. scleritis does NOT blanch with 2.5% phenylephrine

Question 4: Which of the following statements is NOT correct?


a. risk of retinopathy if takillg sta nd ard dose of plaquenil (400 mg/day)
increases if t he patient weighs less t han 135 lbs
b. risk of crystalline re tinopathy with tamoxifen treatment increases if tak
ing> 6.5 Il1g/kg/day for more than 5 years
c. risk of corneal ver t icillata ("whorl keratopathy") with amiodarone is min
imal at 400 mg/ day
d. Fabry 's disease results in corneal vel'ticillata in 90% of cases

Question 5: Which of the following is the MOST appropriate treat


ment for this patient's condition?
a. artificial tears , 1 drop every 2 hours
b. topical decongestant, 1 drop every 4 hours
c. oral steroid s, 60 mg daily
d. mild topical steroid , 1 drop 4x 's/day

Question 6: Which of the following conditions does NOT warrant


concern when prescribing oral steroids?
a. p eptic ulcer disease
b. diabetes
c. pregnancy
d. Addison 's disease

4.3. CASE 19

119

ANSWERS
Question I: correct answer a - nodular e piscleritis
Episcleritis is a benign, se lf-limiting, inflammatio n of the episclera. It is most
common in young ad ul t.s and frequen t recurrences (67%) are common (5). Mi ld
ocul ar discomfort is t he rule, but the chief comp laint is typically associated with
conjunctival hyperemia (6). The cond itio n can be simple (80%) or nodular
(20%); most cases (66%) are unilate ral (5). Episcleri tis is typi ca lly idiopathic
but can be associated (up to 40% of cases) (11) with collagen vascular diseases
(rheumatoid arthriti s), acne rosacea, herpes zoster, herpes simplex and syphilis.
The class ic clinical presenta tion is a unilateral red eye wit h sectoral (70%)
injection (5) .

Summary of other answer choices


Phlyctenular keratoconjunctivitis results from a Type 4 (delayed ) hyper
sensit ivity reaction to staphylococcus (blepharitis is common culprit) and tu
bercu lin protein (tuberc ulosis) . The conditio n results in conjunctival and/or
corneal nodules that are often transient and resolve spont aneously or respond
quickly to treatment . Common symptoms include photophobi a : lac rimation
and blepharospas m (9).
SLK patients typically have hyperemia of the superior bulbar conjunctiva and
limbus, unlike episcleritis, where nasal and/or temporal inj ect ion is expected.
The patient in t his case has a family history of thy roid disease, but no personal
history of t he condit ion. Recall t hat up to 50% of cases of SLK are associated
with thyroid dysfunction (7). This patient is not a contact lens wearer , a nother
common associated condition for SL1<.
Pingueculas res ult from degeneration o f collage n fibrils within the conjuncti
val stro ma (1). They are yell ovv-white , raised areas of conjunctiva., at 3 and/or
9 o'cl ock positions, caused by chroni c dryness a nd /or ultraviolet s unlight ex
posure. P ing ueculas can become irritated and inflamed (pingueculitis) in sec
toral fashion, simi lar to an episcleritis. In this case, t here is no men tion of a
pingueculum in either eye .
Scleritis can present in a simi la r fashion to ep iscleritis. However , deep, bori ng
pain is a hallma rk clinica l find ing in these patients. This patient has rheumatoid
arthritis , a common underlying etiology for scleritis; however: all ot her findings
(unilateral: ac ute onset, mild discomfort, mobile nod ule) are characteristic of
episcleritis.
Question 2: correct answer b - rheumatoid arthritis
An underlying systemic etiology for episcleri tis shou ld be suspected if the con
dition is severe, recurrent or unresponsive to thera py (6). This patient has
a severe (3+ conjun ctiva l hyperemia) episcleritis a nd a known diagnosis of
rheumatoid arthri t is.

CHAPTER

120

4.

EPISCLERA/SCLERA/UVEA

Questi o n 3: correct answer b - scleritis is usually more acute in


o n set
In compar ison to episcleritis, scleri t is is typically more grad ual in onset (6). The
bluish-red hue is best viewed wit h the naked eye under nat ura l lig ht (no sli t
la mp). Approx imately 10-1 5 minutes after instillation of 2.5% phenylephrine,
t he blanching should occ ur in a patient with episcleritis.

Question 4: correct answer c - risk of corneal deposits with amio

d aro n e is minimal at 400 mg/ day

The risk of corneal verti cillata wit h amiodarone is inevitable at 400 mg/day;

at 100-200 mg/day t he patient will have minimal or no corneal pathology (6).

Reca ll that corneal verticillata are bi lateral, light brown epithelial deposi ts

in a horizonta ll y lin ear branching pattern, normally at the inferior third of

t he cornea (6). In severe cases whorl-like keratopathy develops; if this is lo

cated within t he visual axis , sy mptoms of glare, photophobia and colored rings

arou nd lights can result. Reca ll that amiod aro ne can also cause anterior sub

capsul ar lens deposits and non-arteritic ischemic optic neuropathy (1 - 2% of

cases) (3).

Question 5: correct answer - d mild topical steroid, 1 drop 4x's/day

Episcleritis treatment depends on severity. In mild cases, artificial tears and

cold compresses are used for treatment; topical decongestallts can be added

to reduce hyperem ia. In moderate to severe cases (as in this case), treatment

wit h a topical corticosteroid (e .g. lotemax qid) or oral NSAID (e.g. ibuprofen

200-600 mg), or a combination of the above can be used , usually for 5 to 10

days.

Pinguecula's a re often NOT treated unless they become inflamed; artificial

tears can be utilized in mild cases, while mild topical corticosteroids are utilized

in cases of act ive inflammation.

Phlyct e nular keratoconjunctivitis treatment can include topical deconges

tants for mild cases and topica l steroids and/or steroid /a ntibiotic combination

therapy for moderate to advanced cases. Underlying etiology should also be

evaluated and treated appropriately; in most cases t his would involve blephari

tis treatment, but cou ld also warrant PPD testing for evaluation of tuberculosis.

Question 6: correct a n swer d - Addison's disease

Prior to prescribing oral steroid s, you should always inquire abo ut pregna ncy,

peptic ulcer disease and dia betes (6). Corticosteroids are utilized in treatment

for Addison's disease.

4.4

Case 20

4.4. CASE 20

121

Demographics
Age/race/gender: 10 year-old white male
Chief complaint: eyes are sensitive t.o light
History of present illness
Location: OU

Severity: moderate

Nature of onset: gradual

Dw'ation: 3-4 years

Secondary complaints/symptoms: blurry vision


Family ocular history
mother: anterior uveitis
Patient medical history: part ially deaf
Medications taken by patient: none
Patient allergy history: NKDA
Review of systems
Musculoskeletal: mild lower back pain that just started after recen t
baseball game; condit.ion is improving
ENT: difficulty with hearing
Mental stat us
Orientation: oriented t.o time, place, and person
Mood: appropriate
Affect: appropriate
Clinical findings

BVA:
~--~~----~~-Distance
Near
I.

Pupils: PERRL, negative APD

EOl\1s: full, nystagmus noted OD, OS

Confrontation fields: full to finger co unting OD , OS

CHAPTER ,1.

122

Slit lamp
unremarkable OD, OS

OS
OS

cornea: clear

anterior chamber:
iris: mild,

OS

and

transillumination

os

os

lens: clear
vitreous: clear

OS

lOPs: 16
etry

tOllOI11

Fundus OD

OD
unremarkable
Fundus OS
rim tissue
""TPr,r.r

pole: 2+ diffuse

OS

unremarkable
Blood pressure: 1

arm,

Pulse: 68

Question 1:
this

is the MOST
chief

uveitis
h.

uveae

c. ocular albinism
d. corneal abrasion

'-''''.:>C>'VU

2: ""Vhat is the MOST likely cause of this

tion?
a. breakdowll of blood aqueous
b.

amount of

in all melanosomes

decreased numher of melanosomes


d. increased

of

condi

CASE 20

123

is the MOST likely cause of the


decreased
foveal
b< nystagmus
c transillumination defects

macular
4: \Vhat is the MOST
Ar1rrr.n,ron uveae?

threat to vision in a

a< exudative retinal detachment

d< macular edema

5: "Which of the
for this pnC.""""L

is the MOST

man-

a< refer for fluorescein

in 1 year

b< monitor condition


c< refer for

consultation to rule-out associated

d< refer for


with
\vithout contrast
6: Which of the
ance to the sclera?
U<;O",L.VH

of

chiasm and

can cause a blue appear

b. isotretinoin

c ethambutol

d.

ANSWERS
1: correct answer c - ocular albinism
This

includes differentials for

foveal

iris

CHAPTER

124

main types are oculocutaneous


ocular
Oculocutaneous albinism can affect the hair, skin and
the eyes,
while ocular albinism effects are limited

other answer choices


uveae,
referred to as
crest cell disorder characterized
the
in
, a strong
condition is associated with
orders

No

abnormalities or
other differentials,

NY!lO',Onl

and flare

described in this

which

2: correct answer c - decreased nunlber of melanosomes


each melanosome

decreased number of melanosomes


Oculocutaneous albinism
and
Recall that the function of

On a\'erage, lout of every 10 cells in


This cell
called
for the

3: correct answer a foveal

of

for ocular and oculocutaneolls albinism is


with the
of
loss r'r.,'r?>~r'r.r,n

in the
to the amount

4.4. CASE 20
4: correct answer
a condition

5: correct answer c - refer for


rule-out associated

consultation to
two poten

6: correct answer d and corticosteroids


with the
to
Recall that. ethambutol is utilized for t.reatment. of
tuberculosis
include RIPE:
ethambutol. Recall that ethambutol can
neuritis (3)

[Il

KanskL Jack. Clinical


1999.

4th ed. \\loburn: Butterworth and

of Clin
200;3.
Siret D. Clinical Ocular

J. The \Vills

3rd

1999.
I\Iassachuset ts

Ear Infir

mary. 3rd Edition Elsevier. 2009.

[6]
,V. Review of

Friedman, N.

Philadel-

2005.
of variant aniridia.

[9]

K. KI\IK Part One Basic Science


11629-ovcrview

Chapter 5

Vitreous/Retina
21

5.1

white

65 year

Chief

retired

in vision

illness
Location: OS
severe

Nature of onset:

up in the

with

vision

Duration: constant
none

Family ocular
mother: cataracts
Patient medical

of heart

:Medications taken
Patient
unremarkable

Review of
Cardiovascular:
1')7
~,

with

years

128

CHAPTER 5. VITREOUS/RETINA

Mental status
Orientation: oriented to time, place , and person

lVlood: approp ria te

Affect: appropr iate

Clinical findings
BVA:

Near: 20/ 20 OD ; 20/400 OS

Distance: 20/ 20 OD; 20/400 OS

Pupils: PERRL, 2+ APD OS


EOMs: fu ll, no restricti o ns
Confrontation field s: full to finger co unting OD, limi ted and unreli
able OS
Slit lamp
lids / lashes/adnexa: unremarkable OD, OS
conjunctiva: normal OD , OS
cornea: Hudson-Stahli line inferiorly OD , OS
anterior chamber: deep and quiet OD , OS
iris: norm a l OD, OS
le ns: trace PSC OD , t race PS C OS
vitreous: clear OD , OS
lOPs: 16 mmHg OD , 18 mmHg OS
etry

4:30 p rvI by appla nat ion tonom

Fundus OD
C/D: 0.25 H/V with healthy rim tiss ue
macula : normal
posterior pole: normal
periphery: unremarkable
Fundus OS
C / D: 0.1 5 H/V with .3+ diffuse disc edema with collaterals
macula : 2+ cystoid macular edema
poster ior pole: dil ated , tor t uous retinal vei ns with superfic.iil.l ret i
nal hemorrhages and cotton wool spots in all four quadra nts.
periphery: normal

Blood pressure : 115/84 mmHg, right a rm , sitting

Puls e : 78 bpm, regular

5.1. CASE 21

129

Questions
Question 1: 'Which of the following is the MOST likely diagnosis of
this patient's ocular condition?
a. bra nch retinal artery occlusion (BRAO )
b. branch retina l vein occl usion (BRVO )
c. central retinal vein occl us ion (CRVO)
d. o pt ic neuriti s
e. arteritic anteri or ischemic optic ne uropathy (AIO N)
f. central retinal artery occlusion (CRAO)

g. vitreous hemorrhage
h. retinal detachment

Question 2: What is the MOST likely cause of this condition?


a . Hollenhorst plaque
b. calcium pl aque
c. vir us

c1. thromb us

e . sympathet ic ner vo us system hyp eractivity

Question 3: Which of the following listed is the MOST likely final


visual acuity for this patient?
a. 20/50

b. 20/ 100

c. 20/ 30

c1. 20/400

Question 4: What is the function of optic disc collaterals?


a. s hunt blood from re t inal veins to choroidal circul at ion
b. provide al ternat ive rou te for oxygen to ret inal circ ula tion
c. provide alternat ive route for oxygen to fovea
d. shunt choroidal veno us blood to ret inal circul at ion

130

CHAPTER 5. VITREO US/RETINA

Ques tion 5: Which of the following co nditions does NOT have the
pote ntial to cause r e tinal n eovascula rization and vitre ous h e mor
rhage?
a. diabetic re t inopathy
b. retinal vein occl usion
c. retinopathy of premat urity
d . ret ini t is p igmentosa
Questi on 6 : Which of the following is the MOST a ppropria t e treat
ment fo r this p a tient's ocular condition?
a. vitrec tomy
b. PRP

c. carotid do ppler
d. int ravitreal steroid inj ect ion
e. int ravenous (IV) steroids

ANSWERS
Question 1: correct answe r c - Central retina l v ein occlus ion (CRVO)
T his case describes t he classic preselltation of an ischemic CRVO . Since veins
d ra in blood fr om t he retin a, vein occl usio ns p resent wit h intraret in al hemor
rh ages; CRVO fin dings will have hemorrhages in all four q uad ra nts, while a
BRVO will have he morrhages in the area of distribution of the occl uded vessel.

Clinical summary for CRVO's:


CRVO 's can be grouped into no n-isch emic (67%) a nd ischem ic cate
gor ies (6 ).

Papillophle bitis is described by some as a 3rd category of


CRVO 's (.5) (1); th is condi t ion causes marked optic disc edema
in otherwise healt hy yo ung patients .

Vision threatening complicRtions in clude m acular disease (ischemia,


ede ma) a nd n e ovascularization com plications (v it reous hemorr hage,
neovasc ula r gla ucoma , tractio nal retinal detachment). VE GF st imulates
neovasc ularization of the posteri or a nd anterior segme nt a nd has been
proven to cause capillary leakage leading to macular edema (2.5).

5.1. CASE 21

worse
up to
.. Non-ischemic CRVO: Non-ischemic
or better
Overall,
neovascularization

to

cardiovascular dis
Glaucoma is the ocular disease
associated '.vith CRVO's (4) .
.. CRVO's are the 2nd
common vascular
is most common (23) .
.. Clinical appearance: retinal
cotton-vvool
macular edema, and

of vision

diabetic

dilated tortuous retinal


disc edema.

Patients
BRAO's,

all four types of retinal vascular occlusions


are
older
If the macula is involved in the
sudden, unilateral

of other answer choices


Clinical summary for BRVO's:
inciude macular disease
and neovascularization

A
for
retinal arteries and veins
thickened
to compress
the vein (1).

CHAPTER 5. VITREOUS/RE TINA

132

55% of patients have 20/40 vision or better at 1 year (2 1).


Clinical appearance: retinal hemorrhages , cotton-wool spots, macular
edema; .57% of temporal branch occlusion have macular edema (2 1).
Since arteries supply blood to t he retina, arterial occlusions present with is
chemic (white) re t i l1a; CRAO findings will classica lly re veal infarction in a ll
four quadrants, while a BRAO will have illfarction in the area of distr ibution
of the occluded vessel. The edema eve ntuall y resolves and the whi t is h area
disappears (wit hin weeks to months); however , permanent damage remains in
t he area of ischemic retina .

Clinical summary for

CRA~'s:

P at.ients typically have painless loss of vision to 20/400 , unless a cilioreti


nal artery (1 5 - 30% of patients) is present (2 1). If acuity is LP or worse,
strong ly consider ophthalmic a rtery occlu sio n (4).
Only 5% of pat ients develop iris neovascularization and neovascular glau
coma (5) . Pati ents have a 10% risk of episode in the other eye (18).
Risk Factors for CRAO: hy pertension (67%), diabetes mellitus (33%) ,
ca rotid occl usive disease (25%) a nd cardi ac valve disease (25 %) (6).
CRAO may be preceded by transient ischemic attacks (TIA's) of
vis ua l blackout and /o r blur ("a maurosis fugax").
Clini ca l appearance: cherry red spot in the foveala (visibility of choroi d
underneath foveala), superficial whitening of inn er ret.ina l layers (which
returns to norma l color later) narrowed arter ial vascul at ure and Hollen
horst plaques or other emboli (20). A CRAO is the most common cause
of a ch erry red spot (18).

TIA's are te mporary (always less t ha n 24 hours, usually less than


15 minutes) pauses of brain activity due to inadequate perfusion; as
perfusion is restored , patients a re left ,vit h no symptoms (26) (20) .

Clinical summary for BRAO's:


90% caused by emboli (cholesterol) calci um , fibrin, pIa telets) (18) , most
commonly from Holl en horst plaque (6).
10% risk of episode in oth er eye (18).
P atient will have permanent visual field defect in area of damaged retina.

5.1. CASE 21

133

90% of cases involve temporal vessels ; Hollenhorst plaques or other emboli


are found within area of occlusion in 62% of cases (6).
Optic neuritis, in classic cases, results in sudden vision loss in a yo ung fe
male wit h an APD and pain on eye move ment (90% ) (7). Vision loss rapidl y
worsens for 2 weeks then stabilizes before starting to improve 4 weeks after
symptoms (7). On approximately 2/3 of the cases, the optic nerve will appear
normal because the inflammation is retrobulba r; the other 1/3 of patients wi ll
have disc edema (papillitis ).
Arteritic AlON results in sudden, unilateral vision loss with disc ede ma in
a patient over 55 yea rs of age who might have corresponding sy mptoms (non
exhaustive) of temporal headache, tenderness to the temporal artery, scalp
tenderness, jaw cla udication and ma laise.
Vitreous hemorrhages result in sudden , painless vision loss or blurred vision
and/o r black spots t hat can have corresponding fl as hing ligh ts. Mild cases of
vitreous he morrhage will reveal blood that obscures part of the fundus view;
severe cases will not allow any view of t he fundus, while chronic cases ca use
the fund us to appear gray-white and / or yellow in appearance.
Retinal detachments can a lso present with sudden, unilateral , painless visi on
loss and should be includ ed within the differentials for this chief complaint. The
ret inal findings in this case are not co nsistent \vith this diagnosis.

Question 2: correct answer d - Thrombus


CRVO's and BRVO's result from compression of a n a rtery on a vein (classi
cally from HTN and /or Dl\I); this leads to turbulent blood flow, venous vessel
damage and thrombus forma t ion (20 ). In a CRVO, impingement of the central
retinal artery on the central retinal vein leads to thrombus formation , which is
usually located in the area of t he lamina cribosa (6) (2). Thrombus formation
lead s to ischemi a and release of vascular endothelial growth factor (VECF).
CRAO's most commonly arise from heart and/or carotid artery em boli. The
most common e mboli from the heart includ e calcific emboli (large plaques from
heart valves) and thrombi (after a myocardial infarction ). The most common
emboli [rom the carotids incl ude Holl enhorst plaques; these e mboli are smaller
and more likely to occlude sma ller vessels , contributing to BRAO 's (most COI11
mon etiology for BRAO's). E mboli are by far the most co mmon etiology for
a.rterial occlusi ons, other culprits to consider include (non-exhau sti ve list ): gi
ant cell arteritis, coll age n vascular diseases , oral contraceptives and sickl e-cell
disease (20).

134

CHAPTER 5. VITREOUS/RETINA

In young CRVO patients, consider the following conditions for


etiology (non-exhaustive list): oral contraceptive pills, protein
S/protein C/antithrombin III deficiency, factor XII defic iency, an
tiphospholipid antibody syndrome, collagen vasc ul ar di sease and
AIDS (6).

The most commo n ca use of a rteritic AIO N is Giant cell arteritis (GCA) ,
a system ic vasculitis of medium and large-sized blood vessels; occlusion to the
short posterior ci li ary arteries (SPCA 's) results in infarction of the a nterior
portion of the optic disc.
The term optic neuritis simply means inflammation of the optic ner ve; it
does not indicate anyth ing about the etiology of the condition (7). There are
several different types of optic neuritis , but demyelinating is the most common
cause; 50 - 60% of patients with isolated optic neuritis wi ll eventually develop
multiple sclerosis (i\IS) (6).
Question 3: correct answer d - 20 / 400
Fin al visual acuity with a CRVO is typically similar to what the acuity was on
initial presentation. For patient education purposes, remember that the risk of
the fellow eye event ually having a CRVO is approximately 10% (2).
Question 4: correct answer a - shunt blood from retinal velllS to
choroidal circulation
Collateral vessels occur in 50% of patients with a CRVO; they are often not
present at t ime of diagnosis but present months later (6) (2).
Question 5: correct answer d - retinitis pigmentosa
Examples of retinal neovascularization lead ing to vitreous hemorrhage include
diabetic retinopathy, sickle cell retinopathy, retinal vein occlusion, retinopa
thy of prematurity and ocular ischemic syndrome; in each of these cases the
neovasculari zation is preret inal and the newly formed vessels lack endothelial
tight junctions. The location (preretinal) and strength (leaky) of these vessels
creates a situation where vitreous traction can cause shearing of the vessel and
hemorrhage formation.
Diabetes is the most common cause (31 - 54%) of vitreous hemorrhages (2).
The blood in a vitreous hemorrh age typica.lly comes from one of three sources:
vitreous traction (e.g. PVD) a nd/or trauma to normal blood vessels within the
retina, rupture of abnormal blood vessels (retina l neovascularization) or from
another source (e.g. retinal m8croaneurysms, tumors and choroidal neovascu
larization) leaking blood into the area (8).

5.1. CASE 21

UG." .... 'VH

6: correct answer d - intravitreal steroid

Treatment summary for


Neovascularization

patients:
The Central Retinal Vein OcIf
2 clock

terior
neovascu larization
agulation (PRP) is indicated. If no neovascularization is
this
is not benelleia!.

Il1

cvOS demonstrated that focal


CRVO
of the macula does not
visual
ill
with
however, the Standard Care vs. Corticosteroid
Retinal Vein Occlusion
demonstrilted that intravitreal

Macular edema

III

triamci-

Patients should be
, consider

Treatment summary for BRVO


Neovascularization in BRVO

of the occlusion reduces


to .30%
this is not standard of

tent. macular edema


benefit
focal laser
demonstrated a sirnilar benefit in
of macular
with
intravitreal steroid
but the side
from this treatment led
the authors to conclude that focal laser treatment of the macula is the

136

CHAPTER 5. VITREOUS/ RETINA

In itially, patients should be followed every 1-2 mon ths and th en every
3-12 months thereafter; follow-up care should revol ve aro und evaluation
for macu lar edema a nd neovasc ulari zatio n (4) . Always check bloo d pres
sure in t he office. Simil a r to CRVO's, patients sho uld be evaluated for
underlying system ic etiology.

Treatment summary for CRAO and BRAO patients:


Treatment for CRA~ ' s a nd BRAO 's are controversial and ty pica lly ineffective
but might provide some benefit if the m ac ul a is in volved . The overa ll goal of
treatment is to move the embolus as far distally as possible to restore arterial
blood flow. If t he patient presents within 24 hours of vision loss (but best
scenario is within 90 minutes), treatment effectiveness may be enhanced (6) .
Strongly consider ordering ESR in olde r patients with CRA~'s to rule-out
GCA. Discont inue oral contraceptives in young pat ients .
Treatment options include digital ocular massage, ad mission to hospi tal
for carbogen treatment (95% 02 , 5% C02), hyperbaric oxygen chamber
t hera py and therapies ut ilized to lower lOP \vith the goal of increasing
retinal perfusion pressure (glaucoma meds, a nterior chamber paracente
sis (6).

Recall tha t a carotid doppler test is useful in evaluating ocu


la r ischemic syndrome , a conditi on t hat results in carotid a nd /or
ophtha lmi c a rtery blockage; usua lly this is secondary to atheroscle
rosIs.

Vitreous hemorrhages can be treated with pars plana vitreeto my for non
clear ing diabetic vitreous hemorrhages for > 1 month a nd p ersistent (>6 mont hs)
idi opathic vitreo us hemorrhages (6) . Patients wit h vitreous hemorrhages should
avoid aspirin (if health a llows) and ot her ant icoagulants; bedres t and head el
evation is often encouraged to improve likeli hood of hemorrhage set tling infe
riorly.
The Optic Neuritis Treatme nt Trial (ONTT) demonstrated that daily
IV steroids for 3 days, followed by oral prednisone for 11 days sped up t he
visual recovery for patients w ith demyelinating opt ic neuri tis . However, as
has been shown for all t reatment methods with demyelinating optic neuritis,
no treatment has been shown to improve t he final visual outcome (7). For
arteritic AlON, as soo n as a di agnosis is suspected , high-dose steroid t reatment
should be initiated (7); please see neuro section for more deta ils on treatment
of arteri t.ic AlON.

5. 2. CA SE 22

5.2

137

Case 22

Demographics
Age/ race / gender: 18 yea.r-old, whi te mal e; studen t
Chief complaint: decrease in vision
History of present illness
Location: OU

Severity: moderate

Nature of onset: grad ual over the past couple of years

Duration: co nstant

Secondary complaints / symptoms: vision gets worse a t night


Family ocular history
mother: glauco ma
Patient medica l history: Usher's synd rome , di ag nosed early in life
l\1edications taken by patient: none
Patient allergy history: NKDA
Family medical history father: congestive heart fa ilure at age 75
Review of systems
E ar/ Nose/ Throat: hearing im paired
Mental status
Orientation: oriented to time, place , a nd p erson
Mood: appro priate
Affect: appropriate
Distance
Clinical findings

-8.00 - 1.50 x 040


- (.50 - 6 ..50 x 038

PERRL, negat ive APD


PupiJ t M s : full, no restrictions
Confrontation fields: uniform , circular, genera lized constriction of pe
ri pheral vision OD , OS

CHAPTER 5. VITREOUS/RETINA

138
Keratometry:
OD: 42.75
OS: 40.75

030 / 45.00

@
@

042 / 48.25

@
@

120; mires slightly distorted


132; mires severely distorted

Slit lamp
lids/lashes/ adnexa: 2+ erythema of the anterior lid margins OD,
OS
conjunctiva: norm a l OD, OS
cornea: clear OD; Vogt 's striae OS
anterior chamber: deep a nd quiet OD, OS
iris: norm a l OD , OS
lens: trace inferior P SC cataracts OD, OS
vitreous: clear OD , OS
lOPs: 16 mmHg OD, 15 mmHg OS @ 2:30 P.M by applanation tono m
etry
Fundus OD
C/D: 0.10 H/V wit h hyaline bodies of t he optic nerve a nd waxy
opt ic disc pallor
macula: normal
posterior pole: pigment clumping in nud-periphery OD , OS
periphery: unremarkable
Fundus OS
C/D: 0.15 H/V with hyaline bodies of the optic nerve and waxy
optic disc pallor
macula: normal
posterior pole: 1+ attenuated arterioles OD, OS ; pigment clump
ing in mid-periphery OD , OS
periphery: unremarkable
Blood pressure: 102/73 mmHg, right ar m, sitting
Pulse: 68 bpm , regular
Question 1: Which of the following is the MOST likely diagnosis of
this patient 's ocular condition?
a. gyrate atrophy
b. choroideremi a
c. reti nitis pigmentosa
d. fundu s albip unct atus

5.2. CASE 22

139

Question 2: Which of the


condition?

is the most

cause of the

a. mitochondrial euzymc
b. Usher's

membrane metabolism

"\1'7,,,11P

d. inherit.ed from
3: What is
creased vision?

de-

cause of this

bodies

conditions does not cause problindness?


a. fundus
b. choroideremia

c. retinitis
d. gyrate

5: Which of the
of isotretinoin

is NOT a

side effect

lid edema

e.

cerebri

g. cataracts

side effects would be MOST


a.

color defects

d. central retinal

occlusion

CIIAPTER.5. VITREO

140

1: correct answer c - retinitis

nerve, PSC cataracts and


all associaled

macular
si ve contraction of the visual field
mayor
and pe

of other answer choices


very rare,

in the mitochondrial enzyme ornithine

until the
age 10
The patient will report
may notice constricted visual fields. Decreased vision
oc
vision
result of
from macular involvement
curs
the disease "n,)('.''';"

5.2. CASE 22

ln

Choroiderernia is a very
condition characterized by
atrophy of the
retinal pigment
the condition is
in rab
ferase; an enzyme utilized in membrane metabolism (6). Inheritance is X-linked
males
and most commonly presents in the first
recessive
with initial
blindness. By late childhood,
will also
report
and
vision loss and will ha\'e constricted visual
fields. Choroideremia
similar characterist ics to RP
for the

Fundus

punetata
but

2: correct answer b - Usher's


Recall that RP

forms can
or X-linked recessive. RP
more common in I1Tyopes
result of a
in the rnitochondrial
Choroideremia results from
is a

on '''''>;0

ornithine

3: correct answer b
This
The

vision in the left

CHAPTER 5,

142

in this case were very


could not account for the

and

Question 4: correct answer a - fundus albipunctatus


while

Fundus
causes
the other conditions listed cause

5: correct answer g - cataracts

6: correct answer a

a result of oral

.5.3

23

65

retired
floaters

Boaters and flashes of


location

Location: OD
unknown

Nature of onset: sudden


Duration: 48 hours
eyes

ocular
nlother: cataracts

with eye move-

143

5.3. CASE 23
Patient medical history: hypothy roidism
l\!Iedications taken by patient: levothy roxine sodium
Patient allergy history: p enicillin
Family medical history
father: Iung cance r
Review of systems
Endocrine: cold int.olerance , weight gain
Mental status
Orientation: orient.ed to t ime, place, and person

:Mood: appropria.te

Affect: appropriate

Clinical findings

BVA:
I'

----"~D~i~s~tan--c-e-,~~7e-a-r--,

Pupils: PERRL, negative APD

EOMs: full, no restrictions

Confrontation fields: full to finger co unting OD, OS

Slit lamp

lids/lashes / adnexa: unremarkable OD, OS


conjunctiva: normal OD , OS
cornea: mild band kefCItopathy OD, trace ba nd keratopathy OS
anterior chamber: deep and quiet OD, OS
iris: normal OD, OS
lens: clea r OD, OS
vitreous: posterior hyaloid separation from retina with overlying
Weiss ring OD. Normal OS
rops: 16 mmHg OD, 15 mmHg OS
etry

2: 30 Pi'vL by applanation tonom

Fundus OD

CjD: .55 H/V wit h hea lthy rim tissu e


macula: mild RPE mottling
posterior pole: normal

CHAPTER 5. VITREOUS/RETINA

144

periphery: unremarkable
Fundus as
C/D: 0.60 H/V with healthy rim tissue
macula: normal
posterior pole: normal
periphery: unremarkable
Blood pressure: 115/65 mmHg, right arm, sitting
Pulse: 68 bpm, regular
Question 1: Which of the following is the MOST likely diagnosis for
this patient's chief complaint?
a. retinal detachment
b. posterior vitreous detachment (PVD)
c. classic migraine
d. retinitis
e. occipital lobe infarction
Question 2: Which of the following conditions does NOT have the
potential to cause a non-rhegmatogenous retinal detachment?
a. sickle-cell anemia
b. diabetic retinopathy
c. choroidal melanoma
d. macular degeneration
e. none of the above
Question 3: What is the likelihood of a patient with an acute symp
tomatic PVD having a retinal detachment?

a.25%
b.45%
c. 15%
d.1%

Question 4: Which of the following is NOT true regarding migraines?


a. visual aura typically la.'lts less than 60 minutes
b. visual aura takes 5-20 minutes to develop
c. migraine with visual aura is more common than without visual aura
d. recurrent migraines with visual aura signifies a higher risk of stroke

5.3. CASE 23

145

Question 5: What is the likelihood of a patient with lattice degener


ation developing a retinal detachment?
a.25%
b.45%
c. 15%

d.1%

Question 6: Which of the following findings is expected after retinal


detachment surgery with scleral buckle?
a. myopic shift in prescription
b. hyperopic shift in prescription
c. astigmatism shift in prescription
d. no change in prescription

ANSWERS
Question 1: correct answer b - posterior vitreous detachment

This case gives the most common differentials for flashes of light. Photopsia

in eyes with acute PVD is thought to result from traction at the sites of vitre

oretinal adhesion (1). PVD's result from a detachment of the posterior hyaloid

(of the vitreous) from the retina. The hyaluronic acid/collagen complex in the

vitreous is disrupted with age causing the collagen to dump up in bundles. Lib

erated collagen can contract within the complex causing the posterior hyaloid

to detach from the retina, resulting in a PVDj the detachment can be localized,

partial or total (20).

PVD's are quite common - prevalence approximates age, after 50 years old

(50% by age 50, 65% by age 65, etc) (6); however, PVD's occur an average of

.20 years earlier in myopes than in emmetropes (21).

In approximately 10 - 15% of patients with an acute symptomatic


PVD, a retinal break will be present. The risk of a retinal break
increases to 70% if vitreous hemorrhage is present (6). If a PVD
is associated with retinal detachment, progressive field loss is ex
pected (2).

PVD's and degenerative vitreous liquefaction (vitreous changes with age) cre
ate vitreous traction and are the most common causes of retinal tears (2).
Vitreous traction can lead to retinal detachment by initiating the force that

146

CHAPTER 5. VITREOUS/RETINA

allows separation between the retina and RPE and also by creating a poten
tial space for fluid to accumulate and further the separation. Retinal tears
are often associated with floaters and flashing lights (photopsia) but can be
asymptomatic.

Summary of other answer choices


Retinal detachments (RD's) result from separation of the sensory retina
from the underlying retinal pigmented epithelium (RPE). RD's are broken
down into two broad categories (rhegmatogenous and non-rhegmatogenous)
and three overall types (retinal break RD, serous RD, traction RD):
Rhegmatogenous RD's (RRD) result from a retinal break (full-thickness
retinal defect), which by definition includes atrophic holes and tractional
tears.

The term retinal BREAK


atrophic HOLES and traction
TEARS. Thus, an RRD refers to a retinal detachment that was
caused by a hole or tear.

Atrophic holes are round, often small, full-thickness defects that are
not associated with vitreoretinal traction and therefore have a low risk
for subsequent detachment. Holes are caused by chronic atrophy of the
sensory retina; they are most likely to be in the temporal retina (superior
> inferior) (1).
Retinal tears are caused by vitreous traction. Types of retinal tears
include horseshoe (flap) tears and operculated tears. In flap tears the flap
is present because of uneven vitreous traction; vitreoretinal traction often
persists in these cases (vitreous stays attached to the flap) and leads to
an increased risk of retinal detachment as compared to operculated tears.
In operculated tears the initial vitreoretinal traction results in an even,
symmetric tear. After forming the tear, the vitreous pulls away and
vitreoretinal traction no longer persists, reducing the risk for a retinal
detachment.

The superior temporal quadrant (60%) is the most likely location


for a retinal break in patients with an RRD (1). 50% of eyes with
an RRD will have more than one retinal break; in most of these
cases, the break is located within 90 degrees of one another (1).

5.3. CASE 23

147

Non-rhegmatogenous RD's include serous (exudative) RD's and trac


tion RD's. By definition, these RD's are not caused by retinal breaks.
Exudative RD's result from subretinal disorders which damage the
RPE and allow fluid accumulation under the retina (1). Examples include
inflammatory (e.g. scleritis), vascular (e.g. Coats' disease), neoplastic
(e.g. choroidal melanoma) and miscellaneous (e.g. coloboma) causes (2).
The most common cause is ARMD.

'Traction RD's are most commonly caused by proliferative diabetic

retinopathy, retinopathy of prematurity and proliferative sickle cell retinopa

thy (1).

RRD risk factors include:

Lattice degeneration (30%) (6). Patients with Marfan syndrome,


Stickler syndrome and Ehlers-Danlos syndrome can have lattice-like le
sions ("atypical lattice") that increase the patient's risk of a RD (1).
Previous ocular surgery (e.g. cataract surgery). Pseudophakic RD's
often result from small retinal holes at the vitreous base (2).
Posterior vitreous detachment (especially with vitreous hemorrhage).
'Trauma (5 - 10%) (6).
Family history of RRD or previous occurrence of RRD. Both eyes are
eventually involved in about 10% of cases (1).
Myopia: 40% of all RD's occur in myopic eyes (1).
Symptoms for RRD's, ERD's, and TRD's include the following:
Rhegmatogenous RD's (RRD): the patient can be asymptomatic,
but in most cases, symptoms include acute onset of flashes of light,
floaters, shadow or curtain blocking vision and/or decreased vision. Key
signs include retinal elevation (convex retina with folds) with an asso
ciated retinal break (atrophic hole or tractional tear), retinal movement
with eye movement, clear subretinal fluid that does not move eye move
ment, pigment cells in the vitreous (Shafer's sign), hypotony (as com
pared to the non-involved eye) and mild iritis. Chronic RRD cases often
result in a linear pigment demarcation line (takes 3 months or longer to
develop) (1), intraretinal cysts (takes one year or longer to develop) (1),
fixed folds or subretinal precipitates (6).

The quadrant location in which a patient reports flashes of light is


of no value in predicting the location of the primary retinal break;
however, the quadrant location for visual field defects is often valid.
For example, if the field defect is reported in the inferior nasal
quadrant, a superior temporal retinal break is expected (1).

148

CHAPTER 5. HTREOUSjRETINA

Exudative RD's (ERD): patients with ERD's are usually asymp


tomatic unless detachment involves macula; symptoms are similar to
RRD's but are more variable (e.g. vision loss can be minimal to se
vere) (2) .
Tractional RD's (TRD): patients with TRD's do not usually have
complaints of flashes and floaters (1). Symptoms include decrea..,>ed vision
or progressive visual field defect (may remain stationary for months to
years) (1); patients may be asymptomatic.
Retinitis can cause photopsia. For example, many patients with retinitis pig

mentosa report seeing flashes of light (photopsia) and describe them as small,

shimmering, blinking lights similar to the symptoms of an ophthalmic migraine.

Cytomegalovirus retinitis can result in small, peripheral retinal breaks that re

sult in retinal detachments (15 - 50%) (6).

Classical migraine is preceded by a visual aura which develops over 5-20

minutes and lasts less than 60 minutes (6). The aura is most commonly bilateral

and may result in photopsia; patients can report bright spots, zig-zags, jig-saw

puzzle defects, tunnel vision, spreading scotomas, altitudinal field loss and

other visual disturbances (6). These patients will have no vitreous or retinal

abnormalities, unlike other differentials (retinitis, PVD's, retinal detachment)

in this case (1).

Focal occipital lobe infarctions can also result in photopsia (4). This is most

likely to occur in an older patient as a result of a stroke.

Question 2: correct answer e - none of the above

All of the answers listed can cause a non-rhegmatogenous retinal detachment


(RD's). Tractional retinal detachments (commonly from proliferative diabetic
retinopathy) and serous retinal detachments (commonly from ARMD) are com

mon examples of non-rhegmatogenous retinal detachments; recall that these


are RD's that do not originate from retinal breaks (atrophic holes or tractional
tears).

Question 3: correct answer c - 15%

In approximately 10 - 15% of patients with an acute symptomatic PVD, a

retinal break will be prE'.8ent; the risk of a retinal break increases to 70% if

vitreous hemorrhage is present (6).

Question 4: correct answer c - migraine with visual aura is more

common than without visual aura

Most migraines ( 80%) do not have a visual aura (6).

Question 5: correct answer d - 1%

5.3. CASE 23

149

In 25% of cases, lattice degeneration will contain an atrophic hole; retinal


tears can result from vitreoretinal traction on the atrophic, thinned retina (6).
However, as given in this question, only 1% of patients with lattice degeneration
develop a retinal detachment (21).

Lattice degeneration is found in about 6 - 10% of patients


and is more common in myopic eyes. 20 - 33% of patients with
rhegmatogenous retinal detachments will have lattice degeneration.
The condition is bilateral in 33 - 50% of cases (6) and is more com
monly located temporal (than nasally) and superior (than inferi
orly) (1).

Lattice degeneration is typically of little clinical significance because less than


1% of patients with lattice will develop a retinal detachment (2) (21); however,
careful attention to atrophic holes with subretinal fluid and tractional flap tears
is warranted. Treatment considerations for lattice include (6) (1) (2):
Asymptomatic lattice does NOT require treatment. Prophylactic
treatment is not generally indicated unless the fellow eye had lattice
related retinal detachment (2); however, prophylactic treatment can also
be considered for patients with high myopia, aphakia, or strong family
history of retinal detachment (6).
Symptomatic lattice (flashes/floaters) should receive prophylactic treat
ment (cryopexy or laser photocoagulation) (6).

Vitreoretinal tufts are small, focal areas of vitreous traction lo


cated in the retinal periphery. They occur in 5% of the population
and are the second most common peripheral retinal lesion (lattice
is 1st) associated with retinal detachment; less than 1% of patients
with vitreoretinal tufts develop a retinal detachment (18). No pro
phylactic treatment for this condition is indicated.

Question 6: correct answer a - myopic shift in prescription


The following is an overview of treatment and management indications for
retinal detachment, including adverse effects of treatment:
Visual potential is directly related to duration and severity of macular
involvement (1) (2).

150

CHAPTER [i. VITREOUS/RETINA

Chronic asymptomatic RD's can remain stationary, in which case,


treatment is NOT required (2).
Macular detached ("mac-off") RD's usually result in permanent re
duction in vision, even with timely and appropriate treatment; these RD's
should be treated urgently (within 48 96 hours) (6).
Macular threatening ("mac-on") RD's warrant immediate treatment
(within 24 hours) (6).
Tears are more dangerous than holes because of vitreoretinal traction.
Symptomatic tears are more dangerous than those detected during rou
tine examination (1).
Symptomatic tears with persistent traction (horseshoe tears, giant tears)
have a high risk of subsequent detachment and are generally treated
quickly after diagnosis.
Asymptomatic flap-tears have a lower risk for detachment but are typi
cally prophylactically treated. Symptomatic operculated tears also have a
lower risk of detachment; treatment depends on the retinal specialist (2).
Atrophic holes, asymptomatic operculated tears and asymptomatic lat
tice degeneration rarely require prophylactic treatment (2).
Superior breaks are more dangerous than inferior breaks. Retinal breaks
with underlying subretinal fluid should be managed aggressively.
Laser photocoagulation and cryotherapy are not used, in isolation,
for treatment of RD's but instead for treatment of retinal breaks as pro
phylactic therapy to reduce the risk of an RD.
Pneumatic Retinopexy: intravitreal gas bubble is used to temporarily
tamponade the retinal tissue against the RPE, while laser photocoagula
tion or cryotherapy is used to permanently seal the retinal break. This
process has a hlgh success rate and seals the retinal break without uti
lization of a scleral buckle.
Scleral buckle: flexible silicone strip is permanently sutured on or
within the sclera (indenting the sclera) to relieve vitreoretinal traction
on the retinal break. Cryotherapy or laser photocoagulation can then be
used to create permanent adhesion (2). Side effects (non-exhaustive list)
include induced myopia, pain, hemorrhage, infection, diplopia (2).
Vitrectomy: Removal of the vitreous allows release of vitreoretinal trac
tion. Intravitreal gas (retinopexy) or silicone oil is then utilized to tam
ponade the retina before finishing with retinopexy or cryotherapy for per
manent adhesion. Vitrectomy can be performed with or without a scleral
buckle. Common indications for vitrectomy with retinal breaks include
an inability to visualize the break as a result of clouded vitreous (e.g.
blood) and an inability to close retinal breaks through standard tech
niques, typically as a result of very large breaks, posterior breaks that in
clude a macular hole, and severe vitreoretinal traction (1). Post-operative

5.4. CASE 24

151

complications include (non-exhaustive list) elevated rop, cataracts, hem


orrhage, infection, postoperative positioning complications (e.g. patient
should be face-down).

Cryotherapy creates a chorioretinal scar that allows for strong


adhesion between the retina and RPE.

5.4

Case 24

Demographics
Age/race/gender: 70 year-old black male; retired
Chief complaint: decreased vision 'with slight distortion of objects
History of present illness
Location: OD

Severity: unknown; decrease in vision is reported as mild

Nature of onset: gradual

Duration: 4-5 months

Secondary complaints/symptoms: recent onset of floaters that started 2


weeks ago and are now less apparent; no complaints of flashes of light
Patient ocular history: last eye exam 3 years ago; patient wears PAL's
Family ocular history: unremarkable
Patient medical history: chronic bronchitis, diabetes
Medications taken by patient: oral prednisolone
Patient allergy history: NKDA
Review of systems
General/Constitutional: weight gain
Mental status
Orientation: oriented to time, place, and person
Mood: appropriate

CHAPTER 5. VITREOUS/RETINA

152
Affect: appropriate
Clinical findings

BVA:
~I--~=.~--~~~

DIstance

Near

Pupils: PERRL, negative APD

EOMs: full, no restrictions

Confrontation fields: full to finger counting OD, OS

Slit lamp

lids/lashes/adnexa: unremarkable OD, OS


conjunctiva: normal OD, OS
cornea: limbal girdle of Vogt OD, OS
anterior chamber: deep and quiet OD, OS
iris: normal OD, OS
lens: OD: 1+ cortical cataract. OS: trace cortical cataract
vitreous: OD: posterior vitreous detachment with no complication.
Normal OS
lOPs: 16 mmHg OD, 15 mmHg OS @ 2:30 PM by applanation tonom
etry
FUndus OD
C/D: 0.75 H/V with healthy rim tissue
macula: fine, glistening membrane located on inner surface of cen
tral macula
posterior pole: normal
periphery: unremarkable
FUndus OS
C/D: 0.70 H/V with healthy rim tissue
macula: normal
posterior pole: normal
periphery: unremarkable

Blood pressure: 115/65 mmHg, right arm, sitting

Pulse: 68 bpm, regular

Question 1: What is the MOST likely diagnosis of this patient's


posterior segment condition?
a. age-related macular degeneration (AlUv1D)

5.4. CASE 24

153

b. histoplasmosis
c. lacquer cracks
d. epiretinal membrane (ERM)
e. idiopathic central serous chorioretinopathy

Question 2: In this patient, what is the MOST likely cause of his


posterior seglllent condition?
a. related to medication
b. idiopathic
c. high myopia
d. posterior vitreous detachment (PVD)

Question 3: What is the 5 year risk for developing exudative ARMD


in the fellow eye of a patient with exudative ARMD?
a.5%
b.lO%
c. 25 - 35%
d. 40 - 85%

Question 4: What is the likelihood of recurrence


central serous chorioretinopathy?

III

patients with

a.lO%
b.20%
c.2%
d.40%

Question 5: At what visual acuity level is it MOST appropriate to


consider treatlllent for an epiretinallllelllbrane?
a. 20/25
b. 20/30
c. Treatment indicated at any vision level to reduce risk of subsequent
pathology

d. 20/50

154

CHAPTER 5. VITREOUS/RETINA

Question 6: Which of the following is the MOST appropriate treat


ment for this patient's condition OU?
a. laser photocoagulation
b. anti-vascular endothelial growth factor (VEGF) therapy
c. low-vision consultation

d. low-dose antioxidant supplementation

ANSWERS
Question 1: correct answer d - epiretinal membrane (ERM)
This case covers the most common differentials for metamorphopsia. Epireti
nal membrane (ERM) patients are often asymptomatic; mild metamorphop
sia and/or decreased vision are the most commonly reported symptoms. Mild
glial cell proliferation is often referred to as cellophane maculopathy; contrac
tion on this membrane can lead to more severe wrinkling known as macular
pucker. In this case, the location (central) and description (glistening mem
brane) are consistent with cellphone maculopathy and corresponding mild acu
ity loss (20/25-1). Although ERM's are often idiopathic in nature, conditions
that result in breaks in the internal limiting membrane, which allow retinal
glial cells access to the ILM, are causative factors. Risk factors (not exhaustive
list) include posterior vitreous detachment, prior retinal surgery or intraocu
lar surgery (e.g. cataract extraction), trauma, macular holes and intraocular
inflammation. The condition is slightly more common in females (3:2). Preva
lence increases with age: 20% of patients older than 75 will have an ERM (6).

Summary of other answer choices


Idiopathic central serous chorioretinopathy is a condition that results
in RPE and/or choroidal dysfunction; this creates a route for fluid accumula
tion under the macula and subsequent complaints of blurred vision (20/20 to
20/200) and/or metamorphopsia (if macula involved). The condition is most
common in young to middle-aged men (20-50) with a type A personality; it
is associated with stress, pregnancy, steroid use, hypochondrias, hypertension,
others (6). History of similar episodes is common recurrences occur in 40% of
cases (2). Fundus evaluation will reveal a localized, macular serous detachment;
3% of cases will have an RPE detachment as well (6). Fluorescein angiography
will reveal a gradual pooling of fluorescein into a pigment epithelial detach
ment (90% of cases) or "smokestack" appearance (10% of cases) (6). Optical
coherence tomography (OCT) also allows for easy diagnosis of the condition.
Age-related macular degeneration (ARMD) is a common cause of dis
torted vision; both forms, exudative (wet) and non-exudative (dry, atrophic)
can result in metamorphopsia. Macular degeneration is a progressive disease

5.4. CASE 24

155

of the RPE, Bruch's membrane and choriocapillaris (6). The following are a
few noteworthy facts about the condition.
ARMD is the leading cause of blindness in the U.S. population for patients
over 50 years old (6). ARMD is the 2nd leading cause of blindness for
patients 45-64 years old (diabetes is 1st) (13).
Framingham Eye Study revealed that 6.4% of patients 65-74 years old
and 19.7% of patients greater than 75 years old had signs of ARMD (16).
ARMD is more common in Caucasians and females (slightly higher than
males).
Nutritional factors and light toxicity are believed to playa role in patho
genesis (6).
Hyperopia greater than 0.75 diopters (D) increases the risk of exudative
AMD by up to 2.5 times (11).
10 - 20% of patients with ARMD have at least one first-degree family
member with vision loss (11) (12).
Current smokers are more likely to develop ARMD (2.4-2.5 times more
likely) and to have a recurrence of choroidal neovascularization (1.7-2.2
more likely) as compared to those who have never smoked (19).
Increasing age (especially 75 years and older), positive family history,
light iris color, cigarette smoking, hyperopia, hypertension, hypercholes
terolemia, female gender and cardiovascular disease are risk factors for
ARMD (6).

Dry ARMD accounts for 85 - 90% of cases of ARMD (1) (6). Most patients
with dry ARMD do not have vision loss; metamorphopsia, gradual vision loss
(months to years), and blurred vision are common complaints. Dry ARMD is
characterized by the presence of drusen; associated RPE abnormalities (mot
tling, granularity, geographic atrophy, focal hyperpigmentation) may also be
present. Geographic atrophy is the worse form of dry ARMD. 12% of all dry
ARMD patients will develop severe vision loss (defined as loss of > 6 lines) (6);
the majority of these cases results from geographic atrophy. The other major
concern for Dry ARMD patients is progression to Wet ARMD. The Macu
lar Photocoagulation Study Group discovered four risk factors that increase
the likelihood of progression to wet ARMD; these include multiple soft drusen
(especially if confluent), focal hyperpigmentation, hypertension, and smoking.

Confluent drusen and focal hyperpigmentation are risk factors for


exudative ARMD. Some researchers believe that focal hyperpig
mentation may be a choroidal neovascular membrane in early de
velopmental stages (2). Hard drusen (in isolation) is not a risk
factor for more advanced forms of ARMD (6).

156

CHAPTER 5. VITREOUS/RETINA

Wet ARMD accounts for 10 -15% of cases; 88% of legal blindness attributed
to ARMD is caused by the wet form (1) (6). Drusen that are associated with
subretinal fluid (blood or plasma) due to choroidal neovascularization (CNVM)
are characteristic of exudative (Wet) ARMD (2). CNVM's can leak 1) blood
or 2) plasma into two potential places: 1) sub-RPE space 2) subretinal space.
This creates four potential presentations of wet ARJvlD: subretinal hemorrhage
(blood under retina) and sub-RPE hemorrhage (blood under RPE) and sub
retinal detachment (plasma under retina) and sub-RPE detachments (plasma
under RPE) (20). Subretinal detachments are also called serous retinal detach
ments; sub-RPE detachments are also called pigment epithelial detachments
(PED's).
Common symptoms of wet ARJvlD include metamorphopsia, central scotoma
and rapid vision loss. Exudative ARJvlD is the chief cause of vision loss in
patients over the age of 50. The 5-year risk of developing exudative ARMD in
the fellow eye of a patient with exudative ARMD is 40 85% (2).
Histoplasmosis is a fungal infection, caused by Histoplasma capsulatum, with
a classic triad of peripapillary atrophy, multifocallesions in periphery, and mac
ulopathy (including choroidal neovascularization). Most patients are asymp
tomatic; if the macula is involved, the earliest symptom is metamorphopsia (1).
CNV is a late manifestation; if this occurs, incidence is frequently between the
ages of 20 and 45 (1). Recall that this condition is most common in the Ohio
Mississippi River Valley region.
Lacquer cracks represent spontaneous, large linear breaks of Bruch's mem
brane that occur in approximately 5% of high myopes (6) (1); CNV can develop
in association with lacquer cracks, resulting in metamorphopsia and severe vi
sion loss (1).

Uncorrected refractive error, especially the amount and axis of


astigmatism, can cause distortion and warrants careful attention.

Question 2: correct answer d - posterior vitreous detachment (PVD)

Most patients with ERM's have posterior vitreous detachments. ERM's result

from glial cell proliferation onto the internal limiting membrane (ILM). Vitre

oretinal traction during the PVD can create small pores in the 1LM, allowing

intraretinal glial cells access to the front surface of the 1LM (2).

Question 3: correct answer d - 40 - 85%

The incidence of involvement of the fellow eye is estimated to be 28 36%

during the first 2 years, and the annual rate of bilaterality is about 6 - 12% per

year for the next 5 years (11). The overall 5 year risk ranges from 40-85% (2).

5.4. CASE 24

157

Question 4: correct answer d - 40%

Recurrences are common. Some sources report a recurrence rate as high as

50% (18). Most patients improve, without treatment, by 1 to 3 months; 94%

of patients will regain> 20/30 acuity (6). 66% of patients achieve 20/20 vi

sion (18). After resolution of condition, patients often have permanent residual

RPE changes within the macula. In most cases, the condition is observed,

but laser should be considered for treatment in certain scenarios, including:

persistent detachment after 4 months, previous CSR episode that resulted in

permanent vision reduction, patient demand for more immediate visual recov

ery. Laser treatment can expedite recovery process, but does not result in

better final acuity (18).

Question 5: correct answer d - 20/50

Patients with 20/50 visual acuity or worse or patients with intolerable meta

morphopsia complaints may benefit from vitrectomy with membrane peeling

and removal (2).

Question 6: correct answer d - low-dose antioxidant supplementa

tion

The following is an overview of treatment and management for dry and wet

ARMD. Management for dry ARMD involves the following:

Cessation of smoking.
Monitoring of Amsler grid daily.
Consideration of high-dose antioxidants and vitamins for patients with
category 3 (intermediate) and category 4 (advanced) ARMD; the Age
Related Eye Disease Study (AREDS) revealed a modest benefit in these
patients (see box below) but no benefit in patients with category 1 (early)
or category 2 (mild) ARMD.

AREDS demonstrated an absolute risk reduction of6% (29-23%)


in helping patients with more advanced ARMD (categories 3 and
4); these patients lost less than 15 letters (3-lines) of visual acuity
over the 5 year period as compared to placebo (15).

Low-dose antioxidants (iCAPS, ('~ntrum silver) for patients with cate


gory 1 and category 2 ARMD and patients with strong family history of
the condition (6).
Low-vision rehabilitation for patients with functional vision loss.

158

CHAPTER 5. VITREOUS/RETINA

AREDS concluded that current smokers should NOT take beta


carotene at high doses (15 mg) due to increased risk of lung can
cer. The AREDS formula includes vitamin C (500 mg), vitamin E
(400 International Units), beta-carotene (15 mg), zinc (80 mg) and
copper (2 mg) (14).

Prior to a review of wet ARMD management, a quick terminology review is


pertinent:
Subfoveal: directly under foveal avascular zone (FAZ)
Juxtafoveal: 1-199 um's from center of FAZ or ... CNV outside of this
area (in extrafoveal zone) that has leaked fluid within 1-100 urn's of FAZ.
Extrafoveal: 200-2500 um's from center of FAZ

FAZ is an avascular area of approximately 500 um's in diameter in


the center of the fovea (4).

Fluorescein angiography can be used to classify CNV's as classic or occult.


Classic CNV's are characterized by a well-defined membrane which fills
with dye during the early phases of the procedure. Occult CNV's are
characterized by a poorly-defined membrane with later appearing, and
less intense, fluorescein leakage (2) (1).
Most patients with wet ARMD have a CNV that has a combination of
bo~h classic and occult features (2). The term "predominantly classic"
means that over 50% of the entire lesion is composed of classic CNV (6).
Treatment options for wet ARMD include the following:
Thermal laser photoagulation: Proven by the macular photocoagula
tion study (MPS) to be beneficial in subfoveal, jll.xtafoveal and extrafovea
CNV's. However, up to 60% of eyes that undergo treatment will develop
recurrent CNV's, the majority of which will be sub foveal (6). Thus,
the MPS established laser photocoagulation as the treatment of choice
for well-delineated extrafoveal CNV's. Juxtafoveallesions that are sub
foveal or "barely juxtafoveal" (very close to subfoveal region) are typically
treated with photodynamic therapy (PDT) or anti-VEGF agents (2) (6).
The Treatment of ARMD with PDT Trial (TAP) revealed that visu
dyne (verteporfin) is recommended for subfoveal, predominantly classic
CNV's (2) (6) (18). Patients should avoid direct sunlight or bright indoor
light for at least 48 hours after each treatment.

5.5. CASE 25

159

The Verteporfin in PDT Trial (VIP) Verteporfin in PDT-Pathological


Myopia (VIP-PM) Trial revealed that visudyne is recommended in the
management of subfoveal occult but NOT classic CNV when there is
evidence of recent disease progression, especially in specific circumstances
(e.g. baseline vision worse than 20/50) (18) .
Anti-VEGF agents (macugen, avastin, lucentis) are becoming main
stream treatment choices for wet ARMD. Macugen has been shown to
decrease visual loss in all CNV subtypes (18).

5.5

Case 25

Demographics
Age/race/gender: 69 year-old white male; retired
Chief complaint: blurry vision
History of present illness
Location: OD

Severity: moderate

Nature of onset: gradual

Duration: 4-5 months

Secondary complaints/symptoms: none


Family ocular history
mother: retinal macroaneursym
Patient medical history: hypertension; hyperlipidemia
Medications taken by patient: captopril; simvastatin
Patient allergy history: NKDA
Family medical history: cardiac disease
Review of systems
General/Constitutional: patient has not been feeling well the last cou
ple of weeks
Mental status
Orientation: oriented to time, place, and person

GHAPTER 5. VITREOUS/RETINA

160
Mood: appropriate
Affect: appropriate
Clinical findings

Pupils: PERRL, negative APD


EOMs: full, no restrictions
Confrontation fields: full to finger counting aD, as
Slit lamp
lids/lashes/adnexa: unremarkable aD, as
conjunctiva: normal aD, as
cornea: Hudson Stahli line inferiorly aD, as. Corneal arcus aD,
as
anterior chamber: deep and quiet aD, as
iris: normal aD) as
lens: 2+ NS aD, trace cortical changes as
vitreous: posterior vitreous detachment aD with no complication.
Normal as
lOPs: 16 mmHg aD, 15 mmHg as
etry

2:30 PM by applanation tonom

Fundus OD
C/D: 0.10 H/V with healthy rim tissue
macula: normal
posterior pole: arteriovenous (AV) nicking, 2+ diffuse cotton wool
spots (C\VS), mild scattered retinal hemorrhages and hard ex
udates
periphery: circular areas of peripheral atrophy with pigmentation
(cobblestone degeneration)
Fundus OS
C /D: 0.15 II/V with healthy rim tissue
macula: normal
posterior pole: A/V nicking; 3+ diffuse CWS; mild scattered reti
nal hemorrhages and hard exudates

periphery: cobblestone degeneration

Blood pressure: 195/125 mmHg, right arm, sitting


Pulse: 68 bpm, regular

5.5. CASE 25

16]

Question 1. Which of the following is the MOST likely diagnosis of


this patient's posterior segment condition?
a. hypertensive retinopathy
b. diabetes mellitus
c. branch retinal vein occlusion (BRVO)
d. Acquired Immunodeficiency Syndrome (AIDS)
e. myelinated nerve fibers

Question 2: What condition is MOST commonly the etiology for


cotton-wool spots?
a. hypertensive retinopathy
b. diabetes mellitus
c. branch retinal vein occlusion
d. Acquired Immunodeficiency Syndrome (AIDS)
e. myelinated nerve fibers

Question 3: What layer of the retina are cotton-wool spots located?


a. nerve fiber layer (NFL)
b. internal limiting membrane (ILM)
c. outer plexiform layer (OPL)
d. inner plexiform layer (IPL)

Question 4: Which of the following is the MOST appropriate treat


ment for this patient's condition OU?
a. STAT referral to emergency room (ER)
b. consult with primary care physician
c. laser photocoagulation to area of affected retina
d. STAT steroid injection

Question 5: Which of the following drugs can cause a non-arteritic


ischemic optic neuropathy (NAION)
a. sumatriptan
b. isotretinoin
c. tamoxifen

d. bropheniramine

162

CHAPTER 5. VITREOUS/RETINA

Question 6: Topical B-blockers can cause all of the following side


effects EXCEPT
a. erectile dysfunction
b. depression
c. overactive bladder
d. bradycardia
e. bronchoconstriction

ANSWERS
Question 1: correct answer a - Hypertensive Retinopathy

This case overviews the most common causes of cotton-wool spots (CWS). This

case gives a classic description of a patient with grade 3 hypertensive (HTN)

retinopathy. Recall that cotton-wool spots and hard-exudates are categorized

as stage 3 findings; in this stage, diastolic blood pressure is typically at least

110-115 mm Hg and retinal arteries lose their ability to autoregulate blood

flow (2). Cotton-wool spots in HTN retinopathy are usually within 3DD of the

optic nerve head (28).

Recall that HTN retinopathy can result from chronic or acutely elevated (ma

lignant) systemic blood pressure. Hypertension is defined as blood pressure>

140/90 mm Hg. In suspected cases of HTN retinopathy, blood pressure should

be checked to aid in diagnosis. Recall that essential hypertension accounts for

95% of all cases of hypertension and is characterized by elevated blood pres

sure with no known cause (28). HTN retinopathy results from a breakdown of

the blood-retinal barrier (28). Retinal findings are almost always bilateral (4).

The Keith-Wagener-Barker classification system of HTN retinopathy is as fol

lows (2):

Grade 1: retinal arterial narrowing


Grade 2: retinal A/V nicking
Grade 3: retinal hemorrhages, CWS, hard exudates
Grade 4: grade 3 changes plus optic disc swelling

El<lchnig spots are focal areas of choroidal atrophy (from nonper


fusion); they represent past episodes of acute hypertension (4).

Vision loss from HTN retinopathy is rare; however, severe cases can result in
vision loss secondary to optic nerve edema, macular star (macula edema with
hard exudates) and retinal macroaneurysms. HTN is associated with numerous

5.5. CASE 25

163

secondary conditions that can lead to vision loss including vascular occlusions,
retinal macroaneurysm, nonarteritic AION, ocular motor nerve palsies, and
worsening of diahetes (18).

Summary of other answer choices


Diabetic retinopathy (DR) is the most likely culprit for cotton-wool spots (2).
HTN retinopathy is another common cause. HTN retinopathy and DR can ap
pear similar and distinguishing between the effects of each can he difficult when
patients have both conditions (28). However, hypertensive retinopathy is more
likely to present with a "dry" retina (few hemorrhages, rare edema, rare ex
udates, multiple CWS), whereas diabetic retinopathy, in comparison, is more
likely to present with a "wet" retina (multiple hemorrhages, multiple exudates,
more edema, few CWS) with less vessel attenuation (17).
Branch retinal vein occlusions (BRVO's) are another cause of CWS; recall
that HTN and DM are commonly associated with BRVO's and, when present,
are contributors to the occlusion (via thrombus formation). CWS from a BRVO
will be unilateral and in the area of the occluded vessel; this is frequently in
one quadrant only. This description varies from what was presented in this
case.
Cytornegaloviris retinitis is the most common retinal infection in AIDS (6);
it affects 40% of AIDS patients (l) and is most common in patients with CD4
counts of less than 50. CMV results in a hemorrhagic retinitis with thick white
yellow patches of necrotic retina, vascular sheathing, retinal hemorrhages and
cotton-wool spots.
Myelinated nerve fibers are unilateral (80%) (6), feathery, yellow-white
patches of myelination that typically follow the normal course of the nerve
fiber layer. The myelination can also appear as peripapillary myelination or
isolated peripheral patches and can cause confusion for papilledema or cotton
wool spots on cursory examination. Recall that myelination of nerve fibers
does not typically start until after the ganglion cell axons leave the globe and
past posterior to the lamina cribosa. Patients are typically asymptomatic but
can have corresponding visual field defects (6).

Question 2: correct answer b - Diabetes Mellitus


CWS are present in 44% of patients with DR (2).

Question 3: correct answer a - nerve fiber layer (NFL)

Cotton-wool spots (CWS) are microinfarcts in the nerve fiber layer from arteriolar

capillary occlusion and simply represent ischemia within the nerve fiber layer (1).

Recall that flame-shaped hemorrhages (Drance hemes) are also found in the

NFL. Hard exudates are found within the OPL.

164

CHAPTER 5. VITREOUS/RETINA

Question 4: correct answer a - STAT referral to emergency room

(ER)

A patient with blood pressure this high (190/125) needs immediate medical

attention. A diastolic blood pressure reading of 110-120 mm Hg or symptoms

of chest pain, difficulty breathing, change in mental status, and/or decreased

vision with optic disc swelling warrant the immediate referral (6). Ocular

evaluation for HTN retinopathy is warranted every 2-3 months at first, and then

every 6-12 months thereafter (4). Treatment for CWS involves identifying and

controlling the underlying etiology; assuming this occurs, CWS almost always

disappear within 5-7 weeks (28).

Question 5: correct answer a - surnatriptan

Sildenafil (Viagra) , sumatriptan (Imitrex) and amiodarone (Cordarone) can

cause an NAION (3).

Question 6: correct answer c - overactive bladder

Side effects oftopical B-blockers include (non-exhaustive list) dry eye, depres

sion, impotence, bradycardia, bronchoconstriction (3).

5.6

Case 26

Demographics
Age/race/gender: 15 month-old white female; patient born 2 weeks
early with normal birth weight
Chief complaint: mom reports large white spot in the center of her eye
History of present illness
Location: OD

Severity: unknown

Nature of onset: since birth

Duration: constant

Secondary complaints/symptoms: mom reports that her child looks like


she has "crossed" eyes
Family ocular history
mother: glaucoma
Patient medical history: unremarkable

5.6. CASE 26

165

Medications taken by patient: none


Patient allergy history: NKDA
Family medical history
father: hyperthyroidism
Review of systems: unremarkable
Mental status
Orientation: oriented to time, place, and person

Mood: appropriate

Affect: appropriate

Clinical findings
OD: patient unable to fix and follow
OS: patient able to fix and follow
Pupils: white pupillary reflex OD; PERRL OU; 1+ APD OD
Slit lamp
lids/lashes/adnexa: unremarkable OD, OS
conjunctiva: normal OD, OS
cornea: clear OD, OS
anterior chamber: deep and quiet OD, OS
iris: diffuse iris neovascularization OD, normal OS
lens: clear OD, OS
vitreous: clear OD, OS
lOPs: 16 mmHg OD, 15 mmHg OS
etry

2:30 PM by applanation tonom

Fundus OD
C /D: 0.10 H/V with 2+ diffuse disc edema
macula: 2+ diffuse macular edema
posterior pole: 12 mm diameter solid, elevated, yellow-white mass
with overlying dilated tortuous blood vessels
periphery: unremarkable
Fundus OS

C/D: 0.15 H/V with healthy rim tissue

macula: normal

posterior pole: normal

periphery: unremarkable

Blood pressure: 115/65 mmHg, right arm, sitting

Pulse: 68 bpm, regular

166

CHAPTER 5. VITREOUS/RETINA

Question 1: What is the MOST likely diagnosis of this patient's


condition?
a. toxocariasis
b. retinoblastoma
c. Coats disease
d. retinopathy of prematurity

Question 2: What is the MOST likely etiology of this patient's con


dition?
a. idiopathic
b. mutation in tumor suppressing gene
c. low birth weight
d. pica

Question 3: Which of the following listed is a common associated


ocular finding in retinoblastoma?
a. strabismus
b. cortical cataract
c. retinoschisis
d. acute angle-closure glaucoma

Question 4: What is the long-term survival percentage in a patient


with retinoblastoma?
a.lO%

b.2%

c.85%

d.25%

Question 5: Which of the following is the MOST appropriate treat


ment for this patient's condition OU?
a. observe for now and monitor condition again in 2 months
b. refer for pauretinal photocoagulation (PRP) therapy
c. refer with recommendation of laser photocoagulation
d. refer with recommendation of chemotherapy

5.6. CASE 26

167

ANSWERS
Question 1: correct answer b - retinoblastoma

This case gives the most common differentials for leukocoria. The tumor de

scribed in this case is characteristic of a large retinoblastoma. The tumor has

infiltrated the nerve and macula, resulting in an APD and decreased vision,

respectively. Leukocoria in retinoblastoma results from a yellow-white intraoc

ular mass. Small retinoblastomas are white and flat, while large retinoblas

tomas appear solid, elevated and yellow-white, with overlying dilated tortuous

blood vessels. The tumor can arise from inner retina and grow toward vitre

ous (endophytic) or can arise from outer retina and grow toward the choroid

(exophytic) (18).

Retinoblastoma is the most common intraocular malignancy in children

(1/20,000) (18) and is the 2nd most common of all age groups (choroidal

melanoma is most common) (1). 95% of cases are diagnosed by 5 years of

age (6). The condition has no gender or race predilection. The tumor is de

rived from cells in the developing retina (retinoblasts) as a result of mutations

in the retinoblastoma (Rb) tumor-suppressor gene (2). Heritable retinoblas

toma accounts for approximately 40% of cases; only 6% of these patients have

a positive family history of the condition (1). 85% of heritable retinoblas

toma cases are bilateral with multiple tumors; the risk of transmitting this

tumor to offspring is 50% (1). Non-heritable cases account for 60% of cases;

these children typically only develop one tumor. 85% of all unilateral cases

of retinoblastoma are non-heritable (1). Parents with one affected child have

a 6% risk of transmission to another child. Parents with 2 or more affected

children have a 40% risk (18).

Summary of other answer choices


Leukocoria can be present in advanced cases of retinopathy of prematurity
(ROP). The white pupillary reflex in these cases is a result of fibrovascular
scarring secondary to a tractional retinal detachment. ROP is a proliferative
retinopathy that affects premature infants 36 weeks) or infants of low birth
weight 2,000 g) who have received oxygen therapy (2). Retinal vessels do
not develop into the temporal retina until the 9 month of gestation (1); this
area is most susceptible to neovascularization and subsequent tractional retinal
detachments in pre-term infants.
Leukocoria in ocular toxocariasis results from yellow-white granuloma for
mation in children and young adults. Toxocara canis is an intestinal roundworm
that infects dogs and other canids and is transferred to humans by fecal mat
ter (28) (2).The condition usually occurs in children and young adults and is
associated with pica (e.g. eating dirt) and close contact with puppies (28) (6).
Children may present with unilateral iritis and diffuse vitritis (with associated
retinal detachment) and decreased vision or with posterior granuloma and can
be associated with chronic endophthalmitis (28) (6) (2).

CHAPTER 5. VITREOUS/RETINA

168

Leukocoria in Coats disease results from an exudative macular detachment.


Coats disease is an idiopathic peripheral vascular disease that results in unilat
eral telangiectatic, dilated vessels that display a characteristic "light bulb" ap
pearance. Progression of the disease can lead to marked hard exudates (classic
for Coats), intraretinal hemorrhages, exudative retinal detachment, and neo
vascular glaucoma; the latter of which can result in a red, painful eye (20).
Peak incidence in males (85%) prior to age 20; 2/3 of cases will be diagnosed
prior to age 10 (6). Children may present with poor vision, strabismus or
leukocoria.
Question 2: correct answer b - mutation in tumor suppressing gene
As described above, retinoblastomas are derived from cells in the developing
retina (retinoblasts) as a result of mutations in the retinoblastoma (Rb) tumor
suppressor gene (2).
Question 3: correct answer a - strabismus
Leukocoria (60%), strabismus (20%) and iris neovascularization are the most
common presenting signs (1). Iris neovascularization ( 20%) (2) and decreased
vision (5%) are relatively common findings (18).
Question 4: correct answer c - 85%
Individual treatment success varies depending on the number of tumors and
their size and location; however, overall prognosis is good with survival rate
reaching 85 - 90% (6). Early detection is the key. The main determinant for
mortality is optic nerve invasion; optic nerve infiltration can increase mortality
rate to 65% as compared to a mortality rate of only 8% in cases of no optic
nerve involvement. Retinoblastoma has one of the highest cure rates of any
malignanJ, tumor; however, if untreated, the prognosis is grave (21).
Question 5: correct answer d - refer with recommendation of chemother
apy
Large tumors are often treated with chemotherapy and enucleation (1). Addi
tional treatment options for retinoblastoma external beam radiotherapy, laser
photocoagulation, brachytherapy and cryotherapy (1) (6) (2). An experienced
ophthalmic oncologist should perform treatment (6). Genetic counseling should
be considered, especially in familial cases.

5.7

Case 27

Demographics

169

5.7. CASE 27
Age/race/gender: 25 year-old white male
Chief complaint: red eye with decreased vision
History of present illness
Location:

aD

Severity: moderate

Nature of onset: sudden

Duration: 4 days

Secondary complaints/symptoms: floaters in the right eye; no flashes of


light
Family ocular history
mother: retinal detachment
Patient medical history: asthma
Medications taken by patient: advair
Patient allergy history: NKDA
Review of systems
Respiratory: occasional shortness of breath
Mental status
Orientation: oriented to time, place, and person
Mood: appropriate
Affect: appropriate
Clinical findings
BVA:
1'----~D~is-t-an--c-e~~N~e-ar--~

Pupils: PERRL, negative APD

EOMs: full, no restrictions

Confrontation fields: full to finger counting

aD, as

Slit lamp

lids/lashes/adnexa: unremarkable
conjunctiva: normal aD, as

cornea: clear aD, as

aD, as

CHAPTER 5. VITREOUS/RETINA

170

anterior chamber: OD: 1+ cells/1+ flare. Normal OS


iris: normal 0 D, OS
lens: clear OD, OS
vitreous: OD: 2+ vitritis. Clear OS
lOPs: 16 mmHg OD, 15 mmHg OS
etry

2:30 PM by applanation tonom

Fundus OD

CjD: 0.10 H/V with healthy rim tissue


macula: normal
posterior pole: chorioretinal scar with adjacent focal, fluffy, yellow
white retinal lesion
periphery: unremarkable
Fundus OS

CjD: 0.15 H/V with healthy rim tissue


macula: normal
posterior pole: normal
periphery: unremarkable
Blood pressure: 115/65 mmHg, right arm, sitting
Pulse: 68 bpm, regular
Question 1: What is the MOST likely diagnosis of this patient's
posterior segment condition?
a. dominant drusen
b. fundus flavimaculatus
c. Stargardt's disease
d. toxoplasmosis
e. histoplasmosis
f. Best's disease

Question 2: What is the MOST common source of toxoplasmosis


infection?
a. bird droppings
b. undercooked meat
c. house fly
d. tick bite

5.7. CASE 27

171

Question 3: Which of the following conditions is the MOST common


cause of infectious retinitis?
a. dominant drusen
b. fundus flavimaculatus
c. Stargardt's disease
d. toxoplasmosis
e. histoplasmosis

Question 4: Which of the following findings is expected in a patient


with Best's Disease?
a. normal EOG, normal ERG
b. normal EOG, abnormal ERG
c. abnormal EOG, normal ERG
d. abnormal EOG, abnormal ERG

Question 5: Which of the following is the MOST appropriate treat


ment for this patient's posterior segment condition?
a. topical antifungal
b. oral steroids
c. oral pyrimethamine
d. topical antiviral

Question 6: Which of the following drugs can cause pigment deposits


within the macula?
a. indomethacin
b. isoniazid
c. methotrexate
d. tamsulosin

ANSWERS
Question 1. correct answer d - toxoplasmosis

This case overviews the classic presentation of toxoplasmosis. This is a young,

healthy patient with recent onset of unilateral redness, photophobia, floaters,

uveitis, vitritis and decreased vision. Fundus findings are consistent with toxo

plasmosis; a focal, fluffy, yellow-white retinal lesion adjacent to an old, inactive

scar with an overlying vitritis (1) (6).

172

CHAPTER 5. VITREOUS/RETINA

Toxoplasmosis results from Toxoplasmas gondii, an obligate intracellular in


testinal parasite. The patient can present with congenital or acquired disease
or with reactivation of a previously stable lesion. Patients become infected
with the parasite through inhalation of cat waste or through ingestion of un
dercooked lamb, pork or beef and unpasteuri",ed cheeses (21).

Congenital toxoplasmosis is by far the most common form of toxoplasmo


sis (21). Toxoplasmosis is transferred to the fetus in 40% of cases where the
mother contracts the acute form during pregnancy (21). If affected prior to
pregnancy, the baby will not be harmed (1). There is a wide spectrum of
clinical presentation for congenital toxoplasmosis. In 10% of cases, severe sys
temic involvement results in the triad of convulsions, cerebral caldfications and
chorioretinitis (28); mild cases result in small, insignificant chorioretinal scars.

Recurrence of old, stable, congenital ocular toxoplasmosis is the


most common cause of infectious retinitis (1).

Acquired toxoplasmosis is most common in immunocompromised patients. Ac


quired and reactivated congenital lesions present in similar f&'lhion, as overviewed
above for this case. The chorioretinal lesion is often difficult to view clearly
due to the dense overlying vitritis; viewing the lesion in light of this haziness is
often referred to as "headlights in the fog." The average age of onset for these
recurrences is 25 (ranges from 1O-:~5) (1).

Summary of other answer choices


Best's disease is an autosomal dominant inherited macular disorder char
acterized by "egg-yolk" lesions in the macula. The age of onset varies, but is
often in early childhood (5-10 years old). Signs and symptoms vary significantly
among patients.
Stage 1 (pre-vitelliform): characterized by an abnormal EOG in a
normal fundus in an asymptomatic patient.
Stage 2 (vitelliform): egg-yolk macular lesion appears; this is most
likely to occur between ages 3-15 (6).
Stage 3 (pseudohypopyon): entire lesion can become absorbed with
little to no effect on vision.
Stage 4 (vitelliruptive): "egg-yolk" starts to break-up and a "scrambled
egg" appearance can result; mild visual loss is expected in this stage.
Stage 5 (end-stage): characterized by moderate to severe vision loss
as a result of choroidal neovascularization, hemorrhage, atrophy and/or
scarring.

5.7. CASE 27

173

The egg-yolk is thought to be an abnormal accumulation of lipo


fuscin within RPE cells (28). The egg-yolk can remain stable for
years, with only a mild reduction in visual acuity (20/30 to 20/50)
into midlife (28).

Overall, 75% of patients age 40 and under maintain visual acuity better than
20/40 ill at least one eye (28). Adult foveomacular vitelliform dystrophy
presents in patients age 30-50. Signs are similar to Best's disease, but over
all prognosis is better; minimal metamorphopsia, mild acuity loss, normal
EOG and normal ERG with slight tritan color defect is the classic presen
tation (21) (20).
Dominant drusen (e.g. familial drusen) is characterized by scattered drusen
throughout the posterior pole that occurs within the first three decades of
life (6) (28). The drusen are bilateral and symmetric and often appear at the
macula and around the optic nerve head; some suggest that drusen located
nasal to the optic disc is pathognomonic for the condition (1).
Patients are often asymptomatic, unless degenerative changes occur within the
macula. The drusen can increase in size and number with age and become
more confluent and can progress to affect the macula, resulting in a choroidal
neovascular membrane, RPE detachment, or other pathology; in these cases,
decreased acuity is not expected until the fifth to sixth decade. The core
problem in dominant drusen is an abnormal RPE, which allows for choroidal
neovascularization and sensory and/or RPE detachments; findings that are
similar to age-related macular degeneration. Symptoms with dominant drusen
include decreased vision and metamorphopsia; ERG is normal in these patients,
EOG abnormal (21).
Stargardt macular dystrophy and fundus flavimaculatus are considered
variants of the same disorder. Onset is typically in the first or second decade
of life (ages 6 to 20) as a result of decreased acuity. Inheritance is typically
autosomal recessive. Stargardt's disease is the most common hereditary mac
ular dystrophy (6). There is no sex predilection. In early stages, mild, non
specific mottling and a loss of foveal reflex can result in decreased vision that
is often out of proportion with the fundus appearance. As the condition pro
gresses, bilateral yellow flecks appear scattered in a pisciform (fish-tail) configu
ration throughout the posterior pole and mid-periphery. In late stages a classic
"beaten-bronze" macular pattern is apparent ("Bull's eye maculopathy") and
"salt and pepper" pigmentary changes may appear in periphery. Symptoms in
(28).
clude rapid vision loss, color vision abnormalities and complaints of
Unfortunately, in these patients, when acuity starts to drop to 20/40, further
loss to 20/120 or worse occurs quickly (within 4-5 years) (28). Color vision
defects are not present until the advanced stages of the disease; however, this
may aid the practitioner in ruling-out cone dystrophy in diagnosis (28). ERG

174

CHAPTER 5. VITREOUS/RETINA

is normal in early stages of the disease, but becomes abnormal as advanced


stages develop.

Fundus fiavimaculatus diagnosis is reserved for patients with


Stargardt characteristics but no macular dystrophy signs; the con
dition often presents later in life (4th, 5th decade) and patients
are commonly asymptomatic (6) (1). Vision loss can still occur if
fleck-lesions involve the macula.

Question 2. correct answer b - undercooked meat

Humans can become infected with the parasite through eating undercooked

meat that contains tissue cysts (2).

Question 3. correct answer d - toxoplasmosis

Recurrence of old, previously stable, congenital toxoplasmosis is the most com

mon cause of posterior segment infection (infectious retinitis) (1).

Ocular toxoplasmosis is the most common proven cause of chori


oretinitis in the world (21).

Question 4. correct answer c - abnormal EOG, normal ERG


A key sign for diagnosis is an abnormal BOG (even prior to vision loss or fundus

signs) with a normal ERG (28).

Question 5: correct answer c - oral pyrimethamine

Treatment indications for toxoplasmosis depend on location and severity of in

flammation. Small, peripheral lesions may be observed or treated with bactrim.

Moderate to severe vitreou.'l inflammation and sight-threatening lesions (mac

ula, papillomacular bundle, optic nerve) should be treated (6) (18). Treatment

involves a combination of systemic steroids and at least one antitoxoplasmosis

agent (21). Therapies include:

systemic steroids

oral pyrimethamine and oral sulfadiazine

folinic acid (counteracts thrombocytopenia caused by pyrimethamine) (21)


sulfadiazine OR clindamycin OR azithromycin

5.8. CASE 28

175

total course of treatment is 5-6 weeks


Most patients with Stargardt's disease preserve moderate visual acuity (20/70
- 20/200), at least in one eye (2). There is no effective treatment available
for Stargardt's, dominant drusen or Best's disease patients. Patient's with
functional vision loss should be referred for low-vision rehabilitation.
Question 6: correct answer a - indomethacin
Indomethacin causes pigmentary retinopathy, especially within the macula;
thioridazine is another cause of pigmentary retinopathy. Recall that tamoxifen
causes crystallin retinopathy (3). Tamsulosin (flomax) causes intraoperative
floppy iris syndrome. Isoniazid and methotrexate, in rare cases, can cause
optic neuritis (3).

5.8

Case 28

Demographics
Age/race/gender: 64 year-old white male; retired
Chief complaint: vision loss
History of present illness
Location: OD

Severity: unknown

Nature of onset: gradual

Duration: started 3 months ago

Secondary complaints/symptoms: dull ache within and around the right


eye; patient reports several recent incidents of acute vision loss in the
right eye that last a minute or two before returning to normal.
Family ocular history
mother: anterior ischemic optic neuropathy
Patient llledical history: cardiac disease; hypercholesterolemia; previous stroke
7 years ago; severe gout diagnosed 5 years ago; hypertension, moderate
control with meds.
Medications taken by patient: indomethacin, atorvastatin, lisinopril
Patient allergy history: NKDA
Review of systems

CHAPTER 5. VITREOUS/RETINA

176
Musculoskeletal: podagra
Mental status

Orientation: oriented to time, place, and person

Mood: appropriate

Affect: appropriate

Clinical findings

BVAr:__~~~__-'~__~

Distance

Near

Pupils: PERRL, negative APD

EOMs: full, no restrictions

Confrontation fields: full to finger counting OD, OS

Slit lamp

lids/lashes/adnexa: unremarkable OD, OS


conjunctiva: pingueculas, no active inflammation, located nasally
and temporally OD, OS
cornea: farinata OD, OS; 1+ anterior basement membrane dystro
phy superior to visual axis OD.
anterior chamber: deep and quiet OD, OS
iris: OD: sectoral (1 o'clock) neovascularization. Normal OS.
lens: clear OD, OS
vitreous: clear OD, OS
lOPs: 16 mmHg OD, 15 mmHg OS @ 2:30 PM by applanation tonom
etry
Fundus OD
C/D: .30 H/V; neovascularization of the optic disc at 1 o'clock
macula: normal
posterior pole: normal
midperiphery/periphery: dot/blot hemorrhages of the midpe
ripheral fundus, dilated non-tortuous retinal veins, narrowed
retinal arteries
Fundus OS

C/D: 0.35 H/V with healthy rim tissue

macula: normal

posterior pole: normal

midperiphery/periphery: unremarkable

Blood pressure: 145/95 mmHg, right arm, sitting

Pulse: 68 bpm, regular

5.8. CASE 28

177

Question 1: Which of the following is the MOST likely diagnosis for


this patient's chief complaint?
a. ocular ischemic syndrome
b. central retinal vein occlusion (CRVO)
c. central retinal artery occlusion (CRAO)
d. proliferative diabetic retinopathy
Question 2: What is the MOST likely cause of this patient's condi
tion?
a. carotid artery obstruction
b. pericyte damage
c. hypertension
d. medication related
Question 3: Which of the following is the MOST appropriate treat
ment for this patient's rubeosis?
a. low-vision consultation
b. vitrectomy
c. argon laser to rubeosis
d. panretinallaser photocoagulation (PRP) of posterior segment
Question 4: What is the expected prognosis for this patient after
treatment of the rubeosis?
a. good prognosis - stable vision expected
b. outstanding prognosis - improvement in vision expected
c. 25% chance of 20/50 acuity or better at 1 year
d. 90% chance of count fingers or worse acuity at 1 year
Question 5: When is it MOST likely for a patient with an ischemic

CRVO to develop rubeosis?


a. 1 week
b. 12 months
c. 8 months
d. 2-4 months

CHAPTER 5. VITREOUS/RETTNA

178

ANSWERS
Question 1: correct answer a - ocular ischemic syndrome
This case overviews the most common causes of rubeosis. Rubeosis is most
common in proliferative diabetic retinopathy, central retinal vein occlusion and
carotid occlusive disease.
This patient has classic symptoms for ocular ischemic syndrome - gradual
vision loss (90% of cases), periorbital dull pain or headache (40% of cases)
and amaurosis fugax (6). Common signs include unilateral (80%) dot/blot
hemorrhages of the midperipheral fundus, dilated non-tortuous retinal veins,
narrowed retinal arteries and neovascularization of the disc. If a patient has
these retinal findings and carotid artery obstruction, but no anterior segment
signs, the condition is called venous stasis retinopathy. The presence of both
posterior and anterior segment signs and symptoms is referred to as ocular
ischemic syndrome (OIS) (27). Ocular Ischemic Syndrome is most common in
male (2:1) patients 50-70 years old (mean
65 years) (6).
The condition results from carotid and/or ophthalmic artery (less common)
blockage - usually secondary to atherosclerosis (over 90% of cases) and occa
sionally as a result of giant cell arteritis (2). OIS is commonly associated with
systemic hypertension (65%), diabetes mellitus (50%), previous stroke (20%)
and cardiac disease (50%), the latter of which is the most common cause of a
40% 5-yr mortality in these patients (2).

Total cholesterol should be less than 200, HDL should be 40 or


higher, LDL should be less than 100 and triglycerides should be
less than 150.

Summary of other answer choices


Recall that diabetic retinopathy (DR) can be divided into Nonproliferative
Diabetic Retinopathy (NPDR) and Proliferative Diabetic Retinopathy (PDR).
NPDR is also called background diabetic retinopathy (BDR). Proliferative dis
ease indicates the presence of neovascularization; it is associated with a worse
prognosis. Neovascularization is always associated with fibrous tissue, which
is what contributes to occlusion of the angle and secondary angle-closure in
proliferative diabetic disease.

5.8. CASE 28

179

Types of Diabetes and Risk for DR (18) (21):


Type 1: (juvenile onset)
Type 1 is believed to be autoimmune in nature (pancreatic destruction);
weak genetic component, strong concern for diabetic ketoacidosis.
At diagnosis, no BDR is expected. After 5 years: 25% have BDR, PDR
rare. After 20 years: 98% have BDR, 60% have PDR, 30% have CSME.

Type 2: (adult onset)


Type 2 results from insulin-resistant receptor cells or abnormal B
cell production of insulin; strong genetic component, high association
with obesity.

NIDDM Type 2: at diagnosis, 20% have BDR. After 5 years: 30% have
BDR, 2% have PDR. After 20 years: 50% have BDR, 10% have PDR,
20% have CSME.
IDDM Type 2: at diagnosis, 30% have BDR. After 5 years: 40% have
BDR, 2% have PDR. After 20 years: 90% have BDR, 25% have PDR,
40% have CSME.

DR is the leading cause of new cases of blindness in the United


States for adults ages 20-74 (4). The duration of insulin-dependent
diabetes is the main risk factor for presence of diabetic retinopa
thy (21).

Clinical summary of DR
Vision threatening complications of DR include neovascularization compli
cations (vitreous hemorrhage, neovascular glaucoma, tractional retinal detach
ment) and macular disease (ischemia, edema). The Diabetic Retinopathy
Study (DRS) examined when to treat neovascularization.
The DRS Defined High Risk Characteristics (HRC's) as ANY of the
following (21) (18):
- Neovascularization of the Disc (NVD) greater than 1/4 Disc Diam
eter (DD).
- Any NVD or Neovascularization elsewhere (NVE) with a vitreous
or preretinal hemorrhage.
The DRS demonstrated that panretinal photocoagulation (PRP) reduced
the risk of severe vision loss by 50 - 60% in patients with HRC's.

CHAPTER 5. VITREOUS/RETINA

180

The two major concerns with DR are Illacular disease and prolif
erative disease (neovascularization). Macular edema is the most
common reason for legal blindness in DR. 5% of patients with DR
develop proliferative disease (21).

The Early Treatment Diabetic Retinopathy Study (ETDRS) determined when


to treat Illacular disease and when PRP should be utilized for treatment in
patients with NPDR (whether to treat at moderate NPDR, severe NPDR,
etc). The ETDRS defined Clinically Significant Macular Edema (CSME) and
Severe NPDR ("4-2-1 rule"):

CSME
a. Retinal thickening within 500 urn (1/3 DD) of the foveal center OR
b. Hard exudate within 500 urn of the foveal center, with adjacent thickening
OR
c. Retinal thickening of at least 1 DD, within 1 DD of the foveal center

Severe NPDR ("4-2-1 rule")


a. 4 quadrants of hemorrhages/MA's OR
b. 2 quadrants of venous beading OR
c. 1 quadrant of IRMA
The ETDRS demonstrated that focal argon laser treatment of patients with
CSME reduced the risk of moderate vision loss by 50% or more (18); thus,
patients with CSME should be treated. The ETDRS found that patients with
mild to moderate NPDR should not be treated with PRP. Patients with severe
NPDR and early PDR received minimal benefit from treatment.

Severe NPDR (pre-proliferative DR) patients have a 10 - 50% risk


of progression to proliferative disease in 12 months (21).

During vascular occlusions, ischemia can lead to the release of vascular en


dothelial growth factor (VEGF) from the retina and neovascularization and
fibrous tissue can develop in the anterior segment; if left untreated, neovascu
lar glaucoma can result. Although CRAO's result in massive ischemia to the
posterior segment, the inner retina is dead and releases much less VEGF as

5.8. CASE28

lSI

compared to CRVO's, a condition where a sick retina still releases VEGF. lS%
of all CRVO eyes will develop iris neovascularization within the first 4-6 weeks
after obstruction (2). 60% of ischemic CRVO's develop iris neovascularization;
up to 33% of ischemic CRVO patients develop neovascular glaucoma (IS). Only
1- 2% of BRVO patients develop iris neovascularization (2).
Question 2: correct answer a - carotid artery obstruction
Blood flow to the eye is unaffected until carotid or ophthalmic artery blockage
exceeds 70%; ocular ischemic syndrome does not usually occur until the ob
struction reaches 90%; at this level, central retinal artery perfusion decreases
by 50% (6).
Question 3: correct answer d - panretinal laser photocoagulation of
posterior segment
Ocular ischemic syndrome treatment includes the following (6):
PRP to retina if anterior or posterior segment neovascularization is present
Evaluation with carotid doppler; if carotid obstruction is present, a carotid
endarterectomy should be considered.
Glaucoma treatment, depending on lOP and condition of optic nerve
head.
Question 4: correct answer d - 90% of patients will have count fingers
or worse acuity at 1 year
The prognosis for OIS is poor. As mentioned above, the 5-year mortality rate
in these patients is 40% (2). Overall, only 25% of patients will have better
than 20/50 acuity at 1 year (6). If rubeosis is present, 90% of patients will
have count fingers or worse acuity at 1 year (6).
Question 5: correct answer d - 2-4 months
50% of eyes with ischemic CRVO's develop rubeosis; this is most likely to occur
between 2 and 4 months after onset of the condition (1).

References
[1] Kanski, Jack. Clinical Ophthalmology 4th ed. Woburn: Butterworth and
Heinmann, 1999.
[2] Rapuano, Christopher J. Heng, Wee-Jin. Color Atlas and Synopsis of Clin
ical Ophthalmology. Wills Eye Hospital. Singapore: McGraw-Hill, 2003.

182

CHAPTER 5. VITREOUS/RETINA

[3] Bartlett, Jimmy D., Jaanus, Siret D. Clinical Ocular Pharmacology.


Boston: Butterworth, 2008.
[4] Friedbert, Mark A. Rapuano, Christopher J. The Wills Eye Manual, 3rd
edition. Philadelphia: Lippincott Williams and Wilkins, 1999.
[5] Tamesis, Richard R. Ophthalmology Board Review., 2nd Edition.
McGraw-Hill, 2006.
[6] Friedman, ~eil J. Kaiser, Peter K. The Massachusetts Eye and Ear Infir
mary. 3rd Edition. Elsevier, 2009.
[7] Pane,Anthony. The
sevier, 2007.

~euro--ophthalmology

Survival Cuide. 1st Edition. El

[8] Vitreous Hemorrhage: Diagnosis and Treatment. Berdahl, John P,


Mruthyunjaya, Prithvi. MD1 Ophthalmology 1991;98(5 Suppl):741-756.
[9] http://www.emedicine.com/oph/topic85.htm.ParagAMajmudar.MD.
Allergic Conjunctivitis. June 30th, 2008.
[10] Friedbert, Mark A. Rapuano, Christopher J. The Wills Eye Manual, 3rd
edition. Philadelphia: Lippincott Williams and Wilkins, 1999.
[11] Hyman LG, Lilienfeld AM, Ferris FL III, Fine SL. Senile macular degen
eration: a case control study. Am J Epidemiol 1983; 118:213
[12] Gass JD:r..L Drusen and disciform macular detachment and degeneration.
Arch Ophthalmol 1973; 90:206-17.
[13] Kahn HA, Moorehead HB. Statistics on blindness in the model reporting
areas, 1969-1970. DHEW publication no. (NIH) 73-427. Washington DC:
U.S. Government Printing Office, 1973.
[14] http://www.neLnih.gov/amd/summary.asp
[15] http://canadianmedicaljournal.ca/cgij content / fullj 170/4/463
[16] Leibowitz lIM, Krueger DE, Maunder LR, et al. The Framingham Eye
Study Monograph: an ophthalmological and epidemiological study of
cataract, glaucoma, diabetic retinopathy, macular degeneration, and vi
sual acuity in a general population of 2631 adults, 1973-1975. Surv Oph
thalmol1980; 24(suppl):335-61O.
[17J http://cms.revoptom.com/handbook/SECT41b.HTM
[18] Friedman,~. Kaiser, P. Trattler, W. Review of Ophthalmology. Philadel
phia: Elsevier, 2005.
[19] Blumenkranz MS, Russell SR, Robey MG, et al. Risk factors in agerelated
maculopathy complicated by choroidal neovascularization. Ophthalmology
1986; 93:552-8.

5.8. CASE 28

183

[20] Cheatham, K. Cheatham, M. Wood, K. KMK Part One Basic Science


Review Guide. 4th edition. 2009.
[21] Pavan-Langston, Deborah. Manual of Ocular Diagnosis and Therapy, 6th
ed. Philadelphia: Lippincott Williams and Wilkins, 2008.
[22] https://web.emmes.com/study/score/
[23] Priglinger SG, Wolf AH, Kreutzer TC, et al. Intravitreal bevacizumab
injections for treatment of central retinal vein occlusion; six-month results
of a prospective trial. Retina. 2007;27;1004-1012.
[24] Byrnes, Michael J., Combination Therapy for the Treatment of Retinal
Vein Occlusion. Retina Today, March 2009, page 53.
[25] Kooragayala, Lakshmana M. Central Retinal Vein
http://emedicine.medscape.com/article/1223746-overview.

Occlusion.

[26] Tierney, Lawrence M., McPhee, Stephen, and Maxine A. Papadakis Eds.
Current Medical Diagnosis and Treatment, 45th ed. New York, McGraw
Hill, 2006.
[27] Ocular Ischemic Syndrome. Survey of Ophthalmology, Volume 55, Issue 1,
January-February 2010, Pages 2-34.
[28] Alexander, Larry. Primary Care of the Posterior Segment, 3rd ed.

Chapter 6

Optic Nerve / Neuro-ophthalmic


Pathways

Sarah Dougherty Wood, D.D., MS, F.A.A.D.

185

6.1. CASE

189
would have which

cocaine

if the

a.

due to a

b. no

constriction
d.

dilatioll

cause is due

lesion

5: n-eatment for this


toms could be:

case to help alleviate symp

a. aneurysm

b.
discontin uation

d.

removal of lesion

e.

near add

g.

of a test is defined
of a test

b.

that the disease

result

that the

of

is

present

The

present

d. The

absent

the test is

the test

This condition
found ill young
accommodative
blur.

\Vhy the other answers are incorrect:


In this case, anisocoria was found which \Vas more

which eliminates
Robertson and Horner's

marked in dim

full and

A crania! nerve 3
no

present. C'J 3

lJveitis not
uveitis, the

with
cells
the anterior

tends to be smalL

6,2. CASE 30

191

for Horner's:
lesion: no dilation with co
'" Horner's from a
no dilation wilh HA

lesion: no dilation with co

a near add. This


eye because accommoda tion
will
for clearer

is affected, A

vision.
6: correct answer- b.

The
is present

of

test

of a

resulL

that the

result

that the

disease is
Positive Predictive value: The
the
Predictive value: The

6.2

disease

of a disease

absent

Case

year old African American


Chief complaint: vision seems to dim
of present illness
Location:

eyes
moderate

Nature of onset:
Duration: a

seconds

when

secretary

19:3

CASE 30

2: What is the MOST


etiology of the optic nerve
appearance
the
test results?
;'vIRI with contrast:

no intracranial

or

BP
pressure of 260mm \yater with

lumbar puncture:
sit ion

compo

blind spots on visual field


blood count with normal

normal

a.

from

b.

disc drusen

times

neuritis

intracranial

d. Arteritic AIO:\
e.

due to

due to obstructed venous outflow

3: \Vhich of the
a.

is true

are made of

disc drusen?

bodies

b.

with 13 scan and will appear


d. Disc drusen are

nnilateral

e. all of the
f.

and b

is true about spontaneous venous

SVP

All
b.

the SVP is absent. the

If the
d. A normal

has

is present, the

person can have

e. none of the above are

110

SVP

6.2. CASE 30

195

Question 2: correct answer c - idiopathic intracranial hypertension


(aka pseudotumor cerebri) Idiopathic intracranial hypertension (IIH) causes
papilledema and the
pa tient is the
woman of childbearing
age. This condition can also present with:
transient
visual obscurations (loss of vision for a few seconds at a
blind spots on visual field testing.

Characteristics of this patient which ere consistent with IIH: age, gender, tran

scient visual obscurations, obese, and use of birth control pills.

Ill-{ is a diagnosis of exclusion, meaning the following normal findings must be

present:

jvIRI re\'eals no space occupying lesion


The cerebral spinal fluid has a normal composition
l3lood pressure not extrelllely elevated
Normal CBC with no clotting problems
In addition, a lUlnbar puncture must have an elevated opening pressure greater
than 200 mm water (or above 250 rnm water for the obese) (6).

Potential contributers to idiopathic intracranial hypertension:


CANT mnemonic:
Contraceptives
Vitamin A
Naladixic acid (quinolone antibiotic)
Tetracycline

Why the wrong answers are wrong:


Graves disease would show enlarged EOj\IS on i\IRI. Arteritic AION occurs in
those over age 5~) and visioll would be poor. Papillitis from optic neuritis would
be unilateral. Disc drusen call cause pseudopapilledema or true papilledema but
the patient would not have an elevated opening pressure on lumbar puncture.
Question 3: correct answer f
Choices a and b- they are made of hya
line bodies and can be hereditary. Disc drusen are typically bilateral and are
hyperreflective on B scarL
Question 4: correct answer d A norlllal healthy person can have no SVP.
Approximately 20% of normal,
people ha\'e no SVP (8). The lack of a
SVP ill suspected papilledema is not helpful because of the previous statement
but the presence of it is helpful in ruling out papilledema when the nerves are
difficul t to assess.

6.3. CASE 31

197

Accompanying signs / symptoms: pa inful to move her eyes


Patient ocular history: unrem arkable
Family ocular history
mother: diabetic retinopathy
father: red/greell color defi ciency
Patient medical history: anemia
Medications taken by pati e nt: Sing ular pm , iron supplements
Patient allergy history: ;\lKDA, seasonal a lle rgies
Review of systems
Constitutional / general health: fatig ue a nd malai se
Neurologic: t ingling of limbs
Clinical findings
BVA:
OD
OS

Distance
20/20
20/1 00

Pupils: reac tive OU, 2+ AP D OS


EOMs: full bu t pa inful , no diplopia reported
Slit lamp: All within no rmal limits OU
Visual field: OD fuU , OS diffuse centra l loss
Fundus OD
C / D: 0.2, sharp borders, healt hy rim colora tion
vessels: no \'ascul opathy
posterior pole and periphery: wnl
Fundus OD
C / D: 0.2, sh arp borders, healthy rim colora tion
vessels: no vasculopathy
posterior pole and periphery: wnl

63.

CASE 31

199

d. orbital
removal of tumor

e.

f. radio-iodine

5: What is the

visual

pT'o~;nOSlS

in this

>,O(C"COH.C

very poor

b.

reduced contrast

c. 111ay

d.

normal vision within 2-3 months

a and b

e.

answers b, c, and d

answers

11,

b,

Answers:
correct answer a - rvIRr
neuritis:

presents

,. young female with sudden onset, unilateral


to
severe

,.

on eye movement

in

a classic

loss which can be mild

of

,. APD
,. decreased

on

lJ thoff's

.. vision worse when


.. visual field

central scotoma

An JVfRI is indicated to rule out a


nerve lesion.
neuritis and an .f\IRI would also

these conditions may need


be

be ruled out

6.4. CASE 32

201

thyroid dysfunction- Graves disease


cardiac conduction abnormalities- possible in Leber's Optic neuropathy
Question 4: correct answer b - IV steroids and interferon-beta
ies have shown both are potentially helpful.

Stud

First, regarding steroid use: The ONTT treated ON patients with observation,

oral steroids, and IV steroids. The IV steroid group had the best outcome.

Their optic neuritis resolved the fastest and the onset of IvIS symptoms was

delayed compared to the other groups until the two year point when it was

equivocal.

Regarding interferon-beta and steroids: The CHAJ'vIPS


(Controlled High

Risk Subjects A vonex J\1 ultiple Sclerosis Prevention Study) Found combining

steroids with interferon- beta in those with at least two MRI white matter

lesions
the onset of MS and a reduction in lesions at the three year

follow-up (6).

Question 5: correct answer f After the acute onset of optic neuritis, the

vision generally returns to near normal after 2-:3 months. The nerve
to

become pale after approximately 4-6 \\'eeks (11) Contrast


will likely

be poor and the patient may complain of vision which seems washed out. ON

can return i1lld affect either eye but typically in a unilateral acute presentation.

Diplopia lllay develop in these patients, specifically as an INO (internuclear

ophthalmoplegia). This is caused by a lesion of the l\ILF (medial longitudinal

fasciculus, in the brainstem) which blocks conductioll from the contralateral CN

VI nucleus to the ipsilateral CN III nucleus (6). The result is an AD-duction

deficient on the side with the lesion and an AB-duction nystagmus on the

contralateral side. The patient will notice horizontal diplopia when looking in

the direction away from the lesion. It is possible to have bilateral INO where

neither eye can AD-duct. Convergence mayor may not remain intact.

6.4

Case 32

Demographics
Age/race/ gender: 50 year old Caucasian female
Chief complaint: diplopia
History of present illness
Character/signs/symptoms: worse when looking up

CASE 32

203

of this

1: What is the MOST


Brown's

b. Graves disease
c. EO.i\f entrapment

d. Duane's Retraction

intraorbital mass

f. ocular

2: \Vhat is the term for lid

on downward

a.
b. Gunn's

von Graefe's

d. Uthoff"s sign
e.

f. Kocher's

for this

1Ja."'<:'HC

a.

b.

no treatment
d.

diurelic,

the

e.

4: What type of medication is


and what is it used to treat?
calcium

c. ACE

d.

blocker, HTN,

UQ>'a,-,,",rn

or cardiac

HTN
channel

Answers:

Question 1: correct answer b - Grave's disease

ClLUll> ,

Isoptin)

20.5

CASE 32
Duane's Retraction
duction
2) deficients or
retract and the
will narrow
no effect
vertical eye movements as found
An in(raorbilal
not be bilateraL

cause

There is
but vvould

restriction and

2: correct answer c - von Graefe's


The

of the other
with Graves

close the eyes

Gunn's sign: retinal vascular

be found in

Uthoff's

temperature rises, found in idS


with forced lid

Kocher's

deviated

each

and

stare appearance found in Grave's disease


ocular lubricants are

lid

to maintain a heal!
closure. Prism
allevia te

radio-iodine
occurs. orbital
intravenous or

steroids arc used. This

nerve involvement at
Recall
severe:

the mnemonic which lists

N:

0:

of lid retraction. lid

of

time.
from mild

6.5. CASE 33

207

OD
OS

Distance
20/20

CF

Pupils: 4+ APD OS

EOMs: full OU

Confrontation fields: OD full, OS large central scotoma

Slit lamp: wnl OU

lOPs: 17,17 111m Hg

Fundus OD

C/D: 0.7, healthy rim tissue


nlacula: hard drusen without choroidal neovascular membrane
posterior pole: mild vascular crossing changes
Fundus OS
C/D: swollen pale disc, flame-shaped hemorrhages near disc, cup
size cannot be assessed due to swelling
macula: early hard drusen without choroidal neovascular mem
brane
posterior pole: mild vascular crossing changes

Questions:
Question 1: What is the MOST likely diagnosis?
a. CRVO

b. compressive optic nerve lesion


c. non-arteritic ischemic optic neuropathy (NAlON)
d. arteritic ischemic optic neuropathy (A-AlON)
e. optic neuritis (papilli tis)
Question 2: What is the anatomical cause of the vision loss?
a. a primary inflammation of the anterior 1/3 of the optic nerve
b. the central retinal artery compresses the central retinal vein or the vein
gets kinked, blocking outflow of blood from the eye
c. a physical interruption of the axoplasmic flow along the optic nerve from
the eye towards brain which causes swelling
d. the exact cause is unknown
e. occ! usion of the posterior ciliary arteries which profuse the anterior optic
disc

65. CASE 38

209

is there would be retinal


in all four
in CRVO due
the
blood from the blocked vein at the lamina or further
for more details.
. See the retina
acute onset and

because NAIO]\; does occur in this age


unilateral loss of vision
that the

will
normal in :\"AIO:\".
between NAIO:\" and A-AlO\',

Lab
This

medica
of medications may be associated with
NAJO:\" but the evidence
Other

facts to know about .\'AlON:


field

is inferior

but other defects can

with the
nerve
have a
cholesterol.
association with

vftscular disease such


apnea
be

and

initial onset
Optic neuritis
neuritis, when it occurs in the anterior
the
will present with swollen nerve
and the
will Dote
sudden vision loss, The
will be younger, will have
on eye movement (90% of the
and no
cell arteritis symptoms
should be present.
2: correct answer e

due to Giant Cell A.rteritis

vasculitis of medium

cells occlude the

\-\That the other

describe:

artery and prevent

6.6. CASE 34

211

Review of systems
Constitutional/general health: difficulty swallowing
Pulmonary: shortness of breath
Genitourinary: fre quent urination (polyuria)
Clinical findings

BVA:
aD
as

Distance
20 /25
20/2.5

Pupils: PERRLA, no apd


EOMs: restricted upgaze OS, diplopia worse on elevat ion a nd AB-ducti on
Cover test
distance: 46. right hypertropia in primary gaze, 10':" rig ht hyper
tro pi a on upgaze, no horizontal component
Confrontation fields: full to fi nger cou nt OD , OS
Slit lamp
lids/ lashes / a dnexa: O D mild p tosi s
rap: wnl OU
1 minute susta ined upgaze t esting
Palpebral fissur e size:
pre-test: 8 mm OD, 10 mm OS
post-test: 6 mm OD , 9 mm OS

Questions:
Question 1: Wha t is the MOST likely diagnosis?
a. ocular myasthenia
b. CN 4 pal sy
c.

eN 3 pa lsy

d. CN 6 palsy
e. Horner's syndrome
f. Grave's disease
g. I NO

h. CPEO

6.6. CASE 34

21 3

Question 6: Wha t is trea tment for the diagnosis in question 1?


a. no treatment availab le
b. surgical removal of t he t hy roid , radio-iodine therapy, or anti-thyroid med
ications
c. pyridos tigmine (mestinon) and/or immunos uppressa nts
d. a neurys m cl ip
e. orbi ta l decompressio n
f. vision t hera py

g . oc ul a r lubr ica nts


h. increase t he dosage of t he proprano lol
1.

d/ c the lomax

Answers:
Question 1: correct answer a - o cular m y asthenia Eye involvement of
myast henla gravis, JvIG, is termed ocu lar myast henia . J"IG is a n a uto im mune
d isease which occurs in 4-5 per 100,000 (4). It can affect a ny age or race a nd
d iag nosis tends to occur in wo men when t bey a re yo ung a nd in men when t hey
a re elderl y (4). This patients fits t his de mogra phic. In l'vIG , the acety lcholine
(a neurotransmi tt er ) recepto rs at t he neurom uscul ar junction are da maged o r
blocked by a ntibodies. The resul t is wea k skeletal muscle contract ion a nd
fat igue. Sy mpto ms t end to be variable alld wo rse towards the end of the day
or with exertion of t he muscle.
Ocular signs/symptoms of l\IG incl ude:
d iplopia due to EO)vI involve ment
p tosis due to levator superi ori s involvement
This patient had both of these sig ns/sy mpto ms. In t his patient, t he ptosis
got worse wit. h the 1 minute sustained upgaze test. This is a simple cli nica l
test which can be done t.o ca use sufficient fat igue of t he levator to decrease t. he
pa lpebral fissu re size in pat.ients w it h MG. Anot her possible ocula r findin g in
MG: Or bic ul a ris oculi weakness.
Syste rru c sig ns can include: dysart hria (diffi cul ty talking), dysphagia (difficul ty
eat ing), di ffic ul ty holding t he head upright , a nd in t he wo rst cases , respirato ry
fa ilure, a lso know n as myasthenic crisis. in t hi s case, t he patient had diffi culty
wit h swallow ing a nd shor t ness of breat h which may be related to MG and he
sho uld be referred to a neurologist urgent ly.
T his patient was on a beta-blocker (propra nolol) which can act ua lly make MG
worse.

6.6. CASE 34

215

example of a non-comitant deviat.ion: A eN 6 palsy. It will be worse in AB

d uction on the affected side because this is the action of the lateral rect llS

muscle.

If ocular misalignment is the same, no matter the direction of gaze, this is

termed comitant. This occurs in decompensated phorias.

Definition of the wrong choices:

A secondary deviation occurs in non-comitant strabismus and refers to the


amount of ocular misalignment when the paretic eye is fixating. A pri
mary deviation refers to the misalignment vvhen the non-paretic eye is
fixating.
A conjugate rnovernent refers to the movernent of two eyes
same direction.

in the

Sherrington's law states there are paired muscles for an eye, where one
innervated and Olle gets inhibited for an action t.o occur. Example: the
lateral rectus
innervated and the medial rectus of the same eye gets
inhibited for AB-cluction to occur.
Question 3: correct answer c The head tilt should be away from the
affected eye. Remember this: the primary action of the superior oblique is ill
the eye call 110
incyclotort, so with a head
cyclotorsion. 'Vith a SO
tilt away from the affected side, the patient tries to simulate this incyclotorsion
to minimize the diplopia.
Forced d uctions separates a restriction from a palsy. A rest.riction would not
allow the eye to move (like a tumor or entrapment) where a
does move
because there is nothing physically blocking it.
Question 4: correct answer g - Tensilon test, chest CT, TSH, T3,
T4 The Tensilon test helps confirm a diagnosis of i\IG by injecting an
short-acting acetylcholinesterase inhibitor (eclrophonium chloride) into an arm
or hand vein. In those with i\IG, the ptosis or the diplopia should decrease
about 3-4 minutes after the injection (6). Atropine should be available to
use as needed to counter potential cholinergic side effects of the test such as
bradycardia, bronchospasm, or angina (6).

IV ATROPINE is the antidote for potentially dangerous side effects


caused
the Tensiloll test.

217

6.7. CASE 35
mother: diabetes
father: htn
Review of systems
Neurologic: headaches
Mental status
Orientation: wnl
Clini cal findings

BVA:
OD
OS

Dis tance
20 / 20
20/20

Pupils: PERRLA, no apd OU


EOMs: full OU
C onfrontation fields: grossly restricted temporally OU
Slit lamp: all wnl
lOP: 15/15 mmHg
Fundus OU
C / D: 0.2 with bow-tie atrophy

Questions:

Question 1: What is the MOST likely dia gnosis?

3.

pa rietal lobe lesion

b. temporal lobe lesion


c. pi t ui tary adenoma
d. glaucoma
e. toxic/ nu tritional optic new-opathy
f. occipi ta.! lobe lesion

g . tilted disc syndrome

7. CASE 35
severe

h. arteritic
i. Foster

Answers:

which can be; in


feriol',
in
volvernent is
beca.use
compression of these nasal fibers,
in
visual field defects in both
which respect the vertical midline.

adenomas can

The other most common causes of chiasmal


of the
carotid artery,
\Vith chiasma I lesions mayor may not have normal
will
be in horizontal band

may have
Patients may

had her car accident


may
aware of visual field defects.

The visual field defects

from the other conditions listed:

lobe lesion: pie on the


lobe lesion:

in the

If

superior

6.7. CASE 35

221

Question 5: corre ct answer e - junctional scotoma A junctional sco


toma is a chi asmal lesion which has enl arged a nd is not only compressing on
the nasal fibers of both optic nerves but has extended to in clu de a la rge por
tion of one optic ner ve , including the cent ral fibers. This eye wo uld , t herefore,
have a central defect with very poor vision and the other eye (where just t he
nasal fi bers ar e affec ted) would have a temporal defect . The temporal defect is
often denser superiorly beca use the anterior knee of VVild ebrand , which carries
infer ior nasal fiber s, becom es involved due to its close proximity to the lesion.
Why the other answers are incorrect:
Answers a, b ,c a nd i are post-chiasmal an d would be homonymous defects whi ch
"vould not cross the vertical midline. A retinal lesion wou ld only involve o ne
eye. Severe gla ucoma cou ld include ce ntra l vision b ut gla ucoma te nds to give
nasal defects a nd not temporal defects until very late in the disease. AION
has the cl assic visual fi eld with a n inferior alti tudi nal defect. Foster Ke nnedy
syndrome is a res ult of a fr onta l lobe tumor where on e opt ic nerve will appear
swollen a nd one will appear atrophic. The vis ual fi eld will be initially normal
on the swoll en eye a nd involve centra l vision in the at rophic eye.

Visual Field Definitions


c o ngruous : This refers to the simil arity between the visua l field defect in
each eye: t he more posterior , t he more congruous , t herefore, occipi tal
lobe lesions are t he most congruous. This term can only be used if the
defect is incom plete.
complete d e fect: occ upi es the entire half of the field, the location of the
lesion cannot be det ermined fro m the visual fi eld , congruity does no t
apply when t he defec t is com pl ete
incomplete d e fect: occupies less t han the entire half of the field
homonymous : located on t he same side of both eyes (example: rig ht sid e of
both eyes: nasal of O S a nd t empora l of OD )
hemianops ia: a homony mous defect whi ch affects half of the visual fi eld, ei
ther right or left , in each eye
quadra ntopsia : a homo ny mous defect which a ffects only on e quadrant in each
eye
macula-only: A homony mous hemianopi a where ONLY the central 5 degrees
is involved in each eye on t he same side
macula sparing: A homonymous hemianopia where a t leas t 5 degrees in the
m acula area is spared in both eyes (6)

6.7. CASE 35

223

References

[1 ]

Li J, Tripathi R , Tripathi B. Drug-Induced Ocular Disorders. Drug Safety


2008:31 (2): 127- 141.

[2] Rhee D, Pyfer lvI. The YVills Eye l\Ianual , Office a nd Emergency Room
Diagnosis and Treatment of Eye Disease, 3rd edition. Lippincott, Williams,
and Wilkins, 1999.
[3] Kumar V, Cotran R, Robbins S. Basic Pathology, 6th edition. W.B. Saun
ders Company, 1997.
[4] Miller N, et a!. 'Na Is h and Hoyt's Clinical Neuro-O p hth almology: The
Essentials, second ed it ion . Lippincott , \Villiams, a nd Wilkins, 1999.
[5] Cabergoline as a First-Line Treatment in Newly Diagnosed l\ Iacroprolacti
noma s, Pontikides N., Krassas G. , Nikopou lou E. a nd Ka ltsas T , Pituita]")"
Volume 2, i\' umber 4, Ma.J;, 2000.
[6] Kline L. , Bajandas F. Ne llio-Ophthalmology. Rev iew Manual, 5th ed ition.
Slack, 2004.
[7] Atl as of Primary Eyecare Procedures, Casser , Fingeret, Woodcome, 2nd
editio n, 1997, Appleton and Lange,
[8] Legler U, Jonas J. Assessment of the spoutaneo us pulsations of the central
retinal vein in daily ophthalm.ic practi ce, Clin Experiment Ophthalmol.
2007 Dec;35(9) :870-1.
[9] www. \iVebmd.col1l

Glaucoma

Sarah

vVood, O.D.,

F.A.A.O.

226

CHAP TER 7. GLAUC0 1\ IA

227

7.1. CASE 36

7.1

Case 36

Demographics
Age/race/gender: 35 year o ld male
Chief complaint: blurred vision after running
History of present illness
Location: boLh eyes

Severity: mild

Duration: 30 minutes

Frequency: only with exercise

S econdary complaints /s ymptoms: inquiring about LASII{

Patient ocular history: myopic, cont.act lens overwear

Patient medical history: seasonal a ll ergies, GERD , insomnia

Medications taken by patient: finaster ide, montelukast, cimetidine , eszopi

clone
Patient allergy history: ASA, sulfa drugs
Family ocular his tory:
fa ther: retinal det.achm ent w hich left him blind in one eye
R e view of systems
Gastrointestinal: const ipat ion
Clinical findings
BVA:
Distance
OD
OS

20/20
20/20

Pupils: PERRLA, no apd O U


EOMs: full O U
Confrontation fields: full to finger count

Slit lamp
lids/las hes/adnexa: wnl OU

228

CHAPTER 7, GLAUCOJ\IA
conjunctiva: wn l OU
cornea: linear endothelial pigment deposition OU
anterior chamb er: deep a nd quiet OU
iris: iris tra nsilluminat ion defects ou in the mid- peripheral iris, brown
irides OU

lens: trace pig ment on a nterior lens OU

vitreous: cl OU

Fundus OD

Cj D: 0,1
macula: flat
posterior pole: wnl
periphery: lattice degenera tion inferior ly with two at rophic holes,
no detac hments or t ears

Fundus OS

C j D: 0,1
macula: fiat

posterior pole: ,,vnl

periphery: no holes, tears or detachments

Questions:
Question 1: What is the most likely diagnosis causing the chi ef com
plaint for this patient ?
a, mu lt iple sclerosis
b, pigm entary dispersion sy ndrome
c, pseud oexfo liation syn drome
d , Fuch's heterochromi c iridocyclitis
e, Posner Schlossman syndrome
f. Primary open-a ngle glaucoma

Question 2: What test needs to b e done next to confirm the diagno


sis?
a , visual field
b, gonioscopy
c, pachym etry
d, l'vIRI
e,

GDX

1. CASE 36

229

what structure is
3: On
to the trabecular meshwork?

anterior

b. Schwalbe's line

Sehlemrn's canal

d.
e. scleral spur

f.

line

4: What is

mechanism of action to decrease

lOP?
a. increase U\'eoscleral outflolV
b. decreased
c. increased T'vI out.flow

the endothelial pump cells

d, opens up the spaces in t.he Tl\l to

outflow

e.

were to be treated with a


would be contraindicated:
b.
c. Diamox

d.
e. Xalatan
f. choices a and e

choices b,

and d

h. none of the

Answers:

enclothe
.. transillumination iris defects
seen best with

..

of the iris

iris defects

230

CHAPTER 7. GL AUCOMA

den sely pig mented t ra becula r meshwork


ty pica.lly bil ater al
The periphera.l bowing of the iris puts t he iri s in a pposit io n to the lens zonules,
causing rubbing a nd release o f pigment into the a nterior chamber where it is
subsequently deposited in the a ngle a nd o n the endothelium. Pigment release
can be increased with jarring exercise such as running or with dilation. The
pig ment rush t.emporarily blocks the outfl ow in t he trabec ul a r meshwork, caus
ing in creased intraocular pressure which ca n cause short-ter m blurred vision

due to corneal edema.

The cl assi c demographics of pigmentary di spersion syndrome pat ient:

young (20-45 years of age)


myopi c
male (5)
Differential diagnoses and why they do not apply to this case
Multiple sclerosis can cause blurred vision after exercise (or in general
after increase in body temperature), t ermed Ut hoft"s sig n. This patient
has no ot.her findings co nsistent with MS.
Pseudo exfoliation syndrome (often called exfoliation sy ndrome) can
be associated with pig ment release but this condi t ion is found in t he
eld erly. In addition, no pse udoexfoli at ion de posits we re noted on the
pupillary ma rgin or on th e anterior capsu le .
Puch's heterochromic iridocyclitis is ch aracterized by a non-granulomato us
keratic precip itates a nd a low grade, asymptomatic , chronic uveitis which
increa ses t he ri sk of gla ucoma a nd ca taracts . The iris in t he a fT'ected eye
is typically lighte r in color due to iris at rophy from long-term infla m
mation . This case had a quiet a nterior cha mber a nd no he tero chromia.
Posner Schlossman syndrome (aka glaucomatoCJrcl it is cri sis) is char
acterized by unila teral , recu rrent e pi sodes of increased intraocular pres
sure (about 40-60mmHg) due to tra bec uli t is with an open a ngl e on go
nioscopy. The anterior cha mber will have very few ce lls. The pat ient may
have symptoms of blurred vision. The typical pat ient is yo ung to middle
aged. This case ha d oc ular hyp ertension but not as high as 40mmHg,
he repo rt ed the intermi tte nt blur was bilateral , the anterior chamber was
quiet.
POAG (primary open-angl e gla ucoma) : Gonioscopy has no t been done
so it is not known if the angle is open. Also, t he optic nerve is clearly not
gla ucomato us with a c/d ratio of 0.1.

1.

86

Question 2: correct answer


extllnina tion of
ular meshwork. In these
is often seen.
these
are
has been

2.31
will allow for
Schwalbe's
gravity. If
cOllfirma tion of pigment dispersion

range, hut. the

not hBve

Explanation of Incorrect Answers


A visual field is not
or

There is no indication for ?vIRI for t.his


GDX evaluation of the retinal
establish a baseline for
with the
of PDS.
Patients with PDS need to be foilowed to monitor
tary
This condition
ages.

to

of

Question 3: correct answer c - Schlemrn's canal The order of the struc


tures from
to anterior:
Scleral
J Trabecular
Schlelllm's canal. Schwalbe '3 line

4: correct answer d - opens up the spaces in the Tl\1 to


increase outflow
is seldom used
clinical
but

CHAPTER 7.

232

GLAUCOMA

fair game for boards quest ions. Pilo carpine is a cholinergic agonist which wo rks
by pulling on the scleral spur which causes the spaces of t he T }"I to open up .
This a llows for in creased TM outflow. Miosis is a side effect of t he drop but it
is not what causes the decrease in rop.
Mechanisms of action for glaucoma drugs
Prostaglandin a nalogs (Xa latan, Lumigan , Trava tan): increased uveoscle
ra l outflow
Beta-blockers (Timolol , Betagan , Ca rteolol, Betaxolol, Optipranolol, Be
tagan) : decrea sed aqueous prod uc tio n
Adrenergic ago nists (Apraclonidine , Brimonidine) : decreased a queo us
humor production a nd increased u veoscleral outRow
C holinergic agonist (P ilocarpine) : in creased T Il! out Row
Carbonic Anhyd rase inhibitor (Trusopt, Brinzolam ide, Diamox , lVle tha
zolamid e): inhibits carbonic a nhydrase which is a n enzyme in volved in
aq ueous prod uction
Question 5: correct answer g - choices b, c, and d This pat ient has a
sulfa all ergy a nd t rusopt and di amox are carbonic anhyd rase inhibitors which
should not be used with t his allergy.
This patient should a lso not be treated wi t h pilocarpine because of t he lat tice
wi t. h atrophic holes in t he rig ht eye. It is thought miot ic t herapy is linked to
retinal detachme nt. The proposed mecha nism is a nterior displacemen t of the
lens-iris diap hrag m , leading to vitreoretinal traction. This patient would also
not be a n id ea l candid ate for pilocarpine due to the side effe ct of accommod at ive
spasm whi ch is un pleasant for pre-presbyopes (2).
Important facts about PDS
PDS is treated with t he sam e medications as POAG
PDS may be harder to control a nd may have large variations in lOP
depending on t he pigment dispersion
These patients often do \vell ,vith ALT (argon laser trabeculoplasty) be
cause t heir TIl'f is very pigmented

7.2

Case 37

Demogra phics

7.2.

233

CA SE 37
Age/ race/gender: 70 year old woman fr om Finl and

Chief complaint: glare when driving at nig ht


History of present illness
Location: both eyes
Severity: moderate
Duration: several months
Frequency: accompanies dri ving
Accompanying signs / symptoms: diffi culty reading road signs
Secondary complaints/symptoms: dry eye
Patient ocular history: history of refr active a mblyopia OS
Patient medical history: smoker, osteoporosis, asthma
l\1edications taken by patient: fosamax , inhaler
Patient allergy history: simvastatin
Family medic al history:
nlother: gla ucoma

father: dry AMD

sister: glau coma

Review of systems
Constitutional /general health: difficul ty walking
Ear / nose/throat: persistent coug h
Pulmonary: difficulty brea thing
Mental status
Orientation: wnl
Mood: depression
Clinical findings
BVA:

OD
OS

Distance
20/ 40
20/70

Pupils: minimal reactio n OU, no apd OU

CHA PTER 7. GLA UCOMA

234

EOMs: full OU
Confrontation fields: full to fin ger count 00
Slit la mp
lids / lashes/ a dnexa: mild dermatochalasis OU
conjunctiva: trace injectio n OU
cornea: cl ear OU
anterior chamber: mod erately deep/quiet OU
iris: OD w hite/flaky deposit at pupil border, transillumination iris de
fects OD, OS wn l
lens: ea rl y nuclea r sclerosis OU , moderate posterior s ubcaps ular cataract
through the line of sight OU, OD with a bull's eye shaped, white,
fl aky deposit on the anter ior capsul e, phacodonesis OD
vitreous: clear OU

rops: 30, 15 mm Hg
Fundus OD
C/D: 0.7, thinnest inferiorly, no not ching

macula: fl a t, clear

p osterior pole: flat nevus in superior arcad e, 1 DD in size

periphery: no holes, tears or detach ments OU

Fundus OS
C / D: 0.4 with heal t hy, intact rim t issue

macula: flat, clear

posterior pole: wnl

periphery: no holes, tears or detachme nts

Blo o d pressure: 150/85

Questions:
Questio n 1: The rrlOst likely cause of this pati e nt's glaucoma is:
a . phacolytic
b. phac:omorphi c
c. pigmentary dispersion glaucoma
d. true exfoli ation syndrome
e. pseudoex foli ation syndrome

7.2. CASE 37

235

Question 2: This
increased risk for:

of

does NOT put the

at

closure

Cl.

b.

format.ion

d. sunflower catanict
e. poor dilation

3: The initial treatment for this


is as follows:

a.

type of

removal of the lew;

b.

open

c.

d. laser

e.
shunt

g.

h.

Question 4: Which

1S

contraindicated in this

Et.

b. trusopt
c. timolol

d.
azopt

Preservision or Ocuvite to
her

Related Eye Disease


an eye vitan1in such as
hecause of

nrl',,''''''-IT

will decrease
b.

these vitamins should be taken

c.

these
of vitamins pose a

have not been proven to prevent the

d,
her

these \'ital11ins may increase the


eye

A.i\ID

risk of

to prevent eye disease

of

whitish

in

236

CHAPTER 7, GLAUCOMA

Question 6: \Vhich of the following is INCORRECT regarding visual


field interpretation?
a, a stimulus size V is larger th an stim ulus size III
b, a fal se pos iti I'e is where the pati ent respond s to the perim eter making a
noise but t here was no stimulus, thi s is an indi cator of reliability, fal se
positi ves will make the fi eld look better than it act ua l is
c, mean dev iation (MD): t he patients overall performance is compared to
healthy age-match ed norm a ls, this is a n indi cation of a n overall field
depression but is no t s peci fic to g lau coma
d, pattern stand a rd deviation (PSD): This is a measurement of t he shape of
the hill of vision, it will indicate focal a reas of depression whi ch is typical
in glaucoma
e, a high PSD represents a smooth hill of vision
f. the gray scale is helpful wi t h patient educati on and gives t he practitioner
a first glance idea of \vhere the defects may li e bu t the gray scal e should
not be used in making diagnostic dec isions

Answers:
Que stion 1: correct answer e- pse udoexfoliation syndrome (PXF,
sometimes calle d exfoliation syndrome) This age-rela ted condition is
found with highes t p reva lence in t hose of Scandinavi an decent, PXF is charac
t erized by w hitish, fl aky deposits on the pupillary margin , the an te rior lens cap
s ule, lens zonules, a nd t rabec ul ar mes hwork. The pupil usua lly dilates poorly,
This is a systemic co ndi t ion because t he deposits a re found throughout the
body mostly in co nnec ti ve tissue of orga ns such as: lungs, heart and skin, It is
most ofte n unilate ra l but ca n be bil atera l. The fibrillar deposits accumul a ted
in the TM and can cause elevated lO P, The deposits can a lso be fo und on t he
lens zonules which ca n rub on the iris ca using pigment release, This pigment
can also contribu te to the elevation of lOP, This is not considered a form of
p igm enta ry dis persion g la uco ma,
The ri sk for elevated lOP put s those wi t h PXF at ri sk for glaucoma, The risk
of gla uco ma in PXF is 15% wit hin 10 yea rs (4) , PXF is t he most commonl,,,,.
identifiable cause of open-angle g la ucoma,
\Vhy the other answers do not apply to this patie nt
Phacolytic glauco nla: occurs wh en a hypermat ure or mature cataract leaks
lens material whi ch ca uses TM outflow obstruction and resu ltant mar kedl y
increased lOP, The an terior chamber wo uld have cells, flare, an d irides
cent particles, This pat ient has a cataract but none of these sli t lamp
findings, In addition , a cataract this mat ure would not allow for viewing
of the posterior segment,

7.2. CASE 37

237

Phacomorphic glaucoma: occurs Ivhen the lens thickens as a res ul t of catarac t


form ation whi ch pushes the iri s for ward a nd closes the anteri or cha mber
angle. This patient has a moderately dee p angle. The pati ent is at risk
fo r a ngle clos ure but fo r a different reason. PXF causes wea k lens zonul es
an d the lens ca n move forward and block t he pupil cau sing lens block.
The clinical findings show that this patient has weak zonules based on
t he phacodonesis , or a tremul ousness of the lens.
Pigment dispersion syndrome: does ha ve tra nsillum inat ion iri s defects but
this co ndition wi ll occur in a patient much younger and the white/ fl aky
deposits will not be present.
True exfoliation syndrome: occurs when the lens capsul e fl a kes off due to
exposure infrared or thermal rad ia tion, think g lass blower. This is very
rare.
Question 2: correct answe r d - sunflower cataract A sunflower cataract
is found in \ Vil so n's disease , whic h is an d isease of the li ve r. Thi s ca uses copper
depositio n on the le ns and res ultant petaloid shaped pattern on the lens. All
of t he ot her choices can occur in pseudoexfoli ation .
Question 3: correct answer b - same as POAG P seudoexfoliat ion gla u
co ma is managed the sa me as prima ry open angle gla ucoma, although it may
be ha rder to control the lOP. Initial therapy for POAG is typi cally with topical
medi cations a nd where trabec ulectomy and glaucoma s hunts are rese r ved fr om
seve re disease where topical medications are no t s ufficient. Simply remo ving
t he lens with the exfoli a ti ve deposits will not be beneficial because the deposits
a re not made by the lens . T he depos its will cont inue to be produced a nd cause
elevated eye pressure.
Glaucoma surgeries
Peripheral Iridotomy:
Indication: angle closure or hig h risk of angle clos ure
\ Vhat it does: creates a hole in t he iris which allows for aqueous
to flow freely from t he posterior cha mbe r to t he an terior chamber.
This results in th e iris no longe r being pushed forward and blocking
the pupil. This procedure equilibrates the pressure between t he two
cha mbers.
Goniotomy:
Indication : co ngenital glaucoma .
\Vhat it does: makes a n incisi on in the trabecular meshwork to allow
for better outflow.

CHAPTER 7. GLAUCOMA

238

'I\'abeculectomy
Indicat.ion: insufficient lOP con t rol with topica l d ro ps or laser
What it does: This invo lves surgical removal of a small portion
of Ti\I a nd creation of an overly ing sclerotomy site where a queous
outflow can be t.itrated with sutures. The aqu eous outflow elevates
t.he conjunct iva to create a bleb which is typically located superiorly.
The aqueous t. hen dra ins into episcleral veins and the ove rall res ul t
is a lower ing of rop.
Argon laser trabeculoplasty (ALT) or Selective laser trabeculoplasty (SLT):
Indication: insu ffi cient lOP control in open-angle g laucoma
\i\ihat is does : laser to the TM t o cause scar ring which pulls open
parts of the TJ\ I a nd /or st imul a tion of pumping outflow by TM
endoth eli a l cells (mechan ism is not clearly und erstood )
Glaucon1a shunt procedures:
Indication: severe g laucoma.
Vi/hat is does: a tu be is inserted into t he a nterior chamber where
aq ueous is shunted to an ex traocular plate under t he lid

Recall t he definition of the following s uffixes which may hel p in


keep ing t hese procedures st raight:
otom}, = incision
ectomy

oplasty

= repair

removal

Questi on 4: correc t answer c - tim.olol This patient s hou ld not. be given


a beta blocker due to her pulmonary issues . Beta blockers can cau se bron
cho constrict ion.
Question 5: correct answer c These vitam ins are cont raindicated in t his
patient beca use she is a smoker and the beta carotene in these vitamins has been
shown to increase the risk of I ung cancer. There a re formul at ions which have the
beta carotene removed which are for sm okers. In a ddition, t he AREDS st ud y
showed these vitamins do not preve nt the development of AMD . A benefi t was
found in those who a lready have AIVID in categor ies III and IV (more severe at
baselin e). For th ese gro ups, the relative risk reduction of progression of AMD
was 27% but only a 7. 6% absolute risk reduct ion over 5 years (3).

7.3.

CASE 38

239

Relative risk: (Incidence of t he ou tco me in the trea tme nt group)/ (incidence

in the control group)

Absolute risk: Accounts for absolute incidence of the disease (,,''''hat was

the original risk?"), ARR= (incidence of t he outcome in t he control gro up)

(incidence in the trea tment group)

Question 6: correct answer e - a high PSD represents a smooth hill

of vision is incorrect A high PSD r epresents a "lumpy" hill of vision wi t h

foca.l loss typical of gla.ucoma. A low PSD would be a smoot h hill of vision

fo und in hea lt hy patients.

Regarding the stimulus size, there are 5 sizes: I-V. I is the sm a llest and V is

the largest . The default is size III. A large r st imulus may be usee! if a pat ient

has very ad vanced disease and can no longer see t he size III ta rget.

7.3

Case 38

Demographics
Age / race/gender: 55 yea r old African American male
Chief complaint: eyes itch
History of present illness
Location: O U
Severity: modera te
Nature of onset: coincides with blooming of trees / flowers
Duration: 2 weeks
Frequency: constant
Exacerbations/ remiss ions: worse wh en outside
Accompanying signs/symptoms: some tea ring
Patient ocular history: compound myopic a stigmate, several bl ack eyes from
boxi ng
Family ocular history
mother: diabetic retinopat hy
father: visu ally impaired since early childhood , etiology unknown
Patient medical history: hyperte nsion, niddm, depression
Medications taken by patient: HCTZ, glipi zid e

240

CHAPTER 7. GLAUCOMA

Patient allergy history: seasonal , elavil


Family medical history:
mother: diabetic
father: osteoarthritis, high cholesterol
Review of systems: all unremarkable
Clinical findings
BVA:
Distance
OD
OS

20/20
20/20

Pupils: reacti ve OU, irregular pupil shape OS. a!liso OD<OS , +apd OS
EOMs: full OU
Confrontation fields: full to finger count OD , s up/nasal restrictio n OS
Slit lamp
lids/lashes/adnexa: wnl OU

conjunctiva: racial mel a nosis OU

cornea: clear OU

anterior chan1ber: deep and quiet OU

iris: wnl OD, sphincter tear OS

lens: clear OD. Vossius ring OS

vitreous: clea r OU

lOPs: 13, 35 mm Hg
Fundus OD

C j D: 0.2
macula: flat

posterior pole: wnl

periphery: no holes, tears, or detachments

Fundus OS

CjD: 0.75, inferior notch


macula: flat
posterior pole: wnl
periphery: no holes, tears , or detachments

7.3.

CASE 38

241

Questions:
Question 1: "\Vhat is the most likely cause of this patient's glau
coma?
fl. pi g ment

dispersion

b. uveitic gla ucoma


c. angl e recession
d. rubeotic glau coma
e. intermitt.ent a ngle clos ure

Question 2: Which of the following ocular findings would most likely


also be found in this condition?
a. iridodi a lysis
b. neovasculari zation of the angle
c. pse udoexfoli a tion deposits
d. pupil block
e. Kruke nburg spindles

f. Iris tra n sillumina ti on defect s


Question 3: Wha t is the ca use of elevate d lOP with the dia gnosis
from question I?
a . the tra bec ular meshwo rk blocked by the iris which is stuck to it and /o r
iri s b ombe
b. t he t ra becular mes hwork blocked by th e iris due to the contraction of
fi b rous ti ssue which pulls it a nd / or the physical blocking by the fibro us
tiss ue itself
c clogging of the trabec ul a r meshwork a nd damage to t he trabec ul a r mesh
work 's e ndothelial p umping cells
d. physical damage to the TM
e. t he peripheral iris moves forwa rd due to a pressure g radient (posterior
cha mber>anLerior chamb er) blocking the T i\l

Question 4: Blebitis can be a complication of a trabeculectomy


surgery. Which of the following would NOT be a sign/symptom
associated with blebitis?
a. red eye
b. a ching pain
c. decreased vision

242

CHAPTER 7. GLAUCOMA

d. anterior cell
e. hyphemia
f. vitreous cell
g. photophobia
Question 5: Which eye drop has the greatest chance of causing an

lOP spike?
a. lotemax
b. FML

c. Pred forte
d. acular
e. azopt
f. voltaren

g. alrex

Answers:
Question 1: correct answer c - angle recession Angle recession glau
coma is the most likely diagnosis. Patient findings consistent with this diag
nosis: history of blunt trauma, unilateral elevated pressure and optic nerve
damage, iris sphincter tear, and a Vossius ring.
A Vossius ring is a circular deposit of pigment on the anterior capsule due to
iris-lens contact during the trauma.
To definitively diagnose angle recession, gonioscopy must be performed. Angle
recession appears as a very deep angle with an uneven iris insertion, a recessed
iris, and a widened ciliary body. The recession can occur any number of clock
hours but the larger the extent of the damage, the more likely the eye pressure
will be elevated and increase the risk of glaucoma. The recession tends to be
unilateral, therefore, the other eye can be used as a comparison to confirm the
recession.
Explanation of incorrect answers:
Pigment dispersion can present with pigment on the lens but is not in a ring
pattern such as with a Vossius ring. In addition, pigment dispersion would
likely have Kruckenberg spindles and transillumination iris defects.
Uveitic glaucoma would have posterior synechiae and/or peripheral anterior
synechiae (PAS are not visible until gonio is done) which would cause the
elevation of the lOP. During times of inflammation, the iris thickens and gets
sticky. It can adhere to the lens and the TM. Posterior synechiae require 360
attachment of the iris to the lens (iris bombe) to cause the lOP to elevate.

7.3. CASE 38

243

Peripheral anterior synechiae elevate the lOP by blocking outflow with a tent
of iris pulled over the TM. It is possible this patient had some inflammation in
the eye as a result of his ocular trauma.

Rubeotic glaucoma cannot be the etiology because even though he is dia


betic, there are no signs of retinal ischemia and no iris neovascularization.
Intermittent angle closure would show narrowed anterior chamber depth
on slit lamp examination.
Question 2: correct answer a - iridodialysis Iridodialysis can occur
with ocular trauma. This occurs when the iris root is pulled away from its
attachment to the ciliary body. It will appear similar to a peripheral iridotomy.
These eyes can also have a traumatic cataract.
Question 3: correct answer d - physical damage to the TM The
gonioscopic appearance of angle recession is a result of a tear in the ciliary
body between the longitudinal and circular muscle fibers (5). This is an indi
cation that the TM also sustained damage which prevents adequate outflow.
According to The Massachusetts Eye and Ear Infirmary illustrated Manual of
Ophthalmology (4), if >2/3 of the angle is involved, there is a 10% chance of
developing glaucoma.
Other answers and their associations
answer a: cause of uveitic glaucoma: either PAS (iris to TM) or PS (360
adhesion of iris to lens) cause elevated rop
answer b: cause of rubeotic glaucoma: remember whenever there is neo
vascularization in the eye, there is associated fibrous tissue
answer c: cause of pigmentary dispersion glaucoma and pseudoexfoliation
answer e: cause of angle closure glaucoma

Question 4: correct answer e - hyphemia Blebitis is an infection of the


filtering bleb created during a trabeculectomy. Infection can occur at any time
after the surgery (days to years) (5). The treatment depends on the severity
but can range from intensive topical antibiotics to hospitalization with IV an
tibiotics. Careful monitoring is required. All of the listed signs/symptoms can
be associated with blebitis except a hyphemia. There would be no reason for
blood to accumulate in the anterior chamber. A hypopyon, excessive anterior
chamber cell which settles in the anterior chamber, may be present.
Question 5: correct answer c - Pred Forte Pred Forte (predmsolone
acetate 1%) is the most potent topical steroid with the greatest risk of steroid

CHAPTER 7. GLAUCOMA

244

response. Lotemax (loteprednol, 0.5%), FML (fluoromethalone), and Alrex


(loteprednol 0.2%) are all considered "soft steroids" with less chance of lOP
spike. Acular and Voltaren are NSAIDS (non-steroidal anti-inflammatories)
with no chance of IOP spike. Azopt (brinzolamide 1%) is a CAl which actually
is used to lower lOP.

7.4

Case 39

Demographics
Age/race/gender: 66 year old African American female
Chief complaint: annual exam
Patient ocular history: viral conjunctivitis
Family ocular history
mother: POAG
father: cataract
Patient medical history: unremarkable
Medications taken by patient: none
Patient allergy history: PCN
Clinical findings
BVA:
OD
OS

Distance
20/20
20/20

Pupils: PERRLA, no apd


EOMs: full

au

Confrontation fields: full to finger count


Slit lamp: wnl

au

au

lOPs: 26, 26 mm Hg
Fundus OU
C /D: 0.3, pink, healthy rim tissue
other findings: wnl au
Humphrey visual field: full and reliable OU

7.4. CASE 39

245

HRT II: all sectors in the Moorefield Analysis have green check-marks,
standard deviation of reliability is 17 11m
Gonioscopy (4 Inirror): open to ciliary body inferior and superior au,
open to scleral spur nasal and temporal, +iris processes au
PachYInetry: 622 /lm au

This pa:tient has the diagnosis of ocular hypertension (OHTN).


The definition of aHTN is a patient with gonioscopically open,
normal angles, no glaucomatous optic neuropathy, and statistically
high lOP. In the Ocular Hypertension Treatment Study (OHTS),
aHTN was defined as an lOP of 24-32 mm Hg. Clinically, lOP
greater than 22mmHg is often used (5) (7).

Questions:
Question 1: According to the OHTS, after 5 years what is the chance
of progression to POAG in the general untreated study population?

a. 4.4%

b.9.6%

c. 12.2%

d.50%

Question 2: In THIS patient, which of the following is NOT a risk


factor which increases her chance of developing POAG?
a. African American race
b. positive family history
c. lOP
d. corneal thickness
e. age
Question 3: When reading the HRT results, the iInage quality can be
assessed using the topographic standard deviation. For this patient
the standard deviation was 17 /lIn. How should this HRT should be
interpreted? :
a. with caution
b. as reliable
c. as unusable

246

CHAPTER 7. GLAUCOMA

Question 4: Open angle glaucoma can be treated with argon laser


trabeculoplasty (ALT) and selective laser trabeculoplasty (SLT). SLT
differs from ALT in that SLT:
a. has a larger treatment spot size than ALT
b. is more effective long-term in controlling lOP
c. has fewer side effects
d. the laser used is the same one used for pan retinal photocoagulation
(PRP), therefore, no new laser needs to be purchased
e. has less post-operative lOP spikes
Question 5: A drance hemorrhage is usually associated with which
type of glaucoma?
a. POAG
b. uveitic
c. rubeotic
d. normal tension glaucoma (NTG)
e. angle closure
f. PXF

Answers:
Question 1: correct answer b - 9.6% The OHTS randomized those with
OHTN to a treatment group and a control group. The treatment group used
ocular hypertensives to lower the lOP by at least 20%. Progression to POAG
was defined as the development of glaucomatous optic nerve damage and/or
glaucomatous visual field defects. After 5 years, 4.4% of the treatment group
developed glaucoma where 9.6% of the untreated group developed glaucoma.
This represented a relative risk reduction of 50% (4.4/9.6) and an absolute risk
reduction of 5.2% (9.6-4.4) (7).
Question 2: correct answer d - pachymetry readings The average
pachymetry readings (corneal thickness) based on the meta-analysis studies for
white adults is 473-597f.im (95% cr, 474-596) (16). 555f.im is often the number
used clinically. Patients with OHTN on average have thicker corneas, which
may artificially increase the lOP measurements. In the OHTS, thin pachymetry
readings were an independent risk factor for the progression to POAG, even
after controlling for the effect of corneal thickness on lOP readings and other
factors (7). This patient has thick corneas, therefore, this does not represent a
risk factor.
Glaucoma factors present in this patient:

7.4. CASE 39

247

Race is a risk factor for the development of POAG. Compared to Euro


peans and Caucasians, those of African decent, Hispanics, and Latinos
have increased prevalence of POAG (8).
Family history is also a risk factor for glaucoma. Studies have shown it
is associated with a 3-4 fold risk (11) with first-degree relatives (parents,
siblings, children) with the disease.
lOP is a principal risk factor for glaucoma and as lOP elevates, the risk of
glaucoma increases. One study followed OHTN patients untreated over
40 months and found 2.7% of those with lOP between 21-25mmHg, 12%
with lOP between 26-30mmHg, and 41.2% of those with lOP >30mmHg
developed POAG. Eye pressure assigns risk but is not diagnostic because
glaucoma can be present at low pressures and not all people with high
pressures develop glaucoma (6).
Advanced age is a risk factor for glaucoma. In those with glaucoma,
>90% are over the age of 55 (9). This age-dependence holds true across
difference races, as well (8).

Other potential risk factors for glaucoma (not present in this pa


tient) include history of blunt trauma and chronic use of oral steroid
medication.

Question 3: correct answer b - as reliable


ing.com (12):

From heidelbergengineer

To interpret the image quality of the topography standard deviation for HRT:

< lO excellent
10-20 very good
20-30 good
30-40 acceptable
40-50 look for ways to improve
>50 low quality image, don't use for baseline
Glaucoma is an optic neuropathy characterized by retinal ganglion cell death
and corresponding retinal nerve layer (RNFL) loss. Clinically, it is often diffi
cult to detect change in the nerve fiber layer during fundus examination. RNFL
damage (can occur diffusely or focally) must occur for optic nerve damage and
subsequent visual field defects to become apparent. In an effort to detect struc
tural damage to the optic nerve sooner, computerized instruments have been

developed to analyze the RNFL.

Three instruments are currently available for RNFL analysis:

248

CHAPTER 7. GLAUCOMA

Heidelberg Retina Tomograph, HRT (Heidelberg Engineering, Heidelberg,


Germany)
GDx Nerve Fiber Analyzer (Laser Diagnostics Technology, San Diego, CA)
Optical Coherence Tomograph, OCT (Humphrey-Zeiss, Dublin, CA)
Each work differently to provide a quantitative RNFL analysis. Initially, the
patient's results are compared to healthy, normals matched for age and race
to determine if they are statistically normal or abnormal. In subsequent mea
surements, a change-over-time comparison can be made of the patient to their
previous results.
Basic description of how to interpret the findings from each of
these three instruments:

HRT
Interpretation of the RNFL sector indicators on the Moorsfield Anal

ysis:

Green ./ = Normal. The amount of neuroretinal rim falls within 95% limit of

normal. This amount of rim tissue is considered statistically normal.

Yellow ?= Borderline. The amount of neuroretinal rim falls between 95 and


99% of the healthy normals. This means between 1-5% of healthy normals had
this amount of rim tissue.

Red X= Abnormal. The amount of neuroretinal rim is outside the 99% sta

tistical norm. In other words, less than 1% of the healthy, normals had this
amount of rim tissue. This is considered statistically significantly different than
would be expected for this patient.

see www.heidelbergengineering.com for more details (12)

GDX

Nerve fiber indicator (NFl), the overall integrity of the RNFL (in microns),

range from 1-100:

< 30 Normal.
30-50 Suspect.
>50 Glaucoma
The deviation map, reveals the magnitude and location of RNFL defects. It is
color coded and overlies the NFL picture. The color reflects the deviation from
the normal database:
White p>5%
Dark Blue p<5%
Light Blue p<2%
Yellow p<l%

7.4. CASE 39

249

Red p<0.5%

On the deviation map, red colored areas indicates the most risk of
deviation from normal. p<0.5% means the RNFL thickness in this
area has only a 0.5% chance of being normal. Only 5/100 normals
had this finding.

The TSNIT map (temporal, superior, nasal, inferior, temporal) shows the
RNFL thickness around the optic nerve. The map should have a double hump
pattern. The shaded area of the map represents the normal range (95% of
normals) for the normative database. The TSNIT parameters use the same
color coding scheme as the deviation map.
For more details, please see (17)

OCT
For OCT, the RNFL thickness will be broken down in sectors and receive
either a green (normal, the results fall within 95% of the normals), yellow
(borderline, the results fall between the bottom 1% and 5% of the normals) or
red (abnormal- the results fall below 1% of the normals) color.
In general with the interpretation of the RNFL with all of the devices, the infe
rior and superior quadrants are the most important because these are typically
the areas where glaucomatous optic neuropathy occurs, at least initially.
Question 4: correct answer a

SLT uses a larger laser spot size than ALT.

ALT uses an argon laser (the same one used on the retina for PRP) and the
SLT uses a selective double-frequency Nd:YAG laser (would have to be pur
chased specifically for this procedure). They both apply laser to the trabecular
meshwork which results in the lowering of lOP. The exact mechanism by which
this occurs is still not completely understood. The two procedures have equal
efficacy long-term (13) and comparable side effects. The theoretical advantage
of the SLT over ALT is that it can be repeated if further treatment is necessary
because of the apparent reduced thermal damage to the trabecular meshwork.
This advantage is still controversial (14). Both are potentially effective treat
ments for OAG.
Question 5: correct answer d - NTG A drance hemorrhage is associated
with NTG. lOP plays less of a role in the optic neuropathy in this type of glau
coma. Theories propose this disease has vasospastic factors which contribute to
progression. These patients have higher prevalence of vascular disorders such
as: migraines, Raynaud's syndrome, hypercoagulation or low blood pressure
causing poor perfusion. A drance hemorrhage is defined as a hemorrhage on

CHAPTERr GLAUCOMA

250

the disc or edge of the disc. These hemorrhages are relatively uncommon but
when seen may indicate poor control and/or progression (15).

References
[1] Siddiqui Y, Ten Hulzen RD, Cameron JD, Hodge DO, Johnson DH. What
is the risk of developing pigmentary glaucoma from pigment dispersion
syndrome? Am J OphthalmoL 2003 Jun;I35(6):794-9.
[2] Bartlett J., Jannus S. Clinical Ocular Pharmacology, 3rd edition,
Butterworth-Heinemann, 1995.
[3] Kertes P., Johnson T. Evidence-based eye care. Lippincott, Williams, and
Wilkins, 2006.

[4]

Friedman~, Pineda R, Kaiser P. The Massachusetts Eye and Ear In


firmary Illustrated Manual of Ophthalmology. W.B. Saunders Company,
1998.

[5J Rhee D, Pyfer M. The Wills Eye Manual, Office and Emergency Room
Diagnosis and Treatment of Eye Disease, 3rd edition. Lippincott, Williams,
and Wilkins, 1999.
[6J David, R, Livingston DG and Luntz MH (1977), Ocular Hypertension- a
long-term follow-up of treated and untreated patients. BR J Opthalmol
61: 668-74.
[7J Kass MA, Heuer DK, Higginbotham EJ, et a1 (2002). The Ocular Hy
pertension Treatment Study: a randomized trial determines that topical
ocular hypotensive medication delays or prevents the onset of POAG. Arch
Ophthalmol 120: 701-13: discussion 829-30.
[8J Nduaguba C and Lee RK (2006). Glaucoma screening: current trends, eco
nomic issues, technology, and challenges. Curr Opin Ophthalmol 17:142
52.
[9] Quigley HA and Broman AT (2006). The number of people with glaucoma
worldwide in 2010 and 2020. Br J Opthalmol 90: 262-7.
[IOJ Quigley HA, Addicks EM,Green WR. Optic nerve damage in human glau
coma. Arch ophthalmoI1982;100:135-146.
[llJ Tielsch, JM, Katz J, Sommer A, et al (1994). Family history and risk of
POAG. The Baltimore Eye Survey. Arch Ophthalmol 112: 69-73.
[12J Using HRT II: www.heidelberengineering.com
[13] K.F. Damji, et aL Br. J. Ophthalmol. 2006;90(12):1490-1494.

Chapter 8

Emergency/Trauma

Sarah Dougherty Wood, O.D., MS, F.A.A.O.

253

8.1. CASE 40

255

Annually, ove r 2.5 million Americans suffer a n eye injury, a nd globally more
t han half a million blinding injuries occur every year (1).

8.1

Case 40

Demographics
Age/ race / gender : 50 year old Asian ma le
Chief complaint: splashed chemi cal s in left eye
History of present illness
Character/ signs/symptoms: eye is painful and red, vision is blurred
Location: OS
Severity: 7/ 10
Nature of onset: was mix ing cleaning age nt to was h t he floor s a nd
splashed into OS
Accompanying signs/symptoms: photophobia
Patient ocular history: myopia
Patient medical history: unremarka ble
:Medications taken by patient: none
Patient allergy history: Azasite
Mental status
Affect: fl at
Clinica l findings

BVA:
OD
OS

Distance
20 /20
20/50

Pupils: reactive O U, no a pd OU
Slit lamp
lids/lashes/adnexa: wnl OU
conjunctiva: OD wnl, OS 3+ diffu se inj ection, che mosis
cornea: OD gut.tata , no edema, OS diffuse spk

256

CHAPTER 8.

anterior chaxnber:
iris: ,vnIOU

lens: clear OU

and

UMA

OU

Internals: \.\'111 0 U

1: \Vhat should be the first

of t.his

or any other

done in the examination


with a chemical

b.
of the

c.

d. check versions

e. sli t

the best

is:

a. :30 minutes duration


b.

of ocular irritation

is no

c. VA is back t.o normal

d. no SPK are present


in lower cul-de-sac

e. neutral
no

statements is true

3: Which of t.he
chemical burns'?
a. acid is worse than alkali
b. a whit.e eye
c.
d.

eye

more worrisome than a

of the fornices

cement, and
chemical burns can cause the
the
to the "1-'1"'.1"'1

needs to be

nprf,-,rn1Pr\

of acidic chemicals
which are the adhesion of

Question 4: \Vhich of the


would NOT be
treatrnent. of a mild-moderate chemical burn?
b.

AB ointment

of t.he init.ial

CASE clO

257

d. pressure
e. oral

medication, if needed

f.

artificial

5: Which ointment contains the steroid dexamethasone?


3. m3;;:ilrol

b.

tobrex

FML

Answers:
1: correct answer c first
of the

The

time visual
been a chemical

e - neutral
in lower cul-de-sac
h3\'e corneal SP1\:,
if the

After
and
has

of ischemia
This is BAD

L.U,"PlW>LU""Jll

of incorrect answers

Alkali burns are

because

better corneal

of the fornices with a cotton tip


may
necessary if
occurs
Double lid eversion may need to be done
ill the fornices,
are
adbesioll of the bulbar and
adhesions should be broken
prevent
age. A scleral shell can be
4: correct answer c
avoided due to the vasoconstricti ve
All of the
choices may be indicated,

These

should be

CHAPTER 8. EMERGENCY/TRA UMA

258

Question 5: correct answer a - maxitrol


B, a nd dexa met hasone

i'd axi trol is neomycin, poly myx in

Explanation of the incorrect answers


Erythromycin in a n antibiotic wit.hout any steroid
Blephamide contains s ulfa ce ta mide and prednisolone acetate
Tobrex- t obramyc in , antibiot ic without steroid
FML contains flu orom et ha lone

8.2

Case 41

Demographics
Age/ race/gender: 22 year old Africa n American male
Chief complaint: blurred vision, pain
History of present illness
Character /signs/symptoms: pat ient could see blood in his eye after
he got hit
Location: OS
Severity: 5/10
Nature of onset: s udd en, hit by a soccer ball
Duration: sever al hours
Accompanying signs/ symptoms: se nsi t.ive t o light
Patient ocular history: wears glasses for read ing
Family ocular history
mother: dia betic retinopat hy
father: none
Patient medical history: seasonal a llergies
Medications taken by patient: loratad in e
Patient allergy history: seasonal
Family medical history:
mother: dm

8.2. CASE 41

259

father: ht.n
Clinical findings
BVA:
OD
OS

Distance
20/20
20/ 25

Pupils: PERRLA, no apd OU


EOMs: full OU
Confrontation fields: full to finger count OU
Slit lamp
lids/lashes / adnexa: OD wnl , OS ecchymosis below
conjunctiva: OD clear, OS subconjuncti val hemorrhage
cornea: OD clear, OS clea r
anterior chamber: OD deep and quiet , OS deep / 40% hyphem a
iris: OD wnl , OS wn! (t he iris t.hat can be seen)
lens: cl ear OU
vitreous: clea r OU
lOPs: 12, 14 mm Hg
Fundus OD: all wn!
Fundus OS
C/D: 0.4
macula: fla t
posterior pole: wnl
periphery: area of retina l whiting superior/na sal about 3 disc di
ameters in size

8- ball hyphema- total (100%) / ent ire anterior chamber full of black
colored blood

260

CHAPTER 8. ElliERGENCYI TRA UMA

Questions:
Question 1: What oral medicine should be avoided in this patient?
a. ibuprofen/ ASA
b. antibiotic
c. prednisone
d . t.ylenol
e. antiemetic such as Thorazine
f. aminocaproic acid (Ami car )
Question 2: Due to a specific demographic characteristic of this pa
tient and his hyphema, he should be screened for what systemic
disease?
a. DM

b. Hypertension
c. sickle-cell
d. anem..ia
e. leukemia
f. hy perlipidemia

Question 3: What procedure/evaluation should NOT be performed


at the initial exam?
a . dilated fundus exam
b. tonomet.ry
c. gonioscopy
d . pupil evaluation
e. versions
Question 4: The most likely diagnosis of the fundus finding is:
a . commotio retinae
b. sympa thetic ophthalmia
c. cotton wool spo t
d. CRAO

e. BRAO

8.2. CASE

261

Answers:
1: correct answer a
are blood thinners and may make the
The other medicalions which

be indicated:
if the

nauseous-

up may

worse
shown to decrease
and is
An oral antibiotic or
In addilion:

2:

LO use, if needed

may be

- sickle cell

to

Black

Mediterranean
cell trait or disease.

3: correct

done
resolved.

Question 4:
Commotio
outer retina.
is a whitish discoloration of
also
It
due
blunt
Retinal
not affected and the vision nonnal unless the macula is involved. It may
choroidal rupture or retinal
Commotio
be
will resolve on
when their

For more information on cotton


retina section.

spot. CRAO,

the

CHAPTER 8. EIIJERGENCY/TRAUMA

262

8.3

Case 42

Demogra phics
Age/race/gender: 30 year old Caucasian male
Chie f complaint: hit in t he right eye
History of present illness
Character/ signs/symptoms: OD painful, s\-volle n

Loc a tion: OD

Severity: mo derate-severe

Nature of onset: patient was in a fight

Duration: occ urred 3 days ago

Accompanying signs/symptoms: d iplopia

Secondarj' complaints/ symptoms: pain on eye movement


P a tie nt ocular history: wears glasses occasionally for night dri vi ng
Patient medical history: shoulder injured in recent fight , motor vehicle ac
cid ent where he sustained a head injury which required a metal plate in
h.is sku ll
Medications taken by p atient: flexeril
P atie nt allergy history: nkda
Review of systems
Mus culoskeletal: sore shoulder
Neurologic: head ache
Clinical findin gs
BVA:
OD
OS

Distance
20/ 30
20/25

Pupils: react ive OU, no anisocoria or apd


Confrontation fields: full to finger count OU, vertical diplopia whi ch
is worse on upgaze
Slit lamp

263

CASE 42
edema

wnlOS

wn] OS

cornea:
Ot:
anterior chamber: l+ cell
iris: wlli OU
lens: OD stellate
axial
vitreous: clear OU

OS
OS clear

lOPs:

Fundus: All

wnlOU

Questions:
Question 1: You suspect an orbital blow-out fracture. All of the
following tests may be indicated for this patient EXCEPT:
Et.

b. detection of

orbital rim

d. IvIRI

e. versions
g.

Question 2:
clinical test would help

'~ ....."~
U.LOb.U","

of an EOM restriction. What


a palsy from a restriction?

a. versions

b. forced duct-ions

step

c.

d. red

test

e. Hess Lancaster test

3: Which cranial nerve is the most


Ct.

b.

6
c.

to traurna?

264

CHAPTER 8. Ei\IERGENCY/TRAUAfA

Question 4: What is the initial treatment of an orbital blow-out


fracture on day I?
a. imm ediate surgical repair
b. nasal decongestants (i.e. Afrin) and ora l a nt ibiot ic such as Keflex (cephalosporin)
c. hot compresses to t he orbit
d. prism gro und into glasses
Question 5: This patie nt has a concomitant uve itis . What treat
ment/management should be initiated?
a. pilocarpine qid
b. xalatan qhs
c. tobrex bid
d. pred forte qid
e. syst.emic wo rk-u p to determine cause of uveit is
f. choline rgic a ntago nist (such as cyclogel or homat ropine)
g. answers c and e
h. answers d and f

Answers:
Question 1: correct answer d - MRI This patient has a metal plate in
his skull from a previous injury repair. i\jetal is an absolute contraindication
for MRI. An MRI is not ideal for imaging bone anyway, it is better for soft
t iss ue. An x-ray or CT wo uld image bone t o detect a fract ure.
In an orbital blow-o ut fr act ure, the most cornlIlon a rea Lo s ustain damage is
the orbital floor. The eye may drop down a nd t he inferior rect us or inferior
oblique becomes ent rapped in the fract ure (termed ent ra pment). Thi s will
cause limited upgaze and diplopia .
All of the following should be part of a clinical work-up with suspected orbi tal
fracture:
Crepitus also known as subc utaneous emphysema, can be heard by palpating
the eyelids and li stening for a crackling sound . This represents air from
t he nasa l sinuses released du e to t he fr acture which has become trapped
under the skin .
Hypesth esia is a reduction in sensatio n. C heck t he forehead , cheek a nd up
per lip. The most common a rea affected is t he ipsilateral cheek due to
entrapment of t he infraorbital ner ve in a fracture.
Step-off is a n irregular superior orbital rim due to a fracture which can be
felt on examination. It normally sho uld be smo oth and continuous.

8.3. CASE 42

265

Versions noting any pain or diplopia reported by the pa t ient. This may help
determine if an EO/vI is entrapped.
E xophthalmometry which can detect enop htha lmos due to the orbit sin king
partially into the maxillary sinus.

Patients \\l ith a n orbital blow-out fracture should NO T blow their


nose l

Question 2: correct answer b - for ced ductions With a restriction, the

eye ca nnot physically be moved in a certain direction. For example , a n infer ior

rectus entrapment wou ld not a llow for the eye to be moved upward. \Vith a

crani a l nerve pa lsy, a n EO .M is not getti ng innervat ion but the eye cou ld be

physic ally moved.

Duri ng for ced duction test ing, an attempt is made to physically move the eye.

A fo rceps is used to grab an anesthetized conjunctiva and attempted movement

is mad e in t he direction of limited movement. If the eye resists, t his represents

an a nato mical restr iction and if it does not resist, this represents a crani a l nerve

palsy.

Examples of restrict ions are: entrapment , Duane's Synd rome, Brown's Syn

drome, orbital t umor , and Graves di sease.

The other ansvver choi ces: These tests (versions, Park's three step, red lens

test , a nd Hess Lancaster test) can help determine which EorvI is involved but

will not separate a palsy from a restrict ion.

Question 3: correct answer b - eN 4 CN 4, which innervates t he superior

obliqu e, is the crani al nerve most suscep t ible t.o trauma because it has the

longest co urse.

Ca uses of acquired CN 4 palsy (7)

10% neoplasm-aneurysm
20% isc hemi c
30% undetermined
40% tra uma
A pat ient with a CN4 pal sy will have a vertical diplopia and the s uperior
oblique can be isolated by per forming the P ar k's .3 step test (see the binoc ular
vision section fo r review) . The patient will have a bead tilt avvay from the side
of t ile palsy a nd may have a chin-do\Nn position.

266

CHAPTER 8. EMERGENCY/ TRAU1\IA

Bielschowsky's head tilt test: In a CN4 palsy, head tilt towards


the affected side makes t he diplopia worse. Head tilt away from the
affected side decreases the diplopia.

Question 4: correct answer b - nasal decongestants and oral antibiotic


Explanation of incorrect answers
Surgical repair after about 7-14 days if the diplopia persists
Cold compresses, not hot , to decrease the inflammatio n
Presnel prism can be used to decrease the diplopia. The diplopia may
decrease over time so grinding prism into glasses initia lly is not recom
mended.
Question 5: correct answer h - pred forte qid and cholinergic an
tagonist Uveit is can accompany a trauma- termed traum at ic uveitis. The
treatment is pred forte four times a day and a cholinergic antagonist such as
cyclogel 1% four times a day. lOP can be eJevated with uveitis (not so in this
case, typically lOP is lower in uveitis). If this does occur , an ocular hypotensive
can be added. XaJatan would be a poor choice when infla mmation is present
because it may exacerbate t he problem (prostaglandin analog). There is no
need for pilocarpine or a t.opical antibioti c. The cause of the uveitis is known ,
therefore , a systemic work-up is not needed .

8.4

Case 43

Demographics
Age/race/gender: 20 year old male
Chief complaint: pain , t.earing, foreign body se nsation, and deCJeased vision
History of present illness
Location: right eye
Severity: pain 9/10
Nature of onset: s udden; patient was hit in the eye with a tree branch
Duration: severa l hours ago

267

8.4. CASE 4.3


Patient ocular history: not a contact len s wearer
Patie nt medical history: sport s-induced asthma
:Medications take n by patient: inhal er
Patient allergy history: ASA
Clinical findings

BVA:
OD
OS

Distance
20/ 50
20/20

Near
20 /50
20/20

Pupils: PERRL, no apd OU

EOMs: full OU

Slit lamp

lidsflashes / adnexa:
conjunctiva: diffuse injection OD, clear OS
cornea: epithelial defect, 2mm x 2mm slightly tem poral OD, no
infiltrate, clear OS
anterior chamber: mild cell , no flare OD, quiet OS
iris: wnlOU
lens: \V nl OU
vitreous: wnlOU
lOPs: deferred

Questions:
Question 1: Which of the following is NOT a treatment to b e used
for this patient?
a. patch
b. debridement
c. antibiotic ointment such as eryt hromyci n or bacitracin q2-4 hours
d. cycloplegia (1 % cyclogel)
e. sLeroid drops
f. a nt ibioti c drops such as poly trim qid

g. NSAID (topica.l or oral)


h. proparacaine
i. answers a, e, a nd h
J. answers g and h

268

CHAPTER 8. EMERGENCY/ TRA U1VIA

Question 2: After two days, the patient d e velops a fluffy, gray/white


stroma l infiltrate with s a tellite lesions. What is the most likely eti
ology of this finding?
a. viral
b. fun gal
c. sta ph hypersensit ivity
d. acant ha moeba
e. p se udomo nas
f. herpes sim plex
g . sterile ulcer (non-infectious)
Ques tion 3: If the diagnosis from question 2 is verifie d by c ultures,
the initial treatment is
a . Tobradex
b. natamycin 5% q l-2 hours while awa ke , ointment at night a nd cycloplegic

c. virop t ic 9x/ day


d. po ly t rim qlhr
e. vigamox q 1 hr
f. bacitracin oint.me nt and lid hygiene
g. neosporin drop s q lhr

Question 4: After the initial exam, patie nts with corne al abrasions
such a s this one should generally return to clinic in:
a . one day
b. 4 days
c. 1 week
d. no need for follow-u p
Answers:
Question 1: correct answer i-ans wers a, e and h All o f the opt io ns
are accepta ble treat me nts for THIS patie nt's corneal a brasion (a nd often more
t han one is need ed ) excep t a patch , topical steroi d , a nd proparacaine.
A patch is o fte n used for a corneal abrasion to a id in healing and for patient
comfort . In th is case where t he ab rasion was caused by vegetati ve matte r , a
patch should be avoided because o f ri sk of fun gal infection . In general, a p atch
should also be avoided if t he abras ion is t houg ht to be in fect ious (3) or d ue

SA.

CASE ,13

contact lens wear. In contact lens wearers, t.here is risk of a


infection.
no infect.ion
For corneal abrasions
and pressure
can be
have
for
and
BeL
of the BeL is that the
and often tolerates the
abrasions
be ,uC,"'<'.S""U
BeL, and

medium abrasions with


All would benefit from a

steroids should be avoided when an


defect is present
t.his medication can retard
and increase the risk of infection. Once
defect
steroid can then be
if
or tetracaine should NEVER
a treatment to manThe cornea needs sensation to
Debridement would be done with a corneal abrasion if there
loose or
well in the first 24-48 hours. This will
to occur

2; correct answer b If a
and located in the stroma, think
came in contact with
mat.ter.
may also be present. 8ate]
infections of
infiltrates located around the ulcer.

in the
infiltrates
form in the
dear intern'Ll betvveen
would included chronic lid disease
infiltrate. These
Acanthamoeba infections will have classic
a contact lens wearer who
with contact lenses. See cornea section
Pseudomonas infections are

in contact lens wearers

For more details on the above conditions, see the

3; correct answer b

is

section

CHAPTER 8. Ej\IERGENCYj TRA UMA

270

Explanation of incorrect answers


Tobr adex : no steroid should be used ,vith an epitheli al defect
Viroptic is only for herpes simplex epi t helial disease
Vigamox is a fluoroquinolone antibiotic a nd is not indicated
Neomycin is polymyxin/ neomycin/gramacidin and is not indicated
Question 4: correct answer a - one day Close obser vation of cornea l
abrasion patie nts is reco mme nded to watch for signs of infection and to help
with pa in management.

References
[1] eyeportal.org
[2] Cro uch ER Jr, Frenkel i'd. Am J Opht ha lmol. 1976 i'v1ar ;8 1(3):355-60.
Aminocaproi c acid in t he treatment of traumatic hyphema.
[3] Rhee D, P yfer M. Th e Wills Eye i\1a nual , Office and Emergency Room
Diagnosis and Treatment of Eye Disease, 3rd edit ion. Lippincott , Williams,
a nd Wilkins, 1999 .
[4] Ha ndbook on ocul ar disease management - o n-line, Corneal Abrasion and
Recurrent Corneal Erosion section
[.5] J F Acheson, J Joseph , and D J Spalto n, Use of soft contact le nses in a n
eye casua lty department for t he prima ry treatm ent of trauma tic corneal
abrasions. Br J Ophthalmol. 1987 71: 285-289.
[6] Friedman N, Pineda R, Kaiser P. The l\1assachuset ts Eye and Ear In
firm a ry Illustrated Manual of Opht halmology. \V.B. Saunders Company,
1998.
[7]

Miller N, et a l. "Valsb and Hoyt 's Clinical Neuro-Ophthalmology: The


Essentials, second editio n. Lippincott, \'Villiams, a nd Wilkins, 1999.

Chapter 9

Systemic Health

by Kyle M. Cheatham, O.D., F.A.A.O.

271

9.1. CA.SE 4. 4

9.1

273

Case 44

Demographics
Age /race/gender: 22 year-old white male; college student
Chief complaint: blurry vision
History of present illness
Location: OU

Severity: moderate

Nature of onse t: gra.dual

Duration: 4-6 weeks

Secondary complaints/symptoms: does not like his current glasses; woul d


prefer t hinner lenses
Family ocular history
n1other: cat a rac ts
P a tient medical history: unusual skin lesion s over the past year
Medications taken by patient: acet azola mi de (only uses when moun tain
climbing)
Patient allergy history: NKDA
Family medical history: unremarkable
Review of syste ms
General/ Constitutional: poor den tition
Integumentary: epidermoid cysts on face , scalp , a rms
M e ntal status
Orientation: orien ted to t ime , place. a nd person
Mood : appropriate
Affect: appropri ate
Clinical findings
Distance

Near

274

CHAPTER 9. SYSTEMIC HEALTH

as

Confrontation fields: full to

Slit
unremarkable
normal OD, OS
cornea: aD: Imm x Imm anterior
Normal OS
anterior chamber:
aD, OS
and
iris:
OS
lens: clear
OS
vitreous: clear OD, OS

OD, 1.5

lOPs: 16
etry

OS

Pi\I

to line of

tonolll

Fundus OD
with
macula: normal
pole: normal

rim

with

borders and

with

borders and

Fundus OS
0.15 H/V with
macula: normal
post.erior pole: normal
periphery: three
cemral

Blood pressure:

Pulse: 68 bpm,

is the MOST likely diagnosis of


systemic condition?
Goldenhar
b. Gardner's syndrome

disease

9.1. CASE 44

275

Question 2: Which of the following is the MOST likely diagnosis of


this patient's retinal lesion?
a. choroidal nevus
b. congenital hypertrophy of the retinal pigmented epitheliulll (CHRPE)
c. cobblestone degenera tion
d. lattice degeneration
Question 3: Choroidal nevi that are at most risk for malignant trans
formation include all of the following factors EXCEPT?
a. tiJickness > 2rnm
b. size> :3mm
c. proximity to the optic nerve head
d. orange pigment on the sm-face of the tumor
e. presence of subretinal Auicl
Question 4: Which of the following is NOT a side effect of this
patient's medication?
a. metallic taste
b. tingling in back of the neck
c. metabolic acidosis
d. depression
e. aplastic anemia

f. myopic shift in refractive error


Question 5: Which of the following is the lVIOST appropriate man
agernent strategy for this patient?
a. refer to retinal specialist
b. monitor lesion again in 6 months
c. recommend sunglasses for ultraviolet protection
d. refer for endoscopy

ANSWERS
Question 1: correct answer b - Gardner's syndrome
Familial Adenomatous Polyposis (FAP) is an inherited condition (autosomal
dominant) that results in hundreds of polyps throughout the colon in post
puberty patients. If untreated, the patient will develop colon cancer; in most
cases, this occurs by age 50 (1).

276

CHAPTER 9. SYSTE1UIC HEA.LTH

Gardner's Syndrome is a variant of FAP characterized by lllultiple, bi


lateral, atypical congenital hypertrophy of the retina.l pigmented epithelium
(CHRPE's) in the fundus. If a patient has 4 or more CHRPE's in one eye, a
referral to internal rnedicine is warranted to rule-out FAP or Gardner's syn
drorne. The patient in this case is 22 years old, the classic age for diagnosis of
Gardner's
the average age of diagnosis for malignancy in the colon
is 39 (11).

Gardner's syndrome has a triad of multiple intestinal polyps,


skeletal hamartomas and soft tissue tumors (cysts, neurofibromas,
fibromas,
(5). Over 50% of patients with the condition will
have dental anomalies (11).

Summary of other answer choices


Goldenhar syndrome is a rare clinical syndrome that comprises a triad of
ocular dermoid, preauricular skin tags and vertebral dysplasia. Dermoids a.re
choristomas - normal tissue in an abnormal location, that represent the most
common epibulbar (on the eyeball) tumors of childhood (1). They are smooth,
soft, yellowish, subconjunctivaJ masses that are commonly located at the infer
otemporal limbus, but may involve the entire cornea (1)
Crouzan's syndrome is a craniofacial disorder that should be on the list of
differentials for proptosis present at birth. This condition is characterized by
shallow orbits with proptosis, aniridia, blue sclera, strabismus, optic atrophy,
hypertelorism (wide separation of the orbits) and congenital cataracts (1) (4) (5).
Congenital encephalocele can also result in proptosis at birth; the proptosis
increases and becomes pulsatile during crying (4).
Fabry's disease is an X-linked autosomal recessive lysosomal storage disease
that results in cornea verticillata (90% of cases) and posterior spoke-like lens
opacities (50% of cases) (4) (1) (6). In affected males, patients often report
episodes of excruciating pain in the fingers and toes (1); premature death (age
40-50) often results from renal or cardiovascular complications (6).
Question 2: correct answer b - Congenital hypertrophy of the retinal
pigmented epithelium
CHRPE's are benign, pigmented (brown to black), non-progressive lesions
with sharp borders and central hypopigmented lacunae. The lesions are thought
to be congenital with no race or sex predilection (2). CHRPE's are typically
unilateral and solitary, measuring 1-6mm in diameter. Bilateral, multifocal (4
or more) CHRPE's are associated with Gardner's syndrome.

tumor in the choroid that represents


of the population
flat or
ly ele-

not contain the


of
have
Question 3: correct answer b - size> 3mm
Choroidal nevi that are
thickness> 2mm (1), size>
on the surface
nevi

orange

1W"IT!',",C'';'

in back of the neck


:rvletallic taste,

vlliiS"iliS

U"""",UH 5: correct answer d - refer for


risk for colon cancer;

is at
This
warranted.

9.2

45

Demographics
Age/race/gender: 41 year-old white male;
Chief complaint:

vision

and

278

CHAPTER 9. SYSTEjlIIC HEALTH

History of present illness


Location: OU
Severity: seve re
Nature of onset: gradual
Duration: started 3-4 years ago
Secondary complaints/symptoms: none
Patient ocular history: last eye exa m 2 yea rs ago; POAG OU , diagn osed 8
years ago; history of t raum a to t he right eye at age 7
Family ocular history
nlother: gla ucoma
Patient medical history: alcoholis m since age 15; alcoholic cirr hosis with
hypoalbuminemia; hype rtension ; gall- bladder removed .5 years ago
Medications taken by patient: li sinopril; combigan
Patient allergy history: NKDA
Family medical history
father : died at age 55 from lung cancer
Review of systems
G eneral/Constitutional: weight loss
Integumentary: jaundi ce
M e ntal status
Orientat ion: o riented to time, place, and person
l\l{ood: appropriate
Affect: appropriate
Clinical findings
Distance

Near

Pupils : PERRL , negati ve AP D


EOM s: full OD , OS
Confrontation fie lds: full to finger counting OD , OS
Slit lanl.p

279

9.2. CASE 45

lids/lashes/adnexa: unremarkable OD, OS


conjunctiva: normal OD, OS
cornea: OD: 2mm x 2mm a nterior stromal sca r at 6 o 'clock. Nor
ma l OS

anterior chamb e r: deep a nd quiet OD , OS

iris: normal OD , OS

lens: clear OD , OS

vitreous: mild vit reous float ers OD > OS

lOPs: 16 mmHg OD , 18 mmHg OS


etry

4: 30 PlvI by applanat ion tonom

Fundus OD
C/D: .65 Hj. 80Y with notch at 7 o 'clock, 1+ temporal optic di sc
pa llor
macula: normal
posterior pole: norm a l
periphery: pavin gstone degenerat ion inferiorl y
Fundus OS
C/D: .65 H / .80Y wit h thinning at 1 o'clock, 1+ tempora l opti c
disc pa llor
macula: nor mal
posterior pole: normal
periphery: pavingstone degenerati on inferiorly

Blood pressure: 115/ 65 mmHg, ri ght arm, sitting

Pulse: 68 bpm , regula r

Question 1: Which of the following is the MOST likely cause of this


patient's optic disc pallor?
a. gla ucoma
b. traum a
c. a lcoholism
d. related to medication
Question 2: Blood testing on this patient would MOST likely r e veal
which of the following?
a. elevated Alanine transaminase (ALT), decreased B1 levels
b. elevated Hemoglobin A1c (HbA1c) , increased B1 levels
c. decreased Aspart ate tra nsaminase (AST ) , decreased B1 levels
d. decreased glomer ular filtr ation ra te (GFR), increased blood urea nit rogen
(B UN) levels

280

CHAPTER 9. SYSTEIIJIC HEALTH

Question 3: Visual field testing on this patient would MOST likely


reveal which of the foll ow ing?
a. OD: superio r field loss from glaucoma, central or centrocecal field loss
from te mporal optic nerve pallor
b. OS: no field loss from glaucoma, nasal field loss from temporal optic nerve
pallor
c. OD: inferior field loss from glaucoma, nasal fi eld loss from temporal opt ic
nerve pa llor
d. OS: inferior fi eld loss from glauco ma , nasal field loss from temporal opti c
ne rve pallor
Questio n 4: Which of the following is NOT true?
a. For wome n, breast. ca ncer is the most common type of cancer
b. For men , prostate cancer is the most common type of cancer
c. For 'women, lung ca ncer is t he most common ca use of cancer deat h
d. For men , prostate cancer is t he most common cause of cancer death
Question 5: Which of the following is true regarding the patie nt's
ocular medica tion?
a . so lution t hat is a combination of .5% Timolol and 2% Dorzolamide
b. so lu t ion t hat is a combination of .5% Timolo l and .2% Brimonidine
c. solu tion that is a combination of .5% Xalatan and .2% Brimonidine
d. solution t hat is a combin ation of .5% Travatan and 2% Dorzolamid e
ANSWERS
Questi o n 1: correct answer c - alcoholism
T his pati ent has classic findings for a p"tient with severe a lcoholism; tempo ra l
pallor of both optic nerves and compl aints of bila t.era l, pai nIess, progressive
vision loss.
Question 2: correct answer a - elevated Alanine transaminase (ALT) ,
decreased B1 levels
AS T and ALT are enzymes contained with in li ver cells that are released into the
blood in increased co ncentrations in cases of hepatitis, regardl ess of the etiology.
El evation of these enzymes would be expected in th is patient secondary to hi s
liver cirrhosis. GFR, creat.i nine aJ1d BUN would not be relevant in this case
because t hey are monitored in renal disease.

9.3. CASE 46

281

3; correct answer a - aD:


central or centro cecal field loss from

optic nerve

bas notching at 7 o'clock


ill the
to superior
visual field loss. The
nerve is where macular fibers
bundle)
fibers causes central or
visual field loss.

which would
of the
to

Ll"'.:>,",VU 4: correct answer d


For men, prostate cancer is the most
comnlOn cause of cancer death

In women most common cancers in the U.s.


and colon cancer. ?\Iost common
breast and colon.
most common
occurrence
order) are
alld colon canceL ]\105t cornman causes
prost.ate and colon
of cancer death in men are
Uv."""UH 5: correct answer b
Brimonidine

combination of

Timolol and
combination of
Timolol

Case 46

Demographics
white
Chief

vision
of

nr,oc:~'nr

Location:

illness

oe

Severity:

Nature of onset:

Duration: '1-6 months

st.udent

282

CHAPTER 9

SYSTE!lIIC HEALTH

Secondary complaints/ symptoms: none


Patient ocular history: last eye exam 5 years ago; previous contact lens
wearer
Family ocular history
mother: "blind" from retinal detachment
Patient medical history: unremarkable; last visit to his medical doctor was
at age 15 for a upper respiratory infection
l\1edications taken by patient: patanol
Patient allergy history: NKDA ; pollen
Family medical history
mother: died at age 40 from cardiovascular irregularity
Revi e w of systenls
Cardiovascular: hear t palpi tations
Musculoske le tal: joi nt laxity with mul tiple dislocations of joints
M e ntal status
Orientation: oriented to time, place, and person
l\100d: appropriate
Affect: appropriate
Clinical findings

Distance

Near

R e fraction

Distance

-6.50 DS
-6.00 DS

Pupils: PERRL, negative APD


EOMs: full , no restrictions
Confrontation fields: full to finger count ing OD, OS
Slit lamp
lids/ lashes/adnexa: unremarkable OD , OS
conjunctiva: mild diffuse bulbar hyperemia OU

9.3. CASE 46

283

cornea: clear OD , OS
anterior chamber: deep and quiet OD , OS
iris: normal OD , OS
lens: OD : 7 mm ectopia lentis superiorl y ; OS: 5 mm ectopia lentis
superiorl y
vitreo us: clear OD, OS
lOPs: 16 I11mHg OD, 15 mmHg OS ,QJ 2:30 P rvI by a pplana tion t onom
etry
Fundus OD

C /D: 0.10 H/ V with hea lt hy rim tissue

macula : norma l

posterior pole: norm a l

periphery: unrem arka ble

Fundus OS

C/D: 0.1 5H/V wit h heal t hy rim tiss ue

macula: norma l

posterior pole: normal

periphe ry: unremarkable

Blood pressur e : 135/ 75 mmHg , rig ht. arm , sitting

Pulse: 68 bpm, reg ular

Question 1: \Vhich of the following is the MOST like ly diagnosis of


this patient's systemic condition?
a. E hlers-D a nlos syndrome
b. Homocystinuria
c. \ Veill-I\larchesani synd rom e
d. f-.Iarfa n's syndrome

Question 2: Which of the following is the MOST likely e tiology of


this patient's systemic condition?
a . mu tation within t he fibrillin gene
b. deficiency in hydroxylysine
c. defi ciency in cystathionine sy nt hase
d . valine taking t he place of g lutamic acid

284

CHAPTER 9. SYSTEMIC HEALTH

Question 3: What is the T\10ST concerning ocular complication of


Marfan's syndrome?
a. myopia
b. central retinal artery occlusion (CRAO)
c. acute angle closure glauco ma
d. lens subluxation
e. retinal detachment
Question 4: Which of the following is NOT a mast cell stabilizer?
a. cromolyn sodi UIl1
b. alomide
c. alamast
d. alocril
e. optivar
Question 5: Which of the following statements is NOT correct?
a. Osteogenesis imperfecta does NOT cause lens subluxation
b. lvlarfan 's syndrome and ost.eogenesis imperfecta are causes of keratoconus
c. Osteogenesis imperfecta and Ehlers-Danlos sy ndrome resul t in similar
ocular complications, including keratoconus and megalocornea
d. Marfan's syndrome is a common cause of blue sclera
Question 6: Which of the following is the MOST appropriate initial
step in the management for this patient's condition?
a. refer to cardiologist
b. refer to primary ca re physician for better blood pressure control
c. refer to ophthalmologist for evaluatio n of subluxated lens
d. prescribe new glasses and huve patient return in 1 year
ANSWERS
Question 1: correct answer d - T\1arfan's syndrome
This case covers systemic conditions that can cause lens subluxation (ectopic
lent-i s). ivlar fun's syndrome is an inherited (a utosomal dominant) connective
tissue disorder caused by a mutation in the fibrillin gene on chromosome 15 (9).
Clinical feat ures include:

46

for lens subluxation. The

Summary of other answer choices


Ehlers-Danlos
disorder
include
(1):
lens
cornea

features
blue

keratoconus and
mitral

cardiovascular abllormali ties

laxity with dislocation

the enzyme

short stature with

stub

lens sublm::ation

tachment

2: correct al"lswer a - mutation within the fibrillin gene


As described above.

within the fibrillin

Question 3: correct answer e - retinal detachment


In ivlarfan's
retinal detachments
rious ocular complication (1).

and

the

CHAPTER 9. SYSTEi\IIC HEALTH

286

Question 4: correct answer e - optivar

Mast cell stabilizers include cromolyn sodium (Crolom) , lodoxamide (Alomide),

pemirolast (Alamast), nedocromil (Alo cril ).

Question 5: correct answer d - Marfan's syndrome is a common cause

of blue sclera

As described above , blue sclera is NOT a n expected complication with Ivlar

fan's syndrome. Osteogenesis imperfecta and Ehlers- Danlos sy ndrome resul t

in similar ocular complications, including keratoconus , blue sclera and megalo

cornea; however, Ehlers-Danlos can also cause lens subluxation (4). System ic

conditions asso ciated with keratoconus include T\lrner 's syndrome, Down's syn

drome, osteogenesis imperfecta, Ivlarfan's sy ndrome and Ehler's Danlos syn

drome.

Question 6: correct answer a - refer to cardiologist

This patient shou ld be immediately referred to a cardiologist for evalu at ion;

he is at risk for aort ic aneurysm a nd mitra l val ve prolapse. The pat ient has a

family history of early death resulting from card iovascu lar complications and

has not been eva luated by a physician for over 13 years.

References
[1 ]

Kanski , Jack. Clinical Ophtha.lmology 4th ed . Woburn: Butterworth and


Hein mann , 1999.

[2]

Rapuano, Christopher J. Heng, Wee- Jin. Color Atlas and Synopsis of Clin
ical Ophthalmology. Wills Eye Hospital. Singapore: I'vlc Graw-Hill , 2003.

[3]

Bartlett, Jimmy D., Jaanus, Siret D. Clinical Ocular Pharmacology.


Boston: Butterworth, 2008.

[4] Tamesis, Richard R. Ophtha lmology Board Review.,


McGraw-Hill , 2006.

2nd Edition.

[5]

Friedman, Neil J. Kaiser, P eter K. The Massachusetts Eye and Ear Infir
mary. 3rd Editi on. Elsevier, 2009.

[6]

Thomas, Randall. ]'vlelton, Ron. http:// wv\.w.eye update.com/

[7]

Friedman, N. Kaiser, P. Trattler, W. Revi ew of Ophthalmology. Philadel


phia: Elsevier , 2005.

[8]

Cheatham , K. Cheatham, IV1. Vlood, K K]\,lK Part One Basic Science


Review Guide. 4th edition . 2009

287

9.3.

Lawrence :\1.,
Current l\ledical
2006.

and i'lIaxine A.
Eds.
45th ed. NevI' York. l\IcGraw
the Posterior

[111

,","(>Tn,on!

ed.
1093486

Part II

Ocular

ment and

Management

289

10

Pharmacology

by Kyle M.

'-JU<CClCH.:tH

O.D., F.A.A.O.

293

Direct
and bethanechol
the
in this
on the test
outline. Recall that
mechanism
action involves interaction with
muscle receptors, which
the scleral spur
and
opens
up the trabecular meshwork spaces
I' outflow, .L rop.

up 30% lOP reduction.


is

cataracts,

Indirect Cholinergic

Anticholinesterase

Echothiophate (Phospholine): A.llticholinesterase agent


for the
treatment of accommodative
(1) .

Anticholinesterase agent
'..fost common treatment choice
oral tablets every 4 how's

CHAPTER 10. PHARMACOLOGY

Cholinergic

(1) .

blurred \ision can result from

he1'el1ce,

that is the most potent


to 10
available (1). Good eye can be treat.ed
in cases of mild

mouth is the most

blood aqueous
purpose.

upon instillation is the

common ocular side effect.

Drowsiness.

visual hallucinations are t.he most. coml11on

effect.s
less
and less
with the

296

ClIAPTER 10. PHARMACOLOGY

> oj) with


with chronic use
before and after ocular
crease intraocular pressure,
response
(

ervation
clonidine is
to cause dilation in
no effect on the normal

effective
has shown
the past,
because of follicular con-

Brirnonidine

,. Available
concentration ":ith BAK
,15% concentrations with Purite
P),
,. Dosed tid for

and

and bid when used in combination

can

side
Brimonidine is contraindi

on the

neuron to release norepinephrine,


to dilate upon
the

If the patient fails


lesion.

297

Recall that cocaine. when instilled in a healthy eye,


causes
dilation. In Horner's syndrome, instillation of cocaine has no effect
Oll the miotic pupil (regardless of sympathetic pathway lesion 10
catioll). Since apraclonidine is easier to obtain, it is oftell utilized
instead of cocaine. The miotic p\lpil in Horner's Syndrome will
dilate after instillatioll of aprac:lollidine; hydroxyampiletamine is
then used to determine lesion location.

Naphazoline (Naphcon) / Tetrahydrozoline (Visine): Adrenergic ag


onists that constrict conjunctival blood vessels and have the potential to cause
fixed dilated pupils (after
use) due to sympathetic effects on the dila
tor (radial) muscle. Recommended dosage is 1-2 drops per administration that
does not exceed qid.

Systemic Adrenergic Agents


l\1etaproterenol( Alupent), Isoproterenol (Isuprel):
bronchodilator used for asthma treatment.

agonist - potent

Methylphenidate (Ritalin), Dextroamphetamine (Dexedrine): eNS


stimulants that increase dopamine release and are used clinically for the treat
Illellt of ADD, narcolepsy and depression (47). High-dose, chronic
with
these drugs can cause
and dry eyes. In patients with narrow anterior
chamber angles, the
effect can lead to acute or subacute stages of
angle closure (1).
Clonidine (Catapres): 0:2 agonist (inhibitor receptor) which causes sym
pathetic outflow to decrease. Commonly used to treat hypertension because it
decreases vascular resistance and heart rate.
Cocaine (Benzoylmethyl ecgonine): In the Ct\S it blocks the reuptake
of norepinephrine in the reward centers of the brain. It can also be used as a
local anesthetic (by blocking Na channels), vasoconstrictor (stops nose bleeds),
and as a mood elevator in drug abuse. Can be used ophthalmically as a topical
anesthetic and for diagnosis of Horner's syndrome.
Albuterol (Ventolin): Has greater
receptor
(due to Beta
treatmellt of asthma and COPD. Side effects include
1 effects), heart palpitations, nervousness, tremor and nausea. Levalbuterol
(Xopenex), a newer derivative of albuterol, appears to have less side effects.

298

CHAPTER 10. PHARMACOLOGY

Beta-Adrenergic Blocking Agents (B-Blockers)


Parasy mpa thomimetic agents that decrease lOP by ac ting on B-receptors (mainly
/32) in the ciliary ep ithelium to decrease aqueous product ion. Topical agents on
the test outline include timolol (Timoptic), levob unolol (Betagan) , betaxolol
(Betoptic S) , a nd met.ipra nolol (OptiPra nolol). Oral agents on the test outline
include labeto lol (Tra ndate), propranolol (Inderal), atenolol (Tenor min ) a nd
metoprolol (Lopressor). Note t hat all B-blockers end in "0101."

Recall that ,6 1 specific B-blockers minimize lung side effects. Be


taxolol , atenolol and metoprolol a re examples included on the test
outline.

TIMOLOL (Timoptic): Non-selective B-blocker. 1st introd uced B-blocker


in 1978 and co ntinues to be t he most effecti ve at lowering lOP - around 26%
reduct ion (1 ). Also available in gel-fo rm (Timoptic XE).

Timolol sho uld be used cautiously in diabeti cs a nd patients with


hyperthyroidi sm - t he cholinergic effe cts of B-blockers can mask the
clinical signs a nd sy mptoms of these condit ions (1). The pa rasym
pa thet ic effects of myasthenia gravis can be worsened by Timo
101 (1).

Noteworthy cl inical facts about timolol incl ude (4 1) :


Maximum effi cacy is si milar bet.ween .25% a nd .50% solution concentra
t io ns.
Often dosed bid , bu t a once-daily regimen is effective (2 nd drop does not
provide much a ddition al effect) (1) .
If dosed qd. morning dosage is recomme nded (has better daytime effi
cacy) .

Similar to other B-Blockers, unilateral use of Timolol commonly reduces


rop in the contralateral eye (3 4)
Simila r to ot her B-Blockers , extended use of Timolol often results in
long- term drift (rOp starts to gradu all y rise) or short-term escape
(lO P initi ally lowers but returns to norma l within weeks after starting
therapy) (35).

300

CHAPTER 10. PHARMACOLOGY

10.1

Glaucoma Drugs

Glaucoma drugs incl ud e the foll owing:


Cholinergic agonists (e.g. pilocarpine, bethanechol) - these drugs t
Tl\l outfiow
Adrenergic Agonists (e.g. apraclonidine, brimonid ine) - these drugs .l
aqueous production and t uveoscleral outflow
B-blockers (e.g. t imolol , levobunolol , betaxolol, met ipranolol - these
drugs .l- aq ueous production
Combination drugs (cosopt , combigan) - cosopt.l- aqueous production;
combigan .l- a queous prod uction and t u veoscleral o utflow
Carbonic anhydrase inhibitors (azopt, trusopt, a cetazolamide, met
hazolamide, di chlorphenamide) - these drugs .l- aqueous production
Prostaglandins (e.g . xalatan , travata n, lumigan) - these drugs t uveoscle
ral outflow
Hyperosmotics (e.g. glycerine, isoso rbide) - these drugs.l- aqueous pro
duction
Glaucoma drugs included within the autonomic drug section (cholinergic ago
nisLs, adrenergic agon ists , B-blockers, combination drugs) were discussed above.
The following is an overview of the remaining glaucoma drugs, including car
bon ic anhydrase inhibitors, prostaglandins, and hyperosmotics.

Carbonic Anhydrase Inhibitors


Sulfa-based age nts that decrease lOP by inhibiting the production of bicarbon
ate.
Topi cal CAl's: Brinzolamide 1% (Azopt), Dorzolamide 2% (Trusopt)
Trusopt - as monot.herap.!", tiel dosing is sta ndard ; bid dosi ng (21.8
24.4%) slightly less effective as compared to t id dosing (2 2.2 - 26.2%).
Azopt - as mono therapy, bid dosing is standard - effects of bid in com
parison to t id were equivalent.
Adverse effects - Topical CAl's do not have the side effects that occur
with oral CAl's - bitter taste (occurs in 25% of patients taking dorzo
lam ide) is the most common adverse effect (1).

101

301

GLAUC01\fA DRUGS

Oral CAl's: Acetazolamide (Diamox), lYlethazolamide


Acetazolamide
oral
Due to
where short-term use is warranted,
clinical information for acetazolamide

is a summary

.. Available in 12.5
250
sustained-release
500
and 500 mg vial for intravenous administration,
treatment should include a total of 500 mg
rug

.. POAG treatment
6 hI'S 500 mg

recommended due to side

250 mg

hI'S,

.. Reduce aqueous humor formation

..

bands and

and

.. l\Iost serious adverse effects

anemia,

.. Other adverse effects


.. COlltraindications
sider other
disease,

First-line

COll
,"",,-"c<o,-,,

renal

on
27 ;15o/c
allows better diurnal control

302

CHAPTER 10. PHAR!\,IACOLOGY

Prostaglandins increase uveosclera l outflow by activating PGF2a


receptors (ciliary mu scle receptors) for in creased metalloproteinases
act ivity.

Side effects of prostaglandins include:


Iris heteroc hromia (often permanent, eve n after di scontinuation of drug)
Increa sed pigment.ation and growth (lengt h, thickness , a nd number) of
eyelashes
Skin darkening a ro und the eyes (most common in mixed-color irises)
Conjunctival hyperemia (most common with lum igan)
Contraindi cations of prostagla ndins inclu de (1):
Patients with cysto id macula r edema (CrdE) or those at risk for Ci\ IE
(e .g. cataract. s urgery)
Cases of active inflammation (e.g. uveitis)
Patients with previo us episodes of herpes simplex keratitis (1).

10.2

Hyperosmotic Agents

Glycerine (Osmoglyn); High molec ul ar weight, water-soluble compo und


t hat is unab le to cross the blood-aqueous barrier; this creates a n osmotic gra
die nt in \vhich the plasma in the ciliary stroma region is hypertonic to the
aqueo us humor, lowering lO P (8). Noteworthy clinical information includes:
Admi nis tered (4-6 ounces) to lower fluid volume during an acute angle
closure attack.
Ra pidly absorbed a fter ora l admi nistration and is metabolized into car
bohydrates, increasing blood sugar levels. For this reason it is not recom
mended in pat.ients with diabetes - isosorbide solution sho uld be utilized
instead (41).
Sodium Chloride (Muro 128 ) ; Hype rt onic solution that is prescribed for
reduc t ion of corneal edema. Available in t he following forms:
Eye drops (2% and 5% solutio ns) - 5% typically prescribed
dosed qid

In

clin ic

Ointment form (5%) - typically dosed qhs .


Treatment for Fuch 's endothelial dystrophy can include Muro 128
drops qid an d oin tment qhs.

10.3. ANTIlvIICROBIALS

10.3

303

Antimicrobials

DRUGS THAT IMPACT CELL WALL SYNTHESIS


Peptidoglycan provides bac terial cell wall structure . Pept idoglyca n contains
polysaccharide chains that are cross-linked together via transpeptidases. Bac
itracin and Vancomycin act as cell wall synthesis inhibitors by inhibiting pep
t idoglycan synthesis. Penicillins a nd cep halosporins act as cell \-vall inhibitors
t hrough inhibition of tra llspeptidase.

Bacitracin
Peptidoglycan synthesi s inhi bitor - highly selective to gram (+ ) bacteria. i'vlost
commollly prescribed in ointment form for blepharitis .
Polysporin: Broad spectrum a ntibiotic ointment that combin es Baci
tracin 's gram (+) coverage with Polymyxin B 's gram (-) coverage .
N eosporin: P olysporin added to neomycin. P olymyx in , like neomycin ,
is never used as a stand-a lone drug ; it is always found in combinat ion
wit h another agent (40).

Vancomycin
Pept idoglycan synthesis inhibi tor with severe side effects, including ototoxicity
(has caused permanent deafness), nephrotoxicity, and Red Man's Syndrome.
Adm i nistered IV for treatment of bacterial endophthalmit is and is the treat
ment of choice for methicillin resistant. staph infections (i\IRSA) (.54) .

Met hicillin resistant staphylococc i a re difficult to treat because


they a re also resistant t.o cep ha losporins, aminoglycosid es and
macrolides. Vancomycin and its toxic side effects are reserved
for these infections (1).

Natural Penicillin - Penicillin G / Penicillin V


Tra nspeptidase inhibitors prescribed for treatment of gram (+) bacte ri a , in
c! uding streptococcus, syphilis, meningit is, and pneumococcal infections. These
dru gs are NOT penicillinase resistant (47). Penicillin G is given H"I or IV

304

CHAPTER 10. PHAR1\1ACOLOGY

(because unstable in gastric acid). Penicillin V can be given orally. Recall that
oral absorption of most penicillins is poor and is impaired by the presence of
food; they should be taken at least a half an hour before or two hours after
meals.

Aminopenicillins: Ampicillin and Amoxicillin


\!Vider spectrum than natural penicillins (more gram negative) activity but
similar to natural penicillins in that they are NOT penicillinase resistant. Com
monly prescribed to combat otitis media and respiratory infections in chil
dren. Amoxicillin is more commonly prescribed due to its better oral absorp
tion.Clavulonic acid (penicillinase inhibitor, also called B-Iactamase inhibitor)
can be added to amoxicillin to produce the antibiotic Augmentin.

Hypersensitivity Reactions are by far the most common ad


verse effect of penicillins, while anaphylaxis is the most severe acute
effect. If a patient is allergic to penicillin, there is a 1.5% chance
that the patient will also be allergic to a cephalosporin. Penicillins
can render oral contraceptives ineffective (although not a consis
tent response) - they are generally very safe in all trimesters of
pregnancy.

Methicillin, Nafcillin, Dicloxacillin


Penicillinase resistant penicillins. Dicloxacillin can be prescribed for eyelid in
fections such as hordeolums and blepharitis. Orbital cellulitis infections caused
by staph can be treated with IV naJcillin.

Cephalosporins: Cephalexin (Kefiex), Cefuroxime (Ceftin),


Ceftriaxone (Rocephin)
Transpeptidase inhibitors that are less susceptible than penicillin to penicilli
nases (47). Cephalosporins become increasingly more effective against gram
negative bacteria at higher generations .
Keflex - commonly prescribed (2.50-500 mg bid to qid) for dacryoadenitis,
dacryocystitis, and preseptal cellulitis .
Ceftriaxone is the treatment of choice (via IV) for gonococcal conjunc
ti vi tis and orbi tal cell uli tis.

10.3. ANTI1\IICROBIALS

305

Gonococcal conjunctivitis: if cornea is involved, ceftriaxone is ad


ministered 1 gram IV every 12-24 hours; d uratio n of t reatment de
pends on clinical response (typicall y 3-5 days) (35) . If the cornea is
NOT involved systemic ceft riaxone is ad ministered 1 gram INt
Orbi t al celluli tis: the patient is hospitali zed and ceft riaxone is ad
ministered IV fo r 1 week .

DRUGS THAT IMPACT DNA SYNTHESIS


Drugs t hat impact protein sy nt hesis incl ude fiuoroq uinolones , sul fo na mides ,
t rimet hoprim a nd py rimet ha mine.

1. FI uoroquinolones
Ciprofloxacin (Ciloxan, Cipro), Ofloxacin (O c uflox) , Levofloxac in
(Quixin), Gatifloxac in ( Zy m ar), M oxifloxa cin (Vigamox, Avel ox )
Inhibit bacter ial DNA synt hes is by acting 011 DN A gyrase AND Topoisomerase
IV. Recall that DNA gyrase a nd topoisomerase IV a re enzymes uti lized for
bacterial DNA sy nt hesis - they are only fou nd in bacteria.
Examples of clinical uses for fluoroq uinolones include:
Corneal ulcers - every 1-2 hI's fo r sma ll ulcers; fortified antibiotics (every
1 t o 2 hours) fo r larger or sight-t hreatening ulcers.
Simple bacteria l conju nctivitis - ty pically dosed qid (varies with age) for
5-7 days (40).
Corneal erosion/abrasion - dosed bid to qid , depending on size a nd loca
t ion .
Zy mar a nd V igamox , t he 4t h generation fiuoroquinolones, have enhanced po
te ncy towa rd g ra m (+) bacteri a while st ill ma in taining gra m (-) vi rul ence. All
of t he to pical opht ha lmic fl uo roq uinolones, except Quixin , a re approved fo r
use in pat ients 1 year of age a nd old er (1).

Oral AuoroquinoLONES can hurt the attac hments to yo ur BONES


causing tendinitis (5). These d rugs are contraindicat.ed in preg
nancy, clli ldren, and adolescents below t he age of 18 d ue to damage
in carti lage formation and inhibi tion of bone growth (41).

306

CHAPTER 10. PHARJ\JACOLO GY

2. Sulfonamides
Sulfisoxazole (Gantrisin), Sulfacetamide (Sulamyd),
Sulfamethoxazole (Gantanol), Sulfadiazine (lVIicrosulfan)
Agents that inhi bit enzyme used in the first step of folic acid sy nthesis - effec
t ive against gram (+ ) and (-) infections but rarely used anymore for topical
ophth a lmi c treatment due to bacterial resista.nce and t he availability of more
effecti ve agents . Sulfadiazine is used in combination with py rimetha mine to
t reat toxopl as mosis.
Steven Johnson's syndrome can be ca used by t he topical ophthalmi c and
oral su lfonamid es and oral carbonic anhydrase inh ibitors. The most common
ad verse effects of topical ophtha lmic ad minist ration are burning , stinging. con
tact der ma titis a.nd local photosensitization (s unburn on eyelid m argins) (1) .
3. Trimeth oprim (Primsol)
Inhibits enzyme used in the second st.ep of folic acid synthesis. Polytrim is a
combin at ion of trimethoprim a nd polymyxin B .
4. Pyrime thamine (Daraprim)
Same IIIOA as t rimethopri m - inhibits enZYllle used in the second step of folic
aci d synt hesis. P rescribed o rally for toxoplas mosis infections in t he eye .
DRUGS THAT IMPACT PROTEIN SYNTHESIS
Drugs tha.t impact protein sy nthesis includ e t he following:
Aminoglycosides and Tetracyclines (tetracycl ine , doxycyclille, minocy
cl ine) - act o n 30s subunit
Chlorampheni col , Erythromycin, Lincomycin - act on 50s s ubunit
Macrolides (eryt hromycin, az ithromyc in, clarithromyc in) and Clindamyci n
- also act on 50s sub unit

1. Aminoglycosides: G entamicin (Garamycin), Tobramycin


(Tobrex)

Ba cteria.l protein synthesis inhibi tors (act on 30S s ubu nit ) that provide broad
spectrum coverage; however , thei r forte is in t he gram (-) spectrum (40).
TobraDex (Tobra myc in 0.3%, Dexamethasone 0.1 %) is commonly dosed qid
and is prescribed for inflammatory ocula r conditions with associated bacterial
infection (e.g. staph marginal keratitis, corneal infil trate ).

10.3. ANTIMICROBIALS

307

Tobramycin is a lso avail able in topical ophthalmic and ointment form. Gen
tamicin and to bramycin topical ophthalmic solutions are availa ble in forti
fied concentrations , along with fortified cefazolin for the trea tment of sight
threatening cornea l ulcers (1).

Oral aminoglycosides can cause nephrotoxicity and ototoxicity.


Topi cal arninoglycosides are notorious for causing superficial punc
tate keratitis and delayed reepithelializatio n.

2. Tetracyclines: Tetracycline, Doxycycline, Minocycline


Bacterial protein synthesis inhibitors (act on 30S sUb uni t) t hat provide broad
spectrum coverage.

Doxycycline is co mmon ly prescribed for meibomianitis , acne rosacea, chlamy

dial ocular infectio ns (e. g. t rachoma, inclusion conjunctivitis) and aft.er a re

current cornea l erosion to decrease the risk of future recurrences.

l\1eibomianitis - doxycycline 100 mg bid for 4 weeks , t hen qd for 3-6


months (40).
A cne rosacea - doxycycline 100 mg bid un t il symptoms a re relieved
(2-6 weeks) and t hen tapered to 100 mg for several weeks thereafter.
Some patient.s will require long-te rm t.reatment wit h periostat , a low
dose (20mg) doxycycline t.ablet. :M etronidazole (MetroGel) topical
gel (1%) can a lso be prescr ibed for patients with chronic ac ne rosacea.
Chlamydia conjunctivitis - doxycycline 100 mg bid for 10 d ays .
Recurrent corneal erosions - oral doxycy line (.50 mg bid for 2 mo nths)
Tetracyclines are contraindica ted during pregnancy and in children! The
following are noteworthy side effects:
Pseudotumor cerebri , bone growt.h retardation, discoloring of teeth, pho
tose nsitivit.y (looks like severe sunburn), G.!. distress and hypersensitivity
reactions (uncommon).

Recall that Penicilli n, Azith romyci n, T etracycl ine are generally


taken without food - PAT a n empty stomach.

308

CHAPTER 10. PHARJ\IACOLOGY

3. Chloramphenicol, Erythromycin, and Lincomycin


Bacterial protein sy nt hesis inhibitors (act on 50S subuni t) that provide broad
spectrum coverage. Chloramphenicol can be formul ated as an eye drop, in
ointment form , or as all oral agent; however, it is rarely prescribed due to the
possibility of causing irreversible aplastic anemia, optic neuri tis, and Grey baby
sy ndrome.

4. Macrolides (Erythromycin, Azithromycin, Clarithromycin),


Clindamycin
All of t hese drugs a re bacteri al protein synthesis inhibi to rs (act on 50S s ub
unit ); erythromycin , azithromyci n a nd clarithromycin are grouped into a class
of drugs called macrolides. Oral use of the mac rolides can cause gastroin
testinal issues including na usea, diarrhea, abdominal pain. Opht halmic uses
for these drugs incl ude:
Erythromycin ointment has a poor resistance profile a nd is no t com
monly prescribed to combat active infections; it is more commonly ut i
lized in a prophylactic role as a noct urna l lubricant (40). It is a lso
presc ribed (instead of silver ni trate) for prop hylaxis of gonococc al oph
thalmia neonatorum. Oral erythromycin ca n be prescribed to combat
chlamydial infections.
Azithromycin (Zithromax) is commonly prescribed for chlamydial
infect ions (trachoma. a nd inclusion conjunctivitis) because of its co n
venient single I-g ra m dose; it should be take n on an empty stomach.
Azithromycin can a lso be prescribed as four 250 mg capsules or as two
500 mg capsules.
Azithromycin (AzaSite) is a topical solution used to combat blephar
it is - qd dosing is typical - loading dose of qI2hrs for a few days is o ft en
prescribed.

Bacteriostatic agents include (not exha ustive list) tetracyclines ,


trimethoprim , sodium sulfacetamide and, t o some degree, ery
thromycin. Bactericidal agents include (not exhaustive li st) the
penicillins, bacitracin , aminoglycosides, cephalosporins and fl uoro
quinolones (40).

lOA. ANTlFUNGALS

Tuberculosis (TB)
Ethambutol
These
used
combination for treatment of active
ocular side effects include:
can cause

Isoniazid

tears.

can cause

Ethambutol cause
neuritis this
retrobulbar and
bilateral - the initial ocular symptom is reel uced visual

a
very effective
scribed. Rifabutin can cause
endothelial
and

pre

(1) .

, Fluconazole

are oral agents that inhibit


oral
derivative used for

, lVIiconazole

Ketocona

mem bra ne blocker


kerati

CHAPTER 10. PHARMACOLOGY

310

For fungal ulcers, natamycin or amphoteric in B is prescribed ev


ery 1-2 Ius while awake. Systemic antifungals (e.g. ketoconazole)
can be added, and are ad vised , in severe cases. Ora l a nt ifu ngals
are also utilized for treatment of acanthamoeba (e.g. oral keto
co nazole 200 mg for several weeks).

Natamycin (Natacyn): The only topical ophthalmic drug app roved by the
(FDA) to t reat fungal infections. Broad-spectr um agent that is well tolerated
(better t ha n a mphoteri cin B); considered the drug of choice for fungal keratitis.
Has same mechanism of action as amphotericin B. Do not prescribe for patients
who are pregnant.
Nystatin (lVlycostatin): Narrow spectrum (topical only) agent available in
creams, ointments, suppositories, and swish a nd spit form for a pplication to
skin and mucous membranes. Has same mechanism of action as amphotericin
B.

Griseofulvin (Grifulvin): Oral age nt that inhibits fungal mitosis by in


teracting with microtubule formation during cell- wall development (5) . Gri
fuhrin is used to treat infections of the scalp a nd skin, including fingern ail s
a nd toenails. Griseofulvin is t he only antifungal whose MOA does not involve
ergosterol.

10.5

Antiviral Drugs

Thifl uridine (Viroptic)


Drug of choice for primary and recurrent HSV keratitis (types 1 a nd 2) (1) .
Epit helia l keratitis - prescribed every 2 hours, 9 t imes daily until lesion
healed (typically 5-7 days) , t hen 5 times daily for 5-7 more days) (69) (40) .
Stromal keratitis - prescribed qid as prophylactic t reatment with pred
forte qid.

Acyclovir (Zovirax), Valacyclovir (Valtrex), Famciclovir


(Famvir)
Oral agents prescribed for act ive Herpes Zoster ophthalmicus (HZ O) infec tions
and prophylactically for prevention of recurrent Herpes Simplex Virus (HSV)

10.5.

DR

311
for

also be used

.. Active HZO keratitis


800
Famciclovir 500
1
.. Active HSV keratitis
100
5x
500
F<1Jllciciovir 250 mg tid for 7
..

Valtrex
400 mg bid, Valtrex

viral DNA
Oral anti virals can be used to treat
is considered the standard of

Ganciclovir (Cytovene):
tomegalovirus) infections,
lar

IllP('11nl1<';

in IV form for Cl'vlV

Foscarnet (Foscavir):
fails.

(1).

if

of influenza A but

Ribavirin (Copegus or Rebetol):


C. In children, the inhaled

for cases of chronic


ment
include:

Virus

.. Oral rib(1\'irin - blurred vision

.. Inhaled ribavirill- can cause

nd
in

healthcare workers .

Zidovudine (Retrovir) (AZT): The


ther
for AIDS. Acts as a reverse
or
IV. AZT is also used during pregnancy to lower risk
of giving HIV to the fetus.

312

CHAPTER 10. PHARMACOLOGY

ANTIPARASITIC DRUGS
Chloroquine (Aralen)
Drug prescribed orally for t he prophylaxis and treatment of ma laria - causes
toxic heme (breakdown product of hemoglobin) to accumula te in red blood
cells. Also acts as phospholipase-A inhibitor , which provides effecti veness in
inflammatory conditions such as lupus and rheumatoid arthritis; however, it
is ra rely prescribed for these cond itions a nymore because hydroxychloroquine
is an effective treat ment that poses much less risk for bull's eye maculopa
thy (41).
Risk of ret inopathy decreases with dosage less t han J mg/kg of body
weight, treatment d uration less than :) years and normal renal func
tion (1) .
Central and paracentral scotomas are the most C0111mon visual field de
fects with chloroq uine. Color vision is typically normal in the earl y (RPE
mottling) stages of the retinopathy.
Kwell (Lindane)
Agent prescribed for treatment of lice (with sham poo) and scabies (w ith lotion).
Shamp oo treatment should not be used near ocular areas - conjunctivitis can
result if applied to eyelashes.
The following is recommended treatment for ocul ar pediculosis (35).
Removal of lice and eggs with jeweler's forceps.
Topical antibiotic ointment (erythromyci n, bacitrac in) - tid for 1-2 weeks
(smothers lice and nits).
Anti-lice lotion and shampoo (e.g. Kwell) to nonocular a reas .
Clothes and sexual partners sho uld be treated appro priately to avoid
recurrences.

10.6 Major Drugs Acting on the Central Nervous


System

NON-OPIOID ANALGESICS
Acetaminophen (Tylenol)
Agent prescribed for analgesia a nd antipyresis - does NOT have a nt i-inflam matory
properties. Tylenol can be used in patients of a ll ages, including infan ts; it is

10.6. l\JAJOR DRUGS ACTING 01\'eNS


pregnancy,
'which can be fataL

The

313

risk of
overdose
should .'lOT exceed 4,000

OPIOID ANALGESICS

monary edema, sickle cell


tory

for
in

tracranial pressure and

can cause Iniosis.

Codeine
Natural agent 'with
Administered
abuse
with

causes miosis
less

(1,
effect.
addiction and

Tramadol
of codeine used for treatment of moderate to severe
dizziness and nausea,
Adverse effects include

ctLoll1CUl1

(Xanax),

(Valium)

that bind GABA receptors and decrease the


neurotransmi tters, C1 inical uses i!lel ude
and alcohol withdrawaL
side

include:

CHA PTER 10. PHARIIIACOLOGY

314

Mydriasis (anticholinergic act ivity) and nystagmus (1)


Drowsiness, sedation, impaired motor coordination , weakness, dizziness
and confusion
\Vhen combined with alco hol, these drugs can be fatal.

ANTIPSYCHOTICS
Chlorpromazine (Thorazine), Thioridazine (Mellaril)
D2 recept.or antagonists prescribed for treatment of schizoph renia. Adverse
ocu lar side effects i ncl ude:
Hyperpigmentat.ion of the RPE (1). Too much of t hese drugs can result
in Parkinson-like effects (32) .
Pigment on cornea l endothe lium
Anterior stellate cataracts
Dry eye , mydriasis
Increase in lOP

ANTIPARKINSON DRUGS

Prominent Symptoms of Parkinson's Disease - Tremor at rest,


Rigidity, Akinesia, and loss of P ost ural reflexes. Parkinson's is a
TRAP (5).

Sinemet (Carbidopa-Levodopa)
Antiparkinson drug that can also be used to treat benign essential blepharospasm
(BEB). Recall t hat levodopa is converted to dopamine once it crosses the blood
brain barrier and can lead to ad verse effects (e.g . Schizophre nia). Contraindi
cations include schizophrenia, can.lii:!c arrhythmias, melanoma and pat ients at
risk for angle-closure glaucoma.
Bromocriptine (Parlodel)
Dopamine agon ist that can be used to treat benign essential blepha rospasm
(BEB) and prolactin-secreting pituitary adenomas.

l'vIAJOH DHUGS ACTING ON CNS

10.

315

the

Monoamine oxidase inhibitor (i'dAOI) that


their effects.

Amantadine (Symmetrel)
agent that is

to

Dementia l\1edications
Donepezil (Aricept)
dementia. Adverse

inhibitor used to
ocular effects include:

and

irritation
but some consider

contraindication to use

ANTIDEPRESSANTS
There are 3 major

Each class affects a different

neurocransmi ttE'[

Monoamine Oxidase Inhibitors


Phenelzine

of

cheese
cause a lethal
serotonin

cri

CHA PTER 10. PHARMACOLOGY

316

Tricyclic Antidepressants (TCAs)


Amitriptyline (Elavil)
TCA's competitively inhibit NE and Serotonin reuptake at nerve terminals ,
increasing their levels in the synapt ic cleft and allowing clinical use for pain,
fibromyalgia, migraines and sedation. Noteworthy sid e effects include:
Dry eye syndrome and increase in lOP (1 ).
Phenylephrine 10% is cont raindi cated in patients taking T CA's - same
effects can result as with MAOI's.
Sedation, weight gain, orthostat ic hypotension , drowsiness and sexual
dysfunction.
TCA overdose is LIFE-THREATENING! DO NOT give to a suicidal
patient!

Selective Serotonin Reuptake Inhibitors (SSRI's)


Fluoxetine (Prozac), Sertraline (Zoloft)
First-line treatments for depression, panic disord er, OCD, bulimia, P TSD and
premenstrual syndrome. Can ca use dry eye syndrome. Con traind icated with
i\lAOI's du e to risk of serotonin syndrome.

ANTICONVULSANTS
Phe nytoin (Dilantin):
a nd diplopia.

Anticonvul sant with ocul a r side effects of nystagmus

Topiramate (Topamate): Anticonvulsant with ocula r side effects of blurred


vision (most common) and acute second ary angle-closure glaucoma.

Phenobarbital (Luminal Sodium):


oc ular side effects .

Anticonvulsant with no noteworthy

SKELETAL MUSCLE RELAXANTS


Cyclobenzaprine (Flexeril): Skeletal muscle relaxant used for short- te rm
treatme nt of m uscle spasms. Valium ca n also be used as a skeletal muscle
relaxant.

10.7. ]\IIAJOR ENDOCRINE SYSTE1H DRUGS

10.7

317

Major Endocrine System Drugs

Levothyroxine (Synthroid)
T4 hormone used to treat hypothyroidism. Can cause psuedotumor cerebri in
kids.
Sulfonylureas: Glipizide (Glucotrol), Glyburide (Diabeta,
Micronase), Chlorpropalllide (Diabines e), T olbutalllide (Orinase)
Oral agents for NIDD1'vI patients whose disease is not co nt rolled by diet and /or
exercise. These drugs increase secretion of insulin by beta cells, decrease
glucagon release and ca use in creased sensitivity to ins ulin.
Biguanides: Metforlllin (Glucophage)
Oral agent for NIDD lvI t.hat decreases liver glucose production a nd increases
gl ucose uptake.
Thiazolidinediones (the zones): Pioglitazone (Actos), Rosiglitazone
(Avandia)
Oral agents for NIDD M that. com bat insulin resistance (47). These drugs bind
to pel"Oxisome proliferator-activated receptors (PPARs) to increase glucose up
take in muscle and fat t issues. Weight ga in is a cOl11l11on side effect. Ca n be
prescribed in combination wit h met formin.
Dip eptidyl peptidase-4 inhibitors: Sitagliptin (J anuvia)
Newe r group of oral diabetes med ications which inhibi t glucagon release , in
crease insulin secretion and decrease gastr ic emptying (62). Commonly pre
scribed in combination with met formin.

10.8

Major Cardiovascular Drugs

Furoselllide (Lasix): Diur et ic used for t reatment of (non-ex haustive list)


HTN, edema from congesti ve heart failure (CHF), and pulmon ary edema. In
hibits Na 2CI- K+ Co-transport in the thick ascending LOH (47). Can
cause ototoxicity. hypokalemia, dehyd rat ion, sulfa allergies, nephri t is and gout.

CHAPTER 10. PHARIIL4COLOGY

318

Hydrochlorothiazide (Hydrodiuril), Chlorothiazide (Diuril): Diuretic


used for treatment of HTN, CHF , and hypercalciuria. Can cause HyperGLUC
- hyperGlycemia , hy perLipidemi a, hyperUricemia, hyperCalcemia (5). Acts on
t he early distal convoluted tubule (DCT).
Spironolactone (Aldactone): Diuretic used for treatment of hyperaldos
teronism , potassium depletion, and congest ive heart failur e; potassium sparing,
so ca n res ult in hyperka lemia.
Mannitol (Osmitrol): Diuretic that can be ad ministered IV (for approx 45
ll1inutes) to lower lOP in patie nts with acut e angle-closure attack. Contraindi
ca ted in patients with pulmonary edema , dehydration and CHF.
Lisinopril (Prinovil, Zestril), Benazepril (Lotensin), Enalapril
(Vasotec), Captopril (Capoten)
ACE inhibi tors (inhibitors of Angiotensin production) presc ribed for treatment
of HTN. Most common side effect is cough.
Losartan (Cozaar)
Angiotensin II Receptor Antagonist (ARB) prescribed for treatment of HTN.
Does not resul t in cough side effec t.

ARB 's all end with SARTAN. They are often prescribed when
patients cannot take an ACE inhibitor due to the cough side effect.

Propranolol (Inderal), Labetalol (Trandate), M etoprolol


(Lopressor), Atenolol (Tenormin)
Non-selective (3 adrenergic receptor antagonists (,8 blockers) used to treat HTN.
Tamsulosin (Flomax), Prazosin (Minipress)
Alpha adrenergic receptor antagonists (0: blockers) prescribed for t reatment
of HTN and Benign Prostatic Hypertrophy (BPH). Tamsulosin can cause
intraoperative floppy iris sy ndrome.

10.8. MAJOR CARDIOVASCULAR DRUGS

319

0:1 blockers decrease peripheral vasoco ns triction.

Nifedipine (Procardia), Verapamil (C a lan, Isoptin), Dilti a zem


(Cardizem)
Calci um channel blockers prescribed for treat ment of a ngina and HTN; vera
pamil is a lso p rescribed for arrhyt hmias. Some glaucoma specialists recom mend
calcium cha nnel blockers for the treatm ent of low-tension glaucoma; th ese
drugs may increase perfusion to t.he opt ic nerve (34).
Clonidine (Catapres)
0.2 receptor blocker prescribed for t. reatment of HTN. Also used for an algesia,
opioid or tobac co wit hdrawal.
Hydralazine (Apre soline)
Va sodilator used for treatment of severe hypertension and CHF.

The prima ry signs a nd sy mptoms of all types of congestive heart


failur e (CHF) include decreased exercise tolerance, shortness of
breat h, abdominal fullness, peripheral and / or pu lmonary edem a.

Digoxin (Lanoxin)
Agent prescribed for treat ment of congestive heart failure (CHF) - inhi bits
Na+/K+ ATPase enzy me a nd increases int racellula r Ca2 .
Ocular side effects include retrobulbar optic neuri t is. B/Y color defects
and entopi c phenomenon ("snowy" vision, dimming vision, fli ckering lights) (1).

Diuretics, Beta blockers a nd ACE inhibi tors ca.n also be used for
C HF treatment.

CHAPTER W

320

P[-fARMACQI,OGY

Amiodarone
Antiarrhythmic agent (K -;- channel blocker) with

ocular

.. Nonarteritic ischemic
neuropathy
occurs within weeks of starting amiodarone (1).

of

("corneal
- inevitable at 400
the patient will have minimal or no corneal
" Anterior

lens
- commOll
after 6 months of treatment (

in

coronary blood flow


from
is chest
most common reason for this condition is coronary mtery dis
with unstable
and oxygen as .~OOll

Nitrates
edema. Sublingual
used
and
chronic stable
administration is most common - can also be
transdermally, via oral
IV. Oral forms are not
acting.

ANTICOAGULANTS AND THROMBOLYTICS

Vitamin K
should be discontinued 96 to

for
of chrollic
bours (4

10.9. RESPIRATORY DRUGS

321

Other Antiplatelet Drugs (Aspirin and Ibuprofen)


These drugs prevent or reduce arterial platelet aggregation by blocking the
action of cyclooxygenase.

ANTIHYPERLIPIDEMIC AGENTS
Lovastatin (Mevacor), Simvastatin (Zocor), Atorvastatin (Lipitor)
HMG CoA redu ctase inhibitors prescribed to decrea se LDL cholesterol and
tri glyce ride concentrations and to increase HDL.
Fibric Acids
Fenofibrate (TI:icor)
Fibric a cid th a t acts on peroxisome prolifera tor activated receptor (PPAR - 0)
to increases lipoprotein lipase ac tivity for the breakdo'Nn of LDL cholesterol
and triglycerides (47).
Cholestyramine (Questran): Drug t hat prevents absorption of bile in t he
intestine a nd is used in combination with H ~vIG eoA reductase inhibitor s to
lower LDL.

10.9

Respiratory Drugs

Salmeterol (Serevent Diskus): Long ac ting /32 agonist used for chronic
control (not ac ute sympt.oms) of asthma a nd COPD.
Advair Diskus: Long acting {32 agonist that is a combination of salmeterol
(Serevent) and a steroid fiuticasone (Flovent) for ma in tenance trea tm ent
of asthma a.nd COPD a ssocia ted bronchospasm.
Albuterol (Ventolin) , Terbutaline (Brethine): Short ac ting ,62 ago nists
used to treat acute sy mptoms of asthm a, COPD , a nd exercise-induced bron
chospasm - dail y use is not recommend ed.
Cromolyn sodium (Cro lom): P rototy pe of mast cell stabilizer cJass. Does
not a ll eviate ac ute symptoms or exacerbations associated with as thma. Can
be used in combination with a bronchodilato r for lon g-term cont rol of ast hma.
in children and for prevent.a.tive t.reatment prior to unavoida ble exposure to
known allergens.

322

CHAPTER 10. PHAR1\lA COLOGY

Fluticasone (Flonase):

Corticosteroi d nasa l-spray for allergic rhinitis.

Theophylline : i\ Iethylxanthine used for CO PD maintenance . Rarely used


due to safety concerns - has narrow t herapeuti c index (5).
Acetylcysteine 5% (Mucomys t): Mu colytic agent available in oral, in
ha led , a nd solution form. Among eye care practi tio ners, mucomyst is most
commonly prescribed (qid ) for severe dry eye disease, especially when filam ents
prese nt . Also prescrib ed for t reatment of corneal burns.
Zafirlukast (Acco la te): Leukotrie ne antagonist used to treat asthma (in
hibi ts bronchoconstr ict ion).

10.10

Gastrointestinal Agents

Recall that pept ic ulcer disease (PUD) is due to infections with th e gram nega
tive rod Helicobacter pylori, inc reased HCL secreti on, and inadequate defense.
T he following a.re drugs used to trea.t PUD.
Omeprazole (Prilosec), Esomeprazole (Nexium ) : Proton pump inhi bitors
used as first-line t hera py for pep tic ulcer disease and gast ro-esophageal reflu x
di sease (GERD).
Cimetidine (Tagamet), Ranitidine (Zantac), Famotidine (Pepcid):
H2 blockers that block ac id secretio n from pari eta l cells; remember, histamine
stimulates these cell s. These dru gs can be found OTe and a re used for gastr ic
healin g a nd prevention of stomach ulcers and acid reflux.
Sucralfate (C a rafate): Used for ac ut.e management ofPU D to all ow healing
of stomach mucosa . Forms a paste-like substance in t he stomach, thereby
protecting the stomach lining. No well-known ocula r side effects.

10.11

Chemotherapeutic Agents

Methotrexate (Rheumatrex): Foli c acid analog (inhibi ts DNA syn t hesis)


prescr ibed fo r the treat,ment of rheumatoid arthri tis, psori ases, leukemi as, and
lymphomas.

10.12. IMMUNOSUPPRESSIVE AGEt-lTS

323

5 - Fluorouracil (5-FU) (Efudex): Folic acid analog (inhibits DNA sy n


thesis) that inhibits fibroblast proliferation. Ophthalmic use is limited primar
ily to glaucoma filterin g surgery (t rabeculectomy), in which case it keeps the
surgical wound from healing. 5-FU can also be used for treatment of basal cell
carcinom as .

Tamoxifen (Nolvadex): Estrogen a ntagonist used for treatment of breast


cancel' during and for at least five years following the remission of breast cancer.
Noteworthy oc ular side effect incl ud es:
Crystalline retinopathy - most common at doses> 6.5 mgjkgjday for
more than 5 years. The crystals do not typica ll y cause loss of acuity (1).

10.12

Immunosuppressive Agents

Cyclosporine (Neoral or Sandimmune): Sandimmune is the IV formu


latioll , Neoral is oral. These drugs inhibit the production and release of IL-2,
thereby preve llting activation of T-Iymphocytes. Used for prevention of rejec
tion of kidney, li ver, and heart transplants (in conjunction \vith corti costeroids).
Ocular side effect s include cataract, eye pain, (nonspecific) visual disturbance,
conjuncti vitis (non e of these side effects are common) .
Restasis (cyclosporine .05%) inhibits T-cell activation by stopping the
production of interl eukin-2 - dosage is q 12hrs. Most notable adverse
ocular effect is stinging upon instillation.

Azathioprine (Imuran): Purine analog that inserts into DNA and RNA
and stops replicat ion . Systemic uses include treatm ent of rheumatoid arthri
tis and prevention of kidney transplant rejection. No well-known ocular side
effects.

Immunosuppressive therapy (e.g. cyclosporine , azathioprine)


can be used in conjunction with systemic steroids for benign essen
t ia l blepharospasm and for scleritis treatment (when conventional
therapy is not effective) (35).

10. PHARMACOLOGY

10.13

Oral Antihistamines
(Benadryl),
, Promethazine

maleate

effects include

eyes.

Loratadine (Claritin), Fexofenadine (Allegra), Cetirizine (Zyrtec)


antihistamines
Ocular side
Cetirizine

Mast Cell Stabilizers and


HI Antihistamines: Emedastine
maleate

(Emadine), Pheniramine

Mast Cell Stabilizers:


(Alomide),
(Alocril)
do

t11er

10.14. ANTI-INFLA j\ JjIiATORY AGENTS

325

apy. These drugs a re dosed bid a nd can be utilized as preven ta tive treatment
prior to unavoida ble expos ure to knO\vn allergens.
Combination Drugs: Aze las tine (Optivar), Epinastine (Elestat),
Ketotifen (Zaditor), Olopatadine .10% (Patanol), Olopa tadine .20%
(Pataday)
Nlast cell-an t ihi stami ne combination drugs tha t provide effectiveness in ac ute
and chronic cases of allergic conj unc tivitis and ocular itch ing. E xamples of
clini cal uses include:
Seasona l a llergic conjunctivitis - bid fo r on e week (qd for patad ay), then
prn t he reafter (40).
Gi a nt P apillary Conjunctivitis (GP C) - bid unt il symptoms reso lve or
condition clears (qd for pataday) - often 1-4 mon ths (35)

10.14

Anti-Inflammatory Agents

Steroids
Inactivate phospholipase A2 which decreases infl ammato ry mediators by in
hibi ting the arachi do nic acid pathway.
Topical Steroids
Exa mpl es of cli nica l uses for topical steroids inclu de t he following:
Uveiti s - Predforte 1% acetate q 1hl'-2hr5 wit h slow taper (1 drop per day
every 3-7 days) (35).
Central corneal ulcer - If ulcer is in vis ual axis, Predforte 1% acetate can
be added t.o antibacterial agent once t he epithelial defect is nea rl y closed
(red uces ris k of sca rring). No steroid is needed for lesions ou tside the
vis ual axis (40).
Episcleritis - mild steroid , qid for 5-7 d ays
Pingueculi t is - mild steroid, bid to qid fo r 5- 7 days
Ora l a nd IV (e.g . necrotizing) fo r scleri tis
Herpes stromal kera titis - P redforte 1% acetate - dosed qid - steroids
shou ld NO T be prescribed in cases of herpes simplex epithelial keratitis.
Allergic conjunctivitis - mild steroid , q id fo r 7 days, the n bid for 4-8
weeks (40).

CHAPTER 10. PHARMACOLOGY

326

For prescribing pmposes it 's important to categorize topical steroids based on


t heir potency.
Strong steroids: Predniso lone 1% Acetate, Rimexolone (Vexol) and
Dexamet haso ne (Max idex) .1% a re more lik ely to res ult in side effects.
Soft steroids: FI uorometholone (F JdL) 0.1 % a nd Loteprednol (Lotemax)
0 ..5%
Adverse effects (non-exhaustive list) of topical steroids include:
Immunosuppress ion - cause increased risk of secondary infections.
Glaucoma - increased lOP ("steroid response") can res ult from a de
crease in aqueous humor outRow through the trabec ula r meshwork.
PSC cataracts - dose dependent and irreversible; Hispanics appear to
be at highest risk (1).

5% of the genera l popu lation are high steroid responders ; 90%


of patients with established POAG are high steroid responders (34).

Systemic Steroids
Examples of clini cal uses for oral steroid s in clude t he fol lowing:
Scleritis: 60-100 mg qd for 1 wk, then taper to 20 mg qd for 2-3 wks,
then more taper (35) .
Giant cell arteritis: 80-100 mg qd for 2-4 weeks or until symptoms
resolve a nd ESR is normal. This treatment is prescribed AFTER three
d ays of initia l treatment with IV meLhylprednisolone (69) (35).
Toxoplasmosis: 20-40 mg qd - ini tiated 12-24 hours after st arting an
timi crobia l t hera py (35).
Thyroid Eye Disease Optic Neuropa thy: 100 mg qd for 2-1 4 days .

Before prescribing oral steroids, you sho uld a lways inquire a bo ut


preg na ncy, peptic ulcer disease and dia betes (40).

Exampl es of clini cal uses for triamcinolone inj ections include the foll owing (1):
Diabetic macular edema

10.14.

ANTI-INFLAMJ\1ATORY AGENTS

327

Graves ' orbitopathy


Intermediate and non-reso lving posterior uveitis
Chalazion removal
Cystoid macular edema foll owin g cataract surgery (assuming topical treat
ment fails ) .
Cystoid macular edema associat cl with noninfectious posterior uveitis .
Macular edema assoc iated wit h ce ntra l retina l vein occl usion.
Adverse effects (non-exhaustive list) of systemic steroids include:
Diabe tes and /or insulin res istance, weight gain, fat red istr ibution.
Glaucoma, PSC cataracts, decreased immune respo nse and delayed heal
ing time.
Eyel id depigmentation - can occur with chal azion inj ections in dark skinned
patients.

NON-STEROIDAL ANTI-INFLAMMATORY DRUGS


(NSAIDs)
NSAIDs block cyclooxygenase, which decreases inflammat ion by stopping t he
conversion of arach idoni c acid into prostaglandins a nd t hromboxanes.

Topical NSAIDs
Bromfenac (Xibrom), Flurbiprofen ( Ocufen) , Diclofenac sodium
(Voltaren), Ketorolac tromethamine (Acular)
Examples of clini ca l uses for NSAIDS include t he foll ow ing:
P ost-op cataract patients (reduces ri sk of CME) - dosed bid wit h Pred
forte 1% acetate (qid ) .
Recurrent corn eal erosions / corneal ab rasio ns - dosed bid for 2-3 days,
depending on pa in level, until cornea epitheli a lizes.
Allergic conjun ctiviti s - ketorolac is the onl y NSAID approved for to pica l
treatment of seasonal allergic conjunctivitis - dosed bid; a lternative op
tion (bes ides a n NSAID ) should be considered if patient has any corneal
in vol vement .
Noteworthy ocu la r side effects include:
Transient burning a nd stinging upon instilla tion - especially wit h ketoro
lac.
Corneal toxicity - not a consistent occu rrence; corneal melt ing can oc
cur but was most problematic in a gener ic form of Voltaren , which was
subsequentl y withdrawn from the market (1).

CHAPTER 10. PHARMA COLOGY

328
Oral NSAIDs

Oral NSAIDs on the test o utline include as pirin , ibuprofen, naproxen, naproxen
sodiulll , piroxicam, indomethacin a nd celecoxib.
Aspirin (Acety lsalicylic Acid)
Irreve rsible, non-selecti ve Cox inhibitor with a nti-inflamm a tory a nd analgesic
fun ctions (32) (47). Ad verse effects include Reye's syndrome (post-infectious
encephalopathy in children) , GI effects (gastric ulcers and bl eeding) , cardiovas
cular effects a nd respiratory effec ts.
Ibuprofen (Advil, Motr'in), Naproxen (Naprosyn) , Naproxen
Sodium (Aleve), Piroxicam (Feldene)
P ropio nic acid deriva ti ves with similar mechanism of acti on as aspirin; the
important distinction is that they act as reversible non-selective inhibitors
of cyclooxygenase (41). Clinical effec ts include antipyresis, anti-inflamma tory
effects and a na lgesia. Dosage should NOT exceed 2,400 mg/day.
Exampl es of oc ular uses incl ud e:
Scleritis treatment - Ibuprofen (400- 600 mg qid) , Indomethacin (25
mg tid), Naproxen (250-500 mg bid ); length of treatm ent varies - can
range from 1-3 weeks or beyond .
Episcleritis treatment - Ibuprofen (200-600 mg t id for 5-7 days); some
prescribe oral NSAIS instead of topical steroids as initial treatment in
epi scleritis (35).

Ibuprofen can be prescribed in kids 6 months and older (does


NOT ca use Reye's syndrome) , but should not be used during preg
nan cy.

Indomethacin (Indocin)
Potent, non-select ive Cox inhibitor commonly prescribed for gout (32 ). Adverse
ocular effects include whorl keratopathy, ret ina l hemorrhage, a nd retinal
pigmentary ch a nges (especia lly in macula).
Celecoxib (Celebrex)
Cox-2 inhibitor (protects gas tric mucosa) prescribed for treatment of os
teoarthriti s a nd rheumatoid a rthritis.

10.1 5. TOPI CAL OCULAR ANESTHETICS

329

Hydroxychloroquine (Plaquenil)
P hospholipase-A2 inhibi tor prescribed fo r treatment of lupus , rheumatoid a rthri
tis, and mal ari a (prophylaxis a nd treatment). T he most concerning ocular side
effect is bull 's-eye rnac ulopa thy.
Bull's-eye maculopathy - significa nt ly lower risk t ha n with chloro
quin e. Ris k of retin opat hy is low as long as daily dose does not exceed
400mg/day or 6. 5 mg/kg of body weig ht/day (1 ); a nyone weighin g less
t ha n 135 lbs is at increased risk for macul ar loxi cit y if they are ta king
the sta nd a rd 400mg/d ay dosage of plaqu enil (40). n'eatrnent duration
of less than 5 years and norm al renal function are al so importi:ll1t risk
factor s (1).

10.15

Topical Ocular Anesthetics

Topical anesthetics: Tetracaine, Proparacaine, Benoxinate


Ester anest.hetics with a n o nset of 10-20 seconds, whose effects last approxi
mately 10-20 minutes. Fluress so lution is a com binatio n of fluorescein and
benox ina te.

Inj ected anest hetics a re g ive n with epinephri ne so that local blood
vessels are constricted and systemic a bsorption is m inimal. T his
keeps the drug localized, allowing more potent effects (41) .

10.16

Agents for Exudative ARMD

Thi s rev iew foc uses on ly on age nts for ex ud at ive AR]\ID that are li sted on th e
NBEO ou t line (41).
Pegapt a nib (Ma cugen)
Antineop lasti c age nt t ha t decreases ang iogenesis by binding t o a nd inhi bit ing
the actions of vascula r endo theli a l growth factor (VEGP) . l\'Iac ugen is a dmin
istered as an in tra vitreal inj ect ion (41).

CHAPTER 10. PHARMACOLOGY

330
Ranibizumab (Lucentis)

rdonoclonal antibody (Fab por tio n) engineered from mouse an ti bodies that
targets VEGF (1). Lucentis is adm inistered as an intravitreal inj ection (41) .

VEGF is a group of protein s t hat prom ote vascu la r permeabili ty


and new blood vessel forma tion.

Verteporfin (Vis udyne )


Given IV and targets low-density lipoprotein receptors on newly forming vas
cular endothelium. Photodynami c therapy (PDT) uses a specifi c wavelength
of light to cause vasc ular occl usion of choroidal neovasculariza tion . After
treatments pat.ient s are ad vised to avoid sunligh t exposure or bright indoor
light for up to 5 days (1 ) (4 1). Vis udyne (vertepo rfin) is reco mmend ed for
treat ment of su bfoveal, predominantly class ic ch oroi dal neovasc ular membra nes
(CNVi\ I's) (68).

10.17

Toxicology

\Ve now introduce side effects of syste mic and topica l drugs, as they p erta in to
the eye (1, ch. 29,30). The following section is a n excerpt from the K MK Part
One Review Guide (41) .

Ocular Effects from Systemic Drug Administration


?vIany system ic dr ugs can affect the eye - it is important to consider the amoun t,
ro ute and frequency of t he drug as well as th e age, sex , and past history of the
patient. This re view fo cuses only 0 11 the most prominent exampl es . The best
resource for thi s section was the textbook Clinical Ocular Pharmacology by
Bar tlett a nd J aanus.
Drugs affecting the Cornea
Several different effects can occur fro m drug toxicity on the cornea - the most
common include (1, pp . 704):
\Nhorl keratopathy : Amiodarone, Chloroquine, Hydroxychloroquin e, Ta
mox ifen, Indomethacin

10.17. TOXICOLOGY

331

Superficial punctate kerati tis (SPK): Isotretinoin (Accutane)


Endothelial/Descemet's pigmentation: Chlorpromazine
Stromal gold deposits: Gold salts
Delayed healing: Corticosteroids

Anliodarone is an antiarrhythmic drug that can cause "whorl ker


atopathy," a visually insignificant toxicity of the epithelium. It
has also been proven to cause anterior subcapsular lens deposits
and non-arteritic ischemic optic neuropathy (1-2% of cases) (19).
Symptoms of glare, photophobia and colored rings around lights
(most common in Amiodarone) can occur from whorl keratopa
thy (1). Fabry disease is a lysosomal storage disease that can
result in cornea verticillata ("'whorl keratopathy") and spoke-like
lens opacities (34) (19).

Drugs affecting the Lens


The most common effects from drug toxicity on the lens include (1):
Anterior subcapsular effects: Chlorpromazine (stellate cararacts), Amio
darone (deposits), IvIiotics (vacuoles) and Gold salts (gold deposits)
Posterior subcapsular cataracts: Corticosteroids

PSC cataract formation from steroids is dose dependent and irre


versible. Hispanics appear to be at highest risk (1).

Drugs affecting the Conjunctiva and Lids


The most common drugs that cause toxicity of the conjunctiva and eyelids
include (1, pp. 712):
Isotretinoin (Accutane): Blepharoconjunctivitis, dryness, lid edema
NSAIDS: Subconjunctival hemorrhage
Sulfonamides: Ocular findings rare, include: Stevens-Johnson syndrome
and lid edema
Tetracycline: Pigmented cysts on conjunctiva
Sildenafil (Viagra): Subconjunctival hemorrhage, conjunctival hyperemia
Prostaglandin analogs: Increased growth and pigmenta tion

332

CHAPTER 10. PHARMACOLOGY

Drugs affecting the Lacrimal System


Recall that drugs that affect the auto nomi c nervous system can alter tear pro
d uction. Some of the drug cl asses that decrease tear secretion include (1,
pp.714) (19) (2):
Anticholinergics: Atropi ne, scopolamine
Antihistamines: Chlorpheniramine (Chlor- Trim eton) , bropheniramine (Di metane),
diphenhydramine (Benadry I)
Isotretinoin (Accu tane)
B-blockers : i\-iost co mmonly caused by timolol, atenolol , propranolol
Antidepressants: SSRI's (Fluoxetine), amitriptyline (Elavil), imipramine

(Tofranil)

Phenothiazines: Chlorpromazine (Thorazine), t hioridazine (Mellaril)


Hormone therapies: Oral contracepti ves, hormone replacement therapy
CNS st imulants: Methylphenidate (Ritalin), dextroamphetamine (Dexedrine)
Diuretics : Hyd rochlorothiazide (Hydrodiuril)
Drugs affecting Pupil Size
Drugs that can cause mydriasis (1, pp.718):
Anticholinergics: Atropine, sco polamine
Antihistamines: C hlorph eniramine (Chlor-Trimeton), bropheniram ine (Dimetane),
diphenhydramine (Benadryl)
Phenothiazines: Chlorpromazine (Thorazine) , thioridazine (i\/Iellaril)
Cl\TS stimul a nts: Cocaine, methylphenidate (Ritalin) , dextroamphetamine

(Dexedrine)

CNS depressants: Phenobarbital (Luminal) , Antianxiety drugs- diazepam

(Valium)

Drugs that can cause miosis (1, pp.718):


Opiates C'vlorphine, Heroine, Codeine)
Anticholinesterases (neostigmine)

Drugs 'Nith mydriasis side effects can contribute to an a ngle- clos ure
event in patients ,vith narrow anterior chamber a ngles. Anti
cholinesterase agents are found in toxic nerve gases a nd most
insecti cides - exposure can lead to miosis that lasts over a month in
duration (1); The drug Pralidoxime (Proto pam) can be given
intrave nously to counteract this toxicity.

10.17. TOXiCOLOGY

333

Drugs affecting Extraocular Muscles and Eye Movements


Below is a summary of drug-induced extraoc ul ar muscl e ab normaliti es (I,
pp.719 ):
Nys tagmus: Phenytoin (Dilantin) , phenobarbita l (Lum in al), sali cy lates
(NSAIDS)
Diplopia: Antidepressa nts, a nti a nxiet.y age nts
Smoot h pursuit impairment: Alcohol
Oculogyric crisis: Pbenothi azines, cetiri zine (Zyrtec)

Oculogyric crisis occurs vvhen t he extraocu la r muscles undergo


spastic , abnormal muscle contract io ns that leave the eye abnor
mally positioned (typica lly elevated ). Thi s condition occurs most
commonly with phenothiazine tox icity, but can also occ ur with ce
tirizine (1).

Drugs affecting the Sclera and Uvea


Below is a summary of the most co mmon effect.s from drug toxicity on the
sclera and uvea (I, pp.716):
Tamsulosin (Flomax): Intraoperative floppy iris syndrome
Uveitis: Cidofovir (Vistide), mycobutin (Rifabutin), sulfonamides
Blue sclera: Corticosteroid s, minocycline
Drugs affecting the Optic Nerve
The follow ing drugs ca n affect t. he opt ic nerve (I , pp. 736 ):
Digoxin: Retrobulbar op tic ne uritis, BI Y color defects , entopi c phe
nomenon ("snowy" vision, dimming vision, flickering lights)
Ethambutol: Opti c ne uri t is - ty pically retrobulbar and bilateral (29) (1).
Ch lora mphenicol, streptomycin and sulfonamides a re other a nt ibiot ics
th at ca n cause opt ic neuritis (1)
Iso ni azid / Met hotrexate: Unlikely culpri ts of optic neuritis (1)
Sildenafi l (V iag ra), s umat ripta n (lmitrex), amiodarone (Cordaro ne) : a ll
causes of NAIO N
Oral co ntraceptives: Effects are rare - opt.ic neuritis, pap illedema , pseu
dotumor cerebri

334

CHAPTER 10. PHAR1\1ACOLOGY

Drugs affecting the Retina


The following dr ugs can cause retinal toxi city (1, pp. 725):
Chloroquine: Bull's-eye mac ulopathy (much more common than with
hydroxychloroq uine)
Epineph rin e:
eyes)

Cystoid macular edema (topical epinephrine in aphakic

Tamoxifen : White or yellow crystalline deposits (commo nly in macula)


wit h or without macular edema
Thioridazine: Pigmentary retinopathy that can have appearance similar
to Bull's eye maculopathy
Indomet hacin: Retin a l he morrhage , pigmentary cha nges (especially in
macula)
Talc: Ret inopathy (w hite , shiny emboli wit.hin arterioles)
Isotretinoin (Accutane): Color vision Joss, nyctalopia (night blindness)
NSAIDS: Re t inal hemorrhage
Oral co ntracept ives: Vasculopathy - incl uding art erial and vein occl u
sions, retinal hemorrhage
Drugs affecting Intracranial Hypertension
Tetracyclines (minocycline and doxycycline) and isotretinoin (Accutane) can
cause pseudot.ul11or cerebri .
Drugs affecting Intraocular Press ure
Drugs t. hat decrease

rop

include:

Systemic B-blockers, cardi ac glycosides (digoxin), alcohol, cann abinoids.

Marijuana is a cannabinoi d derived from the pla nt Cannabis


sativa . The maximum effect of marijua na on rop occurs 60-90
minu tes afte r inha lat ion a nd lasts approximately 4 hours (1) .

Drugs that increase rop include co r ticosteroid s a nd t he following drugs wit h


anticho linergic activity:
Atropine and scopolamine
Antihistamines: bropheniramine (D imet ane) , diphenhydramine (Benadry l)

10.17. TOXICOLOGY

335

Tricyclic antid epressants: amitripty line (Elav il), imipramine (Tofranil)


Ant ipsychotic agents : phenothiazin es - Chlorpromazine (Thorazine) , thior
idazine (lvIellaril)

Corticosteroids can increase lOP by decreasing aqueous humor


ou t flow. Several mechanisms are believed to occur , including: de
creased ability for Tf'-I cells to replace matrix a nd ph agocy tose
debris a nd a thickening of t ra becula r fibril s and juxtacanali cular
tiss ue (1).

References
[1 ]

Bartlet t, Jimmy D. , J aanus , Siret D. Clini cal Ocul a r Pharmacology.


Boston: Butter worth , 2008 .

[2] Terry, J ack. Ocul a r Disease - detection , diagnosis, and treatment . Spring
field: Thoma s, 1984.
[3] Sugar SH. Pitfalls in the medical treatment of simple glau coma. Ann Oph
thalmol 1979; 11:1041-1050.
[4] Ha vener '-\TH. Ocul a r Pharma cology. St Louis : Mosby, 1978; C ll . 12
[5]

Bhusha n, Vikas , Le, Tao , Amin , Chirag. First Aid for the USMLE Step l.
New York: McGraw-Hill, 2003.

[6] http: //w ww.alco nlabs. com/us/ eo/ products/ OTCother/N aphconA .jhtml
[7] http: / /pedsccm. wustl. ed u/ All-Net /e nglish/ pharmpage/ resp/ cromoly n.html
[8] Ellis, Philip. Ocular Therapeuti cs and Pha rma cology, 5t h ed. St . Louis:
Mos by, 1977.
[9] Sigelman J. The clinical diagnosis of retinal drug toxicity. In: Srinivasan
RD , ed. 0(;11];11" therapeutics. New York: Masson, 1980; Ch. 17
[10] ht tp: //en .wikipedia .org/wiki/ Botulintoxin
[11] Epstein, David L. Cha ndler and Gra nt's Glaucoma, 4th ed. Baltimore:
'Villi ams and Wilkins, 1997.
[12] http://ardb.bjmu .edu.cn / detail/ ARDrug I nfodetail.aspDrugName

Dapiprazol e

336

CHAPTER 10. PHAR1'viACOLOGY

[13] Adler 's Physiology of the Eye, 10th ed . Kaufman, Paul. Aim. Albert. St.
Loui s: ]Vlosby, 2003
[14] Moortby: Ramana S. , Vall uri , Shailaja. Ocular toxicity associated witb
systemi c drug therapy. Current. Opinion in Opbthalmology 1999 , 10: 438
446.
[15] http: //en.wikiped ia.org/wiki/Cant.haxanthin
[16] http://annon c.oxfordjournals.org/ cgi/content / full/15/7 /102:)
[17] Harkins. Timothy. "Dr ug Therapy for Ocular Allergy. " Optometry Clinics
Vol. 2, Number 4, Ocular Pharmacology Update. Eds. John G. Classe.
Norwalk : Appleton an d Lange, 1994.
[18] http://www.rxlist.com/cgi/generic2/ trifluridine.htm
[19] Kanski , Jack. Clinical Ophthalmology 4th ed. Woburn: Butterworth and
Heinmann, 1999.
[20] Chodosh J, Dix R , Howell RC, Stoop \VG , Tseng SCG. Staining charac
teristics alJ d antiviral activities of sulfon hodamine Band Lissamine green.
Invest Ophthall1lol Vis Sci. 1994;35: 1046-58.
[21] Feenstra RBG , Tseng SCG. Compar ison of fluorescein and rose bengal
stai ning. Ophthalmology. 1992;110:984-93.
[22] Kim J, Foulks G. Evaluation of the effect of Lissamine green a nd Rose
Bengal on human cOflleal epithelial cells. Cornea. 1999;183:328-32.
[23] Freedma n, Sharon F. and a ll. "Effects of Ocular Carteolol and Timolol on
plasma high density lipoprotein cholesterol levels." American J ournal of
Ophthalmology. Nov, 1993; 116 ; 600-611.
[24] htt.p: / / www.emedicin e.com/ oph/topic85.htm.ParagAMajmudar . MD.
Allergic Conjunctivit is. June 30th, 2008.

[25] Herni'jndez !VI, Urcola JH , Vecino E. "Retinal ganglion cell neuropro


tection in a rat model of gla 11coma following brimonidille, latanoprost or
combilled treatments'; Exp Eye Res. 2008 )\'lay;86(5):798-806. Epub 2008
Mar 4.
[26] Quinn , Christopher. A Field Guide to Glaucoma Drugs. Rev iew of Op
tometry. 2003 July.
[27] The Eye Digest , Uni versity of Illinois Eye and Ear Infirmary, Chicago, IL.
2003. Agillg Eye and Ear Times.
[28] J G Lawrenso n1 , C Kelly. A L Lawrenson, J Birch. Acquired colour vi
sion deficiency in patients receiving digoxin maintenance therapy. British
Journal of Ophthalmology 2002 ;86 :1259-1261

Part III

Refractive Status/Sensory and

Oculomotor Processes

341

11

rOpla

Sarah

crn">l'rv

Wood, O.D., MS, F.A.A.O.

3cl3

11 .1. CASE 47

11.1

.345

Case 47

DerrlOgraphics
Age/ race / gender: 10 year old fem a le
Chief complaint: headaches when reading
History of present illness
Location: brow pain
Severity: 3/10
Nature of onset: gradual
Duration: persists during reading
Frequency: daily
Accompanying signs / symptoms: eyes get tired and holds print close
r-Iother notes child is doing poorly in school
Patient ocular history: unremarkable
Patient medical history: seizures
Medications taken by patient: dilantin
Patient allergy history: ativan
Clinical findings
Habitual Spectacle Rx
OD: no glasses, 20/20 (distance), 20/2.) (near)
OS: no glasses, 20/20 (distance), 20/25 (near)
Cover test
distance: ortho
near: small esophoria
Subjective refra ction
OD: +0.50 ds, 20/20 (distance), 20/25- (near)
OS: +0.50 ds, 20 / 20 (distance), 20/ 25- (near)
NRA / PRA: +350/ -150
Cycloplegic refraction:
OD: +2.00 ds
OS: +2.00 ds
Ocular hea lth: All findings wnl

.346

CHAPTER 11. AMETROPIA

Questions:
Question 1: This p a ti ent's asthe nopi a when reading is a result of
latent hy peropia. The +0.50ds on dry r e fraction r e presents what ?

a . latent hyperopia
b . mani fest hy pero pia
c. absolu te hy peropia
d . fa.cultative hyperopia
Que stion 2: In ge n e ral , whe r e is the far point o f a p a tient who we ars
+2.00 D glasses with a vertex dis t a nce of 1 5 mm?

a. 50 cm in fro nt o f t he corn ea
b. 50 cm behind t he retina
c. 48.5 cm behin d the re ti na
d . 48 .5 cm beh ind the cornea
e. at th e ret i na.
f. 3.5cm behind t he re ti na

g. infi nity
Question 3 : A h y p e rope st a tes tha t his single VISIOn glasses work
b e tter when h e pulls them frnthe r down on his nose for r eading .
This likely m eans he needs wha t change in h is g lasses powe r ?

a. increase in plus
b . decrease in plu s
c. decrease in cyli nd er power
d . change cylind er axis
e. no cha nge
Que stio n 4 : What prescription should the child in this ca se be give n
initi al ly ?

a. +2.00 ds O U for full-ti me wea r


b. +2.00 ds OU for reading only
c. + 0 ..5 0 ds OU fo r reading only
d. +0 .50 ds OU for di stance o nly
e. no glasses needed

f. +2.00 for d istance only

11.1. CASE-17

347

Question 5: What would be the most likely result of dynamic retinoscopy


on this patient?
a. excessive lag of accommodation
b. lead of accommodat ion
c. norm al lag of a ccommod ation
Question 6: " That of the following is NOT a reason to cycloplege a
patient?
a. suspi cion of laten t hyperopia
b. suspi cio n o f pseudo myopia
c. acco mmodat ive esophoria or esot ropia
d. variable retinoscopy or refrac t ion findings
e. vi sua l ac ui ty not at expected le vel
f. compu ter eye strai n in a pseudophake

g. sy mptoms do not seem to be related to manifest refraction

Answers:
Question 1: correct answer b - manifest hyperopia Clinicall y, late nt
hype ropia is diagnosed when a patient has a n increase in plus on cycloplegic
refraction compared to dry refractio n.
Definitions of types of hyperopia:
Manifes t hyperopia is the a moun t of by perop ia found on subjecti ve re
fraction
Latent hyperopia is the re ma ining hyperopia (2-0 .5= 1..5D) co mpensated
for by the to nicity of t he ciliary body.
Absolute hype ropia is the amou nt whi ch ca nnot be overcome by accom
modation.
Fac ul tati ve hyperopia is the amount that can be overcome by accom mo
dation. For example, if a patient has 4D of hyperopia a nd only 1D of
accommodation , their absolute hyperopia is 3D (4 minus 1) and facul ta
tIve hyperopia IS 1D (4) .
Questio n 2: correct answer d - 48.5 cm behind the cornea \"-Then a
pat ient has been optically corrected for a metropia, t he far poi nt of the eye will
coincide wit h the secondary focal point of t he co rr ecting lens. For hyperopes,
the far point is considered vi rtua l (beh ind t he eye), while for a myope, the far
poin t is real (in front of the eye: between the cornea a nd infinity).

348

CHAPTER 11. AIIiETROPIA

The far point for this patient is located at the secondary focal point of the
+2.00 lens. The fo ca l point of the lens is located at a distance of 1/2.00 m (or
50 cm) from the lens. Since the le ns sits 15 mm (or L5 cm) from the cornea,
the far point is at 50-1. 5 = 48.5 cm from the lens . Because this is a hype rope,
we know the fa r point is behind the eye. Specifically, the far point is 48.5 cm
behind the cornea.

Question 3: correct answer a - increase in plus Increasing ve rtex dis


tance will lea d to a.n increase in the effective plus power of the lens. Therefore ,
a patient who commonly pulls the g lasses farther from his eye could proba
bly benefit from glasses that have more plus power. Recall how a prescription
changes as ve rtex distance cha nges. If the power of t he lens needed at vertex
distance d 1 is F1 , then the power needed at vertex distance d 2 , where d 2 > d 1 ,
is F2 given by

(1l.l)
where L. is d2 - d 1 . This is just an a lgebraic way of saying that the far point
coincides with the secondary foca l point of the correcting lens. From this
equation, one can easily see that less plus power (or equivalently, more minus
power) is needed as the vertex di sta nce is in creased . Therefore, we can infer
that a lens sitting farther from the eye acts as though it has more plus power
than an identical lens sitting at the cornea .

Question 4: correct answer b - +2.00 ds OU for reading only This


patients chief complaint is reading . Therefore, the full cycloplegic refraction
would allow for a normal accommodative demand at near and will likely alle
viate the asthenopia symptoms.
Comment on the other choices:

+2 .00DS for full-time wea r wo uld no t be recommended because this


wo uld blur the child for di stance. The patient does not have a n ac
commodative esotropia. If she did , the full prescription may be required
or possibly a bifoca l.
+O.50DS for reading only is likely not eno ug h plus to a lleviate sy mpto ms.
+O .50DS for distance only wo uld be fine but near wou ld be lefL untreated
and she is not complaining about distance.
No glasses is not an option because the pat ient is symptomat ic a nd school
per form a nce is suffering, therefore, glasses should be prescribed.

+2 .00DS for distance only is a poor choice for reasons previously dis
cussed .
Over time the patient may accept more hyperopic correction at distance.

11.1 CASE 47
Question 5: correct answer a excessive lag of accommodation
nal11lC
measure of the accommodative
near.
estimation method), Bell
and the Nott's method

IvIE:-I is nDrtnrm
on a near target on the
and either with motion or

with their distance refraction and focused


used
motion are seen. The former represents a
used to neutralize but are

used to neutralize
lens will be needed for

>+0.50D.
with +025 to +050D

i':orrnal:

>+O.50D (a

Abnorrnal:

(4)

6: correct answer f computer eye strain in a


are indications for
except computer eye
who is an absolute
has had
have latent hyperopia, accommodative
or vari
able to accommodate.

What

..

for
anesthetic

refraction:

if > 1 year of age

if < 1 year of age


or tropicamide for

.. Perform refraction 30 minut,es later


.. Note: consider using only
for all increased reaction to
t hose with a low birth

350

11.2

CHAPTER 11. AMETROPIA

Case 48

Demographics
Age/ race/gender: 65 year old male
Chief complaint: no longer needs glasses for reading; glasses are two years
old and wants new pair
Secondary complaints/symptoms: some glare with night driving
Patient ocular history: mild non-prolierative diabetic retinopathy OU
Patient medical history: diabetic
IVledications taken by patient: metforlllin
Patient allergy history: sulfa
Clinical findings
Habitual Spectacle Rx and acuities

aD: pI ds, 20/ 80 (distance) , 20 / 30 (pinhole)

as: pI ci s, 20 / 80 (distance ), 20/30 (pinhole)

+250 add

Subj ective refraction

aD: -2.00 ds, 20/ 30 (d istance)

as: -2.00 ds , 20 / 30 (distance)

Slit lamp

lids/ lashes /adnexa: papillom a lower lid OD , cl OU

conjunctiva: cl OU

cornea: cl 0 U

anterior chamber: deep and quiet OU

iris: wnlOU

lens: mod ns and cortical cataracts OU

vitreous: cl OU

All internal findings wnl au

11.2. CASE 48

351

Questions:
Question 1: Which of the following could be a cause of this patient's
myopic shift? (more than one answer can be correct)
a . nu clear scleroti c cat aracts
b. cort ical cataracts
c. inherited from parent (s)
d. blood sugar spike
e. loss of accommocla tion due to presbyopia
f. noct urnal myopi a

g. pseudo myopia
Question 2: The pinhole acuities got the patient from 20/80 to 20 / 30.
The fact that pinhole acuities did not allow the patient to see 20/ 20
is an indication that the distance visual acuity decrease is due to:
a. myopia
b. media opacity (cornea , lens, vitreous) or a n ocular pat hology
c. regular ast ig matism
d. a n unknow n factor but the endpoint of refractio n is expected to be 20/20
e. irreg ul ar astigm atism

Question 3: Without any glasses and assuming absolute presbyopia,


what distance would this patient see most clearly?
a. 20cm

b. 40cm
c. 50C111

d. infinity
Question 4: What would be the best way to correct a high myope
with an axial length of 26mm to minimize relative spectacle magni
fication (RSM) ?
a . glasses
b. conta ct len ses
c. t hey would be t he same

CHAPTER 11. AilIETROPIA

352

Question 5: Which of the following is NOT a finding attributable to


myopic degeneration / p a thologic myopia?
a. angioid streaks
b. post erior stap hy lom a
c. lacquer cracks
d. Fuch's spots
e. obliquely inserted (t il ted ) optic discs

f. latt ice
g. glaucoma
h. retinal detachments
I.

CNVIVI (choroidal neovascula r membr anes)

J. peripapill ary crescents

Ie prema ture cata racts

Answers:
Question 1: correct answer a and d Nucl ear sclerotic cataracts can cause
a myopic shift, even be for e they cause a decrease in the best corrected acuity.
Cortical cataracts, on t he other hand, te nd to cause a hyperopi c shi ft. High
blood sugars level in diabetics can ca use myopic shi fts, as well. These s hift.s tend
to be sym metri c between the two eyes. A pati ent ca nnot have a hyperg lycemic
ind uced myopic shi ft if they are a pseudophake .
Comn1ent on the incorrect answers:
Inherite d myopia J'v'I yopia can be inheri ted from parents . This patient's my
opia was not inherited because it developed late in life.

Below are the ri sks of nlyopia deve lopment in chil dren based on the
presence of parental myopia:
No pare nt : < 10%
One pa rent : 20-25%
Both pa rents: 30-40% (3)

Nocturnal or night myopia occurs in the dark or dimly illuminated sit u


at ions due to t he presence of minimal visual cues to g uide the a mount
o f accommod at ion needed. This can especiall y be a problem with night
driving. Distance vision requires no accommodation bu t wit h minimal

11.2.

CASE <48

3.53

visual cues, accommodation may be suspended at an in te rmedia t.e dis


tan ce causing the eye to be too power ful, thus requiring a more myopi c
prescription (4).
This patient is an absolute presbyope with no accommodation to induce
nocturnal myop ia. Although , this patient could potentially be v ulnerabl e
because nocturna l myopi a can also be induced by increase in spherical
aber rations due to dilated pupils in low ligh t leve ls.
Pseudomyopia occurs when an uncorrected hyperope accommodates to a llow
th emsel ves to see clearly bu t. then acco mmodates too much a nd shifts
t hem into myopia. A dry refraction will y ield a low amo unt of myo pi a
bu t cycloplegic refraction will reveal the uncorrected hyperopia. The
low myopic prescription is not need ed and will actually exacerbate the
problem. A low plus prescription is needed for near only.

Other things which can cause a myopic shift:


scleral buckle s urgery following reti nal detachment
certain medi cat ions such as Diamox and NSAIDS (typ ically transient)

Question 2: correct answer b - media opacity or ocular pathology A


pinhole increases the depth of focus a nd field. The in crease in depth of focus
will im prove the ac uity if the red uction is due to refractive error . This means
with refract.ion shou ld allow the patient to t heoretically arrive at the pinhole
acui ty level. P inhole acu ity measures will be limi ted by med ia opacities s uch as
corneal opaciti es or cataracts or ot her ocular disease (like maculopat hy). This
patient was a ble to be refracted to his pinhole acuity. Due the fact that all
other oc ula r health findings we re within normal limits, it is assu med the ac uity
drop is entirely due to hi s cata racts.

Question 3: correct answer c - 50 cm The range of clear vision extends


from the near poi nt to the far point. In this case, the far point is located in
fron t of the eye at a distance of 2 D, 'vI'hich corresponds t.o 1/2 111 , or .50 cm. A
point object placed at the far point will form a point image on the re ti na .
Th e near point correspond s to t he location at which a point obj ect could be
placed such t hat a point image is formed on t he re ti na, given that the eye is
fully accommodated. In the case o f a complete presbyope, the patient ca nnot
accommodate, so t he near point coincides with t he far point . Hen ce , the range
of clear vi sio n is a single poin t, which is lo cated at .50 cm from the eye.

CHAPTER 1.1. MIiETROPIA

354

The far point is lo cated:


In front of the eye in a myope
Behind the eye in a hyperope
At infini ty for a n em met rope

Question 4: correct answer a - glasses The so urce of the myopi a in


t. his case ca n be assumed to be prima ril y ax ial; recall that the axial lengt h
in Gullstrand 's exact eye is 24 111111. Spectacle magnificat ion considerations
d ictate that a n axial ametrope is best corrected wi th spectacles . Specifically,
Knapp's Law says t hat the relati ve spectacle magnification (RSIVI) for a re
fract ive ametrope is near unity when the lens is placed at the ent ra nce pupil
of the eye (practically speaking, very near the cornea), Vvhile RSj\I for an axial
ametrope is near unity when a lens is placed at the a nterior foca l point of the
eye (abo ut 15 mm from the cornea).
R e lative Spectacle IVlagnification: RSl\I is t he rat io of the ret inal image
size in the corrected eye to the retinal im age size in Gullstrand's em
metropic eye. The goal is often to keep RSi\I~ 1.

Knapp's law, which is based on relative spectacle magnification


considerations, yields t he following ru les :
Ax ial ametropia H glasses
Refractive ametropia

contacts

Question 5: correct answer a - angioid streaks Angio id st reaks a re


simil a r to lacq uer cracks in that they both represent weaknesses in Bruch's
membrane whi ch make the eye suscept ible to CNVM. Angioid streaks have
differen t etiology (see retina section). Answers b-k can be seen in pathologic
myopia. Pathologic myopic is defined as an axial length greater than 32 .5mm
",.. here high myopia is defined greater than 6 diopters and/or a n axial lengt h
great er than 26.5mm. (1)

11.3

Case 49

Demographics

11.3. CASE

~19

Age/race/gender: 8 year old female


Chief complaint: failed school screening, patiellt does not complain of any
symptoms, mot.her not aware of any problems
Patient ocular history: 1st eve exam
Family ocular history
mother: myopia

father: hyperopia

Patient medical history:


T\1edications taken by patient: none
Patient allergy history: seasonal
Family medical history:
nl0ther: depression

father: alcoholism

Clinical findings
Habitual Spectacle Rx

aD: none, 20/80 (distance)

as:

nOlle, 20/80

Pupils: PERRLA, no apd OU

EOlVIs: full OU

Cover test

distance: 2 exophoria

near: 4 exophoria

Confrontation fields: full 1.0 finger count OU

Keratometry

aD: 40.00 at 180, 4500 at 090

as:

40.25 at 180, 45.2.5 at 090

Subjective refraction
aD: 12 00-5.00x090, 20/40

as:

pt would not cooperate for retinoscopy or refraction for the os

All internals and slit lamp findings wnl

au

356

CHAPTER 11. A]11ETROPIA

Questions:
of astigmatism is present in the

1: What

eye?

a.

b.
c. mixed
d.

e. with-the-rule

f.

Question 2: Which type of


in
would blur the
most to least)?

when uncorrected and


the most
in order from

a.
b.

WTR

c.

ATR

d. ATR.

WTR

Question 3: Based on the


much
would be
eye?
-5.00
b.

version of Javal's
in the

x 090

D x 090

c. -5.00 D

180

d. -5.:50 D

090

amount of cylinder in the


from the OD refraction is cut to allow for
What should the
be if the
+2.00-3.50
b.

090

.25-:3.50 x 090

c. +350-3.50 x 090

d.

090

e. +2.

090

11..3. CASE 49
5: In
fraction for the
would the

re
\Vhat

rn",'rI'l1){l.I.

form.
read?

a. -2.00 + 5.00

090

b. +200 + 500

.00 + 5.00 x 090


d. -:300 + 5.00 x 180
e. -0.50

5.00 x 180

f. -300 + 500

090

u."","'UH 6: For the


left eye, a lens rack was used for re
fraction because the
was no
Retinoscopy
OD +3.00 in the 180 meridian (streak oriented vertically),
OS -2.00 in the 90 meridian
oriented horizontally). What is
the prescription in minus cylinder form
the working distance
has already been subtracted out)?
a. +3.00 - 5.00 x 090

h. +3.00

5.00 x 180

c. +3.00

.00 x 180

d. -2.00

5.00 x 180

-2.00 - 5.00

090

+300 - 2.00 x 180


Question 7: Which of the
cylinder lens?

a. + LOO - 250

of a Jackson cross

is an

090

h. +100-lOOx090
- 2.50 x 090
1.00

d.

2.00 x 090

e. +1.00

0.50

090

f. +0.50

0.50

090
is NOT a

8: Which of the

a. III

trend of

tends towards

b.

+0.50 and +1
age 13-14

has the
c. If a child

<YtJlrl""'"

at age

will

remain

age 5-6

CHAPTER 11

358
d. If

child has two

H.efractive

III

with

AMETROPIA

90% chance of
stable

the ages of 30 and 40

Answers:
1: correct aJ:1S\Ver c - mixed IVlixed
is in front
the retina
one is behind.

when

rays enLer
surface with the maximum
The

and
eye. This

line is on the retina and the other is


or behind the retina

retina. The pOlVers in

both have the

and least refractive power


Keratoconus is a an
of a condition

With the rule

the rule: axis of the

in minus

in

no-

is 90

30
in minus

Question 2: correct aJ:1swer b Uncorrected


followed
and then \VTB.. refer to

blurs the

11.3. CASE 49

359

Question 3: correct answer d - (-5.50 x 090) Javal's Rule can be used


to predict the amount of astigmatism anticipated in the glasses prescription
based on the keratometry readings. Javal's rule is an empirical estimate of the
total astigmatism based on the amount of corneal astigmatism.

Javal's rule is given by


ARx = Ac

+ 0.50 ATR astigmatism

(11.2)

\t\ihere ARx is the refractive astigmatism, Ac is the astigmatism measured by


keratometry. Javal's (simple) rule merely says that the total astigmatism will
be the corneal astigmatism plus an additional 0.50 cliopters of against the rule
astigmatism.
For this patient, the K readings were 40.00 at 180, 45.00 at 090. This represents
.5.00 D of against the rule astigmatism. Hence, Javal's rule predicts that the
total astigmatism in this eye will be 5.50 D of ATR astigmatism. In terms of
prescription, this is -5.50 D x 090.

K-readings with more power in the 090 meridian correspond to


ATR corneal astigmatism. K-readings with more power in the 180
meridian correspond to VlTR corneal astigmatism.

Question 4: answer b - (+1.25 - 3.50 x 090) If 1.5D is cut from the cylin
drical part of the prescription, the sphere power must be adjusted to maintain
the spherical equivalent to assure that the circle of least confusion (COL C) falls
on the retina. The COLC is located dioptrically between the two line images
formed by the principal meridians.

In general, the spherical equivalent is given by:

(11.3)
where Se is spherical equivalent. C is cylinder, and S is the sphere
power. This is an algebraic way of saying that the spherical equiva
lent power is the average of the power in the two principal meridians
of the lens.

CHAPTER 11. AAIETROPIA

360

In the original prescription, the spherical equivalent is -0. 50 D. We now will


reduce the cyl power to -3.50 D. To maintain the spherical equivalent, the new
sphere power,S, would therefore be given lJY

350
-0.50 = -_.- + S.
2

(11.4)

So the new sphere power would be +l.25 D. Hence, the final presc ription is
+ l.25 - 350 x 090 .
Question 5: correct answer d One can draw a power cross and easily
convert between plus and minus (yl notation. The power cross for the or iginal
lens shows that the powers are -3 .00 (180) and +2.00 (90). One can write this
two ways. The fir st is to start with the most positive number as t he sphere,
which y ields the minus cylinder notation: +2.00 - 5.00 x 90.
On the other hand , we can also describe the power cross by using the most
negative number as the sphere. In this case, the power cross can be described
by -3.00 + 5.00 x 180. In both cases, the cy linder axis (by definition) gives the
"difference" in the powers in the principal meridians.
Question 6: correct answer b - (+3.00 - 5.00 x 180) A power cross
wou ld have +3.00 D on the 180 meridian a nd -2.00 D on the 090 meridian. We
can describe th is power cross in minus cylinder notation by choosing the most
positive number as the sphere power: +3. 00 - 5.00 x 180.
Question 7: correct answer d - (+1.00 - 2.00 x 090) The power cross
for a Jackso n cross cylinder will have principa l meridians whose powers are
equa l in magnitude but opposite in sign . The result is a spherical equivalent of
o. JCC lenses a re used in the determination of power and axis in those with
astigmatism. JCC lenses are in the phoropter or are hand held.
Question 8: correct answer d The chance of myo pia with two myopic
pa rents is 30-40% (3). Also see Section 1l.2 for more details on myo pia. Refer
to (6) for more informa tion related to t his case.

11.4

Case 50

Demographics
Age/race/gender: 43 year old Caucasian male, electrician
Chief complaint: ar ms a re too short

361

11.4. CASE50

History of

illness
has to hold material further away to

read
mild

Nature of onset:
Duration: the last

months
when tired
is noticed while work-

or function:
Patient ocular

OD

ocular
mother:

father: ca Laracts

Patient medical

unremarkable

Medications taken by
Clinical

UdvHc'Uv

none

HULlU!"-,,

Habitual

OD: +100ds

OS: +lOOds

Cover test

distance:

near: small

eA'up"U1

count 0

Confrontation fields: full to


NRA: +:350
PRA:

np'rtnrnlP(i

a(, 40 em

distance

1..50

Slit

ou
cornea: arcus OU, nasal
anterior chamber:
iris: wnl OU

lens:
OU

vitreous:
OU

Internals: all wlli OU

OD

OU

362

CHA PTER 11. AMETROPIA

Questions:
Question 1: Based on the NRA/PRA findings, what add would likely
be prescribed for this patient for a standard working distance of
40cm?
a.. +250

b. +100
c. +150
d. +200
e. no add is need ed

Question 2: This patient is an electrician and needs to see fine detail


at the working distance of 25 cm. The amplitude of accommodation
is measured to be 4.00 D. What add should be prescribed for this
specific task?
a. no add is needed
b. +2.50

c. + 1.00
d. +200
e.

+4.00

Question 3: Which of the following does NOT occur during accom


modation?
a. the lens is in its thinnest form
b. the ciliary muscle con tracts
c. tension is released on zonula r fibers
d. the anterior s urface of the lens moves forward
e. the pupil constricts

Question 4: Which patie nt would likely be the first to have presby


opic symptoms (assuming all are 42 years old)?
a. uncorrected -1.00 myope
b. uncorrected

+ 1.00

hyperope

c. em metrope with a working distance of 50 cm for reading


d. patient with 2 mm pupils
e. ampli tude of acco mmod atio n of 7.00 D

11.4. CASE 50

363

5: A DIFFERENT
add should
be nr""'"rr.

20 cm. \Vhat

a. +250
b. +1.00
no add needed
d. +500

Answers:
correct answer b -

+1.00
shown plus

relative
is blurred

until near
lenses until

which will allow

For this

for an
The
half of the accommodative
to balance the +3.50 NRA and the -1.50
2.
result is a +1.00 add.

add them

Another method for add determination:


Fused (binocular) cross cylinder method can
add but
assess the accOlllmodative
III a
test is done at 40C111 with the
lens at axis 90
power dialed in) over
both eyes
tentative add for
to each other.

on age, the

of accommodation. In a
levels of
This represents the add to be

correct answer d - 2.00 D The demand of


25cm
of
distance. Half of the accommodative
to
comfortable while
the task.
in reserve,
to be used. A n additional 2D needed in the add to reach the 4D demand.

CHAPTER 11. Ai'viETROPIA

36-1

Accommodati ve deman d (i n diopters) is equaJ to the reciproca l of


wo rking distance (in meters)

Ques tion 3: correct answer a - le n s is thinnest Duri ng non-accommodated


states , tension is on the zonules which Aattens t he lens . During acco mmoda
tion, t he ciliary body cont racts whic h releases the zonular tension on the lens.
This allows for the lens to become thicker , more highly curved a nd more pi us
powered (4).

Near point triad


vergence

pupill a ry constriction, accommodati on and con

Question 4: correct answer b - uncorrected 1.00 D h y p erope An

uncorrected hyperope has to accom modate to clear t heir visio n for dista nce. At

near, they have to accommodate t he norma l amount for their working distance

plus t he add itiona l amount for thei r dista nce vision. This patient is likely to

reach a state of asthenopia at a soon er t ime than the others.

The ot her answers:

An uncorrected myope will be able to avoid presbyopic symptoms longer than

would be expected for their age because their far point is closer than infini ty

(this patient 's is 100cm).

In creased working di stance req uires less accommodati ve demand. A \vorking

di stance of 50 em requires 2.00 D instead of t he norma l 2.50 D for 40 cm.

Sma ll pupils increase the depth of focus.

T he expected amplitude of accommodation (Ao A) is given approx


imately by
A = 18.5 - O.3y ,
(1l.5)
where A is AoA and y is age in years. (7)

11.5. CASE

365
of accommodation of
old has

should be

to

Question 5: correct answer c - no add needed A llcar


the
be seen
accommodated state.
the
can accommodate
D (or
2.50 D of accommodation. This

for this

was used

Please

to it for further

Additional information:
The loss of accommodative
lens.
of

loss of elastic-

t.o

symptoms,

in

Functional
func
medi

tional
etc.
Absolute- no accommodative

11.5

remains

51

20 year old

5t udent

366

CHAPTER 11. AMETROPIA

Chief complaint: distance blur


History of present illness
Location: OU
Severity: mild
Nature of onset: grad ual
Duration: a few mont hs
Relationship to activity or function: notices while trying to see t he
board in class
Secondary complaints/symptoms: eye lid twitch OS
Patient ocular history: OS refractive a mblyopia, patching OD as a child
Family ocular history
mother: amblyopi a
father: does no t know fat her
Patient medical history: amenorrhea
Medications taken by patient: vi t a mins
Clinical findings
Habitual Spectacle Rx

OD: +1.50 DS, 20/25 (distance)

OS: + 350 DS , 20/ 50 (d istance)


EOMs: full OU
Cover test
distance: small exophoria
near: moderate exophoria
Confrontation fields: full to finger count O'C
Subjective refraction
OD: +2 .00 DS, 20/20 (distance)
OS: +4 00 DS , 20/ 40 (dist ance)
AoA: 10 D
ocular health: all findings wnl

11.5. CASE 51

367

Questions:
Question 1: If this patient's retinoscopy findings for the right eye
were +3.50-0.50x090 done at 66 cm. What is the prescription after
the working distance is removed?
a. + 1.50-0. 50x090
b. + 2.00-0. 50x090

c. +6 .00-0.50x090
d. + 6.50-0.50x090

Question 2: The Duochrome test utilizes what type of optical aber


ration?
a. spherical
b. coma
c. chromatic
d. curvat ure of image
e. distortion
Question 3: On the monocular Duochrome test, the patient notes the
red side is more clear. The refraction was +2.00 DS up to this point.
Their final spherical power is most likely which of the following?
a..

+2.50

b. + 2.00

c. +1.50
Question 4: This patient would not be a candidate for binocular
balance. Why not?
a. age

b. differen t ac uities between t he two eyes


c. hyperopia
d. exophoria
e. amount of accommoda tion

Answers:
Question 1: correct answer b Static ret inoscopy is typically done a t 50
cm or 66 cm. Th e reciprocal of the dist.ance is t he amo unt of dioptric powe r
that should be removed after the refl ex is neutralized. In thi s case, we ha ve
1/(0.66 m) = 1. 50 D, therefore +2 .00-0.50x 090 wo uld be the starting point for
th e subjective refraction.

CHAPTER 11. AJlIETROPIA

368

Question 2: correct answer c - chromatic The Duochrome (or Bichrome)


test is done at the end of the refract ion to refi ne the sphere monoc ul arl y. Th e
ac ui ty cha rt is split half red and ha lf green. The patient notes which sid e
appea rs sha rper. Due to chromat ic a berration, the green side focuses before
(t hat is, closer to the front of t he eye t ha n) t he red side. The goal is to have
the retina fall half way between the two targets . If it does, the targets will
a ppear equally clear. If t he patient is ove rminused or overplused, one side will
a ppear clearer because it is closer to the retina.

RAlvI-GAP: red add minus, gree n add plus

This test can also be used as a binoc ula r balance technique if the acuities
a re t he same betwee n the two eyes. The same chart is used a nd t he eyes are
dissociated with prism. T he pa tient is fogged and asked to compare the two
cha rts. The spherical power is modified unLil Lhe two charts look equal a nd the
red and green sides of each cha rt a re equal in clarity.
Binocular balance is typica lly done by prism dissociation and the patient com
pares the two images. The end point is when they appear t he same in clar ity.
Binocular halance ca.n also be don e with the Turville Infinity Bala nce technique.
A se ptum is used so that the right sid e of the chart is seen with the right eye
and the left side with the left eye but the peripheral part of the chart is see n
with bot.h. The refraction is done binoc ularly which allows for a more nat ural
environment for t he patient (4) .
Question 3: correct answer c - +1.50 As disc ussed in t he previo us ques
t ion , the patient is overplussed when t he red stimulus is clear ("RAM : red add
minus") .
Question 4: correct answer - b
Binocula r balance equalizes the accom
modat.ion between the two eyes . The pa.tient must have eqmt! acuities between
the two eyes for this to be possible. This patient is 20/20 OD a nd 20/40 OS
due to amblyopia. The prism dissociated bi-ocular bala nce should be used in
this case because it does not require the two eyes to be compared.

References
[1 ] Friedma n N, P in eda R, Kaiser P. The IVlassachuseLts Eye and Ear In
firmary Illu strated Ma nual of Ophthalmology. W .E. Saunders Company,
1998.

o
Sarah

12

halmic Optics

MS, F.A.A.O.

371

12.1. CASE 52

373

Important prism definitions and concepts

One prism diopter- At a distance of one meter from the lens, light will be

displaced by one centimeter. The light will be bent towards the base of the

lens.

A prism lens is described by the power in prism diopters and the base direction

of the lens.

Through a prism, light is bent towards th e ba se but the object viewed through

the prism will appear to move towards the apex of the prism. Therefore, to

correct a deviation, point the apex of the prism towards the deviation (example:

exotropia is corrected with base-in pri sm).

12.1

Case 52

Demographics
Age/race/gender: 50 year old female
Chief complaint: sees doubl e with new glasses
Clinical findings
New spectacle Rx
OD: -2. 50 DS
OS: -2 .00 DS
Add: +2.00 , flat top 28 bifocal design
Frame PD: 56mm
Subjective refraction
OD: -2.50 DS
OS: -2.00 DS
Monocular patient PD: 28/28 (di sta nce) , 26/26 (near)

Questions:
Question 1: The frame PD matches the pupillary distance of the
patient. What would be the amount and direction of prism if the
fram e PD in the new glasses was actually 60mm?

D BO prism
b. 0.9 D BO prism
a . 9.00

c. 0.9 D BI prism
d. 9 D BI prism

374

CHAPTER

or

OPHTHilLAiIC OPTICS

would
and what ranges
would need to be

a.
b.

fusional

fusional ranges

d. convergence,

fusional ranges

3: What is the
her bifocal?

when this

Uctt,H:)UL

looks into

base up 0'[

base down OU

b. 1

] .25

c.

up

d.

down OS. 1.25

base up OD
down OD

4: This same
out her old iSH""''''''''''
notes worked
fine but the frame is broken. With
are mea<;ured to have
The OD has 3BO and
2BU and the OS 3BO and 3BU. vVhat is the total amount of binoc
ular
in these
U'C'.yv'VU

a. GEO and oEU

b. GEO

OD

lEU over OS

6 BI and lEU over OS

d. 6 EO and lEU over OD


e.

over OS

this patient's ocular


maddox
is done with the red lens over the right eye. To test
the rod is held with the
the horizontal
the
has the
horizontal deviation noted in
to the (right/left) of the
source.
truly has the vertical deviation noted in
horizontal line
of the

d.

12.1. CASE 52

375

e. straight through/below
Question 6: While the new lenses are being ground, fresnel prism is
placed over the left eye to compensate for the ocular misalignment.
What power and orientation of the base are needed?
a. 6.08 prism diopters base 9.46 degrees
b. 12.04 prism diopters base 4.7 degrees
c. 7.8 prism diopters base 39.8
d. 6.08 prism diopters base 350.54 degrees

Answers:
Question 1: correct answer c - 0.9 BI prism Prentice's Rule is used to
determine the prism induced by viewing through a lens at a location other than
the optical center (OC). The further from t he OC, the more prism induced. The
difference in t he patient PD and the fram e PD causes the patient to be looking
through a point away from the optical center; t herefore, prism is induced and
could potentially be a cause of poor adaption to the new glasses.

Prentice's rule is
(12.1 )
where t:,. is the induced prism, cL is the distance from t he optical
center (in cm), alld F is the power of the lens. The direction of the
prism can be inferred from the lens position and lens po-wer (see
below).

If the glasses were made incorrect with the distance PD is too wide by 2 mm
(or 0.2 cm), the prism induced over the OD would be = 0.2 x 2.50 = 0.50"'. III
this case, we have a minus lens decentered outward, so the apex of t he prism
is out. Hence, the prism is base in.

For a minus le ns, the riecentering directio n corresponds to the apex


direction of the ind uced prism. The opposi te is true for pi us lenses.
It is easy to remember this by thinking of a plus lens as two prisms
stacked base to base and a minus lens as two prisms stacked apex
to apex . It is often easie r to use th is rule than to memorize a sign
convention for prisms.

:376

CHAPTER 12

For the OS
to BI

OPHTHAU\IIC OPTICS

0.2 x 2.00
, which
decentered outward.

corre

Since there is BI prism before each


the total binocular
the
powers, This
total of O,g'" BI.
also

the sum of

2: correct answer a In this


induced
fusional ranges
comfortable vision.

Positive fusional vergence range: tested


with BO
or
with plus lenses, Plus lenses induce an
and in
increftsc, This range
BI

fusional vergence range:


with
lenses,
accommodative convergence and in order to maintain
NFV
to
This range

Clinical Note: BI ranges should be measured


BO, Instruct
to
the target clear to
accommodation constant,

au

base down
bifocal line due to the
the induced

its
and the
of the
L\ = 0.5 x 2,00

center
jump induced at the bifocal line
results from
above
down.

the distance between the


center
the seg
mm.
inci,de the executive or Fmnklin seg
seg
and Ribbon
is 7

377

12.1. CASE 52
Question 4: correct a nswer b - 6 EO and 1 EU over OS

Combining Prism for each eye to determine total binocular


prism:
Horizontal: If pri sms over the right and left eye are both
BO or Bl, add them together. If t hey a re opposite, subt ract
the m .
Vertical: If prisms over t he right a nd Jeft eye a re are both
in t he sa me direct ion , s ubt rac t them. If t hey a re in opposi te
directions, add the m together.

Question 5: correct answer b - right/below In this pat ient ne utralized


with BO prism over both eyes and BU over the left eye, she has esotrop ia
and a right hyper tropia. The lvlad dox Rod can measure the di rection a nd
amount of deviat io n (but cannot distingu ish a tropia from a phoria). Recall
a uncrossed de viatio n is caused by an esodeviatio n a nd a crossed dev iation by
a n exodev iatio n. If the patient holds the maddox rod ove r the right eye and
views a poi nt light so urce, the eso tropia will cause the pat ie nt to p erceive the
red line to the right of the light (cso-uncrossed-r ight eye wi ll see the li ght to t he
rig ht- not crossing the midline) . For t he vertica l co mponent, the line will pass
below t he light in a righ t hyper deviation and abo ve for a left hy per deviation.
Th erefore, this patient will see the lin e below the light in this case.
Question 6: correct answer a The pri sm for t he left eye would be 6 BO
a nd 1 B U (see Fig ure 12 .1). One can draw this as a right triangle wit h sides of
leng th 6 and 1 a nd a hy potenuse whi ch represe nts t he resultant prism. Using
the pythagorean t heorem, the hypo tenuse is calculated to be 6.08 (square root
of 36+1).

The magn itude of the prism t hat results from combining two prisms
V (ver t ica l) a nd H (horizontal) is given by
(1 2.2)
The prisll1 directioll

e ,Nill be g iven

by

V
tane = H

(12.3)

CHA.PTER 12. OPHTHALMIC OPTICS

378

6.08 resultant prism


1 base up

6 base out
nose

Left Eye

Figure 12.1: The result of combining of two prisms (6BO and 1BU) over the
left eye

The base orientation is given by tan- 1 (1 /6) = 9.46. Therefore, we require a


6.08 prism diopter Fresnel prism wit h the base 9.46 degrees from horizontal
(going counter-clock wise).

12.2

Case 53

D emographics
Age/race/gend er: 5.5 year old male, iron a nd steel worker
Chief complaint : Rx check: ne,v glasses do not seem right
Clinical findings
Habitual Spectacle Rx

OD (ordered ) : -6.00 - 0.50 x 03.5

OS (ordered) : -6.50 - 2.00 x 048

OD (actual): -6.12 - 0.50 x 039

OS (actual): - 6.50 - 2.00 x 044

Add: lined bifocal, +2. 50

Pantoscopic tilt: 18 degrees

Seg height (ordered): 16 mm

Seg height (actual): 19 mm

12.2. CA.SE 53

:379

Questions:
Question 1: The excessive pantoscopic tilt induces cylinder at which
aXIS (when written in minus cylinder form) for this patient'?
a. 90

b. ,15
c. 3,5

d. 180
e. nOlle of the above

f. no cylinder is induced
Question 2: Which of the following is true regarding pantoscopic
tilt?
a. As the pant.oscopic
b.

t.he lens distort.ions will likely decrease

tilt is helpful for progressive addition lens wearers

c. The best way to induce pantoscopic tilt in a frame is to adjust the


nosepads
d. Pantoscopic tilt refers to the top of the lens being closer to the face than
the bottom of the lens
e. Pantoscopic tilt is required if the eyes are below the optical center of the
lens

Question 3: Do the new glasses meet ANSI standards for refractive


power tolerances for the sphere, cylinder and axis? If not, which
component is wrong'?
a. Yes

b.

sphere

c. 1\'0. cylinder amount

d. No, axis OD
e. No, axis OS

Question 4: This patient's segment height is too high in the new


glasses, What frame adjustment could be made to lower the segment
height?
a. bring the nose pads closer together
b. decrease the pantoscopic tiJt

c. add more padding to the nosepad


d. widen the

if

Imrne

e. lower vertical portion of adjustable nosepads

CHA.PTER 12. OPHTHA.LMIC OPTICS

380

Question 5: Due to the patient 's high amount of myopi a, which fram e
or lens selection should be avoided?
a. as pheric lenses
b. antirefiect ive coati ng
c. sm all eyesize
d . crown glass
e. rounded corn ers
Question 6: What special consideration would b e MOST appropriate
for this patient's glasses/ lenses due to his occupation?
a. transitions
b. ant i-reflective coating
c. safety lenses
d. crow n glass material
e. double-D bifocal

Answe rs:
Question 1: correct answe r d - 180 \Ve can understand t his by remem
bering t hat tilting a spherical lens of power F by an a ngle induces a new
sphere power Fs a nd a new cy l power Fe.

Fs = F (l

+ sin

(J ),

2n
Fe = F(tan 2 (J) ,

(12. 4)

whe re n is the index of refraction of the lens. The axis of the equi valent
(untitted) spb erocy lindri cal le ns will coincide with t he me ridian of rotation.
For pantoscopic tilt , the me riclian of rotation is 180, while for face form tilt the
meridian of rota tion is 90.
Notice t hat if the original lens power is negati ve, t he a bove eq uat io ns wi ll give
the sphere and t he cy linder power of t he effective lens writte n in minus cylinder
form. However, if o ne is tilting a plus lens, t he answer ,,,ill conta.in a positive
value for Fe, meaning it is give n in plus cylinder form and must be conver ted.

In optometry (minus cylind er) notat ion, pantoscopic tilt o f a minus


le ns induce::> an axis at 180. P antoscapic ti lt of a plus le ns induces
a n axis at 90. Faceform t il t o f a minus le ns induces a n axis at 90.
Faceform ti lt of a plus le ns induces an axis at 180.

12. 2. CASE 53

381

Question 2: correct answer b Progress ive wearers benefit from panto


scopic tilt because it moves the read ing portion of the lens closer to the eye.
The result is a n increase in the reading width of the lens.
Explanation of incorrect a nswers:
Pantoscopic tilt will typically increase lens distortions and alter the
cylindrical effect .
P a ntoscopic t il t is req uired if the eyes are above the optic ce nt er of the
lens but not requi red if they are at the optic center.
P a ntoscopic til t is induced by adjusting the temple angle.
R et roscopic til t is just t he opposite of pantoscopic til t: t he bottom edge of t he
lens is further from t he face.
Question 3: correct a nswer e - No, axis OS The ri ght eye sphere powe r
is off by 0.12 D a nd the ax is is off by 4 degrees for O.SOD. The left eye ax is is off
by 4 degrees for 2.00D. This patient's right eye sphere is within the tolerance
stand ards. The OS ax is falls outside of the axis tolerance even though it is off
by the same amount as the OD. This is a result of higher cyl inder power in the
OS lens.

ANSI Standards
Some ANSI standards are pro\ided in Table 12.1 , Table 12.2, a nd Table 12.3 (.5).
Please keep in mind that ANSI standards are reg ul arly upd ated.
Question 4 : correctio n answer d To lower the segment height , the entire
frame needs to be lowered o n the face. To do t hi s , three adjustments can be
made:
in crease t. he distance between nosepads

Sphere Power (D)


<6. -50
> 6.50
Cyl Power
::::; 2.00
2.00-4.50
> 4.50

Tolerance (D)
0.13
29'( sphere pm-ver
Tolerance
0.13
0.1-5
-1 9'( cy I power

Table 12.1: ANSI Z80.1-20 10 Po\\er Tolerances (5)

382

CHAPTER 12. OPHTHALMIC OPTICS

Cyl Power (D)


< 0.25
0.25-0.50
0.50-0.75
0.75-1.50
> 1.50

Axis Tolerance (degrees)


14
7
5
3
2

Table 12.2: ANSI Z80.1-2010 Axis Tolerances (5)

Par amet.er
Thickness
Warpage
Base Curve
Impact

Tolerance
0.3 mm
1.00 D
0.75 D
Resists 5/8 in st.eel ball from 50 in.

Table 12.3: ANSI Z80.1-2010 Tolerances (5)

raise t.he vertical portion of the llosepads


widen t he bridge
Just the opposite would be done to raise the frame , including adding more
padding to the nose pads.
Question 5: correction answer d Crown glass wo uld be very heavy. High
index or polycarbonatc len ses are recommended.
lVlaterials
It is important to keep in mind so me propert ies of the most commonly used
lens materials (4; 1).

Material
Crown glass
CR-39 (Plastic)
Polycarbonate
(Plastic)
High Index Plas
tic

1.523
1.498
1. 586

Pros
does not scratch
lig ht
safety

1.54 166

very thin a nd light

11

Cons
heav:y
scratches easily
hjgh chromatic aber
ration, scratches
high chromatic aber
ration, scratches

:383
for Frame and Lens Selection
and Borish (1) also offer the

frame and lens selection.

roll

cable

lenses

Progressive wearers
ntr.c""",,, tilt

vertical

and

minimal vertex distance

Children

lenses for

sturdy frame
consider transitions
eye

for

no rimless frames

has a risk of
with side shields would be recommended for this
lens
for lenses to be considered
Double-D bifocals lIleall there
usual but also a segment at the top of the
for those who are
to do detailed work which
them such as: electricians and auto mechanics.

384

CHAPTER 12. OPHTHALlIIIC OPTICS

Safety regulations
All lenses must be able to 'withstand impact of a 5/8 inch diameter steel
ball dropped from 50 inches
For safety glasses: The steel ball will be increased to a 1 inch di ameter
for tes ting from th e same di stance
j\'larking for safety lens is Z87
P rescription lenses mus t have a minimum thickness of 3 mm, except for
pI us lenses 2: 3.00 D

All chil dren and monocular patients a nd those using the glasses
for safety or sports are HIGHLY encouraged to get polycarbonate
lenses for safety I

12.3

Case 54

Demographics
Age/race /ge nder: 55 year old fe male
Chief complaint: Sees dou bl e when looking down to read
History of present illness
Character/signs/symptoms: began when she got her new glasses
Seconda ry complaints/symptoms: vision seems distorted when looking to
the side
Patient ocular history: early cataracts
Patient medi cal history: fi bromyalgia
l\1edications taken by pati e nt: none
Clinical findin gs
H a bitual Spectacle Rx

OD: +3.00 DS

OS: -3.00 DS

Add: +2 .50 lined bifocals

12.3. CASE
EOMs: full OU

Cover test

Questions:
Question 1: How much vertical imbalance is induced when the pa
tient looks down 10 mm to read?
a. :3 BU 00

6 BU 00

ED 00
d. 21
OS
c. 3

e. 6 EU OS

2: Which of the following would NOT be a way to correct


for the vertical imbalance?
a. dissimilar segs

and distance

b.

c. slab off
d.

e. contact

Question 3: If slab off was to be used to


cal imbalance, which lens would it be
direction of the prism induced?

for the verti


on and what is the base

OD.

b.

base-down

c.

base-down

d.

4: Which of the
base curve
thickness OS

UC'''i'',UO would be the BEST


Answers are listed as: front
center thickness OD, front base curve OS, center

111111

2 mm

+2.00
+4.00 D.

111m, +4.00

6 mm

+3.00 D, 5

ml11

386

CHA.PTER 12, OPHTHA.LMIC OPTICS

Question 5: The patient notes her vision seems distorted when look
ing through the peripheral part of the lens. \hich of the following
would likely NOT contribute to this complaint?
a, chromatic aberrat ions
b, ,vrong base curve
c, spherical aberrations
d, radial astigmatism
e, curvature of field

Answers:
Question I: correct answer b - 6 BU OD Recall the induced prism from
looking off the optical center is calculated using Prentice's Rule,
OD: +300 x 1 cm

= 3 prism

diopters

OS: -300 x 1 cm = 3 prism diopters


To determine the direction of t he prism , remember looking down is the same as
decentering the lens up. For a plus lens, the decentering direction corresponds
to t he base direc tion of the prism; t herefore, the induced prism is base up,
For a minus lens, it is just the opposite, t he induced prism is base down. In
this case, we have OD 3 BU and OS 3 BD, For vertical imbal a nce , one must
determine the difference in the vertical prism induced between the two eyes,
In this case, the different can be wri tten as 6 BU OD or, equi valently, 6 BD
OS. The power of the add lens can be ignored because the segment powers ancl
heights are identical. As a resul t , any prism t hat results from looking through
the left add is identical to the prism that results from looking through the right
add, Hence, the imbalance due to the acid is zero,

Patients will typically complain of problem s when reading (due to


vertica l imbalance) if the powers in the vertical meridians differ
by more than about 1.5 D, Single vision lenses are not much of a
problem because the patient can simply drop her head whell look ing
clownward, .Multi-focal lenses clo not allow for this adaptation,

Question 2: correct answer d - progressives All of t he following op


tions will work except for progressives lenses , Progressive lenses will also have
ind ucecl vertical imbalan ce,

387

12.3. CASE 54
Explanation of incorrect answers

D issimilar segs met.hods usually use two different fl at top bifocals and
take ad vantage of the different di stances of the optical centers from t he
segment line.
Separate reading glasses work beca use the patient can drop his head when
looking downward a nd therefore continue to look through t he op tical
center of the lens.
Sla b off induces base up pri sm in the most minus (least plus) le ns.
Contact lenses wo rk beca use t he len s moves with the eye. One is forced
to look th rough t he optical center of the lens .
Question 3: correct answer a - OD ED Sla b off removes material in
the lowe r part of the lens , the reby inducing base up prismatic power without. a
change in the diop tric power. It is added to o ne lens and is done on t he lens with
the most mi nus (or least. plus) power in the 90 degree mer idia n. This makes
sense because the minus lens is t he one whi ch induces ba.se down prismat ic
pmver.
Question 4: correct answer c Aniseiko nia refers to different retinal sizes or
shapes be tween the two eyes usual ly due to a nisometropia causing magnificat ion
differences. If large eno ugh, patients can comp lain of eyestrain , head aches, and
image disto rt ions.

As an approximat ion , for each diopter of an isometropia, a 1% dif


fere nce in r et in al image size will be prese nt.. This magnificat.ion
difference will most defini te ly ca use sy mptoms if larger than 10%
because fusion of t he two images wi ll not be possible. Sym ptoms
are possible at lower levels , as well.

Knapp's Law: As a rem inder , Knapp's la,, (see Sect.ion 1l.2) allows us to
infer the following:
axial an isometropi a

correct with spectacles

refractive an isomet.ropia

correct with cont act lenses

Clinical P e arl: T\Iost of the t ime yo u will not know if the pat ient is an ax ial
or refractive a nisometrope. Management of these patie nts shou ld be as follows:
First, fit in a contact lens

388

CHAPTER 12. OPHTHALJI1IC OPTICS


If sympto ms co ntinue, fit in spectacles and manipulate base cu rve and
ce nter t hickness t.o red uce the aniseikonia (2)

Spectacle modification to reduce an iseikon ia


In mos.t pIus lens:
reduce fro nt base curve
decrease thickness
decrease t he vertex distance
In t he most minu s lens:
increase front base curve
increase t hi ckness
in crease the vertex dista nce (3)

Answer choi ce c optimizes these modifications.


Question 5: correct answer c - spherical aberration Spherical a berra
t ions are typica Uy not a problem wit h lenses due to t he small pupil blocking
marginal rays.
Comment on the other answers and lens aberrations in general:
If the base curve is chosen in correctly, t he aberrations will be worse in t he
peri phery of the lens.

Lens aberrations
Chromatic aberra tions: Shor ter wavelengt hs (blues) are refracted more t han
t he longer wavel engths (red) as t hey pass th rou gh an opt ical system ;
therefore, a point image is not formed from a point object. Th e result
may be colored fringes around objects. Chroma ti c aberrations get worse
in the periphery of t he lens and the hjgher t he power of the lens, the
wo rse t he effect .
The chromatic aberration for a lens material is quantified using t he A bbe
val ue, 1/, w hi ch is th e reciprocal of the chroma ti c aberrat ion. Therefore :
as the Abbe value increases t he chromatic abe rration decreases and vice
versa .
Spherical a berr a tions: are d ue to ma rginal rays (not cl ose to the opti cal
axis) forming an image at a different location than t he paraxial rays
(close to the optical axjs). T he result in a non-point image from a pojnt
obj ect .

389

CASE 5el

distortion results from

of the

the lens obliquely


pmver; can
a Iso called
fact that the effective power
the
of the lens

Curvature of field
of
different in the center
Distortion: due to
you are from the

varies

on how far

Practical

of aberrations

aberrations and coma are


is small
to block

because the
lenses (> +7

use the manufacturer's recommendation

To
of base
Corrected curve

aberrations

To decrease chromatic
tilt

shorter vertex

and sufli

Lenses
lenses are lenses which are
curvature
better

corrects lens

that the radius of


lenses include:
lenses

CHAPTER 12. OPHTHALIIIIC OPTICS

390

more comfor table: t he lenses will be lig hter and thinner


cosm esis: the lens will appear more a ttractive because the lens will be
m ade flat ter res ul ting in less magnification, t he eyes will not appear as
big
used for progressive lenses
Field of View
Field of view t hrough lenses will be illcreased with decreased vertex dis
tance.
\Vith plus lenses , the higher the power, th e smaller t he field of view.
\Vi th minus lenses, the higher t he power, the large r the field of view.
The answer to this la st question was based on Brooks a nd Borish (1 ). Please
refer to this reference for further informat ion.

References
[1 ] Brooks, C and Borish, 1. System for Ophthalmic Dispensing, second edi
tion. Butterworth-Heinemann, 1996.

[2] http : //arapaho.nsuok.edu/-salrnonto/vs3_rnaterials/Lecture18.pdf


[3]

Tamesis R. Ophthalmology Boar d Review, second edition. lVlcG raw-Hill ,


2006.

[4] Fannin T. Grosvenor T. Clini cal Optics, second edition . But ter worth
Heinemann , 1996.
[5]

www.opticam pus.com/tools/ a nsi. php

Chapter 13

Contact L

Sarah

O.D., F.A.A.O.

391

13.1. CASE 55

13.1

393

Case 55

Demographics
Age/ race / gender: 30 year old physician assistant, female
Chief complaint: wou ld like to update contact lens prescript ion
Secondary complaints / symptoms: eyes always seem to be red
P at ient ocular history: wears contact lenses about 10 hours/ day, never sleeps
in the lenses , replaces every 2 wee ks, as instructed
Patient m edical history: t hyroid cancer treated with surgery
M edicati o ns taken by patient: levoxyl
Patie nt allergy history: sulfa
Clinical findings
Keratometry
OD: 47.00 at 180 ,45.50 a t 90

OS: 47.00 at 180, 45.50 at 90

Subjective r efraction
OD: -6. :30 - 2.00 x 090,20/20 (dista nce)
OS: -6.50 - 2.00 x 090, 20/20 (dista nce)
Vertex Distanc e: 12 mm
Slit lamp
lids / lashes / adnexa: t at too eyeliner OU
co~junctiva: mild diffuse injection OU

cornea: clear OU
anterior chamb e r: quiet/deep OU
iris: wnl OU
lens: clear OU
vitreous: clear OU

CHAPTER 13. CONTACT LENSES

394

Questions:
1: What would be the contact prescription for this patient,
soft contacts are used?
a. -6.00-1.75

090

b. -6.00-2.00

090

c. -6.50-2.00

090

d. -6.25-2.00 x 090
x 090

e.
-7.00-2.50

090

on the eyes. The following rotation


is noted:
as 10
counterclockwise.
The vision in both eyes is 20/25. How would the axis be modified
based on the rotation in each eye to
to achieve better visual

OD axis

OS axis 90

b. OD axis

axis 100

80

c.OD
d.OD

100

e.OD

OS axis 80

3: Which is true

>!cLlUl.H'"

oxygen

of contact

lenses?

a.

lens thickness

transmission decreases

b. As the Dk

c. Silicone

increases
have Dk values around :30 units

d. As the 'water content increases in silicone


E\UvIA lenses have

the Dk increases

oxygen

Question 4: For the initial fitting, all three of the


base
curves are available. Which base curve would LIKELY be best for
this patient for soft lenses?

8.:3
b. 86
c. 9.0

d. does not matter

13.1. CASE 55

Question 5:
lenses in

the optical features of contact

In

will increase in contact

b. In

will increase in
aniseikonia will

c.

decreased or elimi

nated

will

in contact lenses

will use more accommodation when

A
lenses

L A myope will have to converge

6: The
contact lenses.
multi-purpose solution.
be

d.

red when weru'


has a
to her
option(s) would NOT

solution
to saline sol ution for

the lenses with

e. switch the

to contact

solution

b. switch to a

switch the

at near when

notes her
You
"Vhat

a. try a different

to contact

and

water before insertion

to

Answers:
correct answer a - 1. 75 x 090)
needs to
vertexed when the power
greater than 4
the contact lens will
less power at
cornea.
contact lens
than the
with

must be chosell
focal
of the lens. See

Section 11 1 for more details.


the calculation for each meridian
that the focal
is located

1.2

power needed in the 180 win be -7.75 D. That


,76+ 1
these

12.96 em then
have a

13 = 7.69D or rounded to
of -6.00 -

Ogo.

CHAPTER 13. CONTACT LENSES

396

Question 2: correct answer e - OD axis 100, OS axis 80 Toric soft


contact lenses are needed when the astigmatism is generally 1 diopter for with
the-rule and 0.75 diopters for against-the-rule or oblique astigmatism. The
lens is either prism ballasted or truncated to prevent rotation. Once the lens is
settled, if the lens is rotated as determined by the lens markings, the vision may
be reduced. If so, alteration in the lens axis may be made to compensate. Lens
markings are typically on the bottom of the lens, giving rise to the following
"LARS" rule.

LARS H Left add, Right subtract


That is, a clockwise rotation of the lens means one should increase
the axis value (add), while a counterclockwise rotation means one
should decrease the axis val ue (subtract).

A marking on the bottom of the right lens would have rotated to the left;
therefore 10 degrees is subtracted from 90. A marking on the bottom of the
left lens would have rotated to the right, so 10 degrees is added to the 90
degrees. The new contact lens Rx would be -600-175xlOO and -600-175x080.
The lenses would still sit in the SAT\,fE orientation but the vision should be
improved.
Question 3: correct answer b
Dk, oxygen permeability, is known for all contact lenses and the higher
Dk values are more healthy for the eye.
Comments on the other answers:
Lens thickness also affects transmissibility. The thicker the lens, the worse
the transmission of oxygen.
In traditional hydrogel soft lenses, the Dk increases as water content
increases. In the newer material, silicone hydrogels, as water content
increases, Dk decreases.
Silicone hydrogels have Dk values greater than traditional hydrogels (greater
than 60 units).
PMMA contact lenses, which are rarely used, have no oxygen permeabil
ity (1).
Question 4: correct answer a - 8.3 This patient has fairly steep K's
and remember that steep K's are fit best with small base curves (measured
in 111m). This is simply a statement of the fact that a steeper cornea requires
a steeper lens base curve (measured in diopters), which amounts to a lens

1S.1. CASE 55
whose

:397

curve has
as radius of curvature
curve in
K 'values in

For this
if the 8.:3 be
too
power. In soft contact lenses, base
of the

Otherwise
cornea is flatter
can be tried in
same
effect on the power

5: correct answer c In
with
,viII be decreased or eliminated 'when contacts are chosen.

of Incorrect Answers

distance
will be smaller in contact lenses

creases

de
vertex distance

will be

A corrected In:nt'"n""
"F'CU.cc,",,,,, than \vhen

only the

more accommodation at near


corrected myope
t.han when
calculat

81ld
the vergence of
corrective lenses are
this calculation
for
the scope of this text, and we therefore stress
result.

6: correct answers c and d


no
never be
on soft contact
thamoeba which can cause eye infections.

Saline solution is fine for

Solution sensitivities are common. The


may present with chTonic red
of
upon insertion of lenses.
be

in

different brand

would be

sol utions such


Clear Care neutralize
no chemicals are present when the lens

saline after
inserted.

CHAPTER 13. CONTACT LENSES

398

Da ily disposable lenses , which are worn for one day a nd the n thrown
away, elimina.te the need for contact lens solu t ion a nd may be a good
option.

Hydrogen p erox id e solutions shoul d never go directly into t he eye!


This will cau se a chem ical burn.

13.2

Case 56

Demographics
Age/race/gender: 40 year old Hispanic femal e
Chief complaint: needs new contact lenses, current lenses are slight ly blurred
fo r near
Secondary complaints/ symptoms: occasional itchy eyes
Patient ocular history: ocul ar a llergies
F8..ITlily ocular history
mother: diabetes
fat her: hypertensio n
Patient m e dical history: AIDS , last CD4 count was 50
Medications taken by patient: uses Lobob cleaner for co ntact lenses, HIV
drug cocktail , Elestat bid
Patient allergy history: NKDA
Clinical findings
with H abitual Contact Lens Rx , unknown parameters
OD: 20/ 30
OS: 20/30
Keratometry
OD: 42.00 /43.00 ax is 180
OS: 42 00/ 43.00 axis 180
Subjective r e fraction

13.2. CASE 56

:399

OD: -2.50 - 1.00

OS: -3.50

Slit

wnl

cornea: wnl

anterior chamber:

iris: wnl

lens: wn}

vitreous: wnl

lOPs;

mIn

Internals:

Fundus OD

wnl

macula: wllI

pole: will

lattice ~,.,,,~,,,.,

without holes

Fundus OS

Wll]

rnacula:

Will

wnl

open to anterior trabecular meshwork and scleral spur in


all gazes vvith
no PAS

1: This fJ""v.lt;Uv loves her PMMA lenses which are 15 years


old. Which of the
NOT an
of the materials
to PMMA lenses?
a. more oxygen
b.
c.

better

d. more

2: \Vhich of the
characteristics of this patient
make her a relative contraindication
for contact lens
wear?

CHAPTER. 13. CONTACT LENSES

400
a. lattice degeneration
b. uses allergy eyes drops
c. AIDS
d. oblique astigmatism
e. plays sports

f. narrow angles
o

family history of diabetes

Question 3: During patient education, an explanation is given to


the patient regarding contact lens care, cleaning, disinfecting, and
storage. What does it mean when the tip of a contact lens product
dispenser is red?
a. this is an enzymatic cleaner
b. stop! do not put this directly into the eye
c. only for RGP's
d. use in the eyes when they are red from the contact lenses
e. saline solution
Question 4: To find the parcuneters of this patients Pl\1MA lens, a
radiuscope was used. What information can be gathered from this
device regarding the contact lens?
a. lens diameter
b. optic zone diameter
c. power
d. base curve
e. center thickness

f. lens curve widths

Answers:
Question 1: correct answer: c- better optics The first lenses available
were P]VIl\IA lenses which had Dk of 0 meaning they had no oxygen permeabil
ity. The permeability increases in the contact, the fragility increases, as well.
PMi'vIA lenses were hydrophobic meaning they did not wet well. They did have
excellent optics. Hard lenses in general have better optics than soft lenses, but
PMMA lenses were superior. These lenses are rarely used any longer due to the
advantages of the newer materials. Below gives more details about the contact
lens materials available, as well as the methods for making soft lenses (2)

13.2. CASE
Contact lens materials

than traditional
1\1A

- no oxygen

coated with

Classification of all

nvnrnop

materials by the FDA

\Vater noniouic
1

water, ionic

water, ionic

The

nonionic

made of ionic material tend to absorb substances from the


such as
the Dk increases.
As water content. increases in traditional
true for

of soft contact lenses


a
lathe-cut:

lens

made

molded: soft lens material in the

mold
and
state

with

lathe
a l1lold

or

402

CHAPTER 13. CONTACT LENSES

Question 2: correct answe r c - AIDS AIDS o r a ny immunosuppressive


condi t ion may preclude a pa tie nt from conta ct lens wear due to t heir poor
response to infect ion.
Comment on another a nswer: Elestat is an an t i-allergy eye drop dosed twice a
day and can be used with contact lens wearers. They can put a drop in first
thing in t he morning and wait before putting in t he lenses. After cont.act le ns
removal at night, another dro p can be in serted.
Conta ct lens contraindications / precautions (1)
co rneal infection or inAammation
hi story of contact lens ovenvear
anterior segment disease
excessi ve dry eye
irresponsible p atients
yo ung childre n
systemi c conditions which may involve immunosuppression s uch as AIDS,
diabetes
monoc ula r p atients
fil tering bleb
incom plete blinkers
Therapeutic/Cosmetic Contact le ns indications (1)
keratoconus
soft lenses used as bandage contact le nses
rigid contact lenses get rid of cornea distor tions such as irregula r as tig
matism
tinted or opaque lenses for aniridia , occlu sion to eliminate diplopia, for
cosmesis
orthokeratology to reduce myopia
a phakia
Lenses for specific conditions / patients
Ortho-K: rever se geometry designs
Keratoconus: R ose-K lens, rigid lenses with soft skir t (ex. Softperm), rigid
lens o n top of a soft lens
Presbyopia: mono vision , bifocal co ntact lenses

403

]31 CASE 56
Dry eye: thick
Kids:

or silicone

lenses

GPC: dailies

Heavy depositors;
Stevens-Johnson

or

RGP's

Extended wearers: FDA

mention to
fuL

3: correct answer b Direct

caution to
will be barrn

Soft lens care products

cleaners: cannot go

in

Salines: no disinfectant

sol u tion: safe for

and disinfection

disinfectants:

neutralization tilne

cleaners: remove accumulated

help with

eye symptoms

Contact lens care

\\'ash bands before handling lenses

recommended solutions for contact lenses- no

disinfect lenses as directed

91

a torn or

lens

contact

91

for contact

and leave contact lens case open

do not top off solution

91

as directed

91

..

not

t he eye doctor

in contact lenses unless approved

4: correct answer d - base curve

the eye doctor

CHAPTER 13. CONTACT LENSES

404
RGP Lens verification (2)

Power: use the lensometer with the back surface of the lens up
Base curve: use the radiuscope or keratometer
Overall diameter, optic zone diameter, curve widths: use the measur
ing magnifier (a high powered stand magnifier) or reticule
Center thickness: use the thickness gauge

13.3

Case 57

Demographics
Age/race/gender: 43 year old male, CPA
Chief complaint: red eyes
History of present illness
Character/signs/symptoms: lenses are less comfortable, redness and
burning upon insertion, denies itch upon removal, denies mucous
discharge, denies photophobia
Location: OU

Severity: moderate

Nature of onset: gradual

Duration: several months

Frequency: a!v\'ays present

Secondary complaints/symptoms: difficult to see fine print


Patient ocular history: fit in soft, mont.hly lenses with a base curve of 8.7,
AD\VT: 6 hours, 110 extended wear, both eyes corrected for distance, uses
Renu solution- began a few months ago, was previously using Optifree,
replaces lenses everyone month
Family ocular history
mother: cataracts
father: retinal tear
Patient medical history: seasonal allergies
Medications taken by patient: none

405

57

13.3.

Patient

?\KDA

Clinical

OD:

axis 180

OS:

180

rnovement, well

Lens

no

refraction

OD: -2.00 DS

os:

-2.00 DS

au
au

Slit

au
cornea:
no

infiltrates

anterior chamber:
Dominant eye:

au

aD

1: What is the MOST

cause of this

solution

b. GPC
c. CLARE

infectious corneal \Ilcer

lens
g. SLACH

2: What are initial treatment


antibiotic

b. discontinue lens wear,


c.

to

lenses

least for now

for this

red

CHAPTER 13. CONTACT LENSES

406
d. change lens solu tion
e. patan ol or zaditor

f. rewetting drops
g. fit with looser contact lens
h. fit with a lens wit.h a high er Dk
l.

answers b, c and d

J. answers a and

Question 3: GPC can be caused by:


a. oc ular prost hesis
b. exposed suture
c. soft contact lens
d. RGP

e. foreign body

f. a ll of the above
Question 4: Regarding the patient's near complaints, which of the
following would be true?
a . if fit with monovision, the rig ht. eye would be the near eye
b. translating bifocal lenses have concentric rings of distance and near
c. the patient has to accommodate more in glasses than contact lenses (as
suming they are both -2.00D OU) , otherwise stated: the patient will be
more symptomatic for his presbyopia in his glasses than in his contact
lenses
d. monovision is easier to adapt to by emerging presbyopes than absolute
presbyopes

Answers:
Question 1: correct answer a - solution sensitivity This list of answers
are a ll differentia ls for red eyes in a soft contact lens wearer. Solution sen
sitivity (ty pically to t he preservative) is characleri~ed gradual onset typically
after months of solution use with minimal or even no symptoms . There may
be stinging after lens insertion , redness, and faint , diffuse infiltrates may be
present.
Question 2: correct answer i-choices b, c and d
related red eye differential diagnosis a nd treatment:

Soft contact lens

407

13.3. CASE
Solution
Treatment:
or a

Giant ....."'....'''''_

which has
preser
such as Clear Care which neutralizes
so no
utiOll is needed and no
discontinue lens wear if infiltrat.es
present
if
but non-infectious

GPC

under
time frame before

lens move
to be

discarded
Treatment:
cases

Contact lens overwear or CLARE

lens acute red eye)

Treatment:

Infectious corneal ulcer

corneal infiltrate with


treatment

Soft lens

feel

comfort with extended


in the
punctate keratitis
tein coated lenses

pro

Restasis

408

CHAPTER 13. CONTACT LENSES


lens

comfortable

Treatment: Refit with a flatter lens. Add artificial

temporary break for contact

Chronic

soft lens associated corneal

neovascularization

corneal abra

sion
Treatment: l\;eeds a lens with

For all of
tion

Dk

above

time

Clinical Cases in Contact


the
information. Please

was the reference used for


further information on this

is a mechanical and immune response.


cOlltact lens wear.

4: correct answer d
contact than their
J list the
contact lens case 1.
5 for further

Contact

d isconti n ua

Giant Papillary Con


is most often caused

have to accommodate
is true for a

in their
Please

111

distance vision for both eyes with

over the top

bifocal or llluitifocal contact lenses


dominate eye fitted for distance. non-dominant

for

monovision

multifocal and rnonovisioll choices: Establish a realistic


The vision will be
of the
in low
a restaurant menu.

Bifocal contacts

13.4. CASE 58

409

lens sh ifts to look throug h the near por t ion, the len s is stabilized by
tr unca ting the lens or with prism ballast
Simultane ous vision: t he pupil needs to be large enough so th at about half of
t he field is corrected for dis tance and half for near in eac h eye, concentric
ring design wit h the central portion is typically for rea ding in soft lenses
Monovision
i'vlonovision is best for ea rly prcsbyopcs . Th e la rger the an isometropia, t he more
difficult to tolerate. These patie nts must. have relatively equal best corrected
vision between the two eyes . The pat.ient should no t have vis ual demands which
require excellent stereopsis, like a surgeo n or a seamstress .

13.4

Case 58

D e mographics
Age/ race / gender: 20 year old asian male
Chief complaint: wa nts hard contact lenses
Patient ocular history: has ha d bard and soft lenses in the past, feels the
ha rd lenses gi ve him superior vision
Family ocular history
mother: unremarkable
fa ther: cataracts
Patient medical history: runner 's knee
Medications taken by patient: ibuprofen prn for knee pain
Patient allergy history: ASA
Clinical findings
Habitual Spectacle Rx

OD: -2.00d s 20/ 25 (distance)

OS: -2.00ds 20/25 (distance)

Keratometry

OD: 4500 at 180 / 45.00 at 90

OS: 45. 00 at 180 / 45.00 at 90

CHAPTER 13. CONTACT LENSES

410

refraction

OD:

OS:

Slit

cornea:
OU

anterior chanlber:
iris: wlll OU

lens: clear OU

vitreous: clea.r OU

Ot)

Internals: all \Vnl OU

1: What is the base curve needed for an on-I< fit'?


a. 7.8 mm

b.

111m

c. 8.0 mm

d.

111m

7.0 111m

Question 2: What is the power of the lens needed for an on-K fit'?

<l.

b. -3.25 D

c. -1.25 D
d. -4.50 D
D

e.

3: 'Vhich fluorescein

is consistent with an on-K

fit'?
no fl uorescein

of
b. A green
zone

Bull's

in the

pattern,

a broad

underneath

lens
a.nd no fluorescein in the

in

73,4,

CASE

411

8
d, /'1.
and in the
e,

A dumbbell
and in the

outside that

zone

pattern of touch
zone

outside that

4: The lens is
low and the patient can feel the
of the lens on his lower lid when he blinks. Which of the following
would NOT raise the lens'?
Increase the lens diameter

b,

center

c. minus carrier lenticular

d. flatten

Question 5: This patient has


lids which may make lid attach
ment fit difficult. Which of the following is FALSE
Ik<Ll";UL with
lids?
lens

b,

may demonstrate
out of the

watermelon

and

lids can be common in


and unstable vision

with

effect which results in the


CRuse

lid attachment

bral fit

d,

and

achieve an

will

A smaller lens diameter will help achieve an

6: If the
contact lens base curve was
to 0.1 mm, what would be the new lens
-3,00 D
b, -2,2.5 D

-1.25 D

-2,75 D

Answers:
1: correct answer b - 7.5 mm For an on-I\:
must mateh the corneal curvature in the fh\ttest meridian,
is +45,QOD

the base
this

the

CHAPTER

A useful

for

CONTACT LENSES

to base

is

;3:17.5

1)

and r is the base curve (in


for power of an SSRI
an eIfective corneal index of refraction
on

can convert the

2: correct answer a
it
the power would be the
D.
3: correct answer b

to mm.

\Ve

at

(-2.25
When an on-K fit is chosen,
back surface of the RGP and
cornea
not affect the power needed to correct.
as
measured
refrac

The

answers:

flatter than K

K
cornea
e- fit

a WTR cornea
4: correct answer b

General

To raise a lens:

..

traction between the lens and cornea.


sometimes
will be better.
this
go closer to an on-K fit.
so flatten it.
.. Lens diameter- A
attachmellt fit. If
smaller to
UL","UC;""-

..

diameter

DeCl'ease center thickness


of the

central or lid
the dhlrneter

O.O:3mm which will

minus lens carrier which will be thicker at the


to adhere to upper lid. This will also decreases center

13.4. CASE

413

which will

cornea is
are too steep, movement will be restricted.

curve
O.5-1.0mlll
lens movement to\vard the
curves

To lower a lens: If there is with-the-rule


be

base curve to

to

If lens is

The watermelon seed effect meallS the lens


when there are
upper lids. In
nrr",,'.n1tOnt in the lid attachment fit
needed or an
fit:

center
with

a smaller lens diameter, steepen the ba.se curve


contour
to thin, rounded

flatten the base curve and


lenticular. This fit
or

6: correct answer d
the base curve
As a
means that the power of the tear
becomes
more
olle
to add minus power to the R.GP to compensate.

The

"del minus, flatter add

curve. One
can more
calculate the power of the tear lens
the base curve power
DK
power
for

CHAPTER 13. CONTACT LENSES

414

In t his case, the base cur ve is steepened by 0. 1 mm. As a resul t, the power
of the tear lens cha nges by about 0.5 D. Hence, the new lens power would be
-2.75 D.
One can also calculate the a nswer more precisely by calc ulating t.he front and
back surface powers of the t.ear lens. The back s urface power is the negative of
t.he K-reading (-45 .00 D) . The fro nt surface power is equ al to the base cur ve,
expressed as DK. T he base curve was 7.5 mm for o n-K . but here it is 0.1 111m
steeper, or 7.4 mm. Th is corresponds t.o a DK value of 337. 5/7 .4 ~ 45.6D.
Therefore, t he power of t he tea r lens is -45.00 + 45.6, or approxi mately 0.6
D. Hen ce, we would need to add -0. 6 D to t he prescription to compensate.
Rounding t.o t he nearest quarter, we "vould add -0.5 D.

13.5

Case 59

Demographics
Age/race /ge nder: 20 year old Hispanic female
Chief complaint: wants co nt.act. lenses
Keratometry
OD: 42.50/46.00 axis 180

OS: 42.50/46.00 ax is 180

Subjective refraction
OD: pl-2.00x180
OS: pl-2.00x180
Slit lamp findings a ll wnl OU

Questions:

Question 1: Which type of RGP would be best for this patient?

a. Spherical
b. Back toric
c. Front t.oric
d. Bitoric

13. 5. CASE 59

415

Question 2: Which of the following is FALSE regarding a with-the


rule cornea?
a. If the lens is fit too fl at, it will rid e too high or low
b. The corneal topography pattern will be a dumbb ell sha pe
c. If t he lens sits too hi gh, use a lens \yith a base curve t hat is steeper than
K , a n aspheric or a tori c back surface lens
d. T he fluorescein pattern will be a dumbbell shape pattern of touch
e. Flex ure ca n be helpful
Question 3: \Vhich of the following is true regarding flexure?
a . Flexure can be decreased by decreasing center thickness
b. Fl exure can be detected by doing keratometry over the lenses
c. Flexure can be ca used by t.he bending forc e of t.he upper lid when blinking
d. Flex ure can be caused by a lens \\' hich is too flat
e. Flexure can be reduced by increas ing the optical zone diameter
Question 4:
conus?

Which of the following is FALSE regarding kerato

a. Topography will reveal inferior corneal steepening


b. This condition is associated with atopic individu als
c. The ideal lens fit is one that vaults the apex of the cone and is well
centered
d. This condition is typically bilateral , yet. asy mmet ric
e. Keratonics may be fit wit.h hy brid le nses or i:) piggy back lens. The disad
vantage is that fluorescein cannot be used to evaluate the fit .
Question 5: A diffe rent patient was fit with a spherical RGP. How
much cylinder would be found on the ove r-refraction, given the fol
lowing parameters? K's OD 44.00 at 180, 46.00 a t. 90, Subjective refr ac tion
OD: pl-350x180
a. 1 D
b. 1. 5 D
c. 2D
d. 3.5 D
e. none
f. 5.5 D

416

CHAPTER 13. CONTACT LENSES

Answers:
Question 1: correct answer d- Bitoric \Ve can summari ze some basic
ideas of RGP fitting with the following table. The number given in t he table
(2.50 D ) is a n estimate based on several sources , but it is only a rule of thumb (5;
6). For corneal astigmatism of greater than 2 ..')0 D, it is difficul t to a.chieve a
good physical fit , whi ch is why these lenses must have a back to ric surface.

RGP Generic Rules


Type ofRGP
Astigmatism Type
spherica l
No ne
spherica l
Corneal 2. 50 D)
back t.oric
Cornea l (> 2.50 D)
Le nt.i cular
front toric
fron t toric
Corneal 2.50 D) and lent icular
bitoric
Corneal (> 2.50 D) and lent icular

Question 2: correct answer b \VTR astigmatism has a figure 8 shape on


co rneal t.opog raphy. ATR has a dumbbell shape. The steep part of the cornea
is represe nted by red a reas on topography. In \VTR astigmatism, the steepest
part of t he cornea is verticall y oriented, thus the figure 8 shape.
In genera l, if a cornea has as tigmatism and it is fit too flat, the lens vvill decenter
along the steep meridian. So, in this case of \VTR astigmatism, the lens will
move up or down (will sit too high or too low).
Question 3 : correct answer band c Flexure is the bending of an RGP
which ca n be caused by the bending force of the upper lid when blinking.
Flexure occurs only when t here is corneal astigmatism present and the lens
bend s to rna tch the pat ients corneal curvature. Keratometry readings would
show to ricity eve n though t.he lens has a spherical back surface. This is a
method to clinically dete rmine if flexure is occurring.
Flexure can be decreased by increasing the center thickness of the lens. Flex
ure can occur if the lens is fit too steep or if there is a large optic zone diameter.
The materia l of the lens can be altered to affe ct flexure .
Question 4: correct answer e A high molec ular weight fluorescein can be
used which will not stain the soft lens portion of t hese lenses.
Question 5: correct answer b T he spherical RGP creates a tear lens whose
optical properties exactly compe nsate for corneal astigma tism. Hence , t he
remaining astigmatism expected in over-refraction will be due to non-corneal
sources. The non-cornea l (or "residual") astigmatism is simply t he difference

13.5. CASE 59

417

bet,ween the astigmatism found in refraction (3 ..5 D WTR) and the corneal
ast igmatism found in the K readings (2 .0 D WTR). The difference is 1.5 D
(\J\TTR ), which t her efore represents t,he cyliner expected when a spherical RGP
is used. This pat ient would likely need to be fit wi t h a front toric lens to
compensate for this residua.l astigmatism.

References
[1]

AOA Optometri c Clinical Practice Guidelines , "Care of the Contact Lens


patien t ," AOA.org website, upda ted 2006.

[2J

Grosvenor, T . Primary Care Opt ometry, Fifth edi t ion. Butterwo r th,
Heinemann, Elsevier , 2007.

[3]

\\Ta tana be, R. Clini ca l Cases in Contact Lenses, But te rworth Heillema nn ,
Boston , 2002.

[4]

Fannin T , Grosvenor T. Clinical Optics , second edition. Butterworth


Heinemann , 1996.

[5]

http://www.clspectrum.com/article.aspx?article=12047

[6]

http://www .psyduck . net/school/Bi torics. pdf

Chapter 1

Low Vision

Sarah

O.D., MS, F.A.A.O.

419

14.1. CASE 60

14.1

421

Case 60

Demographics
Age/ race / gender: 78 yea r old Caucasia n female, retired
Chief complaint: diffi cul ty rea.ding newspaper
Secondary complaints / symptoms: current glasses do not seem to work as
well as they used to
Patient ocular history: dry mac ular degeneration OU, ca taract extract ion
2 years a.go OU, corneal g uttata without ede ma
Patient medica l history: hypertens ion, depression , hypercholes te rem ia
Medications taken by patient: HCTZ , Prozac, lipitor
Clinica l findings
BVA: distance: cc OD 20/80, OS 20/ 100
near: cc OD 0.4m/41"1 , OS 0.4m/5j\vl, Lighthouse near card
Manifest Rx
OD: pl-0.75 x 090
OS: -0.50 DS
Add: +250 DS

Questions:
Question 1: What would be the anticipated near add needed to read
1M print based on the OD findings?
a. +4 D

b. + 5 D
c.

+ 25 D

d. +8 D
e.

+10 D

422

CHAPTER 14. LOW VISION

Question 2: With the add calculated in the above equation, where


would the patient hold the reading material, assuming no accommo
dation?
a. 40 cm
b. 20 cm
c. 25 cm
d. 16 cm

Question 3: \Vith the habitual prescription, what s ize print would


you expect this patient to read with the OD if tested at 20 cm with
the Lighthouse near card?
a. 8 i'd

b. 4 i'd
c. 2 }.1I
d. 1 i'd

Question 4: What would be the Snellen equivalent of the near acuity


for the OS based on the Lighthouse testing?
a. 20/100
b. 20/50
c. 20/ 80

d, 20/ 250

Question 5: Distance acuity is often tested on low VISIOn patients


with the ETDRS distance chart . Which of the following are true?
a. testing is typically done at 4 , 2 or 1 me ters
b. On each row , the chart. maintains a consistent number of letters
c. An ad vantage of this chart compared to the Snellen chart is that there
will more letters per line for the low ac uity leve ls . This may result in
more accurate acuity measurement.
d. If a patient reads half the let.ters on the 20 i\I line at 2 meters but none
on the next line, the acuity would be 20/200
e. all of the above

f. none of t.he above


g. a, b) and d

14. . CASE (]O

Answers:
+4 D Recall the Kestenbaum
the near add needed to read 11\1
of the

clinic

2: correct answer c - 25 cm The


add power. In
case.
have
m, or 2.5 cm.

for determination of near add in


8minations:
.. find the

"

refraction wi th

the

V1SlOl1

frame

the Kestenba U111


can be used
1)
or the
near card to determine the tentative add
power

+2.50 add is used


and near
ei
the
will need to hole! the

.. ''\lith the

3: correct answer c 2 M If the


she should ')e
to
half this size at 20 cm
of
This
is based on relati ve distance
rernincls us tlwt the
size "dll increase
factor of
is decreased
a factor of 2.

4: correct answer d - 20/250 The ratio


the Snellen fraction. That is. we have
0.4
5
for

we

that the Snellen fraction

,10 cm,
factor
which

should l1umeri

(14.

CHAPTER 14. LOW VISION

424

The ratios (20/250 and 0.4/5) are equal and both express a n inverse angle,

the minimum angle of resolution (J\lAR). That is, th e reciprocal of the ratio

(250 / 20 or 5/0.4) gives t he smallest angle UvIAR) tha t can be seen by the eye

(in units of arcmin utes) . In this example , the eye ca n see a smallest angle of

a bout 12.5 arcminu tes, which is abou t 12.5 times la rger tha n t he smallest angl e

t ha t can be seen by someone with 20/ 20 vision.

Question 5: correct answer e - all of the above Testing can , in principle,

be done at any dista nce bu t it is ty pically do ne at 4 or 2 meters. If the pa tient

cannot read any letters a t 2 meters, the ch ar t can be broug ht up to 1 meter.

Each row has 5 letters whi ch are evenly spaced whi ch allows for more con siste nt

measurements a t a ll levels.

In addition, the EDTR S chart can allow for more accura te measurement. Why?

On a Snellen cha rt , t here a re no letters betwee n 20 / 200 and 20/100. If the

patient is 20/ 120- 20/ 180, on the Snellen cha r t, their acui ty would likely st ill

measured as 20/ 200 .

With the EDTRS cha rt , the letter sizes a re scaled logari t hmica lly, which means

st.eps are chosen so t hat there is a consta nt mul t iplicative factor relating t he

?dAR on one line to the j\-lAR on a nother line. Hence, the acuiti es for low

vision patients can ofte n be more pre cisely dete r mined. This can also a id in a

more accurate refr action due to refined patient respo nses.

Recording acui ty can be done in t wo way s:


Using met.ers: The numerator is the test distance in meters,
t he de no nlina t.or is the let ter size (met ric) .
Using feet: The numera tor depend s o n the test distance. If
t ested at 4 Ill , use 20 ft. If tested at 2 m, use 10 ft. If tested
at 1 m, use 5 ft. The denomin a to r is the letter size li sted
with Sn ell en notation (in feet) on cha rt.

For t his pat ient, half the 20fvi line was read at 2m, therefore, the ac uity
is 2m/ 20 M o r ot herwise stated as 20/ 200-. References for the above a n
swer: (2) (1)

14.2

Case 61

Demographics

14.2. CA SE 61

425

Age / race /gender: 38 year old African American female, homemaker


Chief complaint: trouble seeing te lev ision
Secondary complaints/symptoms: glare with sunlig ht
Patient ocular history: oculocutaneous albinis m, does not have glasses
Family ocular history
maternal first cousin: albinism
Clinical findings
Habitual Spectacle Rx

OD: none
OS:
BVA:
distance: sc OD 10/140, OS 10/ 100 with Feinbloom chart
near: sc OD 0.4 / 4M , OS 0 .4/3M wit h Lig hthouse near chart
Retinoscopy: difil cult clue to nystagmus

Questions:
Question 1: To determine the best corrected acuity, a trial frame re
fraction is attempted. Because there is no habitual prescription and
retinoscopy was unsuccessful, the refraction begins with no lenses
in the tri a l frame. To determine the best sphere, calculation of the
patient's just noticea ble difference (JND) is required. What would
the JND be for the left eye based on the distance acuity of 10 / 100?
a.

1. 00 D

b. 0.50 D

c 1..50 D

d. 2.00 D
Question 2: When adding the sphere lenses to the trial frame, they
should be put in which weir?
a. front
b. back

c. middle
d . does not matter

426

CHAPTER 14. LOW VISION

Question 3: Once the best sphere is found, the cylinder axis and
power are determined. In this patient, an axis of 180 is established
and now the power is checked. Currently, in the trial frame, -1.00
0.25 x 180 is present. Using the 1.00 D JCC , the patient states the
letters are clearer when the red marking is lined up with axis 180.
What lenses should now be in the trial frame?
a. -1.00 - 1.25 x 180

b. -1.00 - 2.00 x 180


c. -0.50 - 1.25 x 180

d. -1.00 - 0.75 x 090

Question 4: What are the qualifications for legal blindness?


a. Cannot read any letters on t he 20/100 line in t he best eye o r 20 degrees
visual fi eld diameter in t he better eye
b. Cannot read any of the letters on the 20/100 line in the worst eye or 20
degrees visual fi eld diameter in t he better eye
c. Cannot read any lett.ers on the 20/100 line in the best eye or 10 degrees
vi s ual field dia meter in the better eye
d. Cannot read any letters on the 20/1 00 line in t he wo rst eye or 10 degrees
visua l field diameter in ei ther eye

Question 5: This patient was prescribed a 4x, Keplerian telescope.


Are the objective and o cular lenses negative or positive?
a . objective negative, ocular negative
b. objective posit ive, ocular negative
c. objective negative, ocular positive
d. object ive positive, oc ula r positive

Question 6: A Keplerian telescope with a 20D ocular lens and a 5D


objective lens has what magnification?
a. 20x
b. 5x

c. 4x
d. -4x
e. lOx

f. none of the above

14.2. CA SE 61

427

Question 7: The patient wants to use this 4x telescope to also aid


in computer vie wing at 50cm. What power of reading cap will be
required?

a. + 2 D
b. + 3 D
c.

+4 D

d. +5 D
e. +1 .5 D

f +2.5 D

Question 8: In the pre viou s question, what is the dioptri c power of


the system (telescope plus the reading cap)?
a. 4 D

b. 12 D
c.

16 D

d. 8 D
e. 10 D

Answers:
Question 1: correct answer d - 2D The J ND is used when finding the
best sphere power in the tri al frame refraction, which is t he first step of the
process.
J ND estimation= denominator of t he snelle n aculty

(14.2)

This means a 10/100 (equivalent to 20/200 Snelle n) eye should be a bl e to se nse


changes in sp here of about 2.00 diopters. So, clinica lly, use the bracketing
technique with a +1.00 a nd a -1.00 sphere lens and show t he p at ient the two
lenses successively, whi ch is a total differen ce of 2 diopters. If they choose the
-1.00 lens, for exa mple, place twice t hat amou nt the tri al fram e, -2.00 D. Next ,
recheck t he acu ity to see if a smaller JND ca n be used. Contin ue with the JND
until the patient no tes no difference between th e two sp here choices .

Question 2: correct answer b The sphere lens should be position ed the


close to t he eye (the back well) because it is typically the st rongest powe r. An
increased vertex distance, as you get furth er for wa rd in the fr ame, may a lter
t he effective power of t he lens and may produce a n incorrect refracti on.

428

CHAPTER 14. LOIV VISION

3: correct answer c
the choice of 1 or + 1 at axis
notes it is clearer when the -l.00 is at
axis 180.

power.
you must refine the
to
correct answer a
better eye could
due

acuities, the

would be measured
and
with EDTRS chart would be more
sidered
blind. This new definition allows for
blindness.

Question 5: correct answer d


and
plp"n.n"~

are afocal and allow for

ocular lens and a


up tolx
and

ocular
of distance

lens

14.2. CASE 61

429

exit pupil inside t he telescope

image upright

Kepleria n telescopes
positive ocular and a positive objective lens

powers go much higher than 4x

heavi er a nd longer

ex it pupil is outside the t elescope

image inverted

How to interpret markings on a telescope


A telescope label provides informatio n about the magnification and t he objec
tive le ns diam ete r. For exa mple, a 5x20 mark ing indi cates 5x magnification
and 20 mI11 objective lens diameter.

Fro m the markings on the telescope, we can infer th e size of the


ex it pupil of the telescope.
E =d/lI1,

(14.3)

where d is the diameter of the obj ective lens, ]11 is the magnifica
tion, and E is the size of t he ex it pupil. In the previous example,
the exit pupil wo uld be 20;'5, or 4 mm.

It is advantageous to align the exit pupil with the entrance pupil of th e eye
(which is very near the cornea) . Because the exit pupil is outside t he Keplerian
telescope, the image is brighter and t he field is larger.

Question 6: correct answer d - -4x


is given by

The magnification ill of a

~elescop e

(14.4)
where Foc is the power of t he ocular lens and F obj is t he power of the obj ective
lens . Note that the ocular lens is t he lens closest to the eye a nd the objective
lens is the lens closest to t he obj ect to be viewed .
For t his case, the magnification= -20/5 = -4x , where the negati ve sign means
t he image will be inverted. Keplerian telescopes for low vision use will have an
inverting lens o r a prism to make the image right-side up.

CHA.PTER 14. LO\,V VISION

430

Question 7: correct answer a - (+2 D cap) When a telescope is used for


near or intermediate view ing, the acco mmodative demand ca n be very la rge .
To compensate , one can eit her change t he focal lengt h of the telescope or add a
read ing cap. \Vhe n a cap is added to a telescope , it is termed a telem icroscope.

The power of the reading ca p needed is simply the reciprocal of th e


working dista nce in meters. In t his example, t he working distance
is 50 cm, so the power is 1/ 0.5, or 2 D

Question 8: correct answer d


T he sys tem ca n be cha racterized by using
eit her the diopt ric power or t he magni ficat iou .
To find t he dioptri c power , mult iply the dioptri c power of the cap by t he
mag nificat ion of the telescope. In t his case , we have 8 D .
To find the magnification of the system , simply divi de t he diopt ric power
by 4. In this case , the magnifica tion would be 2X.
Reference for t he above case : (3). P lease use this referen ce for further details.

14.3

Case 62

Demographics
Age/race/ gender: 80 year old Ca ucas ian ma le
Chief complaint: wants help wit h reading and watching t elevision
Patient ocular history; wet Al\I D with mac ugen injections OU
Family ocular history
mother: AM D
father: unrema rkable
Patient medical his tory: sm oker

Medicatio ns taken by patient: AREDS vitamins


Patient alle rgy history: shellfish
Mental status
Mood: depr essio n

.14.3. CASE

4.31

Clinical findings

Rx

Habitual

OD:
120
120
OS:
Facial fields: central
refraction
OD: no
OS:
Internals:
Fundus OD

OU
OU

0.2

macula: disciform

Fundus OS
0.2

macula: disciform
pole:

and sub-ret.inal

intact,

Questions:
to
3x
c. 5x
d. lOx

this

e. would not

2: This
How much base-in

eye
b. G BI total
c.

d.

total
BI each

4BI
f.

BI total

to

able to

t.ask

432

CHAPTER 14. LOW VISION

Question 3: What M print size represents standard newspaper print


size?
a. 0.5 I'd

b. 1 i'd
c. 2.M

d. 3 I'd
e. 4 1\1

Question 4: With near acuity testing with the Lighthouse near card,
this patient can see 4 M at the test distance of 40 cm. How is the
near acuity recorded by convention?
a. .4/4l'vI
b. 4/.4m
c. 20/ 200

d. 20/20
e. 40/400

f. 40/ 4i'v'I
Question 5: This patient brings the near acuity card to 10 cm, what
print size should he be able to read now?
a. 5 1\1

b. 2.5 lv1
c. 1 M

d. 10 Tvl

e. 20 1v1
f. 1.25 i'd
Question 6: This patient was shown a CCTV (closed circuit tele vi
sion) to aid in reading his mail. This utilizes what type of magnifi
cation?
a . relative distance mag nification
b. relative size magnifi cation
c. ang ular magn ification
d. tran sverse magn ifi cation
e. co nve ntional magn ifi cation

14.3. CASE 62

433

Question 7: A +12 D hand


has what amount of
cation
it is used as a collimating
at 25 cm)'?
a.

b. 3x
4x
d.

not

information

Answers:

1: correct answer a

3x

2: correct answer d - 8 BI each eye


readers rethe addition of base-in
at powers of -;-4 due to the
the
would see double.

m
or

case
BI.

the

power

6, the BI

needed for each

IS

4.34

CHAPTER 14. LOW VISION

Question 3: correct answer b - 1 lV1 Th e Ivi unit uses the metric syste m
instead of the Snellen system. A li'vi letter s ubtends five arcminutes at one
met.er (.3). To recognize this letter , a patient would have to resolve abou t 1
arcminute (that is, the patiellt wo uld need to see details that are about 5 times
smaller than the letter itself; ot.hen,"ise, he/she could not distinguish one letter
from a nother). Viewing a 1 I'vl letter from 1 m t herefore corresponds to a
minimum angle of resolu tio n (I'vlAR) of 1 arcminute. This is the same as 20/ 20
vision.
On the other hand , when a 11\1 letter is vievved at 40 cm, the acuit.y required
to read the letters is decreased. In going from 1 meter to 40 cm, the distance
has changed by a factor of 2.5. Therefore, the acuity needed to see t he letter
at 40 cm would be 2.5 X "worse" than that needed to see it at 1 m because of
relative distance magnification. Hence , reading a 11\1 letter at 40 cm requires
a l'IiIAR of about 2.5, which is the same as 20/50 vision (3) .
Question 4: correct answer a - 0.4 /4M The nume rat.or is the test dis
tance in meters and the denominator is the prin t size in fvI units. This can be
converted to Snellen acuity, if desired, because the ratios would be the same
(that is, the numerical value of the ratio 0.4/ 4 would be equal to the numerical
value of the Snellen ratio) . The problem with the latter way of recording the
near acui ty is that it does not tell you about the testing distance.
Question 5: correct answer c - 1 M \Vhen t he patient brings the card
in from 40 cm to 10 cm, he will be able to read smaller print due to relative
dista nce magnifi cation. A change from 40 cm to 10 cm represents a factor of
4 change in distance, so the print will appear 4 times larger at 10 cm than at
40 cm. Therefore, if he could originally see 4 I'vl at 40 cm, he should be able to
see 1 fvl at 10 cm.
Question 6: correct answer b - relative size magnification Relative
size magnification is due to a change in the objects actual size wit.hout a change
in its distance from the viewe r. Another example of relative size magnification
which can be utilized by low vision patients is to read large print books or
en large reading material with a photocopier.
CCTVs include a monitor , camera and a movable X-V table. Desired readi ng
mater ial is set on the table, the camera projects the image onto the screen.
T\1agnification , brightness, and contrast can be adj usted by the patient. The
patient will need an add or accommodation which will allow for their desired
viewing distance from the monitor. Often this is the only reading opti on for
patients with advanced vi sual impa irment. Portable CCTVs are now avaiJ
able (1)
Question 7: correct answe r b - 3x A collimating magnifier is a plus
lens positioned so that the object of interest is aligned with the foca l point of

14.3. CASE 62

435

the lens. The magnification of a co llim ating magnifier , ass umin g a reference
distance of 25 cm , is
F
(14.5 )
AI = 4 '
where F is the diop tric power of t he lens.
12/ 4 = 3x .

In th is example, we have

j\,f =

If a mag nifier is used as non- col limating (such as a stand magnifi er where the
object is inside of the foc a l le ngth of the lens) , an add or acco mmodation is
required due to di vergent light leaving t he lens. Here the m aximum mag nifi
cation represents a combination of two factors : the original magnification of
the lens and the addi tio nal magnifi cat ion provided by accommodation. In this
case, we have
F
(14.6)
M= 4+1
Thi s is sometim es te rmed conventional magnification (1).

'''hen a lens is used as a collimating mag nifier, no accommodation


or add power is required. On the other ha nd, if the object of interest
is placed inside the fo cal le ngth of the lens, accommodation or ad d
power is req uired. Th e latter is often true in the case of a stand
magnifier.

References
[1J

Brilli a nt R., Esse nti als of Low Vision Pract ice, Butterworth Heineman n,
1999.

[2]

Cheatham, Ch eat ha m , and Wood. f(MK Part 1 A pplied Science Review


Guide, Lexington, KY, 2009.

[3]

Faye, et a l. The Lighth ouse Oph t ha lmology Resident Training 1\lanual , A


New Look at Low Vision Care, Lighthouse International , 2000, p. 54-8.

[4]

Li ghthouse intern ationa l book.

[5]

Fa nnin T , Grosvenor T. Clinical Opt ics, second ed ition. Butterwo rth


Heinemann, 1996.

Chapter 15

Accommodative / vergence /
oculomotor anomalies

Small

Wood,O.D.,

F.A.A.O.

/5.1.

439

CASE 6.3

15.1

Case 63

29
Chief

old Caucasian

get tired

of present illness
Location: both e:'es
moderate

Nature of onset: when


Duration:

late at
after she received her new

last eye

exam

every

Patient ocular history: last eye exam :3 months ago, mild ocular
which were treated vvith
OTC eye
pm
ocular
lTIother:
father: X-linked retinoschisis
Patient medical
Medications taken by
control

one

'\JKDA

Patient

mother: HTN
father:

cholesterol

Clinical
Habitual
aD: -4.00-2.00x180
as: 75-3.00x180

Distance PD: .58m111


PERRL OC, no

pm for ocular

birth

440

IVON-STRABJS!\lIC BINOCULAR VISION

CHAPTER

EONIs; full ou

Cover test

distance: small

near: moderate

Confrontation fields: full to

count

Von Graefe phorias

distance: 3 "~".nn'v'
near: 7 CAIJi.JIIVl

distance
10 BO

Slit
Ol,
clear

as

iris: wnlOC
lens: wnl au
vitreous: wlll 0 C

1: What is this

calculated

ratio?

/D

a.

b.
c. 7.4.6.
d.

value should be exaIllined to determine if


to compensate for her

I-''''''''''"'' has sufficient vergence


base-in blur at. near
b. base-out break

[1(,

c. base-out blur

distance

d.

near

blur

441

15, , CA.SE 63

LHO,,,,,,-,U

3: What is the most likely

for this

the chief
a,

c. convergence
d.

e, overminused in
accommodative
4.: What binocular vision
normal result?

would

an ab

a. :\RA
b. accommodative

due to slow to

the minus

test,
ct, b
e. all of the above

5: \Vhat would be the average accommodative


of this

",o.v,"'OBv

7 75D

b. 9.8D
c. 4.HD

d, 2.5D
'.'''',''L,'UH 6: \Vhat would be the best way to treat this

a, do
refraction
b, have the

her new

to
to the

read without any


do

d,

the

bifocal

Answers:
1: correct answer b
Calculated

1)

442

CHAPTER 15. NON-STRABIS j\fIC BINOCULAR VISION

where PD is the interpupilla ry distance in cm, h is the distance of the near


ta rget in meters, Pn is the nea r phoria and Pd is the distance phoria. Esophoria
is (+) and exophoria is (-).
For this patient, the calcul ated AC/ A ratio= 5.8+.4 (-7- (-3))=4.2.6./ D
Question 2: correct answer d - base out blur at near Phorias re
quire , according to Sheard's criterion, at least double the compen
sating vergence to have comfortable, single binocular vision. Positive
fusional vergence ranges are tes t ed directly by BO pri sm or indirectly by adding
plus lenses. These ranges must be sufficient to compensat e for exophoria. Neg
a tive fu sional vergence ra nges a re t ested directly by B1 prism or indirectly with
minus lenses. These ranges must be sufficient to compensate for esophori a .
In thi s case , the pat.ient had a n XP at near. So, the near positi ve vergence
va lue is examined. The blur va lue is the importa nt va lue beca use the pa t ient
needs to see clearly, not just single. The amoun t of xp at nea r wa s 7.6. and the
BO blur ve rgence at nea r was 15.6. . This covers t he 14.6. which is required to
compensate for this phori a .
Question 3: correct answer e - overminused in glasses The most crit
ical piece of information to not ice fr om the clinical findings is the presence of
a base-in blur finding during di st a nce tes ting.
Recall during vergence testing, there are three points to record:
blur : when the pati ent notes t he letters blur , t.his represents the point
wh ere accommodation is no longer clear the im age du e to a change in
convergence
break: when the patient notes the letter break in two , thi s presents the
extent of the fusional verge nce
recovery : when the letters come back to single.
The B1 blur at distance re veals t he fact t ha t a patient is accommoda ting at
dista nce. The blur represents t he relaxation of accommo dat ion in response to
the negat ive fusional vergence test ing. If a patients refractive error is corrected,
they sho ul d not be any accommod at ion to relax. If a patient is overminused,
which is th e case for this patient, they will be accommodating a t a ll distances
to see clearl y (1).
The patients symptoms of asthenopia when reading at night is cons istent with
being overminused. Her accommoda tion never gets a rest , accommoda ting at
all distances, bu t near work would be t he 1110st difficult , especia lly when she is
tired.
Question 4: correct answer e - all of the above

15. 1

CA SE 63

443

All of the following tests would be abnormal if a patient was


overminused:
NRA , clearing t he minus on the accommodative faci lit.y t.esting, and the push
up test.

The NRA would be high, that is greater than + 2.5D, if the pati ent is over
minused.
For a target at 40cm, a pre-presbyope shoul d accommoda te no more than
+2.5D for this d is tance. But, if a s ubject is overminused , they have to
accommodate through the extra minus, as well as, the 2.5D. \Vlten they
a re asked to relax thei r accommodat ion, they have more to relax than
2.5D.
Accommodative facility testing may reveal slow clearing of the minus if a
patient is overminused.
This test allows for assessm ent of a subjects dynamic accommodative
abili ty, the ability to change. During test in g, the pre-presbyopic subj ect
is asked to clear +/-2 len ses. The number of cycles (clearing the plus and
minus) in one minut.e.

Th e expected findings for accommoda t ive facili ty testing in a


s ub ject age 13-30:
11 cycles per minute (cpm) monocul arly
8 cpm binocula rly

How to interpr et abnormal results:

If a su bject has inadequate clearing of the plus a nd minus , they

have a facility problem.

If they cannot. clear the pI us, they a re overaccommodating.

If t hey cannot clear the min us, they ha ve diffident accommodation.

In this case , the su bject would act ually appear to have difficient accommo
dation because t hey are using some of their abilit.y to clear the additional
minus, which leaves less for near.

headaches across

Patient ocular
ocular
mother: Fuch's endothelial

father:

detachment

Patient medical history: GERD


Medications taken
Patient
Clinical

Zantac
flonase

lU1LUll",,'"

Habitual

Rx

OD: none

OS:

EOMs: full OU

Cover test

distance: 2

near: III

refraction

and

OS: pI

anel

Von Graefe

distance: 3
near: 10

no vertical

no vertical

130:
130:

NPC:

with

+2.00D

of accommodation: 11.5D

NRA/PRA:

brow

15.2. CASE 64

447

Question 5: Which of the following statements is TRUE regarding


vision therapy?:
a. For accommodati ve problems, first \.\'or k on facili ty and then ampli tude
b. In non-strabismic vision ther apy, t hird degree targets a re used for periph
eral stereopsis and should be worked on fir st, followed by second degree
t hen fir st degree targets.
c. Stereoscopes are not an effective met hod of t herapy for deep suppression
d. Fo r oc ular motor dysfunction t herapy, first work on speed foll owed by
acc uracy
Answers:
Question 1: correct answer c - convergence insufficiency
Convergence ins uffic iency (CI) is Lho ught to be the most common binoc
ular vision problem with a prevalence of about 3-5% of the popu lation.
CI is characte rized by an exophoria greater at near than distance , low
AC / A ratio, receded NPC, nor mal accommodation, low NRA, and low
BO vergence ranges. Th is patien t has a ll of these characteristics.
NPC (near point of convergence) testing assesses t he a bility of the eyes
to converge while maintaining fusion. A near target is brought towards
t he patient un ti l they indicated diplopia or the eyes are observed by the
doctor to deviate. This distance is noted and t hen the target is moved
away from the patien t until the target appears single aga in. The normals
findin gs fo r young adults: 5cm break, 7cm recovery.
Pse udoconvergence ins ufficiency wo uld a lso have 8 receded NPC. Pseudo
C1 has a receded NPC because of accom modative in sufficiency, not an
act ual convergence problem.
To distinguish the CI and pseudo CI: repeat the NPC testing with
a +2.00 D lens in fron t of t he patient eyes. A t rue C1 will have more
trouble converging the eyes t hrough a plus lens because accommodation
wi ll be more relaxed which is linked to convergence. The pse udo C1 will
have a n improved NPC because t he lens makes up for the insuffi cient
accommodation.
Question 2: correct answer a - near BO blur For an exophoria , it is
impor tant to determine if the patient has enough positive fusional vergence
to converge the eyes to com pensate for the XP and still remain fused a nd
asymptomatic. Positive fusional vergence ranges are tested directly by adding
BO or indirectly by add ing plus lenses . Pl us lenses cause an accommodative
d ivergence in order to maintain fu sio n a nd in o rder to maintain fusion , PFY
needs to increase.

64

15.2.

449

-:3.66 If

tor this
number or

The typical treatment for CI is: office or home-based VT with the


NPC and PFV

of

VT includes:
Brock's

ups.
computer
Studies have shown office-based

Hart

Reconl1nended treatment for non-strabisnlic BV anomalies


CI -

if no

CE

lens

DI

BI
near, then VT

BO

based on Sheard's criterion


needed

refer if

DE - VI',
Basic XP - BI

,\IT

Basic EP - BO

VT

Vertical imbalance - prism

5: correct answer- b

The answers

below

for

Vision

and fusional vergence


as described in the Scheiman book: (2)

of

Broad

bar

computer
ofPFV

ami decrease
and NFV
-one

the

parts

sees

conditions
-works well with shallow to moderate
2,

accommodative and vergence


the direction of the
ex,

and rnir

fusional vergence, accommodative


3,

Le, aperture
and
decrease
part of the target and the ot.her
-treatment

ofPVF
the other

shallOlv to moderate

used after the


.1.

two

miscellaneous tasks- life saver cards, brock


three-dot card
and accurate
stereoscope, \Vheatstone stereoscope,
increase the

and

and decrease the

of PFV and NFV

-divides the space


-has 1st. 2nd and 3rd

visible

each eye

targets

-works even with


not a first-line
6, After

and

Vision therapy or Ocular lVlotor Dysfunction


work on accuracy, then on
and move to
fine movements,

Start work on

~L'JL~'''''~

tech-

453

15.3. CASE 65
Duration: a few months

Frequency: daily

Exacerbations/remissions: worse when tired

Relationship to activity or function: during class

Secondary complaints/ symptoms: loses her place whil e reading


Patient ocular history: last eye exam o ne year ago
Family ocular history
nlother: unremarka ble
father: color blind
Clinical findings
Habitual Spectacle Rx

OD: none

OS:

BVA:
OD
OS

Distance
20/20
20/20

Near
20/20
20/20

Pupils: PERRLA , no apd OU


EOMs: versions full with endpoint nystagmus OU
Confrontation fields : full to finger count OU
Subjective r efraction
OD: + l.OOD 20/20 (distance)

OS: + 100D 20/20 (distance)

Von Graefe phorias


distance: 2 exophori a
near : 5 exophori a

AC / A ratio: 4: 1
MEM retinoscopy: +050D
Accommodative facility: +/-2.00 flipper s:
binocular a nd monocular 5 cpm, slow on - and

+ equa lly

Fused (bin ocular) cross cylinder: + 0.75D


NRA / PRA: + 1.00/ -1. 00, same result 'when done monocula rly
Push-up test: 13.5D
NPC: 4 cm

All ocular h ealth findings Wi\L OU

15.3. CASE

455

5: EOMS versions
noted. This represents a
(would/would

would

b.

woulo not
would

d.

would not

Answers:
correct answer b - accommodative facility Accommoda
characterized
a slow accommodative response to ,.W.'Hi~V"
in accommodative stimulus. Patients will often
about dist.ance blur
after
work. The
reduced and the
will fail
on plus
rninus on lens
on monocular and binocular
vvill be normal. In this case, all of
Accommodative
consistent with accommodative

How to

Accommodative

is shown
indicates when the
is
etc. The number of

with

+/-2D
The

accommodative
See the first
in this section
of the incorrect answers:
of accommodation
Also present: a
AccomnlOdative spasm
would fail
Ill-sustained accommodation would
after
minus on

CASE 66
has a different

OKT\ nystagmus
caloric
rotational nystagmus

wit h nystagmus may have a null


nystagmus
the minimum
timaL Prism
be used to attempt to
minimize the movement.
Latent

which is

at
when one

occluded.

Clinical connection:
instead
binocular cond! tions
methods.

mus, increases with fixation


nystagmus
the same in all
can either
or inherited,

of gaze,

horizontal and

ocular al
Alexander's law- VVith gaze in the direction of the fast
mus ,vill
m

15.4

66

.,")0 year old white

Chief

double \'isiol1

mechanic

15.4. CA.SE

1: \Vhat is the most

cause of this

a. Brmvn's

b. eN 4
eN
e. Left inferior rectus

Question 2: What is the most likely cause of this condition m this


a. ischemic
b.
c.
d.

trauma

f. inflammation of trochlea
receptors

g.

h.

Loa

or inelastic

3: \Vhat is likely the best initial treatment for this pa


tient'?
surgery

b.
c. VT
d. occlusion

atropine

f.
g.

steroids
orbital

4: If a patient has a left


vergence is the
or
eye and this is the (base-up or base-down) finding?
supravergence,
b. supravergence, base-down
c.

d.

base-down

461

.15.J. CASE 66

D ifferential

LUCl;;.H';'~'

for vertical

.. Brown's
.. Skew deviation
a vertical
clear disturbance (4)

torsional component,

supranu

.. eN 3

.. orbital

trauma, tumor,

floor

.
.. oeular
the

the eye is
in
gaze. EOi\IS will be most restricted on elevation during AD-duction because of
the restriction of the muscle. This condition is almost
unilateraL The
may
a chin up posture.

Question 2: correct answer b


C::\ 4
trauma and

of

due to the vertical ranges.

answer b - prism

vert ica I
How much to

Vertical imbalances

correct the vertical

15.4. CASE 66

463

Extraocular Muscle Actions


Lateral rectus: AB-duction
Medial rectus: AD-duction
Superior rectus: elevation , incyclotorsion , AD-duction
Inferior rectus : depression , excyclotorsion, AD-d uction
Superior oblique: incycloto rsion , depression , AB-duction
Inferior oblique: excyclotorsion, elevation, AB-duction

The IRteral and medial recti are easy, to aid


mnemonic: RAD SIN: Recti AD-duct

111

the last 4, remember the

Superiors In tort

References
[1]

Grosvenor , T. Primary Care Opt.ometry, Fifth ed it.ion. Butterworth,


Heinemann , Elsevier, 2007.

[2] Scheiman, M. Wick, B. Clinical Management of Binocular Vision , Het


erophoric, Accommodative, and Eye Movement Disorders, Third edition.
Lippincott, Williams, and Wilkins , 2008.
[3] AOA Optometric Clinical Practice Guidelines, "Care of Patient with Am
blyopiaO, AOA website, updated 2004.
[4] Klin e L, Bajanda, F. Neuro-Ophtha lmology, Rev iew Manual, Fifth edition.
Slack Incorporated, 2004.
:,i1 Randomized clinical triall of treatment.s for symptomatic convergence in
sufficiency in children. Convergence insufficiency Treatment Trial St.udy
Group. Arch Ophthalmol. 2008 Oct;126(10):1336-49.

[6] Borish I, Clinical Refraction , 3rd edition. Butterworth-Heinemann ;1970.


[7] Kurtz, Carlson. Clinical Procedures in Ocular Examination. McGraw-Hili ,
2004.

Chapter 16

Amblyopia/ Strabi

Sarah Dougherty Wood, O.D.,

F.A.A.O.

16.1. CASE 67

469

Question 3: \\That result of the Worth 4 Dot Test would be expected


for this patient if he is suppressing the right eye?
a. 2 dots
b. 3 dots
c. 4 dots
d. 5 dots

Question 4: What would likely be the best INITIAL treatment for


this patient?
a. Brock 's string
b. EO lv! surgery
c. full hyperopia co rrect ion
d. patching of OS
e. atropine treatment o f OS
f. patching of OD

g. prism

Question 5: The initial treatment given was full hyperopic correction


and the ET was still present with adequate time given. Occlusion
therapy (patching) was then initiated. For this patients level of a m
blyopia, patching therapy should be how often and for how long?
a. 6 hours/ day
b. full time patching
c. 2 hours/day
d. would no t be effecti ve for this patient , t he patie nt is too old for t his
treatment to be effi ca cious

Question 6: Which of the following is NOT a risk factor for strabis


mus?
a . Dow n's Sy ndrome
b. family history
c. cere bral palsy
d. epicanthal folds
e. diabetes
f. head trauma

CHAPTER 16. AMBDYOPIA/STRABISMUS

470

Question 7: Which strabismus patient would have the greatest like


lihood of developing amblyopia? Strabismus that is:
a. intermittent, alternating
b. intermittent, unilateral
c. constant, alternating
d. constant, unilateral
Answers:
Question 1: correct answer b - accommodative ET This patient has
accommodative ET based on age of onset (cannot be infantile or sensory), full
versions (cannot be neurological acute or mechanical), unremarkable ocular
health examination (sensory), no history of EOM surgery (cannot be consecu
tive), and size of the ET (cannot be micro ET).
Strabismic misalignment which is convergent in nature is esotropia. There are
four main categories of esotropia: 1) infantile 2) acquired 3) secondary 4) micro.
Infantile ET ,sometimes termed congenital ET, occurs prior to 6 months of
age and the cause is usually unknown. This will involve a large ET, about
40 - 60 A Inferior oblique overaction (hyperdeviation when adducting),
dissociated vertical deviation (DVD, both eyes move up when covered),
and latent nystagmus may be present. Typically treated with surgery, if
non-accommodative.
Acquired ET occurs after 6 months of age. There are three types of acquired
ET: accommodative, acute, and mechanical.
Accommodative esotropia is a result of either a high amount of hy
peropia and/or a high AC / A ratio and is associated with accom
modation. Treatment is typically with corrective lenses, perhaps
bifocals. Prism, VT, or surgery are considered if this is not effec
tive.
Acute ET has a sudden onset and is a result of either a neurological
problem (i.e. CN 6 palsy) or a decompensated phoria. Depending on
the cause, treatment may include patching or surgery. Some cases
will resolve over time.
Mechanical ET is caused by a physical restriction of the EOM (i.e.
Duane's Syndrome type 1).
Secondary ET is due to either sensory deprivation or consecutive ET.
Sensory deprivation occurs after the age of 5 and is a result of re
duced visual acuity in one eye.
Consecutive ET is iatrogenic, usually due to an overcorrection of an
XT during EOM surgery. This is treated with prism, VT, or lenses.

l71

goes undetected wi th conventional cover test


be confirmed with the 80 test (see
in
case
Vision is
mildly reduced. Treatment: consicler VT or prisms.

Question 2: correct

and
ratio or

increase in ET

ET at distance and increases to


near is consistent. with the

of

accomrnodative ET.

Question 3: correct answer b - 3 dots


the presence of
and
40 seconds

The \Vorth

Dot test evaluates

two green
The left eye will

of Worth 4 Dot

OS

2 dots

OD

3 dots

normal

fusion and no

diplopia:

to decrease the accommodative demand


ratio.

CHAPTER 16. AMBLYOPIA/STRABISl\,fUS

472

Amblyopia Treatment- consider adding one treatment at a time


a. correction of refractive error
b. Occlusion therapy

-patching

-atropine

c. structured near activities/vision therapy (3)


Question 5: correct answer c - 2 hours/day

Based on PEDIG:

Amount of patching needed (for children ages 3-6) for amblyopia


treatment
For moderate amblyopia (20/40-20/80)
near activity

2 hours/day with one hour of

For severe amblyopia (20-100-20/400)= 6 hours/day with one hour of


near activity (4) (5)
Amount of atropine dosing (for children ages 3-6) for amblyopia
treatment
For moderate amblyopia= 2 days/week
For severe amblyopia= 2 days/week may be effective too (6)
Atropine vs. Patching
Atropine and patching have similar results for moderate amblyopia. After 6
months, the amblyopic eye was 20/30 or better and/or improved 3 or more lines
from baseline in about 75% in both groups. Either modality has a treatment
of up to 6 months with recommended follow-ups every 4-6 weeks. (7)

Safety of Atropine

Possible side affects are: raised blood pressure, mental confusion,

increased pulse, dryness of mouth and throat, and loss of neuro

muscular coordination.

These effects are rare and the medication is generally tolerated. If

patient does have side effects, 5% homatropine is recommended as

an alternative. (9)

16. 2. CASE 68

473

Question 6: correct answer d - epicanthal folds Epica nthal folds are


seen most often in East Indian a nd Na.tive America n infants. They a re promi
nent vertica l nasal skin folds which may ma ke it appear the eyes are in an eso
positi on. The eyes are a ctually straight and the re is no risk of strabismus. All
of the other answers do put th e patient at risk for strabismus.
Question 7: correct answer d - constant , unilateral

16.2

Case 68

D e mographics
Age/race/ge nd er: 5 year old , African Ameri can boy
Chief complaint: patient has none , moth er no tes eye turn
History of present illness
Characte r/signs/symptoms: eye drifts out

Location: OD

Nature of onset: intermittent

Duration: a few seconds at a time

Frequency: several times a day

Exacerbations/ remissions: worse \vhen he is tired

Patient ocular history: first eye exam last year, got glasses which he was
s uppo~ed to wear full-time
Family ocular history
mother: history of double vision related to 1\IS
father: unremarkable
Pati e nt medical history: ast.hma, full-term birth
l\1edica tions taken by patient: inhaler prn
Patient allergy history: NKDA
Family medical history:
mother: l'vIS
father: diabetes ty pe 2
Clinical findings

CHAPTER 16. AMBLYOPIA/STRABISMUS

474

Habitual Spectacle Rx

aD: -150-150x090

as:

-150-150-090

Pupils: PERRL, no apd ou

EOMs: full ou, no pain or diplopia reported

Cover test

distance: 12xp with intermittent 18 prism diopter RXT


near: 4xp
NPC: bridge of nose
Subjective refraction

aD: -150-150x090 20/20

as: -150-150x090 20/20

Von Graefe phorias Patient did not seem to cooperate


Slit lamp
lidsjlashes j adnexa: cl

au

conjunctiva: racial melanosis

au

cornea: cl au
anterior chamber: posterior embryotoxin au, deep and quiet
iris: wnl

au

au

lens: cl au
vitreous: cl

au

lOPs: soft to palpation

au

Internals:

Fundus aD

CjD: 0.1
macula: flat
posterior pole: wnl
periphery: CHRPE 1 disc diameter in size superior/temporal, in
tact

Fundus

as

CjD: 0.2
macula: flat
posterior pole: wnl
periphery: white without pressure inferior/nasal, intact

16.2. CASE

475

1: Which of the following could


of diplopia when the right eye becomes

does not
a.

b. harmonious anomalous retinal


c. unharrnonious anomalous
d.

anomalous

visual confusion

horror

two of the above

2: Which type of

<,v,nTT'"

does this

have?

a. infantile

XT

b.

XT

c. intermittent

XT

d. acute

XT

e. mechanical

XT

consecutive XT

h. micro XT
f. sensory

1.

two

the above

3: What would be a

treatment option(s) for this

a.
b.
c. base-out

d, over minus his habitual distance

under

in his habitual

f. surgery

g. all of the

a, b, d, and f

h,

L choices

choices

c\

and f

and f

476

CHAPTER 16. AMBLYOPIA/STRABISMUS

Question 4: Which of the following can test for suppression?


a. Bagolini lenses
b. Worth 4 Dot
c. 4 Base-out test
d. Visuoscopy
e. Bruckner
f. all of the above
g. choices a, b, c
h. choices b, c, e
i. choices b. c

Question 5: Which of the following is generally a FALSE statement


regarding surgical intervention for strabismus?
a. Surgery should be considered in ET when wearing the full spectacle cor
rection, the amount of deviation is greater than 15-20 prism diopters at
distance and near
b. XT which exceeds 20-25 prism diopters shonld be considered for surgery
c. Patients with infantile ET can wait to have muscle surgery until age 5
d. Those with totally accommodative ET are at risk for iatrogenic XT (con
secutive) with surgery, therefore, surgery should generally be avoided
e. Botox injections can be done as an alternative to surgery to decrease
ocular misalignment

Answers:
Question 1: correct answer g - suppression and harmonious anoma
lous retinal correspondence The brain does not like diplopia, it causes
visual confusion. This occurs when the maculas view two different images and
the images are superimposed. To counteract diplopia, the brain may suppress
the image from one eye or anomalous retinal correspondence (ARC) may occur.
Normal retinal correspondence is present in those whose foveas are lined up in
the same visual direction. ARC occurs when the fovea of one eye becomes
rewired to match up with a non-foveal area of the other eye. This is a sensory
adaptation and must occur before the age of 5.
The difference between the objective and subjective tropia is the angle of
anomoly. The goal of the creation of a "new" foveal location is for it to be
lined up with the fovea of the other eye such that the patient no longer has
subjective diplopia. Thus, the angle of anomaly would be equal to the angle of
deviation. This is termed harmonious ARC, which is the most common type.
If there is a movement of the foveal location but it does not move quite enough

477

CASE 68
to resolve the
this
of the fovea occurs in the wrong
is termed

HARC:
of
Unharrnonious
present but
Paradoxical ARC:
with,

Horror fU8ionis: This occurs in


be obtained with
The
closer

or
exodeviation is
the deviation is

worse, this

of

than it

to start

with a
will report that
are
over each other and

in
6 lllonths of age. Half of these cases have basic
and the other half have either convergence
"",'Cie,nft> excess
the
near. In convergence
to

Infantile or

will be decreased,

Acute

and is constant,

This could be

ria,

Mechanical
Duane's

restriction of an EOJ\l
etc.) Versions will not be full,

XT occurs after the


of and is mllch more cornman
adults
with
vision loss in one
in no stimulus to fuse, For
treatment. the callse of the vision loss should be
if

478

CHAPTER 16. AMBLYOPIA/STRABISMUS

Consecutive XT is iatrogenic, typically an overcorrection of the ET during


surgery.
Micro XT would be less than 10 prism diopters and would be difficult to
appreciate on cover test. This is not common. (1)
Question 3: correct answer h Vision therapy, base-in prism and overmi
Busing at distance are viable options. If these do not work or the deviation is
too large, surgery is also an option. The VT may include increasing positive
fusional vergence ranges and facility. Remember that to correct with prism,
the apex goes towards the deviation! So, in an XT, the apex is out, therefore
they are corrected with base-in prism. Over minusing (about I-3D) the pa
tient at distance will cause the patient to accommodate and converge. This
will help keep the eyes from deviating. Whichever treatment option is selected,
this patient needs to be followed closely.
Question 4: correct answer g Bagolini lenses, Worth 4 Dot, and the 4
Base out test can detect suppression.

Four levels of sensory fusion by Worth


Zero-degree - no fusion, monocular/suppression
First-degree
(superimposition targets)- patient will have
diplopia, testing uses two completely different targets, dif
ficult to suppress, used in antisuppression therapy
Second-degree - (flat fusion targets)- patient will have no
diplopia, uses identical targets which have suppression checks
added, motor
Third-degree targets - stereopsis, ultimate sensory fusion, sen
sory and motor

Testing:
Bagolini lenses: test for suppression and retinal correspondence. These lenses
are made of striated, plano, clear lenses. With these glasses on, if the patient
views a point light source, each eye will see a line which is 90 degrees from the
direction of the striations in the lens (similar to a maddox rod). The lenses are
positions such that the right eyes sees the line oriented as / and the left eye as
\. Ask the patient what they see.

16.2. CASE 68

479

Interpret at ion of the Bagolini testing:


/ = s uppress ing OS
\ = suppression OD
X = norm al ret in a l correspondence, as long as cover test shows no
tropia
V = esotrop ia
1\ = exotropia

"Vorth 4 Dot testing is described in a mblyopia case 1.


4 Base out test: If a sma ll s uppression area at the fovea of on e eye is present ,

this can result in a microtropia. This if often hard to detect on cover


tcst. The patient will likely have a mild visual acuity reduction with no
explanatioll found on comprehensive exam . The 4 BO test will expose
this small suppression a rea. A 4~ loose lens is used and placed in front
of one eye.

Interpretation of results:
Outward movement and refixation with the other eye= no suppres
sion 0 f either eye
Outward movement a nd no refi xation with other eye= suppressio n
of non-covered eye
No outward movement or refi xatio n with other eye = suppression
of covered eye

Key to understanding: In t he eye wi th the s uppression, 4 prism cliopters


ca uses the image to move such a small a mount that the image still falls
within the scotoma and the suppressed eye can not detect its movement .
Therefore, no movement occurs when the lens is placed in hont of this eye.
\'Vhen placed over the good eye , there is an initial outward movement due
to Hering's Law (yoked moveme nt) but no refixation because this small
movement is !lot detected by the bad eye .
Visuoscopy This test is done to detect ecce ntric fi xat ion which is a monocular
phenomenon when a non-fovea l area is used to fi xate. To perform this
test , the ophthalmoscope is used. The pat ient is instru cted to look at
the ce nter of the grid in the scope a nd the posit ion of the center of the
g rid is noted in relationship to the foveal refl ex. If no eccentric fixation is
prese nt , the two wi ll be a lig ned. Others test which can detect eccentric
fi xat ion: l\ Iaxwell's spot , Haidinger 's Brushes: monocular Hirschberg.

G"HAPTER 16. AMBLYOPIA/STRABISMUS

480

Bruckner Test This test is used on infants to detect strabismus, anisometropia


and media opacities. The ophthalmoscope is used at 80-100cm and the
red reflex is compared between the two eyes. The two reflexes should
appear equaL If not, the darker one is the fixating eye or the one with
the media opacity or with the higher refractive error.
Question 5: correct answer c Those with infantile ET should have surgery
prior to 2 years of age. This will increase the chances of the patient having
binocular vision and stereopsis.
General Theatment Guidelines for strabismus:
Options available (can be combined):
Optical correction- Generally want a clear image on the retina. Need to
take into account what the correction will do to the angle of the strabis
mus and fusion.
Plus or minus lenses- those with high AC/A ratios (convergence excess
and divergent excess) will respond to changes in sphere. Minus lenses
increase convergence and plus lenses increase divergence. Bifocals can be
used when more plus is needed at near than distance- this is often helpful
in accommodative esotropia.
Vision therapy- factors to consider when determining if VT should be pre
scribed: age, motiV'o.tion of patient, time and financial restraints. VT has
the potential to improve sensory and motor fusion, oculomotor control,
increase accommodation and vergence ranges, and eliminate amblyopia.
Prism- may be helpful if the deviation is less than 206.. This should be
avoided when amblyopia or suppression are present. Fresnel prism can
be used if the deviation is expected to change over time. Base-out is used
for esodeviations and base-in for exodeviations.
Patching and atropine therapy to the non-amblyopic eye- see amblyopia
section, first case
EOM surgery- consider non-surgical options first, indicated if cosmesis is
not acceptable to patient or if compliance with treatment is poor (1)

16.3

Case 69

Demographics
Age/race/gender: 7 year old, American Indian female, 2nd grader

16.3. CASE 69

48 1

Chief complaint: fail ed school screening, patient ha s no complaints, mother


notes patient does average in school and has no t noticed any problems
Patient ocular history: unremarkable
Patient medical history: congenital heart defect
M e dications taken by patie nt: none
Patient allergy history: NKDA
Clinical findings
Habitual Spectacle Rx
OD: none
OS:
uncorrected VA:
OD
OS

Distance
20/5 0
20/ 70

Near
20/60
20/80

Pupils: PERRL OU, no apd


EOMs: full ou
Subjective refraction
OD: +200-150x180 20/25
OS: +3 25-325x180 20;'50
Von Graefe phorias
distance: ortho
near: 6 EP
Hirschberg 0. 5m m nasally displaced OU
Stereo testing +stereofiy, 2/5 \iVirt rings , 3/3 animals
Slit lamp All findings wnl OU
lOPs: soft to palpation OU
Internals:
Fundus OU
C/D: 0.35 , healthy rim tissue OU
macula: fiat au
posterior pole: vessel tor tuosity OU
periphery: intact au

482

CHAPTER 16. AMBLYOPIA/STRABISMUS

Questions:
Question 1: This patient has refractive amblyopia. Which of the
following refractive errors has the GREATEST risk of developing
refractive amblyopia?
a. OD pl-300x180, OS pl-300x180
b. OD +400-050x180, OS +400-050x180
c. OD -500-050x180, OS -700-050x180
d. OD pl-050x180, OS +075-125x180

Question 2: What is the interpretation of this patient's Hirschberg


findings?
a. esotropia
b. no deviation
c. eccentric fixation
d. exotropia
e. hypotropia

Question 3: Which of the following is TRUE regarding stereo testing


on this patient?
a. These tests are considered global stereo tests
b. The patients stereopsis is considered normal
c. It is possible to get all of these test correct without any stereopsis
d. Random dot stereopsis tests have monocular cues
e. The patient does not need to wear her habitual spectacles for stereo
testing
f. The contour test would be better than the global test at detecting con
stant strabismus.

Question 4: This patient has a 6.0. esophoria at near. Which of the


following would occur on CT at near?
a. When one eye is covered, the uncovered eye moves out
b. When one eye is covered, the uncovered eye moves in
c. With one eye covered, the cover is then moved to the other eye, the eye
that was just covered will move out
d. With one eye covered, the cover is then moved to the other eye, the eye
that was just covered will move in
e. On unilateral cover test, the covered eye will not move

483

16.8. CASE 69
f. On unilateral

are

h. three answers are correct

5: Which of the

is NOT a dissociated test?

maddox rod
b.

fil ter

c. cover test

d. Mallet

use of vertical

to

the

6: Which condition will cause


diplopia or

to

t,vo

to have

harmonious anomalous retinal

fixation

d.

e.

Answers:

is
further out. Both eyes have one
In
eye will

is

even if the refractive error is

because the error is so

either eye.

484

CHAPTER 16. AMBLYOPIA/STRABISMUS

Potentially amblyogenic refractive errors


Hyperopia, > ID aniso, >5D OU
Myopia, >3D aniso, >8D OU
Astigmatism, >1.5D aniso, >2.5D OU (8)

Clinical note: Important patient education for this patient: glasses need to
be worn full-time!
Question 2: correct answer b - no deviation The Hirschberg test is a
gross measure of ocular alignment. It is done at 50cm with a point light source
and the corneal reflex (line of sight) is observed in relationship to the pupillary
axis. The difference in tbis angle is termed angle lambda.
Interpretation:
Normal angle lambda: slight nasal displacement by about 0.5mm
Nasal displacement from normal represents an exotropia, temporal an esotropia,
superior is hypotropia, and inferior is hypertropia.
A deviation by Imm represents about a 22 prism diopter deviation. If this
deviation is actually measured with prism, it is termed the Krimsky test.
Question 3: correct answer c This patient's stereopsis was tested with all
contour targets and monocular cues can allow the patient to perform better
than would be expected.
There are two categories of tests for the evaluation of stereopsis: contour or
local and global stereopsis.
Contour testing uses laterally displaced targets which allows for monocular
cues, which allows those even with no stereopsis to guess correctly. Examples
are the Titmus fly, animals, and \Virt circles. Contour testing is better for
detection of peripheral stereopsis (>60 sec of arc).
Global testing uses random dot targets which have no monocular cues. This
testing is very good at detecting constant strabismus, even with gross random
dot targets.
For all types of testing, patient habitual refraction should be worn in addition
to the stereo glasses because stereopsis can be negatively affected by blur.
Stereopsis can also be tested using anaglyphs and polaroid targets. (9)
Normal stereopsis is considered 20 seconds of arc with contour testing and
appreciations for gross random dot targets. (9)
Question 4: correct answer g - answers c and f are correct Cover
test is a dissociated test meaning it breaks fusion. If a patient has a phoria,
when fusion is broken, the covered eye will move to its tonic position. In an

16.3. CASE 69
esophori a , this position will be a converge nt posi tion a nd in exophoria, it is
a divergent position. While doing unil atera l cover tes t, t he non-covered eye
should rem ain steady but if there is a phoria, t he covered eye will move behind
the paddle.
On t his pati ent, each eye will become slightly con ve rgent when covered. ''''' hen
it is uncovered a nd the paddle moves to the other eye, it will appear to move
from co nvergent to s traight- a ppearing to move ou t.
Question 5: correct answer d - Mallet b ox The iVIall et box, used for
fi xati on dispari ty tes ting, has bi nocu la r cues for mot.or fusi o n, therefore , it is
not dissocia ting. The maj ority of the fi eld of view is see n by bot h eyes which
serves as fusion locks . It is termed an associated test. Other fi xation disparity
tests: AO Vectographic slide , Bernell lantern, ,,"\lesso n fixatio n ca rd , Sheedy
dispa romet.er .
Question 6: correct answer d - decompen sated phoria Deco mpensated
phorias are la rge phorias which can no longer be ove rcome a nd ac utely become
a t ropi a- either constant or intermittent. Diplopia wi ll be present.
Comments on the incorrect answers:
ARC - See amblyopia case 2
Congenital nystagmus does not generally cause su bjective complaints such
as oscillopsia where acquired nystagmus may have this complaint .
Saccadic suppression occurs during a saccade t.o suppress the vision elimi
na ting t he possi bility of oscillopsia.
Fixation disparity (unit is minutes of arc) is t he small misa lign ment of t he
visual axes o f the two eyes wh ich does not cause double vision. The
corresponding retinal points between t he two eyes where images must
fa ll to ha ve single visi on in cl ud e a small area on either side to allow fo r
flexibility. This area is termed Panum's fus iona l area.
Clinically, fixation dispar ity test.ing may be indica ted if no dipl opia is
present but t he pat ient expe riences as thenopia . If a fix ation disparity is
noted, neutralizing prism is used to a nd thi s a mo unt of prism is termed
the a ssocia ted p horia . Th is is often the amount of pri sm prescri bed to
reli eve a palient.s sy mpto ms. prj
There are 4 ty pes of fixati on dis parity curves which can be plotted where the y
axis is the a mount o f fixation dispa rity and the x ax is is the associated phoria:
A ty pe 1 curve is t he most common , sig moid a l in shape, and t he pat ient
is generally asy mptomat ic .
A ty pe 2 cur ve is a resu lt of an esodi sparit.y

486

CHAPTER 16. AMBLYOPIA/STRABISMUS

A type 3 curve is an exodisparity


A type 4 curve is a result of unstable binocularity
If a patient has a dissociated phoria in the opposite direction of the associated
phoria, it is termed a paradoxical fixation disparity. This is not typical.

References
[1] AOA Optometric Clinical Practice Guidelines, "Care of the Patient with
Strabismus: Esotropia and Exotropia", AOA website, updated 200 1l.
[2) Cotter S; Pediatric Eye Disease Investigator Group. Treatment of ani
sometropia amblyopia in children with refractive correction. Ophthalmol
ogy. 2006 Jun; 113 (6):895-903.
[3) Scheiman M. Lecture at NECO, March 2010.
[4) Repka, Beck, Holmes, Birch, Chandler, Cotter, HertIe, Kraker, Moke,
Quinn, Scheiman A randomized trial of patching regimens for treatment
of moderate amblyopia in children. Archives of Ophthalmology, Volume
121, Number 5 603-611.
[5] Holmes, Kraker, Beck, Birch, Cotter, Everett, Hertle, Quinn, Repka,
Scheiman, Wallace; Pediatric Eye Disease Investigator Group. A random
ized trial of prescri bed patching regimens for treatment of severe amblyopia
in children. Ophthalmology. 2003 NovjllO(ll): 2075-87.
[6] The Pediatric Eye Disease Investigator Group. A Randomized Trial of
Atropine Regimens for Treatment of Moderate Amblyopia in Children.
Ophthalmology 2004; 111 :2076-2085.
[7] Pediatric
Disease Investigator Group. A randomized trial of atropine
versus patching for treatment of moderate amblyopia in children. Arch
Ophthalmology. 2002 Mar;120(3): 387-8.
[8] Weissberg E. Essentials of Clinical Binocular Vision, Butterworth. 2004.
[9] Scheiman, M. Wick, B. Clinical Management of Binocular Vision, Het
erophoric, Accommodative, and Eye Movement Disorders, Third edition.
Lippincott, Williams, and Wilkins, 2008.

17

Percept

function/ olor Vision

F.A.A.O.

Sarah

487

489

17.1. CASE 70

17.1

Case70

Demographics
Age/ race/ gender: 42 yea r old male
Chief complaint: blur at. Ilear
History of present illness
Character/signs/symptoms: cannot see fine print

Location: OU

Severity: mild

Nature of onset: g rad ua l

Duration: a few months

Exacerbations/remissions: especially when tired

Patient ocular history: pati ent knows he has difficulty with matching col
ors, does not wear glasses
Patient medical history: unremarkable
Family ocular history: does not know father , mother had normal color vi
sion
l\1edications taken by patient: none
Patient allergy history: NKDA
Clinical findings
BVA:
OD
OS

Distance
20/ 20
20/ 20

Near
20/40
20/ 40

Subjective refraction
OD: pi ds
OS: pI ds , add +125 OU
Nagal anomoloscope required more red to match the yellow stimulus

490

CHAPTER 17. PERCEPTUAL FUNCTION/COLOR VISION

Questions:
Question 1: From the examination, what is the most likely diagnosis
of the color vision defect?
a. protanomolous trichromat
b. deuteranomolous trichromat
c. protanope
d. deuteranope
e. tritanomaly
f. monochromatism

g. normal color vision


h. tritanope
Question 2: Which one of the following is FALSE about color vision
testing?
a. all individuals should be able to see the first plate of the Ishihara test
b. pseudoisochromatic plates require specific lighting to be a valid test. Test
ing should be done with daylight illumination or a Macbeth llluminant
Clamp
c. testing should be done binocularly
d. A Nagai anomoloscope can distinguish an anomalous trichromat from a
dichromat
Question 3: With your diagnosis from question 1, which test or tests
would pick up the abnormality?
a. Ishihara pseudoisochromatic plates
b. Farnsworth D-15 test
c. HRR pseudoisochromatic plates
d. Farnsworth-Munsell 100-hue test
e. all of the above
f. none of the above

Question 4: Which answer is most likely true regarding the inheri


tance of this patient's color vision defect?
a. X-linked- acquired from father
b. X-linked- acquired from mother who was a carrier
c. autosomal inheritance

70

7.1,

491

more common in
III

t.han males

of

f. for a female to be color


tive gene

her mother needs

(,0

have a defec

Answers:
1: correct answer a - protanornolous trichromat
the
of the 0.'ageJ

The

.. A trichromat has three types of

and L

trichromat

.. The deuteranomolous trichromat


them

a spectrum shifted

(.0

If an
three
more red
1110re gree n to

and it deuteranornolous trichromat will

., Tritanomaly is a form of itnomalous trichromatism which is (In inherited


specCrum shift. This is more rare thitl1
anomalies which are inherited .

..
achieve a match .
.. A dichromat only has t\\"O
more difficult because the two
An

\II

hich makes

,JUV"'J,J'C",U

do not cover
distill

492

CHAPTER 17. PERCEPTUAL FUNCTION/COLOR VISION

There are three types of dichromats:


Protanopes are red deficient (lack erythrolabe), therefore, their sensitivity is
reduced in the higher wavelengths
Deuteranopes are green deficient (lack chlorolabe)
Thitanopes have inherited blue-yellow deficiency
Monochromatism (achromatopsia) is a rare condition in which the individ
ual cannot distinguish colors. They have only one type of retinal cone
(cone monochromat) or lack all cone function (rod monochromat) (1).

General color vision trends


Congenital red/green: bilateral, male, stable
Acquired blue/yellow: due to outer retinal disease
Acquired red/green: due to inner retina/optic nerve or visual path
way abnormalities, asymmetric or unilateral, progressive (2)

A thorough description of color vision anomalies can be found in (1; 2).


Question 2: correct answer c - testing should be done binocularly
is FALSE- all the other choices are true Color vision testing should be
done monocularly. This will allow for detection of unilateral or asymmetric
color vision abnormalities.
The first plate of the Ishihara test should be seen by all individuals, regardless
of their color vision. It can serve as a test for possible malingering.
Question 3: correct answer e - all of the above All of these tests
can detect red-green defects, like the one in this patient. The Ishihara plates
CANNOT detect a blue-yellow defect. Note that while these tests will all detect
a red-green defect, not all tests can distinguish between a dichromat (such as
a protanope) and anomalous trichromacy.

X-chrom contact lenses can be used to "enhance" color vision in


anomalous trichromats and dichromats. These lenses are red-tinted
and serve as a long pass filter. When worn over one eye only, the
spectral sensitivity shifts and if viewing is alternated between the
two eyes, color perception can possibly be enhanced (2).

17.2. CASE 71

493

Question 4: correct answer b - X-linkedfrom mother who


was a carrier Inherited color vision abnormalities are X-linked recessive.
This means the gene variation
carried on the X chromosome. It must be
present on both
in women to be manifest because it is
trait and
the color
normal X will dominant. A woman would have to receive a
color vision abnormal X from both parents to manifest a color vision defect.
have one chromosome which
receive from tileir mother,
Since men
if she was a carrier or had a color vision defect. the SOll will have a color
vision defect.. As a
males have color vision defects much more often
normal color vision in caucasian males is
and
caucasian females
Protanomalous trichromats are present in 1% of
rnales, (I)

17.2

71

Ca ucasian female

30 year

Chief complaint:
of

nrp"<Pl",j"

in low

conditions

illness

Location: Ot:

mild

Nature of onset: since

neuritis

Duration: 1

OD

to activity or function: in low

conditions

does not seem as clear


she would like it to be

Patient ocular

of

neuritis OS

year ago, OD

ocular history

mother: retinoschisis

father:
Patient medical
:Medications taken
Clinical

sclerosis

interferon beta treatment

years

494

CHAPTER 17. PERCEPTUAL FUNCTION/COLOR VISION


Habitual Spectacle Rx

aD: +400D8 20/20 (distance)

as: +400D8 20/20 (distance)

Pupils: PERRL, no apd OU

EOMs: full ou

Cover test

distance: 2xp

near: lOxp

Subjective refraction

aD: +400D8 20/20 (distance)

as: +400D8 20/20 (distance)

Slit lamp

lids/lashes/adnexa: cl OU

conjunctiva: cl OU

cornea: cl 0 U

anterior chamber: deep and quiet OU

iris: iris nevus OD, will 08

lens: cl OU

vitreous: cl OU

lOPs: 14 mmHg OD

Internals:

Fundus au

C/D: 0.2 diffuse rim pallor

macula: flat

posterior pole: will

periphery: intact

Questions:
Question 1: This patient has good acuity but has functional visual
complaints. Wbat is the MOST likely cause of her complaints?
a.
b.
c.
d.
e.

early cataracts
spectacle lens aberrations
reduced contrast sensitivity
dry eye
early presbyopia symptoms
f. functional vision loss
g. subclinical macular damage
h. convergence insufficiency

495

17.2. CASE 71
cut-off of 60

b.

d.

3: The reduced Snellen fraction of 0.2 represents what


acuity?
a,

b.

d. 20/80
c.
f.

would be the BEST treatment


with their chief complaint?
CI.

b.

surgery

c. artiflcial tears
d.

colored filter

e.

bifocal

f. vision
h.

vitamins

i. no treatment

vision is to visual
to:
C\.

critical flicker fusion

b. contrast

in

d. vernier
c.

f resolution

as temporal vision is

496

CHAPTER 17. PERCEPTUAL FUNCTION/COLOR VISION

Answers:
Question 1: correct answer c - reduced contrast sensitivity This
patient likely has a decrease in low and/or moderate frequency contrast sen
sitivity. Therefore, her visual acuity on high contrast chart is good but the
quality of the vision she reports is poorer than would be expected.
More on contrast sensitivity:
Recording of visual acuities in an eye exam is typically done on a high con
trast chart, black letters on a white background. Visual acuity represents the
high frequency cutoff of the contrast sensitivity function. The real world is
not typically black and white but contains lower contrast objects, often in low
illumination settings. Therefore, visual acuity is not always a good representa
tion of a patients true visual function. Low vision patients often perform worse
than would be expected for their level of acuity due to poor contrast sensitivity
due to their ocular disease (typically due to retina, optic nerve, lens, or corneal
damage). A contrast sensitivity curve can be plotted for patients and may be
more representative of the full range of visual sensitivity.

Clinical Pearl
Patients with history of optic neuritis may have 20/20 visual acuity
but often complain of blurred vision. This may be a result of poor
contrast due to optic nerve atrophy. These patients may benefit
from good lighting and filters.

Contrast is defined as the ratio of two quantities: 1) the difference between


the peak luminance and the average luminance and 2) the average luminance.
It is typically represented by a number between 0 and 1 (or sometimes 0 and
100 percent); the higher the number, the more contrast. The Snellen chart has
virtually 100% contrast.
Measuring the Contrast Sensitivity Clinically
With Sinusoidal Gratings: The patient is presented with vertically oriented
gratings at various contrast levels with various spatial frequencies (space be
tween the gratings).
Charts available: The Arden Plate Test, Vistech Chart
The curve will be plotted with spatial frequency on the x-axis (cycles per
degree) and contrast sensitivity on the y-axis (decibels). The plot (see Fig
ure 17.1) will have a single maximum with the peak at around 3-6 cycles per
degree (1).

498

CHAPTER 17. PERCEPTUAL FUNCTION/COLOR VISION

Question 4: correct answer d - a colored filter A filter can aid in


increasing contrast. An increase in lighting would also be a good option for
this patient.
Filters allow some light to pass through and some light to be absorbed depend
ing on wavelength. Often filters are used for low vision patients to decrease
glare, enhance patient comfort, and/or increase contrast.
Clinically Useful Filters
Narrow band filter- only lets a small range of wavelengths through the filter,
the rest are absorbed
Interference filter- only transmit one wavelength
Broad band filter- allow a large band of wavelengths to pass
Long-pass filter- Allows only long wavelength light to pass (to be transmit
ted)
Neutral density filter- transmits all wavelengths but decreases the amount
of light entering eye equally, gray in color, can be used as a sunglass lens,
do not alter color perception
Low Vision application Blue Blockers- These are amber tinted lenses
which block short wavelengths. Low vision patients often find these helpful
in reducing discomfort glare outside which is a result of shorter wavelengths.
They alter color perception (4). This type of color vision distortion is termed
a chromatopsia (2).
Filters come in a variety of colors and the transmission is specified as a %,
meaning the amount of light which is allowed through the lens. Therefore, a
20% lens is darker and lets half the light through compared to a lens which is
a 40%.

Clinical pearl- Neutral density filters can be used to measure the


severity of an afferent pupillary defect (APD). Filters of increasing
density can be put over the good eye until the APD is no longer
visible on the bad eye. (2)

Question 5: correct answer a - critical flicker fusion frequency (CFF)


Spatial vision is the ability of the eye to detect changes in brightness and to
detect fine details (related to perception of space). It is measured clinically
with visual acuity and contrast sensiti>'ity.

17.3. CASE 72

499

Temporal vision is used to detect mot.ion (rela ted to percept ion of time) . The
CFF is measured by showing a pa tienL a light which cont inues to flicker faster
and faster. Eventually the light no longer appears to flicker and just appears
continuous. This is the high te mporal resolution limi t which is measured in
hertz. One hertz is eq ual to one cycle per second. (2)

17.3

Case 72

This section reviews important Visual Perce ption concepts

Questions:
Question 1: Which pathway is likely damaged first in glau coma?
a. mag nocellular pat hway
b. parvocellular pathway
c. equally da maged
Question 2: Which of the following is a characteristic of scotopic
vision?
a. goo d color discriminat ion
b. good acuity
c. uses cones
d. high contrast sensitivity
e. sensitive to dim lights
Ques tion 3: If a patient's pupil was moved off-center but rem a ined
the same diameter, how would the patient's perception of the bright
ness be affected?
a. increased
b. decreased
c. stay t he same
Question 4: Eye movements such as microsaccades while reading
prevent which of the following effects:
a. Stiles-Crawford effect
b. Troxler effect

CHAPTER 17. PERCEPTUAL FUNCTION/COLOR VISION

500

c. t richromacy
d. Broca-Sulzer effect
e. Brucke-Bar tley effect
f. masking

Answers:
Question 1: correct answer a - magnocellular pathway
have two major classes : magnocellula r and pa rvocellular.

Ganglion cells

Magno cells - involved wit h Motion- t hink i'vlagno, Motion


The where syste m- for detecting and a lerting
peripheral retina
respond to high temporal fr equencies, low spat ial frequencies
have larger receptor fi elds- t hink Magno, large
more se nsitive to la rge sti muli than spatial u.etails
fast
project t o Area rvIT in the vis ual cortex
Parvo cells - color perception
t he what system- fin e details
resp ond to low tempora l frequencies
have hig h spatial fr equency resolution
most ly foveal
have smaller receptor fi elds
slower
Clinical Application r-Iagno cells are thought to be damaged fir st in glau
coma. The FDT (frequen cy doubling perimetry, specifically The Matrix) is a
peri p her al vision test which selectively ta rgets magno cell s. T he patient views
fli cker ing sine wave grat ings which is a bove the CFF of the parvo system. This
allows for isol ation of the magno pat hway to detect early damage. (2)
Question 2: correct answer e - sensit ive to dim lights
answers a re all characteristics of t he photopic syste m.

The other

Human Light Perception


Scotopic vision o ccurs in dim lighting conditions where the rods a re being
used . As a resu lt , color di scrimination a nd visual acuity are poor. There
is a great li ght sensiti vity due to high spatia l summatio n. This system is
maximally sensit ive at 507 um. (2)

7.3. CASE 72

501

Photopic vision occurs III


visual
summation but
system is

used. As

Mesopic conditions use both


be
television in a dark room; in this
be stirnulatecL

An

would
would

both

Practical Application \\'hen


If a person looks at the
rods are used for night vision and

3: correct answer b - decreased

that strikes cones


will

!lon-orthogonal
an

to hi t the cones at an angle different than


dimmer.

fixaUon to

Care Optometry, Fifth edition.


Heinemann,
Schwartz S. Visual
1999.

A Clinical Orientation. 2nd

and \VooeL I\:\Ih Part 1


2009.
Brilliant R.
1999.

Science Review

of Lmv Vision Practice. Butterworth

HCH'CHWX'"

Chapter 18

Visual

Sarah

Human Develop

O.D., MS, F.A.A.O.

1B.1. CASE

505

18.1

73

8
Chief complaint: loses
hind

when

illness
moderate

Duration: whenever
Patient ocular history: umemarkable
Patient medical

ADHD

Medications taken by

Ritalin

Clinical findings
Habitual

refraction
OD: pi DS
distance and near
OS: pi DS
distance
near
Cover test
distance: ortho
near:
Results fell in the 13th nD',,"D.n'.

DEM

All ocular health

were wnl

eye movement is the DEM


test
a.
b,

d,

c. ductions

f. vestibulo-ocular reflex

he is

.506

CHAPTER 18. VISUAL AND HUi\IAN DEVELOPl'vIENT

Question 2: Which of the following is a FALSE statement regarding


learning disabilities and learning related vision problems?
a . they are often found coexisting in children with ADHD
b. symptoms may include distraction, red uced read ing ra te, poor compre
hension, tas k avoidance
c. la nguage delays are often common
d. dyslexia is a term used by teachers and other non-medi cal professions and
is not a recognized problem by medical professionals
e. treatment may require correcting of refractive error and binocular prob
lems, as well as, in-office vision t herapy for improvement of vi sual infor
mation processing
Question 3: Which of the following is NOT a skill of VIP (visual
information processing) testing?
a. directionality
b. visual motor integra tion
c. vergence
d. bilateral integrat ion
e. fine-motor coordina tion
Question 4: What testes) are used to assess VMI (visual motor in
tegration)?:
a. , VoId sentence copy ing test
b. Developmental test of VrdI
c. DEl'd

d. R-L awareness test


e. L-R reversal test
f. King- Devick

Answers:
Question I: correct answer b - s a ccades Saccades a re the very rapid ,
yoked eye movements which are used during reading. They can also be ini t i
ated by a su dd en visua l, auditory or peripheral stimulus. They can be tested
clinica lly with the DEi\r[ test. During t esting t he patient will call off a series
of numbers (in either a horizontal or vert ical series) as q ui ck as possible. The
accuracy a nd speed will be recorded.
How to interpret: Any score below t he 15th percentile is considered signifi
cantly poor. (1) O t her t ests of saccadi c function: King-Devick, NSUCO

18.

CltSE 73

of the other eye movements:

Smooth
move the
The ?\SDCO test has

can

Versions refers to the

movement of the eyes.

directions

refers to

Ductions

monocular

followed.

lllovements.

Vestibulo-ocular reflex prevents


movements
than :30 seconds.
Optokinetic nystagmus stabilizes
than 30

across the reti l1a


[or

head

movements

the processes of
the term should
(\\lord ,,",;uvH1\.
left confusion. (2)

are

formatioll
considered
3: correct answer c

All

choices are

vergences.
related vision
from

Visual efficiency

visual

processes

for VIP

508

CHAPTER 18. VISUAL AND HUMAN DEVELOP]\lENT


a. refractive error
b. accommodation
c. vergence
d. motility

Visual information processing - higher brain fllnctions- giving meani ng to


what is seen
a . vis ua l spatia l or ientation ski lls- the awa reness of one's position rel
ative to t he envi ronment
bilateral integration- using both sides of the body
latera lity- knowing one's own right an d left
directionality- know right and left in the environment
b. Visual analysis ski lls
non-motor skills- vi sual perception
Includes: visual discrimination, visua l figure ground discrimina
tion, visual closure, visua l memory, visualization
VlvII (vis ua l motor integration) - processing of visual infor mation
and responding with fine motor activity
Includes: visual analysis of t he stimulus, fine-motor control,
visual conceptua lization
fine-motor coordination
a udi tory-visual integration
c. Rapid naming- quicldy recognizing a visual symbol
d. Exec ut ive functions- brain processes needed to acco mplish a ny goal
directed behavior
In general, early detection of LRVP is critical to avoid deJays in academic
achievement, During school, the vis ual demands increase th rougho ut the years
and problems will only get exacerbated.

The Denver Developmental Screening Test should be used if t here


is a history of developmental delay, This can detect learn ing related
vision problems.

Evaluation of Visual Efficiency


Testing involves t he visual acuities, refract ion , ocu lar motility a nd alignment,
and accommodati ve-vergence Eu nction.

CASE 74

Important tests to know: NSUCO


and SCCO 4+
ane! saccades

of

DEM

l\Iovemen(

Saccade Test-

ane!

both

Treatnlent of visual

deficits: see binocular vision section

Evaluation of Visual Information

Awareness Test,

Visual
tal Test

Executive function- Children's Color Trail

ll'eatment of VIP deficits:


been done.
in-office
for about
such as

20<m minutes.

Goal is to
home. At

Learn

correct answer a and b The VVoid sentence


of Vl\1I are
to evaluate
motor
above.

74

18.2

month old infant with mother

Chief

mother is concerned because

of

illness

the

seem to

CHAPTER 18. VISUAL AIVD HU1IIAN DEVELOPj\.fENT

510

Location: not always t he same eye

Duration: for a short period of ti me

Frequency: a few times a week

Secondary complaints/symptoms: excessive tearing a nd discharge OD


Patient ocular history: none
Family ocular history
n1other: none
father: none
Patient medical history: normal term baby, no birth complications
Medications taken by patient: none
:M e ntal status
Mood: happy a nd a ler t
Clinical findings
E01\1s: fixes a nd fotlows OU
Cover test no objection to covering of either eye, no eye t urn seen upon
examination
Bruckner test equal reflex between the two eyes
Hirschberg displaced slightly nasally OU
Retinoscopy +300-050x090 OU
Slit lamp
lids/ lashes/adnexa: large lacr ima l lake, mucus extr ud ed from punc
tum upon application of pressure OD, wnl OS
conjunctiva: white OU

Questions:
Question 1: What is the treatment and management of this infant 's
eye turn?
a. prism
b. referral for EO}';1 muscle surgery
c. glasses
d. no treatment is needed
e. patching

CASE 7q

511

2: \Vhat is the most

cause of the

tearing

b, nasolacrimal cl uet obstruction


c,

neonatorium

d, viral infection

3: The mother asks: In general, when should a child be


in for an eye examination?
a.

before the start of school

b, not unle",s

are

symptoms

c, age 8

d, (:j

3 years and
4:

When do you

to school
a child to reach adult level of

resolution
a. at

b, 6

year

d. 6-8
5: What age, on average, is a child

to

to

walk?
a, 6 months

b.

c.
d. 18 months
e. 24 months

Answers:
answer d - no treatment is needed
the
be
, cover test,
or asymmetry
t.he eyes. Concern should
more constant. Parental education should be done to
jf

js

t he same

512

CHAPTER 18.

VISUAL AND HUMA]-l DEVELOPIIIENT

Question 2: correct answer b - nasal lacrimal duct obstruction Naso


lacrimal duct obstructions can be congenital or acquired in origin. Congenital
cases generally occurs due to a blockage of the valve of Hasner by a membrane.
This membrane usually spontaneous ly opens at. about 1-2 months. Treatment
is generally conservative until 6-1.3 months if spontaneous resolution has not
occurred, at which time probing is necessary. (3) Until that time, erythromycin
ointment and massage can be used.
This mother was very at.tentive to her chi ld and should be commended for
bringing her child in to get an eye examination.
Parents should be educated to watch for the following signs in babies:
excessive tearing
red or crusty eye lids
eye turn which is fairly constant
a white pupil
extreme light sensiti vity (q)

Question 3: correct answer d The AOA recommends eye examinations


at 6 months, 3 yea rs of age and prior to starting school.

Question 4: correct answer c - 3-5 years A newborn is expected to have


a round < 2.5 diopt.ers of hyperopia ancl an acuity of around 20/600. Astigma
tism of up to two diopters is also common in infants. As t he child grows, typi
cally emmetropization decreases the amount of hyperopia. Emmetropization is
an active process believed to adjust the axial length and anterior segment power
to try to achieve emmetropia at a higher rate than would occur by chance. It
typically stops by about 18 months of age. (6) By age one, the acuity is about
20/100. The adult level of visual acuity is typically reached around age 3-5.
Vernier acuity (the ability to detect very small misalignment of lines) is the
slowest visual function to develop.
The measuremellt of grating acuity in an infant can be performed using: Op
tokinetic nyst agmus (OKN) drum, preferential looking using Teller Acuity
cards, and visual evoked potentials (VEP). (5)

Guidelines for axnounts of hyperopia outside normal limits:


infant: greater than or equal to +3.50D
1- 7 years: gTeater than or equal to +2 .00D
8-12 years: greater than or equal to +1.50D (7)

18.3.

lVlodes of

HOTV

Preschool age

Question 5: correct answer c

cards

broken wheel

12 months

J'v1ajor non-visual milestone in childhood development:

3 months- lifts head and

when

on stomach

5-6 months- rolls oyer

the

is

will

grasp your

6 years- knows

from left

9-13 111011ths- crawls

1 year-

to walk

18 mont.hs- knows several words


2 years-

18.3

to run

Case

Age/race/gender: 6 year old


Chief complaint: no

female

no eye turn or

per

Patient ocular history: first eye exam


ocular history unknown
Patient medical history; mental retardation
l\1edications taken by patient:
Patient

history: NKDA
distract-able, moderate

Clinical

difficult exam due to

e",')n,pr"

",vmrw"

mother

CHAPTER 18. VISUAL AND HU1IIAN DEVELOPl\IENT

514

Dry retinoscopy
OD: +200 ds

OS: +200ds

Facial characteristics: Rat mid face, short nose, small palpebral fissures, thin
upper lip

CT: RET
versions: full ou

Questions:
Question 1: Based on the patient's facial characteristics, what is the
most likely cause of the mental retardation?
a. Down 's syndrome
b. Fragile X syndrome
c. Fetal Alcohol Syndrome
d. Cerebral Palsy
e. Autism

f. Asperger's syndrome
Question 2: \Vhat optic nerve disorder may also be found
patient?

III

this

a. pseudotul11or cerebri
b. morning glory sy ndrome
c. optic nerve coloboma
d. Leber 's optic neuropat hy
e. optic nerve hypoplasia

f. Foster Kennedy sy ndrome


g. melanocy toma
Question 3: Which of the following is FALSE regarding Down's Syn
drome?
a. It is a result of trisomy 21
b. lvIay have an association with keratoconus
c. High refractive error (typically myopia) may be associated
d. There is increased risk of strabismus
e. Occurs in approximately 1/10,000 births

f. The risk of Down 's increases with maternal age

18.3. CASE 75

.515

Question 4: What is considered average on an IQ test?


a.

70

b. 85
c. 100

d. 120
e. 140
Question 5: What condition is a result of an absent X chromosome?
a. Klinefelter's syndrome

b. Edward's syndrome
c. Turner's syndrome
d. Down's syndrome

Answers:
Question 1: correct answer c - Fetal Alcohol Syndrome
Disorders which impact child development
Fetal Alcohol Syndrome- due to consumption of alcohol by mothers during
pregnancy, varying degrees of mental retardation, abnormal neurobehav
ioral development, and facial abnormalities (flat midface. short nose, thin
upper lip, small palpebral fissures). Leading cause of mental retardation.

(9)
Cerebral Palsy- a group of motor disorders characterized
impaired volun
tary movement. from prena tal developmental abnormalities or postnatal
eNS damage before age of 5. Demonstrate non progressive spasticity,
ataxia, or involuntary movements. (9) Strabismus . accommodative insuf
ficiency, and refractive error are common .. (10)
Down's Syndrorne- see

Fragile X Syndrome- most common cause of familial mental retardation,


enlarged testicles, X-linked recessive but males can just be carriers 20%
of the time (12).
Autisrll- A syndrome characterized
extreme alooflless, speech and language
compulsive phenomena, and poor intellectual development and
even mental retardation. More common in
and manifests before the
age of 3. (9)
Asperger's syndrome- more highly functioning form of autism

CHAPTER 18. VISUAL AND HWIiA N DEVELOPJIIEN T

516

Question 2: correct answer e - optic nerve hypoplasia Optic nerve

hypoplasia is an incomplete development of the optic nerve, the nerves a re

small and surrounded by a ring of scler a and ring of hyperpigmentation (double

ring sign). It is associated with maternal alcohol or drug use.

Question 3: correct answer e Down's synd rome , trisomy 21 , is the most

cOl11mon chromosomal disorder, approximately 1/ 1000 births (ll). It is char

acterized by mental retardation (all have an 1Q below 65), prominent epicant ha l

folds, a sim ia n crease, a Aa t face, and increased risk of dementia, leukemia, and

conge nital heart disease. lII aternal age has a strong inAuence. High refractive

error and strabismus are common. There is an associat ion with keratoconus (3).

It is unclear in t.he literat ure if Down's syndrome patients have increased sen

sitivity to at ropi ne.

Question 4: corre ct answer c - 100 1Q testing- tests various as pects of

cogn iti ve ability. The most commo n test is t he W e chsler test (WISe). This

test allows for comparison of verba l comprehension and perceptual reasoning. If

perceptual reasoning is much lower t han verbal comprehensio n, vision therapy

may be indicated to enhance visua l processing sk ills.

Average IQ= 100 with a standard deviat ion of 15

Therefore , 66% of people fa ll bet'ween 85-115 and 95% fall within 70-130. (1)

Question 5: correct answer c - 'IUrner's syndrome


is XO and effects 1/ 3000 females.

Turner's synd rome

Klinefelter's syndrome is XXY a nd occurs with male hypogonadism .

Ed ward 's sy ndrome is trisomy 18 a nd occurs in 1/8000 children .

References
[1 ]

Scheiman , M. Wick , B. Clinical Management of Binocular Vision, Het


erophoric, Accommodative, and Eye lIIovement. Disorders, Third edition.
Lippincott , Vv"i llia ms, and \~i ilkins , 2008.

[2] AOA Optometric Clinical Practice Guidelines , aCare of t he Pat ient wit h
Learning Related Visual ProblemsO. AOA website , revised 2008.
[3]

Friedman N, P ineda R, Kaiser P. The lIIassachusetts Eye and Ea r In


firmary Illu strated Manual of Ophthalmology. W.B. Saunders Company,
1998.

[4] AOA Infant Vision: Birth to 24 Months of Age . AOA.org/ x9429.xml

18.3. CASE

517

Schwartz S. Visual
1999.
Mutti,

A Clinical

Oriel1ta~ion.

2nd edition.

a!. Refractive

m Human Infants.

Volume 81, Issue 10.

~Iarsh-Tootle

WL and Frazier i\IG,

30 ill Borish's Clinical


Second Edition, Eds.

and Borish t\L Elsevier, St Louis ]1([0, 2006.

[8] Scheiman and Rouse,

of

vision

2006
[9] Beers 'vI, Berkow R ?vIerck I\lanuRl of
I\ferck Research Laboratories, HJ99.

edition.

V, Bennett

[11]

J
Cotran R, Robbins S. Basic
1997.

6th edition, 'V.B. Saun

Chapter 19

Legal Issues,
h

hies,

Sarah

19

ublie

19.1. CASE 76

521

Case

Recommend
Standards of Conduct and the

the AOA

The
Code of Ethics.

Questions:
1: The "Contact Lens Law" which has been established
~,.,,~,~v~ the release of contact lens
prescriptions. Which of the
is TRUE
the law'?
must be released to the patient once the
b. Contact lens

valid for 2 years


has to be

c. The
ask

d. The contact lens law does


e. In no

of the individual
the

if

to cosmetic contact lenses

is an

contact lenses

will order their contact lenses from


and they are free to do so. The
contact lens
verification requests froITl contact lens sellers to prescribers, which
of the
is true?
a. Once the seller has contacted tile
the
has
hours to respond
b.

does not

the

in the alloted time. the seller has to

c.

d. A seller is required to
e.

recei \'ed the verification

seller can contact


not automated
call

3: \Vhich of the
to follow the HIPPA
a. H.i\JO's

direct

is an

which is not

call but

522

CHAPTER 19. LEGAL AND ETHICAL ISSUES

b. 1l'Iedicare
c. Healthcare p ro viders
d. nursing homes
e. employers

f. none of t he above
Question 4: According to HIPPA, which of the following would b e a
violation of the release of protected health information by a health
care provider?
a. release of yo ur medication Jist when requested by your employer
b. reporting of epi demic keratoconjunctivitis (EKe) episode to a govern
ment agency whi ch t racks outbreaks in t he area
c. sharing a patients medical condit ion wit h friends or fa mily, if t he patient
agrees
d. health infor mat ion can be shared to a llo\\' for pay ment to doctors or
hospita ls
Question 5: There are 18 patient protected hea lth information (PHI)
elements which must be removed in order for the information to be
disclosed and used freely. This assures that the p a ti ent will not
be able to be identified by any of the information. Which of the
following is NOT considered PHI:
a.. name

b. socia l security numb er


c. e-mail address
d. phone num ber
e. medi cal record number
f. full face photograph

g. [ax number
b. post al address
i. birth date
J. a ll of the above are considered PHI

Question 6 : During a job intervi ew , which subject is allowed to be


addressed with the interviewee?
a. marital/ famil y status
b. disability
c. birthplace

19./. CASE

(1.

for terrnination from

r. race
g. sex
L

of the above

U",,,,CIVll

7: The Federal Patient's Bill of

includes all of the

following
to make informed decisions about their health,
understood i nfofma tion

have the

consurners

to

b.

to fully
to be '-"""'''''0''"
members
have confidence that their
health information
with health care
e.

must make
obtainment of healthcare seryices

to meet financial

f. all of the above are true

8: Which of t.he

is t.he definition of nonmalfi

.
?
Clence.

a. an individuals right to

their own decisions

b. do no harm

others

d. distribution of
and services
cases which are similar

\V

hieh is

and the

Question 9: Informed consent includes all of t.he


", Children and those with insufficient mental competency
to
consent
b. the

the
d, the

or
l>r"n""pn

risks and benefits of

for this

treat,ment

treatment of

EXCEPT:
SUlTO

.524

CHAPTER 19. LE GAL AND ETHICAL ISSUES

e. a lterna tive t rea tment options

f. mus t. always be writ ten and signed by the patient

Question 10: If an optometrist refers a pati e nt to a sp e cialist, which


of the following is NOT true?
a. The optometris t rema ins responsible for proper t rea tment of the pat ient
once t he referral has bee n made
b. In many states , it is illega l to receive mo ney for ma king a referral
c. The optometrist should provide t he patients medical history and records
to the referring doctor
d. The optometrist should ha ve adequate knowledge of the s pec ialist to as
sure that they are compete nt to manage and t reat the pa t ient
e. The specialist is no t required to send the pa tient back to the optometris t
a fter t he treatment is comple te

Question 11: An optometrist refers a lVledicare pati e nt to a cataract


surgeon. The optometrist is a part-owner of the practice of the
surgeon. This is a violation of which law?
a . The St ark Law
b. Anti -trust law
c. HIPPA
d. Patient Bill of Rig hts

Question 12: Which of the following is FALSE regarding owne rship


of patient records?
a . An optometr ist is obli gat.ed to keep a adequa te record of all patients seen
b. If an op tom etri st. is a n owner or pa rt ner in a practice and decid es to
lea ve t he pra ct ice, they have joint owners hip over a ll of t he records in t he
practice. This inc! udes patie nts which t he opt omet rist did not ma nage.
c. An indep endent contractor has a legal ownership over Lhe p atient records
but only t he pat ients seen by t his optomet ri st.
d. A p atient does no t own t heir phy sical record but they have a righ t to the
information in the record
e. An optometri st who is a n e mployee in a practice. has the lega l right to
the pati ent records bu t only the pa tients see n by this optometrist.

19.1. CA SE 76

525

Question 13: A patient calls the office and talks to the fr ont desk staff
complaining of n e w onse t double vision with a very painful headache.
The patient is scheduled for an appointment in 1 week. The patient's
diplopia and headache were caused by a brain aneurysm which rup
tured before the appointment occurs. \Vho is at fault?
a . the front desk staff

b. the doctor
c. the patient
d. no o ne's fa ult, this is simply bad luck

Answers:
Question 1: correct answer a A contact lens prescription must be released
to t he patient after t he fi tt ing is complete rega rdless if the pat ient asked for it or
not. The deci sion regar ding when t he fit ting is complete is based on necessicy
of follow-ups a nd on the doctor's judgement. Th e contact lens prescript io n
is valid for a minimum of one year. If t he spec ific stat.e has a law where t he
prescription is valid for a longer period of time, t he states law is upheld. T he
minimum is one year unless there is a medically indicatio n for a shorter period
of ti me.
The law applies to any type of contact lens whether it is for vision correction
o r cosmetic purposes.

A prescriber can be a n ophtha lmologist, optometr ist or possibly opticians, if

the state la ws allow for it. (1)

Question 2: correct answer a A presc riber has eight business hours to


respond to communication from t he seller. If this does not occur , t he seller
can fill t he prescription without anot her attempt. The seller is not required
to acknow ledge receip t of the rep ly. HIP PA states patient authorization is not
required here because it is req uired by law to disclose tllis protected health
information to the t.h ird party if th e information is for t rea tment. Cont act lens
verification is considered a treatment.
All the forms of communication a re valid to verify a prescription: fax, e- mail ,
direct phone call or a utomated phone call. (1)

Question 3: correct answer e From the website of U.S. Department of


Healt h and Human Services: "The HIPAA Privacy Rule provides fed era l pro
tections fo r perso nal heal t h informat ion held by covered enti t ies a nd gives pa
tients an array of rights with respect t.o that informatio n. At the same tim e,
t he Privacy Rule is bal anced so that it permits the disclosure of perso nal health
information needed for patient. care and other im portant p urposes.

526

CHAPTER 19. LEGAL AND ETHICAL ISSUES

The Security Rule specifies a series of administrative, physical , and tech nical
safeg uards for covered enti t ies to use to assure the confidentia lity, integrity,
and availability of electron ic protected hea lth information."
Those required to comply to HIPPA: Health Plans, most Hea lth Care P rovide rs
and Health Care Clearinghouses (an entity which receives and processes health
inform ation such as a billing service).
Those not req uired to comply: life insurers , employers , workers comp ensation
carriers, ma ny schools and school d is tricts, many state agencies like child pro
tective service agenc ies , many law enforcement agencies , and many municipal
offices. (2)
Question 4: correct answer a
Question 5: correct answer j The other 9 elements include: account
number , health plan beneficiary numbers, cer tificate/ license numbers , veh icle
identi fiers, device identifiers and serial num bers , URLs, biometric identifiers ,
internet protocol address numbers, or a ny other unique id enti fying number,
characterist ic , or code (3)
Question 6: correct answer d - reason for termination from previous
employment All of t he other choices are not allowed by law to be asked at
a job inte rview. In addition , t he hiring process may not be influenced by these
fa ctors.
Question 7: correct answer f - all of the above are true The P a
tients' Bill of Rights a nd Responsibilities has t hree major objectives: First,
to strengt hen consumer confidence by assuring the health care system is fair
and responsive to co nsumers' needs, provides consumers with credible and ef
fective mecha nisms to address their concerns, and encourages consumers to
ta ke an active role in improving a nd assuring t heir health. Second , to reaf
firm t he importa nce of a st rong relationship between pat ients a nd t heir health
care professiona ls. Third , to reaffirm the critical role consumers play in safe
guarding their own hea lth by establishing both rig ht s a nd responsibiliti es for
a ll participants in improving heal t h status . (4)
Question 8: correct answer b - do no harm These four concep ts make
up a se t of critical eth ica l principles in hea lthca re with which all optometrists
s hould be familiar.
Choice a is au tonomy, choi ce c is beneficence , choice d is justice (5)
Question 9: correct answer f - must always be written and signed
by the patient All of t he above a re true regarding informed consent except

19.1. CASE 76

527

the requirement to be written and signed by the patient. Depending on the


risk or severity of the procedure/diagnosis, a simple consent may be all that is
needed. For example, for the low risk procedure of dilation, a verbal explanation
and consent can be received from the patient without written authorization.
In the chart, the consent can be documented but written authorization is not
required. If a surgery is to be performed, the risks are much higher and written
documentation would be required in this circumstance. (5)

Question 10: correct answer a The specialist is responsible for the treat
ment of the patient once the referral has been made. The optometrist should
follow the progress of the patient but is not longer responsible. The optometrist
does need to send any patient medical records to assist in the care of the patient.
(6)
Question 11: correct answer a - The Stark Law The Stark Law states:
It is illegal to refer a medicare or medicaid patient to a designated health service
for which the referring doctor or a member of their immediate family have a
financial relationship such as ownership or a compensation arrangement. There
are exceptions to this rule.
Anti-trust Laws protect consumers from actions that restrain trade and main
tain the free market economy. These laws are applicable to optometrists. First,
it prohibits price-fixing. Optometrists cannot collaborate to set their fees for
optometric good and services. The fees should be independently determined
for each office. In addition, optometrists cannot ban together to group boy
cott a particular business or supplier. For example, based on the Anti-trust
laws, it would be illegal for the AOA, a group of competing optometrists, to
recommend avoiding a specific frame supplier.
HIPPA and the Patient Bill of Rights have been previously discussed. (6)

Question 12: correct answer d An employee has no legal right to any of


the patient records upon departure from the practice.
Question 13: correct answer b - the doctor The doctor is responsible
for their own staff. It is the doctor's job to train them how to triage calls from
patients.

References
------ -- ---------- --.-------
...

[1]

www.ftc.gov/bcp/edu/pubs/business/health/bus62.shtm

[2]

www.hhs.gov/ocr/privacy/hipaa/understanding/index.html

[3]

vy'Ww.AOA.org website under Regulatory Issues/HIPPA

Index

adult inclusion conjunctivitis

treatment of, 45

age-related macular degeneration

dry ARMD, 155

management of dry ARMD, 157

overview of, 154

treatment of wet ARMD, 158

wet ARMD, 156

AIDS, 402

AION, 196, 200, 208

alamasL, 324

Alanine transaminase (ALT), 280

albinism

overview, 124

albuterol, 321

mechanism of action, 297

alcoholism

case on, 280

Alexander's law, 457

allegra, 324

alocril, 324

alomide, 324

alphagan, 296

alphagan P, see alphagan

Alrex,244

amantadine, 315

amblyopia, 467, 484

refractive, 482

strabismic, 467

treatment, 472

ametropia

axial, 354

4 base out test, 478, 479

5-fluorouracil, 322

8-ball hyphema, 259

Abbe value, 388

aberration

chromatic, 388, 389

spherical, 388, 389

ACj A ratio, 441

acanthamoeba, 23, 67, 269, 397

accommodative

amplitude, 364, 444, 448

demand, 364, 456

facility, 443, 455

ill-sustained, 455

accutane,8

adverse effects of, 330-332, 334

acetaminophen, 312

acetazolamide, see Diamox

acetylcysteine, 322

achromatopsia, 492

acne rosacea

treatment of, 52

acular, 327

acyclovir, 310

ADHD,507

Adie's tonic pupil, 189

adrenergic agonists, 232, 295

adrenergic antagonists

systemic drugs, 297

topical ophthalmic summary of,

297

529

INDEX

530
refractive, 354

aminocaproic acid, 261

aminoglycosides, 306

amiodarone, 120, 319

adverse effects of, 330, 331

adverse effects summary of, 319

amitriptyline, 316

amoxicillin, 304

amphotericin B, 309

ampicillin, 304

anaglyphs, 451

anesthetics, 329

angioid streaks, 354

angiotensin converting enzyme (ACE)

inhibitors, 318

angiotensin II

receptor antagonists (ARB's),

318

angle of anomaly, 477

angle recession, 243

aniseikonia, 387, 388

anisometropia, 387, 483

ankylosing spondylitis, 108, 190

anomalous retinal correspondence,

476

anterior basement membrane dys

trophy (ABMD), 76

anti-trust law, 527

anticholinergics

adverse effects of, 332

anticoagulants, 320

antidepressants

adverse effects of, 332

antihistamines

adverse effects of, 332

antihyperlipidemic agents, 321

aphakia, 383

ARC, see anomalous retinal corre


spondence

Arden plate test, 496

area MT, 500

AREDS, 157, 239

argon laser trabeculoplasty, ALT,

232, 238, 249

Argyll Robertson pupil, 189

Aspartate transaminase (AST) , 280

Asperger's syndrome, 515

aspirin, 328

astigmatic keratotomy, AK, 89

astigmatism

against the rule, 359

compound, 358

irregular, 358

mixed,358

oblique, 358, 389

simple, 358

with the rule, 358

atopic dermatitis, 37

atopic keratoconjunctivitis (AKC),

36

atorvastatin, 321

autism, 515

azithromycin, 308

Azopt,244

b-scan ultrasonography, 85

bacitracin, 303

Bagolini lenses, 478, 479

Bailey-Lovie chart, 497

band keratopathy, 51

basal cell carcinoma, 16

base curve, 404

Behcet's disease, 108

beneficence, 526

benoxinate, 329

benzapril,318

Best's disease, 172

beta blockers, 232, 238, 297

bichrome, see duochrome test

Bielschowsky's head tilt test, 266

bifocals

double-D, 383

biguanides, 317

BIMjBOP, 452

binocular balance, 368

binocular cross cylinder, see fused

cross cylinder

blebitis, 241, 243

blephamide, 258

blepharitis

anterior, 22

posterior, 22

INDEX

seborrheic, 22

branch retinal artery occlusion

overview of, 132

branch retinal vein occlusion

overview of, 131

treatment of, 135, 136

brimonidine, see alphagan

Brock's string, 449

bromocriptine, 314

Brown's syndrome, 204, 265, 461

Bruckner test, 480, 513

bull's-eye maculopathy, 312, 334

Busacca nodule, 107

calcium channel blockers, 319

canaliculitis, 32

cancer

most common in men and women,

281

captopril, 318

carbonic anhydrase inhibitors, 232,

300

carotid cavernous fistula, 7

cataract

drugs causing, 62

nuclear sclerosis, 77

posterior subcapsular, 331

surgery, 92

cataracts, 352

CCTV,434

ceftriaxone, 304

cefuroxime, 304

celebrex, 328

celecoxib, 328

cellophane maculopathy, 154

central retinal artery occlusion

overview of, 132

treatment of, 136

central retinal vein occlusion, 208

overview of, 130

treatment of, 134

central serous chorioretinopathy, 154

prognosis and treatment of, 156

cephalexin, 304

cephalosporins, 303, 304

cerebral palsy, 349, 515

531
cetirizine, 324

adverse effects of, 333

CHAMPS study, 201

chemical burn, 257

chlamydial conjunctivitis

treatment of, 45

chloramphenicol, 307

chloroquine, 312

adverse effects of, 330, 334

chlorpromazine, 314

adverse effects of, 314, 330-332

cholesterol

normal values for, 178

cholestyramine, 321

cholinergic agonists, 232

indirect agents (AchE Inhibitors),

293

choroidal detachment, 95

choroidal nevus, 276

choroideremia, 140

cidofovir

adverse effects of, 333

cimetidine, 322, 324

ciprofloxacin, 303

circle of least confusion, 359

CLARE,407

clarithromycin, 308

claritin, 324

clear lens extraction, 89

clindamycin, 308

clinically significant macular edema

(CSME), 180

clonidine, 297, 319

CN 1, 222

CN 10, 222

CN 11,222

CN 12,222

CN 3, 190, 222

palsy, 214

CN 4, 222, 265, 460

palsy, 2H, 265, 460

CN 5,222

CN 6, 196, 222

palsy, 214, 215, 462

CN 7, 222

CN 8, 222

532
CN 9,222

coats disease, 167

cobblestone degeneration, 277

cocaine, 190

mechanism of action, 297

codeine, 313

Cogan's lid twitch, 216

combigan, 281

commotio retinae, 261

conductive keratoplasty, CK, 89

Congenital hypertrophy of the reti


nal pigmented epithelium

(CHRPE),276

congruity, 221

conjunctival intraepithelial neopla

sia, 60

conjunctival nevus, 61

conjunctivitis

adult inclusion, 43

allergic, 37

bacterial, 43

chlamydial, 43

giant papillary, 37, 407, 408

gonococcal, 43

trachoma, 43

viral,42

contact lens, 393, 398

care, 403

classification, 401

law, 521, 525

ortho-k,402

RGP, 401

. soft, 395

solution, 398, 403, 406

therapeutic, 402

verification, 403

x-chrom, 492

contrast sensitivity, 496

convergence insufficiency, 447

cornea

macular dystrophy of, 56

corneal abrasion, 20, 268

corneal erosion, 20, 67

treatment of, 22

corneal ulcer, 20

acanthamoeba, 23

INDEX
bacterial, 23

fungal,23

corneal verticillata, 120

drugs that cause, 77

corticosteroids

adverse effects of, 331, 333

cosopt, 281

cotton wool spots

differentials for, 162

coumadin, 85, 320

cover test, 485

CPEO,214

crepitus, 264

critical flicker fusion frequency, 498

cromolyn sodium, 324

Crouzan's syndrome, 276

cryotherapy, 151

curvature of field, 389

Cushing's syndrome, 210

cyclobenzaprine, 316

cyclopentalate, 294

cyclosporine, 51, 323

cytomegalovirus, 109

dacryocystitis, 32

Dalen Fuch's nodules, 261

debridement, 269

DEM, 506, 509

Denver Developmental Screening Test,

508

deuteranope, 492

Devic's disease, 199

dexamethasone, 325

dextroamphetamine, 297

diabetes

cataract formation, 80

diabetes mellitus

oral medications for, 317

diabetic retinopathy

clinically significant macular edema

(CSME), 180

high risk characteristics (HRC's),

179

overview of, 178

The Diabetic Retinopathy Study

(DRS), 179

INDEX
The Early Treatment Diabetic

Retinopathy Study (ETDRS),

180

vision threatening complications

of, 179

diamox, 196

side effects of, 277

diazepam, 313

dichromat, 491

diclofenac sodium, 327

dicloxacillin, 304

digitalis, 85, 142

digoxin, 85, 142, 319

adverse effects of, 319, 333

dilantin, 316

diltiazem, 319

disciform keratitis, 56

distortion

barrel, 389

pincushion, 389

Dk, 396,400

DLK,90

dominant drusen, 173

Donepezil, 315

Down's syndrome, 349, 515

doxycycline, 307

drance hemorrhage, 249

dry eye syndrome, 32

keratoconjunctivitis sicca, 49

treatment of, 51

Duane's syndrome, 204, 265, 462,

477

ductions, 507

duochrome test, 367

dyslexia, 507, 509

ECCE, 92

ectropion, 32

ectropion uveae, 124

Ehlers-Danlos syndrome, 285

elestat, 325

Elschnig spots, 162

emedastine, 324

emmetropization, 512

enalapril, 318

endophthalrnitis, 94

533
entrapment, 265

epicanthal folds, 473

epidemic keratoconjunctivitis, EKC,

42, 269

epinephrine

adverse effects of, 334

epiretinal membrane, 154

epithelial basement membrane dys


trophy (EBNlD), 76

erythromycin, 258, 308

esomeprazole, 322

esotropia, see ET

ET

accommodative, 470, 471

acquired, 470

acute, 470

consecutive, 470

infantile, 470

mechanical, 470

micro, 471

sensory, 470

ethambutol, 125, 309

adverse effects of, 333

exfoliation syndrome, see pseudoex


foliation

exit pupil, 429

exophthalmometry, 204, 265

exotropia, see XT

exposure keratopathy, 67

treatment of, 69

Fabry's disease, 276

face form tilt, 380

famotidine, 324

far point, 347, 353

FDT,500

fetal alcohol syndrome, 515

fexofenadine, 324

field of view, 390

filters, 498

blue blockers, 498

neutral density, 498

fixation disparity, 485

curves, 485

paradoxical, 486

flashes of light

534
differentials for, 145

flat fusion, 478

Bexeril, 316

flomax, 216, 299

adverse effects of, 333

floppy iris syndrome, 216

Buconazole, 309

fluoress, 329

fluorometholone (FML), 325

fluoroquinolones, 303

fluoxetine, 316

Buticasone, 321

FML, 244, 258

forced ductions, 204, 215, 265

foscarnet,311

fragile x syndrome, 515

Fransworth D-15, 490

Fransworth-MunselllOO-hue test, 490

Fuch's endothelial dystrophy, 56

treatment of, 57

fundus albipunctatus, 141

fundus flavimaculatus, 174

furosemide, 317

fused cross cylinder, 363,448

fusion

chiastopic, 452

orthopic, 452

fusional vergence

negative, 376

positive, 376

ganciclovir, 311

Gardner's syndrome, 275

gatifloxacin, 303

GDX, 231, 247

gentamicin, 306

giant cell arteritis, 209, 210

giant papillary conjunctivitis, 37

glasses

safety, 383

glaucoma

angle closure, 243

angle-closure, 107

congenital, 237

normal tension, 249

phacolytic, 236

INDEX
primary open-angle, POAG, 230

rubeotic, 243

uveitic, 243

glaucoma drugs

adrenergic agonists, 295

adrenergic antagonists, 297

carbonic anhydrase inhibitors,

300

hyperosmotic agents, 302

prostaglandin analogs, 301

summary of, 300

glaucomatocyclitis crisis, see Pos


ner Schlossman syndrome

glipizide, 317

glyburide, 317

glycerine, 302

Goldenhar syndrome, 276

gonioscopy, 230

goniotomy, 237

gout, 51

Graves disease, 195, 196, 200, 203,

214, 265

griseofulvin, 310

Gunn's sign, 205

gyrate atrophy, 140

Haidinger's brushes, 451

HEMA,400

heparin, 320

Hering's law, 479

herpes simplex virus

overview of, 66

treatment of, 68

herpes zoster virus

treatment of, 68

high risk characteristics (HRC's), 179

HIPPA, 525, 526

Hirschberg, 484, 513

histoplasmosis, 108, 156

homatropine, 294

homocystinuria, 285

hormone therapies

adverse effects of, 332

Horner's syndrome, 189, 190, 214,

462

horror fusionis, 477

535

INDEX
HRR pseudoisochromatic plates, 490

HRT, 247

human leukocyte antigen (HLA B27),

109

hydralazine, 319

hydrochlorothiazide, 317

hydroxyamphetamine, 190

hydroxy chloroquine, 329

adverse effects of, 330

hyperopia

absolute, 347

facultative, 347

high, 383

latent, 347

manifest, 347

hyperosmotic agents, 302

hypertension

malignant, 194

hypertensive retinopathy, 162

hypopyon, 243

hypotony, 94

ICCE,92

ice test, 216

idiopathic intracranial hypertension,

194, 334

imitrex

adverse effects of, 333

indomethacin, 328

adverse effects of, 328, 330, 334

inferior oblique, 222, 463

inferior rectus, 222, 463

inflammatory bowel disease, 108

INO, 201, 214

interferon, 201

interstitial keratitis, 56

IOL

IIls-UxeQ, 92

options, 93

subluxation, 95

IQ testing, 516

iridodialysis, 243

iridotomy, 237

iris

cyst, 114

metastatic lesions of, 115

nevus, 114

irritable bowel syndrome, 115

Ishihara plates, 492

isometropia, 483

isoniazid, 125, 309

adverse effects of, 333

isoproterenol, 297

isotretinoin

adverse effects of, 330, 332, 334

.lackson cross cylinder, 360, 428

januvia, 317

.laval's rule, 359

.lones testing, 33

just noticeable difference, .lND, 425,

427

keflex, 304

kenalog, 326

keratitis

disciform, 56

interstitial, 56

neurotrophic, 46

rosacea, 50

keratoacanthoma, 12

keratoconjunctivitis

phlyctenular, 119

keratoconjunctivitis sicca, 49

treatment of, 51

keratoconus, 402

Kestenbaum equation, 423

ketoconazole, 309

ketorolac tromethamine, 327

King-Devick,506

Knapp's law, 354, 387

Koch's sign, 205

Koeppe nodule, 107, 115

Krimsky,513

Krukenberg's spindle, 229

kwell,312

lacquer cracks, 156, 354

lagophthalmos, 205

LARS, 395

LASEK, 89

laser

536
Nd:YAG,249
LASIK,89

flap, 88

impact on lOP measurements,

90

retreatment, 90

lasix, 317

lateral rectus, 222, 463

lattice degeneration, 147, 277

overview of, 148

learning disability, 507

Leber's optic neuropathy, 200

legal blindness, 426

lens

aspheric, 383, 389

materials, 382

progressive, 365

rose-k, 403

lens subluxation, 284

leukocoria, 167

levodopa, 314

levofloxacin, 303

levothyroxine, 317

lindane, 312

lipitor, 321

Lisch nodule, 114

lisinopril, 318

localization, 452

loratadine, 324

losartan, 318

lotemax, 244, 325

lovastatin, 321

lucentis, 330

lumigan, 301

lupus, 199

Lyme disease, 108, 199

lymphoma

of conjunctiva, 61

M unit, 433

macrolides, 308

macugen, 329

macular dystrophy, 56

macular edema

Irvin Gass, 96

macular pucker, 154

INDEX
Maddox rod, 377

magnification

conventional, 435

relative size, 434

relative spectacle, 354

magnifier
collimating, 435

magnocellular, 500

malignant melanoma, 16

of conjunctiva, 61

mannitol, 318

Marfan's syndrome, 284

mast cell stabilizers, 38, 286, 324

maxitrol, 258

Maxwell's spot, 451

medial rectus, 222, 463

Meperidine, 313

metaproterenol, 297

metformin, 317

methicillin, 304

methotrexate, 322

adverse effects of, 333

methylphenidate, 297

metrogel, 307

metronidazole, 307

miconazole, 309

migraine

classical, 148

minimum angle ofresolution, MAR,

424,434

minipress, 318

minocycline, 125, 307

adverse effects of, 333, 334

miotics

adverse effects of, 331

MLF,201

molluscum contagiosum, 12

monoamine oxidase inhibitors, 315

monochromatism, 492

monocular estimation method, MEM,

348

monovision, 409

Morgan's Norms, 448

morphine, 313

moxifloxacin, 303

mucomyst, 322

537

INDEX
multiple sclerosis, 214, 230

Muro 128, 302

myasthenia gravis, MG, 213

myopia, 352

heredity of, 360

high, 354, 383

night, 352

pseudo, 353

shift,353

myotonic dystrophy, 80

Nagel anomoloscope, 491

nasolacrimal duct obstruction, 31,

512

congenital, 33

natamycin,310

near point, 353

near point of convergence, NPC, 447

near point triad, 364

negative predictive value, 191

negative relative accommodation, 363

negative relative accommodation, NRA,

448

nerve fiber layer, 247

neurotrophic keratitis, 46

nexium, 322

nifedipine, 319

nitroglycerin, 320

NO SPECS, 205

non-arteritic ischemic optic neuropa

thy

drugs that can cause, 164

nonmalficience, 526

norephinephrine, 190

Nott's method, 349

NSAIDS

adverse effects of, 331, 334

nystagmus, 456

congenital, 457, 485

drugs causing, 62

end-point, 456

jerk,457

latent, 457

OKN, 457, 507, 512

pathologic, 457

physiologic, 457

nystatin, 310

OCT, 247

ocuilox,303

ocular hypertension treatment trial,

231, 245

ocular ischemic syndrome

overview of, 178

prognosis and treatment of, 181

ocular myasthenia, 205, 213, 216,

462

ocular toxocariasis, 167

oculogyric crisis, 333

ofioxacin, 303

OHTN, 245, 246

oiopatadine, 325

omeprazole, 322

optic nerve, 222

optic nerve head drusen, 196

optic nerve hypoplasia, 516

optic nerve sheath meningiomas, 200

optic neuritis, 196, 199, 200, 209

optic neuritis treatment trial, 200

optivar) 325

oral contraceptives

adverse effects of, 333,334

orbital blow-out fracture, 265

orbital cellulitis, 7

orbital pseudotumor, 7

oscillopsia, 485

oseltamivir,311

osmoglyn, 302

oxygen permeability, see Dk

pachymetry, 231

pantoscopic tilt, 380

Panum's fusional area, 485

papillae, 38

papilledema, 219

papillitis, 200

papilloma, 12

Park's three step, 460

pars planitis, 109

parvocellular, 500

patients' bill of rights, 526

pattern standard deviation, 239

INDEX

538
pegaptanib, 329

Pelli Robson, 497

penicillin, 303

Percival's criterion, 448

phacoemulsification, 92

pharyngoconjunctival fever (PCF),

42

phenelzine, 315

phenobarbital, 316

phenylephrine, 106

phenytoin, 316

adverse effects of, 333

phoria

associated, 486

decompensated, 485

photopsia

differentials for, 145

photorefractive keratectomy, PRK,

88

pigmentary dispersion, 229,231,232,

237, 243

pigmentary retinopathy

drugs that cause, 175

pilocarpine, 231, 293

pingueculitis, 119

pinhole testing, 353

pituitary adenoma, 219

plaquenil, 329

PMMA, 396, 400

polycarbonate, 383

positive predictive value, 191

positive relative accommodation, 363

positive relative accommodation, PRA,

448

Posner Schlossman syndrome, 230

posterior capsule opacification, 97

posterior polymorphous dystrophy,

55

posterior vitreous detachment, 145

prazosin, 299, 318

Pred Forte, 244

Prentice's rule, 375

presbyopia, 365, 403

absolute, 365

contact lens, 408

emerging, 365

functional, 365

primary acquired melanosis (PAM),

61

prism, 373

diopter, 373

Fresnel, 266, 375

proparacaine, 329

prostaglandins, 232, 301

adverse effects of, 331

protanope, 492

prozac, 316

pseudo convergence insufficiency, 447

pseudoexfoliation, 95, 230, 236, 237,

243

pseudomonas, 269

pseudotumor cerebri, see idiopathic

intracranial hypertension

pupillary block, 95

pursuits, 451

push-up test, 444

pyogenic granuloma, 61

pyrazinamide, 125, 309

pyridostigmine, 216

pyrimethamine, 306

quixin, 303

racial melanosis, 61

radial keratometry, RK, 88

RAM GAP, 368

ranibizumab, 330

ranitidine, 322, 324

rapid card reagent (RPR), 110

Raynaud's syndrome, 250

Reiter's syndrome, 108

restasis, 51

retinal detachment

drugs causing, 62

overview of, 146

treatment of, 149

retinitis, 148

retinitis pigmentosa, 140

retinoblastoma, 167

prognosis and treatment of, 168

retinopathy of prematurity (ROP),

167

INDEX
retinoscopy

Bell, 348

dynamic, ,'348

static, 367

retroscopic tilt, 381

retrovir,311

Reye's syndrome, 328

rheumatoid arthritis

juvenile, 108

Ribavirin,311

rifabutin

adverse effects of, 333

rifampin, 125, 309

ritalin, 297

adverse effects of, 332

adverse effects on pupil size, 332

rocephin, 304

rosacea keratitis, 50

rubeosis

causes of, 178

saccades, 451, 485, 506

salmeterol, 321

Salzmann's nodular degeneration, 51

treatment of, 52

Sands of the Sahara, see DLK

sarcoidosis, 109, 199

satellite lesions, 269

sclera

causes of blue sclera, 125, 286

scleral buckle, 150

scleritis, lO6

comparison to episcleritis, 119

scleromalacia perforans, 107

sebaceous gland carcinoma, 16

seborrheic keratosis, 12

segment height, 382

Seidel sign, 93

selective laser trabeculoplasty, SLT,

238

selegeline, 315

sensitivity, 191

sensory fusion, 478

serotonin selective reuptake inhibitors

(SSRl's)

adverse effects of, 332

539
sertraline, 316

Sheard's criterion, 442, 448

Sherrington's law, 215

sickle cell, 261

side effects

summary of topical drugs, 330

sildenafil, 84

adverse effects of, 331

SILO,452

simvastatin, 321

sitagliptin, 317

slab off, 387

specificity, 191

spironolactone, 318

squamous cell carcinoma, 16

conjunctival, 61

staphylococcal marginal keratitis, 20

Stargardt macular dystrophy, 173

Stark law, 527

stereopsis, 478, 484

stereoscopes, 451

steroids, 325

mechanism for increasing lOP,

335

ocular adverse effects of, 326

triamcinolone, 326

Stevens-Johnson syndrome, 403

Stiles-Crawford effect, 501

succinylcholine, 295

sulfonamides, 303, 306

adverse effects of, 331, 333

sulfonylureas, 317

superior limbic keratoconjunctivitis

(SLK), 42, 119

treatment of, 44

superior oblique, 222, 463

Superior oblique tendon sheath syn

drome, see Brown's syn

drome

superior rectus, 222, 463

suppression, 451, 467, 476, 478

supraciliary effusion, 190

SVP,195

symblepharons, 257

sympathetic ophthalmia, 261

synechiae

INDEX

540
peripheral anterior, 243

posterior, 243

synthroid, 317

syphilis, 109

congenital, 57

rapid card reagent(RPR), 110

venereal disease research labo


ratory(VDRL), 110

systemic lupus erythematosus, 51

talc
adverse effects of, 334

tamifiu, 311

tamoxifen, 323

adverse effects of, 330, 334

tamsulosin, 299, 318

adverse effects of, 333

telescope, 426, 428, 431

magnification, 429

reading cap, 427

temporal arteritis, see giant cell ar


teritis

tensilon test, 215

terbutaline, 321

tetracycline, 8, 307

adverse effects of, 331

contraindications of, 46

The Diabetic Retinopathy Study (DRS),

179

The Early Treatment Diabetic Retinopa

thy Study (ETDRS), 180

theophylline, 322

thioridazine

adverse effects of, 334

Thygeson's superficial punctate ker


atopathy, 67

thymus gland, 216

thyroid eye disease, 7

timolol, 298

tobramycin, 306

tobrex, 258

Topamax, 190

toxoplasmosis, 108

overview of, 171

treatment of, 174

trabeculectomy, 238

trachoma, 43

tramadol,313

transient ischemic attack (TLA), 132

travatan, 301

triamcinolone, 326

trichromat, 491

deuteranomolous, 491

protanomolous, 491

tricyclic antidepressants, 316

trifiuridine, 310

trimethoprim, 303, 306

tritanope, 492

Troxler effect, 501

true exfoliation syndrome, 237

Tscherning ellipse, 389

tuberculosis

treatment of, 125

Turville infinity balance, 368

tylenol, 312

UGH syndrome, 97

usher's syndrome, 141

Uthoff's sign, 199, 205

uveitis, 106

anterior, 107

posterior, 108

traumatic, 266

treatment of, 111

valium, 313

vancomycin, 303

vascular endothelial growth factor

(VEGF),330
venereal disease research laboratory
(VDRL),110

VEP, 512

verapamil, 206, 319

vergences, 507

vernal keratoconjunctivitis (VKC),

37

versions, 507

verteporfin , 330

vertex distance, 348

vertical imbalance, 386

viagra, 84

adverse effects of, 331, 333

541

INDEX
vigamox,303

VIP, see visual information process

ing

viroptic, 310

vision

photopic, 500

scotopic, 500

spatial, 498

temporal, 498

Vistech chart, 496

visual acuity testing, 424, 513

EDTRS,424

visual efficiency, 507

visual field, 231

visual field defects

bitemporal hemianopsias, 219

Bjerrum scotomas, 220

central/centrocecal, 220

chiasmallesions, 219

homonymous, 220

macula only, 220

macular sparing, 220

quadrantopsia, 219

visual information processing, 507

visual motor integration, 508, 509

visudyne, 330

visuoscopy, 479

vitrectomy, 150

vitreoretinal tuft, 149

vitreous hemorrhage, 133

voltaren, 327

von Graefe's sign, 205

VOR,507

Vossius ring, 242

warfarin, 85, 320

Weill-Marchesani syndrome, 285

Wernicke's encephalopathy, 280

whorl keratopathy, 120, 330

drugs that cause, 77

Wilson's disease, 80

Worth 4 dot, 471,478

xalatan, 301

xanax,313

xanthelasma, 12

xanthogranulomas

juvenile, 115

XT,477

acquired, 477

consecutive, 477

infantile, 477

mechanical, 477

micro, 478

sensory, 477

zaditor, 325

Zafirlukast, 322

zantae, 322, 324

zidovudine, 311

zaear, 321

zoloft, 316

.zymar, 303

zyrtee, 324

You might also like